Sei sulla pagina 1di 320

DANYLO HALYTSKYI

LVIV NATIONAL MEDICAL UNIVERSITY

DEPARTMENT OF BIOLOGICAL CHEMISTRY

MCQs
in biochemistry

for students of medical, dentistry and pharmaceutical faculty

LVIV – 2012

1
Prepared by:
Prof. Sklayrov A.Ya., M.D., Ph.D
Prof. Lutsik M.D., M.D., Ph.D
Fomenko I.S., Ph.D
Klymyshin D.O., Ph.D
Nasadyuk Ch.M., M.D.

Editor: prof. Sklayrov A.Ya., M.D., Ph.D.

Reviewed by: prof. Pinyazhko O.R., M.D., Ph.D


prof. Komarytsia J.D., M.D., Ph.D

2
FOREWORD

Biological chemistry is a fundamental medical discipline. The understanding


of the processes of proteins, carbohydrates, lipids and water turnover as well as
the role of vitamins, trace elements and regulatory mechanisms in the organism is
the background for the interpretation of the laboratory indices in health and
disease in clinical practice.
The obtaining of the proper knowledge, based on the use of the different
methodological approaches, i.p. test control, is one of the main tutorial elements
for the students of the higher educational medical institutions.
Aimed at the improving of the students’ knowledge and their preparation for
the practical exercises, modul controls and taking an integral examination “Krok-1”
the department prepared a guide, which according to the program, includes: tests,
clinical cases, schemes of chemical transformations on key questions, which are
presented in Chapter 13 of the guide. The list of references includes the main
guides on biochemistry, accessible in the library of the university.
The authors hope that the use of this guide will improve the level of students’
knowledge and be helpful in mastering of the learning material.

3
CONTENTS
FOREWORD
Section І Control of the initial level knowledge. Biochemical
constituents of the cell. Methods of biochemical
investigations 5

Section II Enzymes and coenzymes. Regulation of metabolism. 19

Section III Metabolic pathways and bioenergetics. Tricarboxylic


acid cycle. Biological oxidation and oxidative 40
phopshorylation

Section IV Structure and metabolism of carbohydrates 63


Section V Structure and metabolism of lipids 90
Section VI Structure and metabolism of amino acids 116
Section VII Principles of molecular biology and molecular genetics 136
Section VIII Molecular mechanisms of hormone action on target
cells. Biochemistry of hormonal regulation 155
Section IX Biochemistry of the nervous tissue 170

Section X Biochemistry of the muscular tissue 187

Section XI Biochemistry of nutrition 198

Section XII Functional role of water soluble and fat soluble vitamins
in metabolism and providement of cell functions. 208

Section XIII Biochemistry and pathobiochemistry of blood. 221

Section XIV Functional and clinical biochemistry of liver tissue.


Biotransformation of xenobiotics and endogenous toxic
compounds. 235

Section XV Water and mineral metabolism. 243

Section XVI Functional role of kidneys in urinogenesis. Normal and


pathological constituents of urine 251

Section XVII Biochemical constituents of connective tissue.


262
Section XVIII Biochemistry of saliva and tooth tissue 268
Section XIX Biochemical reactions
References: 283

4
SECTION I
CONTROL OF THE INITIAL LEVEL KNOWLEDGE. BIOCHEMICAL
CONSTITUENTS OF THE CELL. METHODS OF BIOCHEMICAL
INVESTIGATIONS.
1. All proteins contain the:
A. Same 20 amino acids D. Same 5 amino acids
B. Different amino acids E. Only a few amino acids
C. 300 Amino acids occurring in nature

2. Proteins contain:
A. Only L- - amino acids C. D, L-Amino acids
B. Only D-amino acids D. Only L- β- amino acids
E. Only D- β- amino acids

3. In proteins the -helix and β-pleated sheet are examples of:


A. Secondary structure D. Quaternary structure
B. Primary structure E. All of these
C. Tertiary structure

4. Living cells have the unique ability to synthesize only ___ the form
ofoptical isomer due to __:
A. ‘D’ form, stereospecific enzymes D. ‘L’ form, DNA
B. ‘L’ form stereospecific enzymes E. ‘D’ form, RNA
C. ‘D’ form, DNA

5. Isoelectric pH of an amino acid is that pH at which it has a:


A. Nil net charge C. Positive charge
B. Negative charge D. None of these

6. A Zwitter ion is one which has in aqueous solution:


A. No electrical charges at all D. One positive charge and one
B. Two positive charges and one negative charge
negative charge E. One positive charge
C. Two negative charges and one
positive charge

7. This amino acid cannot have optical isomers:


A. Glycine D. Alanine
B. Histidine E. Arginine
C. Threonine

8. The amino acid containing hydroxyl group:


A. Threonine D. Alanine
B. Isoleucine E. Ornithine
C. Arginine

5
9. An aromatic amino acid is:
A. Tyrosine D. Arginine
B. Lysine E. Alanine
C. Taurine

10. Primary structure of a protein is formed by:


A. Peptide bonds D. All of these
B. Hydrogen bonds E. None of these
C. Disulphide bonds

11. Two amino groups are present in:


A. Lysine D. Threonine
B. Glutamate E. Alanine
C. Leucine

12. Branched chain amino acids are:


A. Valine, Leucine and Isoleucine D. Cysteine and cystine
B. Tyrosine and Tryptophan E. Glutamine and Asparagine
C. Glycine and Serine

13. The sulphur containing amino acid:


A. Methionine D. Valine
B. Serine E. Tyrosine
C. Homoserine

14. Which of the following statement about the peptide bond is true?
A. It is planar C. It is a carbon-carbon bond
B. It has cis hydrogen and oxygen D. It has rotational freedom
groups

15. An amino acid not found in proteins is:


A. β-Alanine D. Histidine
B. Proline E. Arginine
C. Lysine

16. Starch is a:
A. Polysaccharide D. Protein
B. Monosaccharide E. None of these
C. Disaccharide

17. Polysaccharides are:


A. Polymers D. Oils
B. Acids E. Fats
C. Proteins

18. The general formula of monosaccharides is:


6
A. CnH2nOn D. CnH2nO2n
B. C2nH2On E. CnO2n
C. CnH2O2n

19. A polysacchharide which is often called animal starch is:


A. Glycogen D. Dextrin
B. Starch E. Glucose
C. Inulin

20. The sugar found in milk is:


A. Lactose D. Galactose
B. Glucose E. Ribise
C. Fructose

21. A triose sugar is:


A. Glycerose D. Fructose
B. Ribose E. Glucose
C. Erythrose

22. An example of a saturated fatty acid is:


A. Palmitic acid D. Erucic acid
B. Oleic acid E. Arachidonic acid
C. Linoleic acid

23. The cholesterol molecule is:


A. Steroid D. Straight chain acid
B. Quinoline derivative E. Amino acid
C. Benzene derivative

24. Vitamins are:


A. Accessory food factors D. Proteins in nature
B. Generally synthesized in the body E. Carbohydrates
C. Produced in endocrine glands

25. A nucleoside consists of:


A. Purine or pyrimidine base + sugar D. Purine + pyrimidine base +
B. Nitrogenous base sugar +phosphorous
C. Purine or pyrimidine base + E. Sugar + phosphorous
phosphorous

26. The sugar moiety present in RNA is:


A. Ribose D. Deoxyribose
B. Arabinose E. Glucose
C. Ribulose

27. The nitrogenous base present in the RNA molecule is:


7
A. Uracil C. Xanthine
B. Thymine D. Hypoxanthine

28. The sugar found in DNA is:


A. Deoxyribose D. Ribulose
B. Ribose E. Glucose
C. Xylose

29. The most active site of protein synthesis is the:


A. Ribosome D. Cell sap
B. Nucleus E. The Golgi complex:
C. Mitochondrion

30. A nucleotide consists of:


A. Purine or pyrimidine base + sugar + D. Purine or pyrimidine base +
phosphorous phosphorous
B. A nitrogenous base like choline E. Sugar + phosphorous
C. Purine or pyrimidine base + sugar

31. A purine nucleotide is:


A. AMP D. TMP
B. UMP E. TDP
C. CMP

32. A pyrimidine nucleotide is:


A. CMP D. IMP
B. AMP E. ATP
C. GMP

33. The most abundant free nucleotide in mammalian cells is:


A. ATP D. FAD
B. NAD E. FMN
C. GTP

34. RNA does not contain:


A. Thymine D. Ribose
B. Adenine E. Phosphorous
C. Uracil

35. Genetic code is:


A. Collection of codons D. Collection of pyrimidine
B. Collection of amino acids nucleotide
C. Collection of purine nucleotide

36. Genetic information of nuclear DNA is transmitted to the site of protein


synthesis by:
8
A. mRNA D. Polysomes
B. rRNA E. DNA
C. tRNA

37. Synthesis of DNA is also known as:


A. Replication D. Translation
B. Duplication E. Mutation
C. Transcription

38. Nucleic acids are biopolymers, which are formed from the next
monomers:
A. Mononucleotides D. Isoprene units
B. Amino acids E. Fatty acids
C. Carbohydrates

39. Genetic information flows from:


A. DNA to RNA D. DNA to cellular proteins
B. DNA to DNA E. RNA to DNA
C. RNA to cellular proteins

40. Oxidation of which substance in the body yields the most calories:
A. Lipids D. Glucose
B. Glycogen E. Vitamins
C. Protein

41. Eukaryotes have defined cells, which exhibit the next structural
peculiarity:
A. Genetic information is stored in D. Genetic information is stored
DNA, organized as nuclear chromatin in DNA, uniformly distributed
B. The cell possess a cell wall throughout the cytoplasm
C. The cell contains specific E. Genetic information is stored
particles, responsible for cell in messenger RNA
respiration

42. Lysosomes are cellular organelles, which have the following functional
significance:
A. Degradation of complex C. Post-translational
biomolecules (proteins, nucleic acids, modification of proteins
oligosaccharides etc.) D. Oxygen consumption by
B. Production of energy the cell (respiration)
(biosynthesis of ATP) E. Cell movement

43. Proteasomes are subcellular particles responsible for the next process
in the cell:
A. Hydrolysis of polypeptides and B. Specific folding of protein
proteins polypeptide chain
9
C. Post-translational modification of E. Degradation of oligosaccharide
proteins part of glycoproteins
D. Degradation of damaged nucleic
acds

44. The cellular organelles called “suicide bags” are:


A. Lysosomes D. Golgi’s bodies
B. Ribosomes E. Mitochondria
C. Nucleolus

45. The following chemical constituent stores genetic information in the cell:
A. Nuclear DNA D. Heterochromatin
B. Messenger RNA E. Ribosomal RNA collected
C. Euchromatin in nucleoli

46. Biosynthesis of protein in the cell takes place in specialized subcellular


particles which are called:
A. Ribosomes D. Centrosomes
B. Nucleosomes E. Peroxysomes
C. Proteasomes

47. The following substances are cell inclusions except:


A. Centrosome D. Melanin
B. Glycogen E. Vitamins
C. Lipids

48. The power house of the cell is:


A. Mitochondria D. Lysosomes
B. Cell membrane E. Peroxysomes
C. Nucleus

49. The digestive enzymes of cellular compounds are confined to:


A. Lysosomes D. Polysomes
B. Ribosomes E. Mitochondria
C. Peroxisomes

50. In determination of concentration of hormones in blood the next method


is routinely used in clinical laboratory investigations:
A. Immunoenzymatic assay D. Spetrophotometry
B. Immunoprecipitation E. Polarography
C. Chromatography

51. In determination of protein concentration in blood plasma the next


method is the most convenient and most frequently used in laboratory
practice:
A. Colorimetry
10
B. Precipitation by salts of heavy D. Electrophoresis
metals E. Polarography
C. Precipitation with strong acids

52. The movement of charged particles towards one of the electrodes under
the influence of electrical current is:
A. Electrophoresis D. Gel filtration
B. Molecular sieving E. Spectrometry
C. Gas liquid chromatography

53. Detection of protein in biological fluids, e.g. in urine, is achieved with the
next methodical approach:
A. Precipitation with strong C. Immunoprecipitation with
inorganic or organic acids specific antiserum
B. Amino acid analysis after acid D. Determination of optical
hydrolysis of sample. density at 280 nm
E. Polarimetry

54. Indicate optical method of investigation, which is used in clinical


biochemistry:
А. Photocolorimetric D. Electrophoresis
В. Affinity chromatography Е. Immunoenzyme assay
С. Salting out

55. The sorting out of molecules according to size and shape may be
adapted to protein purification in this technique:
A. Gel filtration chromatography D. Electrophoresis
B. Adsorption chromatography E. Immunoenzyme assay
C. Paper chromatography

56. Separation of molecules according to their molecular mass is achieved


by the next chromatographic method:
A. Gel filtration chromatography D. Partition chromatography
B. Ion exchange chromatography E. Affinity chromatography
C. Absorption chromatography

57. Biuret reaction is specific for:


A. –CONH-linkages D. –SH groups
B. –CSNH2 group E. All of these
C. –(NH)NH2 group

58. Which of the following techniques purifies proteins:


A. Ion exchange chromatography D. Fluorescent analysis
B. Photocolorimetry E. Immunoenzyme assay
C. Polarography;

11
59. The solubility of most proteins is lowered at high salt concentrations
process called:
A. Salting out process C. Isoelectric focussing
B. Salting in process D. None of these

60. The degradative processess are categorized under the heading of:
A. Catabolism D. Amphbolism
B. Anabolism E. None of the above
C. Metabolism

EXAMPLES OF KROK 1 TESTS

61. Out of 200 different amino acids form in nature the number of amino
acids present in protein:
A. 20 D. 35
B. 25 E. 100
C. 40

62. Choose from listed below methods ONE, which is used for fractionation
of protein mixtures and isolation of individual proteins (enzyme, hormone,
toxin etc):
A. Affinity chromatography D. Proteolysis
B. Precipitation with nitric acid E. Radioimmunoassay
C. Boiling of extracts

63. Determination of C-reactive protein (CRP) in blood plasma is conducted


with the use of antisera, containing specific antibodies against CRP. What
type of analytical method is used in this case?
A. Immunoprecipitation D. Chromatography
B. Spectrophotomenry E. Polarography
C. Electrophoresis

64. Proteins are biopolymers of principal significance in cell building, they


are composed from amino acids as monomers, which are connected into
chain by the next main type of chemical bond:
A. Peptide bond
B. Phosphodiester bond
C. Ionic bond
D. Hydrogen bond
E. Glycosidic bond

12
65. Protein preparations from human blood plasma are frequently used in
clinical medicine for treatment of many diseases. Fractionation of blood
plasma and preparation of distinct protein fractions is achieved by the next
method:
A. Fractional precipitation with C. Precipitation with salts of heavy
ammonium sulfate metals
B. Fractional precipitation with ethanol D. Electrophoresis in agarose gel
by Cohn YI method E. Ultracentrifugation

66. Determination of proportion between protein fractions in blood plasma


or serum has an important clinical and diagnostic significance. The
following routine method for obtaining results of this sort is most frequently
used in clinical laboratories:
A. Salting out with neutral salts D. Electrophoresis in agar gel
B. Absorption chromatography or on acetyl-cellulose films
C. Precipitation with strong acids E. Immunoprecipitation

67. For determination of DNA synthesis in the cell usually is used


measurement of incorporation of H3-thymidine into cellular biopolymers.
The next type of analysis is used in this specific case.
A. Radioisotope method C. Electrophoresis
B. Polymerase chain reaction D. Radioimmunoassay
(PCR) E. Affinity chromatography

68. Proteins are biopolymers of principal significance in cell building, they


are composed from amino acids as monomers, which are connected into
chain by the next main type of chemical bond:
A. Peptide bond D. Hydrogen bond
B. Phosphodiester bond E. Glycosidic bond
C. Ionic bond

CLINICAL CASES AND SITUATIONAL TASKS

69. A protein solution was heated to 50 0C and lost all its properties. How can
this be explained?

ANSWER: During heating there was denaturizing of protein.

70. With the use of salting out protein was separated from the biological
solution. This protein will be used for treatment. What needs to be done to
separate the protein from other molecular additives?

ANSWER: The separation of protein from the other molecular additives can
happen with dialysis or gel-filtration
71. The total amount of protein in human blood is 50g/L. What can you
conclude from this information, and what can cause this state?

ANSWER: If the total amount of protein in blood is 50g/L, this means that that
person has hypoproteinemia. This state can be observed during the deficit of
proteins in food, starvation, cachexia, stop of synthesis of proteins in the liver and
in other states as well.

72. During analysis of urine, a positive reaction with Sulfosalicylic acid was
observed. In total, 0.253% of protein was observed. What does this signify?
What effects can this condition have on the organism?

ANSWER: If during analysis of urine, a positive reaction with Sulfosalicylic acid


was observed, then this means that there is protein present in the urine. This
occurs during cachexia, infectious sickness, nephritis, and so on. This condition
can lead to hypoproteinemia, lowering of oncotic pressure and signs of
inflammation.

73. The total amount of protein in blood of the patient is 95g/L. Is this
normal? What is the cause of this state?

ANSWER: If the total amount of protein in human blood is 95g/L, then the
condition observed is known as hyperproteinemia. This is observed during water
loss by the organism, hepatitis, chronic inflammatory states, collagenosis,
lymphma, endotheliomas and so on.

74. Name compartments of an animal cell:


Answer:

75. Discribe the method, shown at the picture:

Answer: Dialysis is a passage of solutes through a semi-permeable membrane


(cellulose tube). Molecules smaller than the pore size pass through the membrane
(water, salts, protein fragments); protein stays in the tube.

76. Discribe the method, shown at the picture:


Answer: Size exclusion chromatography (gel filtration chromatography) -
separation based upon molecular size. Column is filled with semi-solid beads of a
polymeric gel that will admit ions and small molecules into their interior but not
large ones. When a mixture of molecules and ions dissolved in a solvent is
applied to the top of the column, the smaller molecules (and ions) are distributed
through a larger volume of solvent than is available to the large molecules.
Consequently, the large molecules move more rapidly through the column, and in
this way the mixture can be separated (fractionated) into its components. The
porosity of the gel can be adjusted to exclude all molecules above a certain size.
Sephadex, Sepharose or Sephacryl, which are fine porous beads, are trade
names for gels that are available commercially in a broad range of porosities on
the basis of their density and sedimentation velocity

77. Discribe the method, shown at the picture:

Answer: Differential centrifugation is used to separate certain organelles from


whole cells for further analysis of specific parts of cells. In the process, a tissue
sample is first homogenised to break the cell membranes and mix up the cell
contents. The homogenate is then subjected to repeated centrifugations, each
time removing the pellet and increasing the centrifugal force. Finally, purification
may be done through equilibrium sedimentation, and the desired layer is extracted
for further analysis.
Separation is based on size and density, with larger and denser particles pelleting
at lower centrifugal forces. As an example, unbroken whole cells will pellet at low
speeds and short intervals such as 1,000g for 5 minutes. Smaller cell fragments
and organelles remain in the supernatant and require more force and greater
times to pellet. In general, one can enrich for the following cell components, in the
separating order in actual application:
 Whole cells and nuclei;
 Mitochondria, lysosomes and peroxisomes;
 Microsomes (vesicles of disrupted endoplasmic reticulum); and
 Ribosomes and cytosol.

78. Discribe the method, shown at the picture:

Answer: polyacrylamide gel electrophoresis (PAGE) - separation of molecules


on the by applying an electric field. Successful separation can be accomplished
by electrophoresis in various gels (semisolid suspensions in water) rather than
in a liquid solution. Gels are cast between a pair of glass plates by polymerizing
a solution of acrylamide monomers into polyacrylamide chains and
simultaneously cross-linking the chains into a semisolid matrix. Gel pore size
can be varied by adjusting the concentrations of polyacrylamide and the cross-
linking reagent. Highly cross-linked polyacrylamide gel = pores are quite small.
Such a gel could resolve small proteins and peptides, but large proteins would
not be able to move through it smaller proteins migrate faster than larger
proteins through the gel. Gel's pore size and strength of the electric field
influence the rate of movement.

79. Discribe stages of DNA sample electrophoresis:


Answer:
SECTION ІІ
ENZYMES, STRUCTURE AND CLASSIFICATION.
REGULATION OF METABOLISM

1. Chose a correct statement about common feature of enzymes and


inorganic catalysts.
A. Acceleration of catalyzed reaction
thermodynamically permitted D. Specific dependence on
reactions substrate concentration
B. Dependence of activity on pH E. Dependence on the
of medium presence of cofactors
C. High selectivity to type of

2. Enzymes differ from inorganic catalysts by the next property:


A. Thermolability D. Thermostability
B. Acceleration of reaction E. No selectivity to type of
equilibrium achievement catalyzed reaction
C. Sensitivity to catalyst poisons

4. Enzymes accelerate chemical reaction due the to next effect:


A. Enzymes lower the energetic barrier of chemical reaction
B. Enzymes shift the position of equilibrium of chemical reaction.
C. Enzymes make possible endergonic reaction without energy supply

4. Enzymes accelerate the rate of reactions by:


A. Decreasing the energy of D. Decreasing the free energy
activation change of the reaction
B. Increasing the equilibrium E. Decreasing the equilibrium
constant of reactions constant of reactions
C. Increasing the energy of activation

5. The energy required to start an enzymatic reaction is called:


A. Activation energy D. Potential energy
B. Chemical energy E. Free energy
C. Metabolic energy

6. Chose from listed below enzymes ONE which exhibits selectivity to


stereochemical epimers of substrate:
A. Urease D. Alcohol dehydrogenase
B. Trypsin E. Lactate dehydrogenase
C. Aminopeptidase

7. An enzyme is a:
A. Protein C. Lipid
B. Carbohydrate D. Nucleic acid
E. Amino acid

8. Which of the following is not true regarding enzymes?


A. They are destroyed after the C. They remain active even after
completion of the reaction they separation from the source
catalyse D. They are irreversibly
B. They catalyze only a particular destroyed at high temperature
type of reaction E. Their activity depends on the
pH of the solution

9. Active center of enzyme can be defined as follows:


А. Site on the enzyme molecule, which С. Nonprotein component of
binds substrate and provides its further enzyme molecule
transformation. D. Site for binding of allosteric
В. Part of the molecule, which easily effector.
splits from apoenzyme. Е. Whatever part of polypeptide
chain of enzyme molecule.

10. As cofactors of enzymes the most frequently met are the next
compounds:
A. Vitamins, modified by cell enzymes C. Hormons, e.g.thyroxine
(vitamin derivatives) D. Carbohydrates
B. Native vitamins E. Polynucleotides

11. Nonprotein part of complex (conjugated) enzyme is called as:


A. Coenzyme D. Allosteric activator
B. Holoenzyme E. Allosteric suppressor
C. Apoenzyme

12. Chose the correct statement about allosteric site of enzyme:


A. It modulates reversibly D. It irreversibly inhibits
enzymatic activity after binding with enzymatic activity after binding
ligand with ligand
B. It is identical with active center. E. It induces chemical
C. It serves for attachment of modification of enzyme
enzyme to biomembrane molecule

13. The active site of an enzyme is formed by:


A. R group of amino acids D. Sulphur bonds which are
B. NH2 group of amino acids exposed
C. CO group of amino acids E. Peptide bonds

14. Coenzymes are:


A. Heat stable, B. Soluble,
dialyzable, non protein organic colloidal, protein molecules
molecules
C. Structural E. Inorganic
analogue of enzymes catalists
D. Different forms of
enzymes
A. Allosteric suppressor
15. Coenzymes combine with:
A. Apoenzymes D. Antienzymes
B. Proenzymes E. Cofactors
C. Holoenzymes

16. Active centers in nonconjugated (simple) enzymes, e.g.trypsin, are


formed by the next constituents of enzyme molecule:
A. Amino acid side chains only C. Nucleotides
B. Peptide bonds between D. Carbohydrates
selected amino acids E. Phospholipids

17. Chose the amino acid, which is frequently involved in formation of active
center in different enzymes.
A. Histidine D. Valine
B. Leucine E. Glycine
C. Proline

18. Which from the listed below is coenzyme?


A. All of these D. CoQ and CoA
B. ATP E. Thiamine
C. Vitamin B and C

19. How are called enzymes which catalyze the same reaction, are of the
same origin but differ in chemical properties?
A. Izoenzymes C. Holoenzyme
A. Apoenzyme D. Zymogen
B. Coenzyme

20. Multiple forms of the same enzymes are known as:


A. Isoenzymes D. Pre-enzymes
B. Zymogens E. Coenzymes
C. Proenzymes

21. Enzymes activity is controlled by


A. All of these D. Concentration of the enzyme
B. pH of the solution E. Concentration of the
C. Temperature substrate

22. Boiling destroys the activity of enzyme irreversibly due to the next
process:
A. Change in conformation of enzyme molecule
B. Cleavage of peptide bonds in enzyme molecule
polypeptide chain of enzyme. E. Formation of
C. Cleavage of disulphide bonds intramolecular or intermolecular
in enzyme molecule cross-links in enzyme molecule
D. Cleavage of ionic bonds in

23. In humans most enzymes have an optimal temperature of action at:


A. 37 oC D. 30 oC
B. 0 oC E. 100 oC
C. 20 oC

24. What happens with enzymes at 90 oC?


A. Denaturation D. Alteration of tertiary
B. Hydrolysis structure
C. Alteration of primary E. Tight binding of
structure substrate in active site

25. The pH optima of most of the enzymes is:


A. Between 6 and 8 D. Between 8 and 12
B. Between 1 and 3 E. Above 12
C. Between 4 and 6

26. Pepsin has the next pH optimum for enzymatic activity:


A. pH 1,5 D. pH 8,0
B. pH 5,1 E. pH is non significant
C. pH 7,5

27. Trypsin exhibits maximal catalytic activity at the next pH value:


A. pH 7,5 D. pH 9,5
B. pH 5,1 E. pH is non significant
C. pH 2,5

28. Chose from listed below enzymes, ONE which represents class
hydrolases:
A. Pepsin D. Phenol oxidase
B. Aldolase E. ATP synthase
C. Glucokinase

29. Enzymes, providing polysaccharide hydrolysis are defined as:


A. Glycosidases D. Catalase
B. Lipase E. Proteases
C. Phosphatase

30. The first position in classification of enzymes is occupied by:


A. Oxidoreductases C. Isomerases
B. Transferases D. Hydrolases
E. Ligases

31. The fourth position in classification of enzymes is occupied by:


A. Lyases D. Hydrolases
B. Transferases E. Ligases
C. Isomerases

32. The code number of each individual and well characterized enzyme
contains the next ranges of figures:
A. Four D. Five
B. Two E. Six
C. Three

33. Chose from list below enzymes, which exhibits specificity to peptide
bonds:
A. Chymotrypsin D. Arginase
B. Urease E. Cellulase
C. Alcohol dehydrogenase

34. What type of reactions presented below is catalyzed by hydrolases?


A. A-B + H2O = A-OH + B-H D. A-B = A + B
B. AH2 + B = A + BH2 E. A-R-B = A-B-R
C. A+B+ATP= A-B + ADP + Pi

35. What type of presented below reactions is catalyzed by


dehydrogenases?
A. AH2 + B = A + BH2 D. A-B = A + B
B. A+B+ATP= A-B + ADP + Pi E. A-R-B = A-B-R
C. A-B + H2O = A-OH + B-H

36. The best example of extracellular enzymes (exoenzyme) is:


A. Digestive enzymes D. Malatedehydrogenase
B. Nucleases E. None of these
C. Succinic dehydrogenase

37. The substrate for amylase is:


A. Starch D. Ribose
B. Cane sugar E. Maltose
C. Lactose

38. In cell, digestive enzymes are found mainly in:


A. Lyzosomes D. Nucleus
B. Mitochondria E. Vacuoles
C. Ribosomes

39. A mitochondrial marker enzyme is:


A. Succinate dehydrogenase D. Pyruvate dehydrogenase
B. Aldolase E. DNA-polymerase
C. Amylase

40. The third position in classification of enzymes is occupied by:


A. Hydrolases D. Isomerases
B. Transferases E. Ligases
C. Lyases

41. In human saliva there is an enzyme able to hydrolyze the α[1→4]


glucosidic bonds in the molecule of starch. Name this enzyme:
A. α-Amylase D. β-Galactosidase
B. Phosphatase E. Lysozyme
C. Fructofuranosidase

42. In the cell, enzymes are located in subsequent organelles, providing


their specific functioning. Note enzymes located in lysosomes.
A. Cathepsins and biosynthesis
glucosaminidase D. Enzymes of urea synthesis
B. Fatty acid synthesis enzyme E. Glycogen synthetase and
complex branching enzyme
C. Enzymes of protein

43. Cytochrome c participates in transport of electrons in respiratory chain


of the cell and is located in the next cellular compartment:
A. Mitochondria D. Golgi vesicles
B. Nucleus E. Lysosomes
C. Cytoplasm

44. In a patient was detected disorder in digestion of protein in stomach and


small intestines. What group of enzymes may cause this disorder?
А. Proteinases D. Lyases
В. Amylase Е. Aminotransferases
С. Lipase

45. Chose from list below enzymes located in nucleus:


A. RNA polymerases E. Glycogen synthetase and
B. Enzymes of protein biosynthesis branching enzyme
C. Enzymes of tricarboxylic acid cycle
D. Cathepsins and glucosaminidase

46. The enzymes of the citric acid cycle are located in:
A. Mitochondrial matrix C. Nucleus
B. Extramitochondrial soluble fraction D. Endoplasmic reticulum
of the cell E. Lysosomes
47. Chose from list below, enzyme belonging to class of hydrolases:
A. Thrombin D. Phenol oxidase
B. Lactate dehydrogenase E. Aldolase
C. Glucokinase

48. Chose from listed below enzymes, ONE which exhibits specificity to
peptide bonds:
A. Chymotrypsin D. Arginase
B. Urease E. Cellulase
C. Alcohol dehydrogenase

49. Activity of multienzyme complexes is regulated by the next type of


regulation:
A. Feedback inhibition D. Non competitive inhibition
B. Catabolic repression E. Competitive inhibition
C. Limited proteolysis

50. Proenzymes:
A. All of these D. Proelastase
B. Chymotrysinogen E. None of these
C. Pepsinogen

51. An allosteric modulator influences enzyme activity by:


A. Binding to a site on the enzyme C. Changing the nature of the
molecule distinct from the catalytic product formed
site D. Changing the specificity of the
B. Competing for the catalytic site with enzyme for its substrate
the substrate E. Covalent modification of enzyme

52. In active center of enzymes which catalyze hydrolysis of substrate


(hydrolases) usually is present the next amino acid residue:
A. Histidine D. Proline
B. Leucine E. Valine
C. Phenylalanine

53. In active center of enzyme very frequently is present side chain of the
next amino acid:
A. Histidine D. Proline
B. Leucine E. Valine
C. Phenylalanine

54. Activity of many enzymes depends from the presence of free thiol
groups in active center. What amino acid residue provides presence of
these groups in enzyme molecule?
A. Cysteine C. Tryptophan
B. Lysine D. Methionine
E. Serine

55. Michaelis-Menten constant (Km) reflects the next property of enzyme:


A. Affinity to substrate E. Sensitivity to competitive
B. Thermolability inhibitors
C. Sensitivity to pH of medium
D. Affinity to a product of reaction

56. Michaelis-Menten constant corresponds to:


A. Substrate concentration at which D. Concentration of substrate, at
reaction rate is half maximal which rate of reaction reach
B. Optimal pH for activity of enzyme maximal value
C. Enzyme concentration, which E. Ionic strength of medium
provides half maximal velocity of favoring maximal activity of
reaction enzyme

57. When [s] is equal to Km, which of the following conditions exist?
A. Half the enzyme molecules are D. Enzyme is completely
bound to substrate saturated with substrate
B. The velocity of the reaction is E. The velocity of the reaction is
equal to Vmax independent of enzyme
C. The velocity of the reaction is concentration
independent of substrate
concentration

58. Michaelis-Menten constants of two enzymes are 1,3x10 -5 M/l and 2,3x10-3
M/l subsequently. Indicate true statement about the affinity of these
enzymes to substrate.
A. The second enzyme has higher D. For decision an information
affinity to substrate on concentration of enzyme is
B. Enzymes possess equal affinity to needed
substrate E. Data are incomplete and it is
C. The first enzyme has higher impossible to draw a conclusion
affinity to substrate

59. In an enzyme assay the substrate concentration was taken much higher
than Km. In this conditions the rate of the reaction will be as follows:
A. Shows zero-order kinetics D. Is independent of enzyme
B. Approaches 50 % value of Vmax concentration
C. Is proportional to substrate E. Is independent of temperature
concentration

60. The Michaehis-Menten hypothesis:


A. Postulates the formation of an B. Enables us to calculate the
enzyme substrate complex isoelectric point of an enzyme
C. States that the rate of a chemical D. States that the reaction rate is
reaction may be independent of proportional to substrate
substrate concentration concentration

61. ‘Lock’ and ‘Key’ model of enzyme action proposed by Fisher implies
that:
A. The active site is complementary D. The active site is not changed
in shape to that of the substrate E. Substrates change
B. The active site is flexible and conformation prior to active site
adjusts to substrate interaction
C. The active site requires removal of
PO4 group

62. According to international convention (EC) as a unit of enzymatic


activity is accepted 1 catal, which can be defined as:
A. Quantity of enzyme which transform 1 umole of substrate in
transform 1 mole of substrate in 1 1 second
second D. Activity of 1 mg of pure
B. Quantity of enzyme which enzyme
transform 1 umole of substrate in 1 E. Number of substrate
minute molecules transformed in 1
C. Quantity of enzyme which minute

63. The specific activity of an enzyme is:


The number of enzyme units per transformation of one micromole
miсrogram of enzyme protein of substrate per minute under
The amount of enzyme that produces 1 standard conditions
mole of product per second under The activity of an enzyme in the
standard conditions presence of its preferred
The activity of an enzyme in relation to a substrate
standard preparation of the enzyme molmin-1 of protein -1 or U/mg
The amount of enzyme causing

64. Acetylcholin esterase cleaves acetylcholin hydrolytically. Insecticides,


pesticides and nerve gases of fluorophosphates structure irreversibly
inhibit acetylcholin esterase. What type of inhibition is it?
A. Inhibitors bind with serine residue in allosteric center
residue in active center D. Inhibitors form complex
B. Inhibitors are structural analogs with acetylcholine
of true substrate E. Inhibitors induce
C. Inhibitors bind with histidine denaturation of enzyme

65. Trypsin and related to it proteolytic enzymes (chymotrypsin, thrombin et


al.) contain in active center a specific amino acid residue, which is
covalently modified by fluorophosphate compounds with irreversible lost of
enzymatic activity. What is this amino acid residue?
A. Serine D. Aspartate
B. Tyrosine E. Methionine
C. Threonine

66. Pancreatic juice has a great number of enzymes. Some of them are
secreted in inactive form. What these enzymes are?
A. Trypsinogen, chymotrypsinogen D. Lipase, phosphatase
B. Amylase, sucrase E. DNA-ase, aminopeptidase
C. Ribonuclease, pepsin

67. Zymogens of proteolytic enzymes are activated by the next process:


A. Limited proteolysis D. Decarboxylacion of
B. Phosphorylation of serine aspartic acid side chain
residues in protein molecule E. Hydroxylation of lysine
C. Carboxylation of glutamic acid

68. Note substance, which induce transformation of pepsinogen to pepsin:


A. Hydrochloric acid D. Bile acids
B. Enterokinase E. Trypsin
C. Adenosine triphosphate

69. Trypsinogen is produced in exocrine /endocrine (chose the correct


position) part of pancreatic gland and excreted to duodenum, where it is
activated by the next factor:
A. Enteropeptidase D. Cholecystopancreozymine
B. Secretin E. Chymotrypsinogen
C. Gastrin

70. In course of chymotrypsinogen activation the next effect is achieved:


A. Formation of active center (coenzyme) determining
B. Formation of allosteric center enzymatic activity
C. Elimination of allosteric E. Modification of amino acid
inhibitory sites residues involved into active
D. Attachment of specific cofactor center

71. The activation of chymotrypsinogen is achieved by the next process:


A. Limited proteolysis groups
B. Phosphorylation of serine D. ADP-ribosylation
residues with the use ofATP E. Carboxylation of aspartic
C. Acetylation of free amino acid residues

72. Enzymes requiring NAD as co-substrate can be assayed by measuring


change in absorbance at:
A. 340 nm D. 365 nm
B. 210 nm E. 690 nm
C. 290 nm
73. Chose from listed below organs one producing and secreting amylase:
A. Salivary gland parotis D. Intestinal mucosa
B. Liver E. Spleen
C. Kidneys

74. What diagnostic significance has enhanced activity of amylase in urine?


A. Indicates on damage of exocrine D. Indicates on riquets
pancreas tissue E. Indicates on vitamin C deficiency
B. Indicates on disorder in liver tissue
C. Indicates on some disease of bones

75. What is the mechanism of inhibition of folic acid synthesis by


sulfanylamides?
A. Competitive D. Uncompetitive
B. Irreversible E. Allosteric inhibition
C. Due to enzyme denaturation

76. An enzyme in saliva which hydrolyzes starch is:


α-Amylase Malate
Pepsinogen Maltase
Chymotrysin

77. Enzymes, which are produced in inactive form in the living cells, are
called:
Proenzymes Apoenzymes
Papain Izozymes
Lysozymes

78. The presence of a non competitive inhibitor:


A. Leads to a decrease in the C. Leads to a decrease in Km
observed Vmax and Vmax
B. Leads to both an increase in D. Leads to an increase in
the Vmax of a reaction and an Km without affecting Vmax
increase in Km E. Leads to a increase in Km
and Vmax

79. An allosteric enzyme influences the enzyme activity by:


A. Changing the conformation of C. Changing the specificity of
the enzyme by binding to a site other the enzyme for the substrate
than catalytic site D. Changing the nature of the
B. Competiting for the catalytic products formed
site with the substrate E. Changing the nature of the
substrates

80. In reversible noncompetitive enzyme activity inhibition:


A. Vmax is reduced D. Vmax is not changed
B. Km is increased E. Concentration of active
C. Km is decreased enzyme is reduced

81. Which of the following regulatory reactions involves a reversible


covalent modification of an enzyme?
A. Phosphorylation of serine OH C. Competitive inhibition
on the enzyme D. Non-competitive inhibition
B. Allosteric modulation E. Feedback inhibition

82. Feedback inhibition of enzyme action is affected by:


A. End products D. Allosteric inhibitor
B. Enzyme E. None of these
C. Substrate

83. How many polipeptide chains (subunits) contains molecule of creatine


kinase?
A. Two D. Four
B. One E. Six
C. Three

84. Albinism is due to deficiency of the enzyme:


A. Phenylalanine hydroxylase D. Tyrosine dehydrogenase
B. Tyrosinase E. Lactate dehydrogenase
C. p-Hydroxyphenylpyruvic acid
oxidase

85. Competitive inhibitor of succinate dehydrogenase is the next substance:


A. Malonate D. Fumarate
B. Oxaloacetate E. α-Ketoglutarate
C. Alanine

86. In rickets the next enzyme activity is highly elevated in blood serum:
A. Alkaline phosphatase D. Alanyl aminotransferase
B. Lactate dehydrogenase E. Amylase
C. Aldolase

87. The following statements describe coenzymes and prosthetic groups:


A. Some enzymes contain organic C. Coenzymes are small
molecules that are derived from polypeptides that are involved in
vitamins and that are essential for transfer reactions catalysed by
activity. enzymes
B. Some enzymes contain metal ions D. Сoenzymes are required in
such as zinc or copper that are some enzyme-catalysed
essential for their activity reaction as carried of acyl
groups or phosphate groups
E. Coenzymes are required in some reduction reactions involving
enzyme-catalysed oxidation- hydrogen transfer

EXAMPLES OF KROK 1 TESTS

88. By consumption of fresh milk in kid developed disorders of digestive


tract. Consumption of other sugar-containing food-stuffs did not cause
similar disorders. Genetically determined insufficiency of what enzyme can
be considered in this case?
A. Lactase D. Hexokinase
B. Phosphoglucomutase E. Glucose-6 phosphate isomerase
C. Glycogen synthetase

89. In a patient was detected disorder in digestion of protein in stomach and


small intestines. What group of enzymes may cause this disorder?
А. Proteinases D. Lyases
В. Amylase Е. Aminotransferases
С. Lipase

90. In dietology in cases of children milk intolerance a milk is used in which


lactose content is diminished enzymatically. What enzyme is used for this
purpose?
A. -galactosidase D. –amylase
B. –amylase E. –glucosidasde
C. –glucosidase

91. Ethylene glycol, the major ingredient in antifreeze, is occasionally


consumed by alcoholics as a substitute of true alcohol beverages. In
metabolism of ethylene glycol a highly toxic aldehyde is produced, much
more toxic than acetaldehyde, produced from ethanol. Ethanol is often
administered as a treatment in cases of ethylene glycol poisoning. What is
the most likely reason that ethanol is an effective treatment for ethylene
glycol poisoning?
A. Alcohol dehydrogenase ADH with ethylene glycol and
(ADH) exhibits a much lower Michaelis converts it to a harmless product
constant (Km) for ethanol than for D. Acetaldehyde, which
ethylene glycol is produced by the reaction of ADH
B. Ethanol is an allosteric with ethanol, is of therapeutic value
effector of ADH in addition to being a E. Ethanol induces
substrate another enzyme which effectively
C. Ethanol combines with a metabolize ethylene glycol
toxic product formed by the reaction of

92. In a treatement of some infections diseases, sulfanilamide drugs are


used, which inhibit the growth of bacteria. What is the mechanism of their
action?
A. They are structural analogs of p- D. Inhibit the absorption of folic
aminobenzoic acid, needed for acid
biosynthesis of folic acid E. Irreversibly inhibit
B. Allosteric inhibition of bacterial biosynthesis of folic acid,
enzymes needed for normal bacterial
C. They are involved in oxidative- survival
reductive reactions

93. In blood serum of a patient a marked increase in activity of trypsine,


alpha-amylase and lipase was detected. What disease can be suggested?
A. Acute pancreatitis D. Malignant tumors
B. Cholelythiasis E. Poisoning with insecticides
C. Chronic hepatitis

94. Pharmaceuticals, containing mercury, arsen or other heavy metals, are


inhibiting enzymes, posessing sulfhydril groups. What amino acid is used
for reactivation of these enzymes?
A. Cysteine D. Aspartic acid
B. Histidine E. Glycine
C. Isoleucine

95. In recognition of hepatitis the determination the following enzymes


activity in blood has diagnostic significance:
A. Amino transferases D. Aldolase
B. Amylase E. Creatin kinase
C. Lactate dehydrogenase

96. Researchers isolated 5 isoenzymic forms of lactate dehydrogenase from


the human blood serum and studied their properties. What property
indicates that the isoenzymic forms were isolated from the same enzyme?
A. Catalyzation of the same reaction D. Tissue localization
B. The same molecular weight E. The same electrophoretic
C. The same physicochemical mobility
properties

97. A patient presents high activity of LDH1,2, aspartate aminotransferase,


creatine phosphokinase. In what organ (organs) is the development of a
pathological process the most probable?
A. In the heart muscle (initial stage of C. In kidneys and adrenals
myocardium infarction) D. In connective tissue
B. In skeletal muscles (dystrophy, E. In liver and kidneys
atrophy)

98. Marked increase of activity of МВ-forms of CPK (creatinephosphokinase)


and LDH-1 were revealed on the examination of the patient's blood. What is
the most likely pathology?
A. Miocardial infarction D. Pancreatitis
B. Hepatitis E. Cholecystitis
C. Rheumatism

99. 12 hours after an accute attack of retrosternal pain a patient presented


an increase of aspartate aminotransferase activity in blood serum. What
pathology is this deviation typical for?
A. Myocardium infarction D. Diabetes mellitus
B. Viral hepatitis E. Diabetes insipidus
C. Collagenosis

100. Desulfiram is widely used in medical practice to prevent alcocholism. It


inhibits aldehyde dehydrogenase. Increased level of what metabolite causes
aversion to alcochol?
A. Acetaldehyde D. Propionic aldehyde
B. Ethanol E. Methanol
C. Malonyl aldehyde

101. A 49-year-old driver complains about unbearable constricting pain


behind the breastbone irradiating to the neck. The pain arose 2 hours ago.
Objectively: the patient’s condition is grave, he is pale, heart tones are
decreased. Laboratory studies revealed high activity of creatine kinase and
LDH1. What disease are these symptoms typical for?
A. Acute myocardial infarction D. Cholelithiasis
B. Acute pancreatitis E. Diabetes mellitus
C. Stenocardia

102. In case of enterobiasis acrihine - the structural analogue of vitamin B 2 -


is administered. The synthesis disorder of which enzymes does this
medicine cause in microorganisms?
A.FAD-dependent dehydrogenases D. NAD-dependet dehydrogenases
B. Cytochromeoxidases E. Aminotransferases
C. Peptidases

103. Hydroxylation of endogenous substrates and xenobiotics requires a


donor of protons. Which of the following vitamins can play this role?
A. Vitamin C D. Vitamin E
B. Vitamin P E. Vitamin A
C. Vitamin B6

104. A newborn child has convulsions that have been observed after
prescription of vitamin B6. This most probable cause of this effect is that
vitamin B6 is a component of the following enzyme:
A. Glutamate decarboxylase C. Netoglubarate dehydromine
B. Pyruvate dehydrostase D. Aminolevulinate synthase
E. Glycogen phosphorylase

105. In a patient with complaints on pain in cardiac area a myocardial


infarction was recognized after estimation of enzymes activity in blood.
Indicate, please, what enzyme activities were determined?
A. LDH, creatine kinase, D. Trypsin, lysozyme, citrate
aminotransferase synthase
B. Amylase, lipase, phosphatase E. Aldolase, succinate
C. Peptidase, arginase, glucokinase dehydrogenase, hexokinase

106. In course of tuberculosis treatement a patient was administered


isoniazide - a structural analogue of nicotinamide and pyridoxine. What type
of inhibition by mechanism of action exhibits isoniazide?
A. Competitive D. Allosteric
B. Irreversible E. Noncompetitive
C. Uncompetitive

107. After laboratory investigation in blood of patient an increase of LDH


activity was detected, which is characteristic symptom of heart, liver or
kidney diseases. What additional biochemical investigation must be
performed in differential diagnostics?
A. Determination of LDH isozymes D. Determination of blood
B. Estimation of blood glucose level cholesterol level
C. Ketone bodies level in blood E. Amylase activity in blood

108. In a patient a preliminary diagnosis of myocardial infarction was


proposed. A characteristic feature of this disease is a marked increase in
blood in the first 24 hours of the next enzyme:
A. Creatine-phosphokinase D. Glc-6-P dehydrogenase
B. Arginase E. Alpha-amylase
C. Catalase

109. Ambulance delivered a patient to the hospital with a preliminary


diagnosis “acute pancreatitis”. What enzyme activity must be estimated in
blood and urine in order to support this diagnosis?
A. Alpha-amylase D. Gamma-amylase
B. AlAT (GPT) E. Lactate dehydrogenase
C. AsAT (GOT)

110. In dietology, in cases of children milk intolerance a milk is used in


which lactose content is diminished enzymatically. What enzyme is used for
this purpose?
A. -galactosidase D. –glucosidase
B. –glucosidasde E. –amylase
C. –amylase
111. In cases of chronic pancreatitis a decrease of trypsin production and
secretion is observed. Digestion and absorption of what substances is
injured in this case?
A. Cleavage of proteins D. Cleavage of disaccharides
B. Cleavage of polysaccharides E. Cleavage of lipids
C. Cleavage of nucleic acids

112. After the addition of an extract of pancreatic gland to the tube with
starch solution, a blue coloration of the sample with iodine have
disappeared, which indicates on starch hydrolysis. What pancreatic enzyme
is involved in this reaction?
A. Amylase D. Aldolase
B. Chymotrypsin E. Trypsin
C. Lipase

113 During the investigation of pancreatic juice was detected a great


number of enzymes. Some of them are secreted in inactive form. What are
these enzymes?
A. Trypsinogen, chymotrypsinogen D. Amylase, lipase
B. Sucrase, amylase E. DNA-ase, aminopeptidase
C. Ribonuclease, pepsin

114. Phosphororganic compound diisopropyl-fluorophosphate is a


dangerous toxin as it inhibits cholinesterase. What is the mechanism of this
inhibition?
A. Irreversible D. Uncomoetitive
B. Reversible E. Noncompetitive
C. Competitive

115. In blood serum of a patient a marked increase in activity of trypsine,


alpha-amylase and lipase was detected. What disease can be suggested?
A. Acute pancreatitis D. Malignant tumors
B. Cholelythiasis E. Poisoning with insecticides
C. Chronic hepatitis

116. In a patient a preliminary diagnosis of myocardial infarction was


proposed. A characteristic feature of this disease is a marked increase in
blood in the first 24 hours of the next enzyme:
A. Creatine kinase D. Glc-6-P dehydrogenase
B. Arginase E. Alpha-amylase
C. Catalase

117. During the surgery after injection of a drug, which cause the
myorelaxation, in patient occures a prolonged stoppage of resptration (more
then 5 min). What enzyme insufficiency may be responcible for this
accident?
A. Acetylcholine esterase dehydrogenase
B. Catalase D. Monoaminoxidase
C. Glucose-6-phosphate E. Acetyl-transferase

118. In blood serum of a patient a marked increase in activity of trypsine,


alpha-amylase and lipase was detected. What disease can be suggested?
A. Acute pancreatitis D. Malignant tumors
B. Cholestasis E. Insecticide poisoning
C. Chronic hepatitis

119. In blood serum of a patient a marked increase of activity of creatine


phosphokinase and lactate dehydrogenase was detected. . What disease
can be suggested as a cause?
A. Myocardial infarction D. Hemolytic jaundice
B. Acute pancreatitis E. Nephrosis
C. Chronic pancreatitis

120. In blood serum of a patient a high activity of isozyme LDH-1 was


detected. In what organ pathological changes occure?
A. Heart D. Pancreas
B. Liver E. Kidney
C. Sceletal muscles

121. During investigation of gastric secretory function was detected


decrease of hydrochloric acid content in gastric juice. What enzyme activity
will decrease in this case?
A. Pepsin D. Amylase
B. Lipase E. Carboxypeptidase
C. Hexokinase

122. In malignant tumor of prostata a marked increase in activity of the next


enzyme in blood serum is observed:
A. Acid phosphatase D. Lactate dehydrogenase
B. Aldolase E. Alkaline phosphatase
C. Alanyl aminotransferase

123. On the third day following onset of acute myocardial infarction, which
enzyme estimation will have the best predictive value?
A. Serum LDH D. Serum ALT
B. Serum AST E. Serum amylase
C. Serum CK

124. What enzyme may be used in substitute therapy in cases of gastric


secretion insufficiency?
A. Pepsin C. Hyaluronidase
B. Trypsin D. Collagenase
E. Ribonuclease

125. In a patient the disorder of proteins digestion in stomach and small


intestine is observed. What enzymes insufficiency cause this disorder?
A. Peptidases D. Lipases
B. Oxido-reductases E. Aminotransferases
C. Amylases

126. Trypsinogen is produced in exocrine part of pancreatic gland and


excreted to duodenum, where it is activated by the next factor:
A. Enteropeptidase D. Cholecystopancreozymine
B. Secretin E. Chymotrypsinogen
C. Gastrin

127. In diagnostics of myocardial infarction the next isoform of lactate


dehydrogenase in blood has diagnostic significance:
A. H4 (iso 1) D. HM3 (iso 4)
B. H3M (iso 2) E. M4 (iso 5)
C. H2M2 (iso 3)

128. In diagnostics of an acute viral hepatitis estimation of the next


enzymatic activity in blood serum is the most valuable:
A. Alanyl aminotransferase D. Amylase
B. Glutathion peroxidase E. Alkaline phosphatase
C. Creatine kinase

129. Different forms of lympholeukoses are effectively cured with enzyme


preparation called:
A. Asparaginase (tPA)
B. Plasmin D. Hyaluronidase
C. Tissue plasminogen activator E. Streptokinase
130. A child manifests epileptic seizures caused by vitamin B6 deficiency.
This is conditioned by the decrease of the 7-aminobutyrate level in the
nervous tissue which acts as an inhibiting neurotransmitter. The
activity of which enzyme is decreased in this case?
A. Alanine aminotransferase. D. Glutamate decarboxylase.
B. Pyridoxal kinase. E. Glutamate synthetase
C. Glutamate dehydrogenase.

131. After the addition of an extract of pancreatic gland to the tube with
starch solution a blue coloration of the sample with iodine have
disappeared, which indicates on starch hydrolysis. What pancreatic enzyme
is involved in this reaction?
A. Amylase C. Chymotrypsin
B. Trypsin D. Lipase
E. Aldolase

CLINICAL CASES AND SITUATIONAL TASKS

132. What enzyme is present in the solution, when during the addition of
ionized H2O2, O2 bubbles are observed?

ANSWER: The presence of catalase causes the breakdown of H 2O2 into oxygen
and water.

133. In a child, urine with a high amount of homogentisic acid is observed.


This acid darkens once has contact with air. With what enzyme pathology is
this condition associated with?

ANSWER: Alkaptonuria- an inborn condition during which with the excretion of


urine a large amount of homogentisic acid is also excreted. The person's urine
turns a dark due to the oxidation of homogentisic acid. The reason of this
condition is the absence of the enzyme homogentisate 1,2-dioxygenase

134. What enzyme has an effect on connective tissue by repairing scars,


hematomas, exudates and transudates in pleural and abdominal cavity?

ANSWER: Hyaluronidase is responsible for the separation of α-andβ-


1,4glycosidic bonds between disaccharides in hyaluronic acid, which is a
component of connective tissues. This enzyme causes an increase in permeability
in tissues and facilitates the movement of substances between tissues.

135. During acute respiratory illnesses, in the air ways, a large amount of
sticky mucosa makes it difficult to breathe. What enzyme is used to thin the
mucosa?

ANSWER: Trypsin and chymotrypsin are proteolytic enzymes, which take part in
hydrolysis of proteins, and have proteolytic and mucolytic activity and ability to
dissolve elastin. These enzymes facilitate the thinning of mucosa and ease its
evacuation.

136. In clinical practice, for treatment of acute and chronic pancreatitis and
other damages of the pancreas, the pharmacological substance Kontrical is
used. What is the mechanism behind this drug?

ANSWER: Kontrical is an inhibitor of proteases, and in its action decreases the


activity of trypsin, plasmin, and kallikrein.

136. Which types of inhibitoipn are shown below:


Competitive inhibition

Vmax
1 /V + inhibitor
+ inhibitor
V
Vmax/
2 uninhibited

KM KM, app 1/[S]


[S]

Non-competitive
1 inhibition

V max
1 /V + inhibitor

V
Vmax /
2 uninhibited
+ inhibitor

KM 1/ [S]
[S]

2
Answer: 1- rewersible competitive 2- reversible noncompetitive

137. Explain the type of inhibition shown below:


Aswer: Feedback inhibition is, where the product of a metabolic pathway inhibits
is own synthesis at the beginning or first committed step in the pathway CTP is
the product of this pathway and it is also a precursor for the synthesis of DNA and
RNA (nucleic acids). The rapid synthesis of DNA and/or RNA depletes the CTP
pool in the cell, causing CTP to be released from ATCase and increasing its
activity. When the activity of ATCase is greater than the need for CTP, CTP
concentrations rise rapidly and rebinds to the enzyme to inhibit the activity. ATP
activates ATCase. Purines and Pyrimidines are needed in equal amounts. When
ATP concentrations are greater than CTP, ATP binds to ATCase activating the
enzyme until the levels of ATP and CTP are about the same.
SECTION ІІ
METABOLIC PATHWAYS AND BIOENERGETICS. TRICARBOXYLIC ACID
CYCLE. BIOLOGICAL OXIDATION AND OXIDATIVE PHOPSHORYLATION

1. When ATP forms AMP:


A. Inorganic pyrophosphate is C. Phsophagen is produced
produced D. ADP is produced
B. Inorganic phosphorous is E. No energy is produced
produced

2. Standard free energy (ΔG°) of hydrolysis of ATP to ADP + Pi is:


A. –30.5 KJ/mol D. –20.9 KJ/mol
B. –49.3 KJ/mol E. – 2.5 KJ/mol
C. –4.93 KJ/mol

3. Standard free energy (ΔG°) of hydrolysis of ADP to AMP + Pi is:


A. –27.6 KJ/mol D. –15.9 KJ/mol
B. –43.3 KJ/mol E. 2.5 KJ/mol
C. –30.5 KJ/mol

4. Standard free energy (ΔG°) of hydrolysis of phosphoenolpyruvate is:


A. –61.9 KJ/mol D. –9.2 KJ/mol
B. –43.1 KJ/mol E. –4.2 KJ/mol
C. –14.2 KJ/mol

5. Standard free energy (ΔG°) of hydrolysis of creatine phosphate is:


A. –43.1 KJ/mol D. –15.9 KJ/mol
B. –51.4 KJ/mol E. –9.2 KJ/mol
C. –30.5 KJ/mol

6. Which of the following compounds would you expect to liberate the


least free energy on hydrolysis?
A. AMP D. Phosphoenolpyruvate
B. ATP E. Phosphocreatine
C. ADP

7. Which of the following compounds would you expect to liberate the


highest free energy on hydrolysis?
A. Phosphoenolpyruvate D. AMP
B. ATP E. Phosphocreatine
C. ADP

8. The degradative processess are categorized under the heading of:


A. Catabolism D. Amphibolism
B. Anabolism E. None of the above
C. Metabolism

9. Most of the metabolic pathways are either anabolic or catabolic.


Which of the following pathways is considered as “amphibolic” in
nature?
A. TCA cycle B. Glycolytic pathway
C. Lipolysis E. Pentosophosphate pathway
D. Glycogenesis

10. Enzymes of tricaboxylic acid cycle are located:


A. In the mitochondrial matrix D. In the inner mitochondrial
B. On the outer surface of the outer membrane
mitochondrial membrane E. In the intermembrane space
C. On the inner surface of the outer
mitochondrial membrane

11. Which of the following statements regarding TCA cycle is true?


A. It is amphibolic in nature D. It is an anaerobic process
B. It occurs in cytosol E. It occurs in nucleus
C. It contains no intermediates for
Gluconeogenesis

12. Before pyruvic acid enters the TCA cycle it must be converted to:
A. Acetyl CoA D. Citrate
B. Lactate E. Succinate
C. -ketoglutarate

13. The formation of citrate from oxaloacetate and acetyl CoA is:
A. Condensation D. Hydrolysis
B. Reduction E. Isomerisation
C. Oxidation

14. Tricarboxylic acid cycle to be continuous requires the regeneration


of:
A. Oxaloacetic acid D. Malic acid
B. Pyruvic acid E. Succinic acid
C. α-oxoglutaric acid

15. The next principal substrate is involved into oxidation in


tricarboxylic acid cycle:
A. Acetyl-CoA D. Glucose
B. Pyruvate E. Glutamate
C. Lactate

16. What substance is the main fuel material for TCA cycle?
A. Acetyl-CoA D. Fatty acids
B. Glucose E. Succinyl-CoA
C. Amino acids

17. Malate dehydrogenase is an enzyme dependent from the presence


of the next cofactor:
A. NAD+ B. TPP
C. Coenzyme A E. FMN
D. NADP

18. Substrate level phosphorylation in TCA cycle is in step:


A. Succinate thiokinase D. Isocitrate dehydrogenase
B. Malate dehydrogenase E. Succinate dehydrogenase
C. Aconitase

19. α-Ketoglutarate dehydrogenase complex contains derivatives of the


following vitamin:
A. Thiamine D. Pyridoxine
B. Folic acid E. Ubiquinone
C. Retinol

20. Which of the following enzymes catalyze convetion of succinate to


fumarate:
A. Succinate D. Citrate synthase
dehydrogenase E. Isocitrate
B. Aconitase dehydrogenase
C. Fumarase

21. An allosteric enzyme responsible for controlling the rate of TCA


cycle is:
A. Isocitrate dehydrogenase D. Aconitase
B. Malate dehydrogenase E. Succinate dehydrogenase
C. Fumarase

22. A principle of succinate dehydrogenase activity determination is


based on the reduction of methylene blue with reduced form of the
coenzyme. What coenzyme is incorporated into the structure of
succinate dehydrogenase?
A. FAD D. TPP
B. NAD E. PALP
C. FMN

23. Out of 24 mols of ATP formed in TCA cycle, 2 molecules of ATP can
be formed at “substrate level” by which of the following reaction?
A. Succinyl-CoA→Succinic acid D. Citric acid → Isocitric acid
B. Isocitrate→Oxaloacetate E. Fumarate→Malate
C. Succinic acid→Fumarate

24. Which of the following enzymes catalyze reaction: Acetyl-CoA +


oxaloacetate → citrate + CoASH:
A. Citrate B. Succinate
synthase dehydrogenase
C. Fumarase
D. Aconitase E. Isocitrate
dehydrogenase

25. Citrate is converted to isocitrate by aconitase which contains:


A. Fe2+ D. Mg2+
B. Ca2+ E. Mn2+
C. Zn2+

26. If all the enzymes, intermediates and cofactors of the citric acid
cycle as well as an excess of the starting substrate acetyl-CoA are
present and functional in an organelle free solution at the appropriate
pH, which of the following factors of the citric acid cycle would prove to
be rate limiting?
A. Molecular oxygen C. Turnover of intermediates
B. Half life of enzyme D. Reduction of cofactors

27. In TCA cycle, oxalosuccinate is converted to α-ketoglutarate by the


enzyme:
A. Isocitrate dehydrogenase D. Succinase
B. Fumarase E. Succinate dehydrogenase
C. Aconitase

28. In citric acid cycle, GDP is phosphorylated by:


A. Succinate dehydrogenase D. Fumarase
B. Aconitase E. Isocitrate dehydrogenase
C. Succinate thiokinase

29. All of the following are intermediates of citric acid cycle except:
A. Pyruvate D. Fumarate
B. Oxaloacetate E. Citrate
C. Oxalosuccinate

30. The reaction succinyl-CoA to succinate requires:


A. GDP D. NADP+
B. ADP E. NAD+
C. CDP

31. An aneplerotic reaction which sustains the availability of


oxaloacetate is the carboxylation of:
A. Pyruvate D. Succinate
B. Glutamate E. Aconitate
C. Citrate

32. Which from listed below substances is used as an inhibitor in


studies of TCA cycle functioning?
A. Malonate B. ATP
C. NAD E. Isocitrate
D. Aconitate

33. A specific inhibitor for succinate dehydrogenase is:


A. Malonate D. Cyanide
B. Succinate E. Aconitate
C. Citrate

34. The inhibition of isocitrate dehydrogenase by NADH is:


A. Feedback inhibition D. Competitive inhibition
B. Non-competitive inhibition E. Irreversible
C. Uncompetitive inhibition

35. Which of the following substances activate isocitrate


dehydrogenase:
A. ADP D. ATP
B. FADH2 E. Mg2+
C. NADH+H+

36. The positive allosteric modifier of the enzyme pyruvate


carboxylase?
A. Acetyl CoA D. ATP
B. Biotin E. ADP
C. Oxaloacetate

37. How many moles of FADH2 are produced in process of oxidation of


0,25 mole of acetyl-CoA in tricarboxylic acid cycle?
A. 0,25 D. 1,0
B. 0,1 E. 1,5
C. 0,5

38. How many moles of NADH+H+ are produced in process of oxidation


of 0,25 mole of acetyl-CoA in tricarboxylic acid cycle?
A. 0,75 D. 0,5
B. 0,25 E. 1,0
C. 0,1

39. The number of molecules of ATP produced by the total oxidation of


acetyl CoA in TCA cycle is:
A. 12 D. 6
B. 8 E. 14
C. 10

40. Tricarboxylic acid cycle (TCA) generates reduced forms of NAD and
FAD which are used in:
A. Respiratory chain of enzymes D. Biosynthesis of ATP by
in mitochondria oxidative phosphorylation
B. Synthesis of fatty acids E. Biosynthesis of purine
C. Biosynthesis of ATP by nucleotides
substrate phosphorylation

41. The inhibition of succinate dehydrogenase by malonate is:


A. Competitive inhibition D. Feedback inhibition
B. Non-competitive inhibition E. Irreversible
C. Uncompetitive inhibition

42. The number of ATP produced in the succinate dehydrogenase step


is:
A. 2 D. 4
B. 1 E. 5
C. 3

43. Most of protons and electrons which are further involved into
mitochondrial respiratory chain are supplied from the next metabolic
pathways:
A. Tricarboxylic acid (CTA) C. Pyruvate dehydrogenase
cycle complex
B. Glycolysis D. Oxidation of fatty acids
E. Cleavage of proteins

44. In citric acid cycle, NAD is reduced in:


A. Three reactions D. Four reactions
B. Two reactions E. Five reactions
C. One reactions

45. The mitochondrial electron transport chain carriers are located in:
A. Inner mitochondrial D. On the inner surface of the
membrane external mitochondrial
B. Mitochondrial matrix membrane
C. Intermembranous space of E. On the outer surface of the
mitochondria external mitochondrial
membrane

46. Most of protons and electrons which are further involved into
mitochondrial respiratory chain are supplied from the next metabolic
pathways:
A. Tricarboxylic acid (CTA) cycle D. Oxidation of fatty acids
B. Glycolysis E. Cleavage of proteins
C. Pyruvate dehydrogenase
complex
47. The oxidation-reduction system having the highest redox potential
is:
A. NAD+/NADH D. Fe3+ cytochrome b/Fe2+
B. Ubiquinone ox/red E. Fe3+ cytochrome c1/Fe2+
C. Fe3+ cytochrome a/Fe2+

48. Redox potential (EO volts) of NAD+/NADH is:


A. –0.32 D. +0.03
B. –0.67 E. –0.88
C. –0.12

49. Redox potential (EO volts) of ubiquinone, OX/RED system is:


A. +0.04 D. +0.29
B. +0.08 E. + 0.35
C. +0.10

50. The oxidation-reduction system having the lowest redox potential


is:
A. Fe3+ cytochrome a/Fe2+ D. Fe3+ cytochrome c1/Fe2+
B. Ubiquinone OX/RED E. NAD+/NADH
3+ 2+
C. Fe cytochrome b/Fe

51. The correct sequence of cytochrome carriers in respiratory chain is:


A. Cyt b→cyt c1→cyt c→cyt aa3 D. Cyt b→cyt aa3→cyt c→ cyt c
B. Cyt aa3→ cyt b→cyt c→cyt c1 E. Cyt aa3→ cyt c1→cyt c→ cyt b
C. Cyt b→cyt c→cyt c1→cyt aa3

52. The sequence of the redox carrier in respiratory chain is:


A. NAD→FMN→CoQ→cyt D. NAD→FMN→CoQ→cyt
b→cyt c1→cyt c→cytaa3 → O2 b→ cyt aa3 cyt c1→cyt
B. FMN→CoQ →NAD→cyt E. FMN→NAD→ CoQ→cyt
b→cyt aa3→cyt c1→cyt c → O2 b→ cyt aa3 cyt c1→cyt
C. NAD→FMN→CoQ→ cyt
c1→cyt c→cyt b→cytaa3 → O2

53. All of the following electron carriers are components of the


mitochondrial electron transport chain EXEPT:
A. NADP+ D. FAD
B. NAD E. Coenzyme Q10
C. FMN

54. Cytochrome c of mitochondrial respiratory chain transfers


electrons:
A. To cytochrome oxidase D. From ubiquinon
B. From FAD-H2 E. To cytochrome b2
C. From NADP-H2
55. In respirastory chain electrons only are transported by:
A. Cytochrome c D. Ubiquinon
B. FAD E. Succinate dehydrogenase
C. NAD

56. Dehydrogenases are enzymes which catalyze transfer of the next


structural elements of the molecule:
A. Hydrogen D. Amino group
B. Phosphate group E. Oxygen anion
C. Methyl group

57. Which of the following vitamins is precursor of flavine


mononucleotide?
A. Vitamin B2 D. Vitamin C
B. Vitamin A E. Vitamin D
C. Vitamin B6

58. A component of the respiratory chain in mitochondria is;


A. Coenzyme Q D. Coenzyme containing thiamin
B. Coenzyme A E. Coenzyme B
C. Acetyl coenzyme

59. The component of respiratory chain enzymes cytochrome oxidase


(cytochrome a+a3), operates as a transporter of:
A. Electrons D. Cations
B. Protons E. Hydrogen
C. Anions

60. A copper containing oxidase is:


A. Cytochrome oxidase D. Xanthine oxidase
B. Flavin mononucleotide E. Lysine oxidase
C. Flavin adenine
dinucleotide

61. Peroxidase belongs to the next class of enzymes:


A. Oxido-reductases D. Lyases
B. Transferases E. Isomerases
C. Hydrolases

62. The redox carriers are grouped into respiratory chain complex:
A. In the inner mitochondrial D. On the inner surface of outer
membrane mitochondrial membrane
B. In mitochondiral matrix E. Cytosol
C. On the outer mitochondrial
membrane
63. The next enzyme complex serves as a point of entry for most of the
electrons generated by the action of the citric acid cycle:.
A. Complex I D. Complex IY
B. Complex II E. ATP synthase
C. Complex III

64. Reducing equivalents from succinate enter the mitochondrial


respiratory chain at:
A. FAD D. Cyt c
B. NAD E. Cyt b
C. Coenzyme Q

65. If the reducing equivalents enter from NAD in the respiratory chain,
the phosphate:oxygen ration (P:O) is:
A. 3 D. 4
B. 1 E. 5
C. 2

66. If the reducing equivalents enter from FAD in the respiratory chain,
the phosphate:oxygen ration (P:O) is:
A. 2 D. 4
B. 1 E. 5
C. 3

67. Reducing equivalents from succinate enter the mitochondrial


respiratory chain at:
A. FAD D. Cyt c
B. Coenzyme Q E. Cyt b
C. NAD

68. Some of the free energy released in the mitochondrial electron


transport chain can be harnessed to the formation of ATP. How many
moles of ATP can be formed per pair of electrons transferred from
reduced FAD to oxygen?
A. 2 D. 3
B. 0 E. 4
C. 1

69. Some of the free energy released in the mitochondrial electron


transport chain can be harnessed to the formation of ATP. How many
moles of ATP can be formed per pair of electrons transferred from
reduced NAD to oxygen?
A. 3 D. 0
B. 1 E. 4
C. 2
70. Cytochrome oxidase is an enzyme of respiratory chain with the next
characteristics:
A. Contains copper ions D. Contains two iron sulfur
B. Trasfers electrons to centers
cytochrome c E. Is a simple protein
C. Is not inhibited by cyanides

71. An enzyme catalyzing oxidoreduction, using oxygen as hydrogen


acceptor is:
A. Cytochrome oxidase D. Succinate dehydrogenase
B. Lactate dehydrogenase E. Fumarase
C. Malate dehydrogenase

72. Which of the following enzymes is tightly associated with inner


mitochondrial membrane?
A. Succinate dehydrogenase D. Fumarase
B. Citrate synthase E. Malate dehydrogenase
C. Alpha-ketoglutarate
dehydrogenase

73. Reducing equivalents from pyruvate enter the mitochondrial


respiratory chain at:
A. NAD D. Cyt b
B. FMN E. Cyt aa3
C. Coenzyme Q

74. Brown adipose tissue is:


A. Characterised by high C. Associated with high
content of mitochondria activity of ATP synthase
B. A prominent tissue in D. Characterised by low
human content of cytochromes
E. TCA doesn’t occur here

75. One of the respiratory chain enzymes, namely cytyochrome c, is a


transporter of:
A. Electrons D. Hydrogen atoms
B. Protons E. Oxygen anion
C. Cations

76. Most of hydrogen atoms (protons and electrons) involved in process


of tissue respiration are transported to the respiratory chain by the next
compound:
A. NADH+H+ D. FADH2
+
B. NADPH+H E. Pyruvate
C. FMNH2
77. Which from the respiratory chain enzymes needs FAD as
coenzyme?
A. Succinate C. Ubiquinone-cytochrome
dehydrogenase c reductase
B. NAD H2-ubiquinone D. Cytochrome oxidase
reductase E. Complex 3 enzymes

78. Dehydrogenases utilize, as coenzymes, all of the following except:


A. FH4 D. NAD+
B. NADP+ E. FMN
C. FAD

79. A mitochondrial marker enzyme is:


A. Succinate dehydrogenase D. Pyruvate dehydrogenase
B. Amylase E. Pepsin
C. Aldolase

80. Activity of what enzyme can be detected due to the color


development of benzidine after its oxidation?
А. Phenol oxidase D. Peroxidase
В. Cytochrome c Е. Lactate dehydrogenase
С. Aldehyde dehydrogenase

81. What reaction conditions are needed for oxidation of


pyrocatechin by molecular oxygen in presence of
phenoloxidase and production of oxidation products with
brown color?
А. In presence of pyrocatechin С. Absence of pyrocatechin
and potatoe juice D. In presence of Na2S
Е. In strong acidic medium
В. After boiling of potatoe juice

82. Which of the following actions describes the uncoupling of oxidative


phosphorylation in a mitochondrial system?
A. The phosphorylation of ADP continues but oxygen
ADP stops but oxygen uptake uptake stops
continues D. Oxygen uptake stops
B. The phosphorylation of E. Produced ATP is not
ADP to ATP accelerates translocated across inner
C. The phosphorylation of membrane to cytoplasm

83. Chemiosmotic theory for oxidative phosphorylation has been


proposed by:
A. P. Mitchell D. Chance and Williams
B. Pauling and Corey E. H. Krebs
C. S. Waugh

84. Activity of cytochrome oxidase is inhibited by:


A. Cyanide D. Sulphite
B. Sulphate E. Chloride
C. Arsenite

85. The chemical inhibiting oxidative phosphorylation, a dependent on


the transport of adenine nucleotides across the inner mitochondrial
membrane is:
A. Oligomycin C. Dinitrophenol
B. Atractyloside D. Pentachlorophenol

86. ATP synthetase is a multichain enzyme complex with the next


characteristic features:
A. It possess ion selective D. It is integral membrane
channel which selectively protein associated with outer
conduct protons membrane of mitochondria
B. Cyanides inhibit its E. It is inhibited by high
enzymatic activity irreversibly concentration of ADP and
C. It contains four iron-sulfur inorganic phosphate
centers

87. Note a compound which is considered as natural uncoupler of


oxidative phosphorylation:
A. Thermogenin D. Sapogenin
B. Progesteron E. Stearic acid
C. Succinic acid

88. Reactive oxygen species (ROS) include the next substances EXEPT:
A. Hydrogen ions D. Hydroxyl radical
B. Hydrogen peroxide E. Organic hydroperoxides
C. Superoxide anion

89. Uncouplers of oxidative phosphorylation are acting as follows:


A. Collapse proton gradient and ATP synthesis
across outer mitochondrial simultaneously
membrane D. Competitively inhibit
B. Allow electron transport FMN-dependent ubiquinone
to proceed and block ATP reductase
synthesis E. Inhibit the electron
C. Arrest electron transport transport in cytochromes chain
90. An enzyme catalyzing oxidoreduction, using oxygen as hydrogen
acceptor is:
A. Cytochrome oxidase D. Succinate dehydrogenase
B. Lactate dehydrogenase E. Fumarase
C. Malate dehydrogenase

91. The chemiosmotic hypothesis of oxidative phosphorylation


suggests that ATP is formed because:
A. Formation of proton energy bonds in mitochondrial
gradient across the inner proteins
membrane D. ADP is pumped out of the
B. The change in the matrix into the intermembrane
permeability of inner space
mitochondrial membrane toward E. Protons are pumped into
ADP and inorganic phosphate the mitochondrial matrix
C. Due to formation of high

92. Some hormones are acting as uncouplers of oxidative


phosphorylation. Chose from listed below hormones one considered as
potent uncoupler.
A. Thyroxine D. Insulin
B. Norepinephrine E. Cortisol
C. Testosterone

93. Catalase represents the next class of enzymes:


A. Oxidoreductases D. Lyases
B. Transferases E. Isomerases
C. Hydrolases

94. Rotenone inhibits the respiratory chain at:


A. FMN → coenzyme Q D. Cyt b → Cyt c1
B. NAD → FMN E. Cyt c1→Cyt a3
C. Coenzyme Q → cyt b

95. The transfer of reducing equivalents from NADH to coenzyme Q is


specifically inhibited by:
A. Piericidin A D. Rotenone
B. Oligomycin E. Theroxine
C. Carboxin

96. Note an endogenous compound which induces heat production due


to uncoupling of oxidative phosphorylation
A. Thermogenin D. Progesteron
B. Succinate E. Sapogenin
C. Stearate
97. Chose a toxic substance acting as potent uncoupler of oxidative
phosphorylation:
A. Dinitrophenol D. Sodium cyanide
B. Phenol E. Heme
C. Picric acid

98. Cychrome aa3 belongs to the next class of enzymes:


A. Oxido-reductases D. Lyases
B. Transferases E. Isomerases
C. Hydrolases

99. Catalase is an enzyme with the next properties:


A. It catalyses the C. Catalyses interaction of
decomposition of hydrogen CO2 and water and produce
peroxide with formation of water oxygen and formaldehyde as
and oxygen as products final products.
B. It reduces hydrogen D. Contains FMN as
peroxide using substances coenzyme
providing hydrogen atoms with E. It is NAD-dependent
formation of water and oxidized enzyme
substance

100. The driving force for ATP synthesis in mitochondria is created due
to:
A. Asymmetric distribution of energy release
protons across inner D. Generation of high energy
mitochondrial membrane intermediates during electron
B. Transport of electrons transport
along the chain of cytochromes E. Difference in electric
C. Reduction of oxygen and charge between inner and outer
production of water with free mitochondrial membranes

101. What products are produced from superoxide anion under the
action of superoxide dismutase?
A. Free oxygen D. NAD
B. Hydroxyl radical E. FMN reduced
C. Protons

102. Cytochrome oxidase is an enzyme of respiratory chain with the


next characteristics:
A. It contains copper ions D. It contains two iron sulfur
B. It trasfers electrons to centers
cytochrome c E. It is an integral membrane
C. It is not inhibited by protein associated with outer
cyanides mitochondrial membrane
103. What enzyme can decompose hydrogen peroxide without
involvement of organic compounds as donors of hydrogen?
A. Catalase D. Cytochrome P450
B. Peroxidase E. Lipoxygenase
C. Monooxygenase

104. Superoxide radicals can be detoxified by:


A. Cytochrome c D. Superoxide dismutase
B. Cytochrome b E. Peroxidase
C. Cytochrome a

105. Rate of tissue respiration is raised when the intracellular


concentration of:
A. ADP increases D. AMP decreases
B. ATP increases E. None of these
C. ADP decreases

106. Hydrogen peroxide may be detoxified in the absence of an oxygen


acceptor by:
A. Peroxidase D. Superoxide dismutase
B. Catalase E. Oxygenase
C. Cytochrome a

EXAMPLES OF KROK 1 TESTS

107. Mitochondria are subcellular organelles and are present in a


cytoplasm of every cell exept mature red blood cells, bacteria, blue-
green algae. What method is used principally for their isolation?
A. Differential centrifugation D. Spectrophotometry
B. Chromatography E. Gel-filtration
C. Electrophoresis

108. Enzymes of tricarboxylic acids cycle oxidize acetyl-CoA and


produce 3 molecules of reduced NAD and one molecule of reduced
FAD. Where are localized these enzymes?
A. In mitochondrial matrix D. In cell cytoplasm
B. On plasma membrane E. On inner mitochondrial
C. On external mitochondrial membrane
membrane

109. A patient was admitted into hospital with a diagnosis diabetes


mellitus type I. In metabolic changes the decrease of oxaloacetate
synthesis rate is detected. What metabolic passway is damaged as a
result?
A. Tricarboxylic acid cycle C. Cholesterol biosynthesis
B. Glycolysis D. Glycogen mobilization
E. Urea synthesis

110. Substrate phosphorylation is a process of phosphate residue


transfer from macroergic donor substance to ADP or some other
nucleoside diphosphate. What enzyme of tricarboxylic acid cycle
participates in reaction of substrate phosphorylation?
A. Succinyl thiokinase D. Fumarase
B. Citrate synthase E. Alpha-ketoglutarate
C. Succinate dehydrogenase complex
dehydrogenase

111. In a patient are manifested symptoms of intoxication with arsenic


compounds. What metabolic process is damaged taking into account
that arsen containing substances inactivate lipoic acid?
A. Oxidative decarboxylation of α- D. Coupling of oxidation and
ketoglutarate phopsphorylation
B. Fatty acids biosynthesis E. Microsomal oxidation
C. Neutralization of superoxide
anions

112. During metabolic process active forms of the oxygen including


superoxide anion radical are formed in the human body. With help of
what enzyme is this anion activated?
A. Superoxide dismutase D. Glutathioneperoxidase
B. Catalase E. Glutathionereductase
C. Peroxidase

113. ATP synthesis is totaly blocked in a cell. How will the value of
membrane rest potential change?
A. It will disappear decrease
B. It will be slightly increased E. First it will decrease, then
C. It will be considerably increased increase
D. First it will increase, then

114. Profuse foam appeared when dentist put hydrogen peroxide on the
mucous of the oral cavity. What enzyme caused such activity?
A. Catalase D. Glucose-6-
B. Cholinesterase phosphatdehydrogenase
C. Acetyltransferase E. Methemoglobinreductase

115. The next process occurs in suspension of mitochondria with


ruptured inner membrane and provided with malate and oxygen:
A. Transport of electrons C. Decrease of pH in the
along enzymes of respiratory external medium
chain D. Increase of pH in
B. Phosphorylation of ADP mitochondrial matrix
E. Oxydative phosphorylation will take place

116. During the necropsy of a 20-year-old girl a pathologist concluded


that the death of the patient had resulted from poisoning by cyanides.
The activity of what enzyme is mostly inhibited by cyanides?
A. Malate dehydrogenase D. Aspartate aminotransferase
B. Cytochrome oxydase E. Lactate dehydrogenase
C. Heme synthase

117. CO is extremely dangerous poison as it irreversibly blocks


repiratory chain of enzymes. At which point is arrested electron
transport in presence of CO?
A. Cytochrome oxidase D. Respiratory complex III
B. Succinate dehydrogenase E. NADH2-ubiquinon
C. Ubiquinon-cytochrome c reductase
reductase

118. Some hormones are acting as uncouplers of oxidative


phosphorylation. Chose from listed below hormones one which is
considered as the best uncoupler;
A. Thyroxine D. Insulin
B. Norepinephrine E. Cortisol
C. Testosterone

119. Activation of membrane lipid peroxydation is one of the basic


mechanisms of membrane structure and functions damage as well as
the death of a cell. The cause of this pathology is:
A. Vitamin E deficiency D. B1-hypervitaminosis
B. B12-hypervitaminosis E. B12-deficiency
C. B1-deficiency

120. The production of thyroid hormones is stimulated under


thyrotoxicosis. It leads to body weigh loss, tachycardia, and rise of
psychic irritability. Choose the biochemical mechanism by which
thyroid hormones affect the tissue bioenergetics from the listed below:
A. Uncoupling of oxidation D. Blockage of substrate level
and oxidative phosphorylation phosphorylation
B. Blockage of mitochondrial E. Activation of oxidation and
respiratory chain oxidative phosphorylation
C. Activation of substrate
level phosphorylation

121. Hydrogen peroxide is harmful and extremely toxic to living cells.


Chose an enzyme which is used by cells for neutralization of hydrogen
peroxide:
A. Glutathion peroxidase B. Cytochrome oxidase
C. NADP-H2-oxidase E. Monoamine oxidase
D. Cyclooxygenase

122. Superoxide anion is generated in course of tissue respiration and


is a strong oxidant, possessing a harmful effect. What enzyme is
involved in neutralization of this compound?
A. Superoxide dismutase D. Catalase
B. Monoamine oxidase E. Xanthine oxidase
C. Peroxidase

123. In diseases which are accompanied by hypoxia an incomplete


reduction of oxygen molecule in respiratory chain and accumulation of
hydrogen peroxide occurs. Note an enzyme which provides
neutralization of hydrogen peroxide:
A. Glutathion peroxidase D. Glutathion reductase
B. Cytochrome oxidase E. Oxidase of reduced NADP
C. Succinate dehydrogenase

124. Cyanides are extremely dangerous poisons as they irreversibly


block repiratory chain of enzymes. At which point is arrested electron
transport in presence of cyanides?
A. Cytochrome oxidase D. Respiratory complex III
B. Succinate dehydrogenase enzymes
C. Ubiquinon-cytochrome c E. NADH2-ubiquinon
reductase reductase

125. High resistance of “winter-swimmers” (so-called “walruses”) to low


temperatures is explained by increased production of certain hormones
that stimulate the processes of biological oxidation and heat formation
in the cells through the uncoupling of mitochondrial electron transfer
and the oxidative phosphorylation. Choose the name of these
hormones:
A. Thyroid hormones D. Insulin
B. Glucagone E. Corticosteroids
C. Adrenaline

126. During the necropsy of a 17-year-old girl a pathologist concluded


that the death of the patient had resulted from poisoning by cyanides.
The disturbance of what process became the most credible cause of the
girl’s death?
A. Tissue respiration hemoglobin
B. Synthesis of vhemoglobin D. Urea synthesis
C. Transport of oxygen by E. Atherosclerosis

127. Infant death occurs due to cyanide poisoning. What is the


biochemical mechanism of cyanides’ unfavorable action on the
molecular level?
A. Inhibition of cytochrome dehydrogenase
oxidase D. Inactivation of oxygen
B. Chemical bounding to the molecule
substrates of TCA E. Inhibition of cytochrome b
C. Blockage of succinate

128. Cyanides are cellular poisons, inhibiting electron transport on


terminal segment of respiratory chain in mitochondrias. What is the
mechanism of their toxic effect?
A. Formation of a complex with Fe+3 D. Inhibition of ATP-synthase
form of cytochrome oxidase function
B. Block up of electron transport on E. Uncoupling of respiration and
a level of NAD H –coenzyme Q- oxidative phosphorylation in
reductase. mitochondria
C. Block up of electron transport
from cytochrome bc1.

129. Enzymes of respiratory chain perform oxidation of substrates and


transfer of reductive equivalents to oxygen with production of water
molecules. Where they are located?
A. On inner mitochondrial D. On outer mitochondrial
membrane. membrane
B. On cytoplasmic membrane E. In nucleus
C. In cytoplasm

130. Patient P. is working in chemical industry, connected with cyanic


acid production. He complains in attacks of suffocation and dizziness.
What is the mechanism of cyanides effect upon tissue respiration?
A. Interaction with heme of C. Block the activity of
cytochrome a3 cytochrome c
B. Attacment to nitrogen atom in D. Block bonding with an oxygen
pyrimidine cycle of NAD E. Block the transfer of electrons
from cytochrome bc1

131. Coupling of tissue respiration and oxydative phosphorylation takes


place in the presence of electrochemical gradient of H + ions between
mitochondrial matrix and intermembranous space. What substance
from listed below can uncouple respiration and phosphorylation?
A. Rothenon D. Dinitrophenol
B. Somatotropin E. Glucose
C. Cyanides

132. Cytochromes are components of respiratory chain in


mitochondrias, which transfer electrons from ubiquinon to molecular
oxygen. What part of cytochrome molecule take part in oxydative-
reductive reactions?
A. Iron atom D. Proteinous part
B. Vynil residue E. Methen bridge
C. Pyrrole cycle
CLINICAL CASES AND SITUATIONAL TASKS

134. Amital is pharmaceutical, which is used in pharmacology as a hypnotic


agent. What is the mechanism of its actions on the processes of tissue
respiration?

Answer: Amital (aminobarbital) - a derivative of barbituric acid, which inhibits


cellular respiration by blocking electron transport ,at the level provided with the-
coenzyme Q-reductase

135. A patient with overactivity of the thyroid gland. Why biological


oxidation in cells is disturbed?

Answer: Thyroid hormones (thyroxine, triiodothyronine) activate Na+, K+-ATP-ase.


With their action, they observed the active absorption of O2 mitochondria without a
corresponding increase in the synthesis of ATP.

136. The patient, who had been using phenobarbital to treat Gilbert’s
disease, developed addiction to this drug. How can you explain the
mechanism of this addiction?

Answer: The addiction to phenobarbital induction explained by the synthesis of


cytochrome P-450. The phenobarbital oxidized mono-oxygenase system of
endoplasmic reticulum with cytochrome P-450.

137. Why do patients with cardiovascular failure and signs of acidosis to


administered the cocarboxilase (thiamine pirophosphate) drug?

Answer: Cocarboxilase (thiamine pirophosphate) is coenzyme form of vitamin B1,


which is a part of enzymes involved in oxidative dekabooxydation α-ketoacids (eg
pyruvate dehydrogenase). Cocarboxilase prescribed for acidosis origin of
diabetes, hepatic and renal failure, respiratory acidosis, chronic pulmonary heart
failure and coronary insufficiency and a number of different processes that
improve energy metabolism.

138. A postoperative patient on intravenous fluids develops lesions in the


mouth (angular stomatitis). Urinalysis indicates an excretion of 15 μg
riboflavin/mg creatinine, which is abnormally low. Which of the following
TCA cycle enzymes is most likely to be affected?

Answer: The patient has demonstrated a deficiency in riboflavin (urinary excretion


of less than 30 μg/mg creatinine is considered clinically deficient). Riboflavin is a
component of the cofactor FAD (flavin adenine dinucleotide), which is required for
the conversion of succinate to fumarate by succinate dehydrogenase.
139. After excessive drinking over an extended period of time while eating
poorly, a middle-aged man is admitted to the hospital with “high output”
heart failure. Which of the following enzymes is most likely inhibited?

Answer: This patient has exhibited symptoms of beri beri heart disease, which is
a result of a nutritional deficiency in vitamin B1 (thiamine). The active form of the
vitamin, thiamine pyrophosphate, is a required cofactor for α-ketoglutarate
dehydrogenase.

140. ATP may be hydrolysed to form ADP and P i (orthophosphate) or AMP


and PPi (pyrophosphate). Pyrophosphate may be subsequently hydrolyzed
to orthophosphate releasing additional free energy. Note the ΔG o values (1,
2 and 3), which are released in each reaction.
1
ATP + H2O ADP + Pi

+ 2 +
ATP H2O AMP PPi

3
PPi + 2 Pi
H2O

Answer: 1 - DGo=-7.3 kcal (-30.5 kJ/mol), 2 - DGo'=-7.7 kcal (-32.2 kJ/mol), 3 -


DGo'=-8 kcal (-33.5 kJ/mol)

141. A 16-month-old girl was found to have ingested approimately 30 mL of


an acetonitrile-based cosmetic nail remover when she vomited 15 minutes
postingestion. The poison control center was contacted, but no treatment
was recommended because it was confused with an acetone-based nail
polish remover. The child was put to bed at her normal time, which was 2
hours postingestion. Respiratory distress developed sometime after the
child was put to bed, and she was found dead the next morning. Inhibition of
which of the following enzymes was the most likely cause of this child’s
death?

Answer: The culprit here is cyanide produced from acetonitrile. Cyanide inhibits
the electron transport chain of cytochrome oxidase.

142. Several molecules are known to specifically inhibit the electron


transport process. Used in conjunction with reduction potential
measurements, inhibitors have been invaluable in the determination of the
correct order of electron-transport chain components. Insert the
corresponding inhibitors (at the scheme) to the sites of their action.
NADH NAD+(-0.315 V)
2e-
ADP + Pi
Complex I ?1
D Go’=-69kJmol-1
ATP
(+0.030 V)
Complex CoQ (+0.045 V)
Succinate FADH2
II

Fumarate ADP + Pi
Complex III
DGo’=-36.7kJmol-1 ?2
ATP
Cytochrome c
(+0.235V)

ADP + Pi
Complex IV
DGo’=-112kJmol-1 ?3
2e- ATP
2H+ +1/2O2 H2O (+0.815V)

Answer: 1- rotenone (a plant toxin used by Amazonian Indians to poison fish and
is also used as an insecticide), amital (a barbiturate); 2- Antimicin A, 3- monoxide
(CO), azide (N3-) and cyanide

143. Name the respiratory inhibitors, shown below:


OCH3
CH3O O
O CH2-CH2-CH(CH3)2
H HN
CH2-CH3
O

O N
O O H
H
CH2 2
C
1 H CH3
O O CH3
O
C - NH
O-C-CH2-CH(CH3)2
-C N H3C
O

3 OH
O
O (CH2)5-CH3
NH-CHO
4

Answer:1 – rotenone, 2 – amytal, 3 – cyanide, 4 – antymycin A.

144. Explain the role of ATP-synthase components, shown below:


Inner
membrane



Fo   
 
 F1
Intermembrane
Matrix
space

Answer: The F1 headpiece includes three α and three β subunits one copy each
of three other subunits (γ, δ and ε. F 0 includes a cluster of 9-12 copies of a small
peptide, which appears to form a transmembrane channel for protons.

145. Name enzymes of citric acid cycle:

CH3
O
C=
H2O S oA
NАD+ NАDН2
Acetyl-CoA
C H2-COOH
CH2 -COOH
CO-COOH
HO-CH-COOH 10 Oxaloacetate HSCoA
H2O L-malate 1
9 CH2-COOH
CH-COOH HO C COOH
=

CH-COOH CН2- COOH


FАDН2 Fumarate
Citrate
8 2 H2O
FAD
CH2 -COOH
CH2 -COOH
CH2 -COOH COOH
C
Succinate
=

CH-COOH
GTP Cis-aconitate
H2O
HSCoA 3
7 CH2-COOH

GDP CH-COOH
Pi HO-CH-COOH
CH2 -COOH
O = isocitrate
CH2 C NAD+
SCoA 4
Succinyl - CоА
NADН2
6 CH2-COOH
CH2 -COOH 5 CH-COOH
СО2 CH2
NADН2 NAD+ CO-COOH
CO-COOH Oxalosuccinate
a - Ketoglutarate
HSCoA СО2

Answer:1 – Citrate synthase, 2, 3 – aconitase, 4, 5 – isocitrate dehydrogenase,


6 – a-ketoglutarate dehydrogenase complex, 7 – succinate thiokinase, 8 –
succinate dehydrogenase, 9 – fumarase, 10 - malate dehydrogenase.
146. Fill in the blanks:

Answer: a – acetyl CoA, citrate syntase; b – isocitrate dehydrogenase, CO 2 ; c –


malat dehydrogenase

147. Inhibition of oxidative phosphorylation by cyanide ion leads to


increases in which of processes?
Answer: Gluconeogenesis requires ATP, which is in short supply, turning up the
catabolism of glucose to lactate in the absence of an intact electron transport
chain. ADP cannot be transported into the mitochondrion because ATP, its
antiporter partner, isn’t made by oxidative phosphorylation as a result of cyanide
inhibition of cytochrome oxidase. Metabolism of fatty acids and ketone bodies
requires a functional electron transport chain for their metabolism, and these
possibilities are also ruled out.

148. Name the following compounds of Krebs cycle:

Answer: 1 – oxaloacetate, 2 – malate, 3 – fumarate, 4 – citrate


SECTION ІV
STRUCTURE AND METABOLISM OF CARBOHYDRATES

1. The irreversible reactions of glycolysis include that catalyzed by:


A. 6-phosphofructo-1-kinase. D. Glyceraldehyde-3-phosphate
B. Fructose-bisphosphate aldolase.se. dehydrogenase.
C. Phosphoglucose isomerase. E. Phosphoglycerate kinase.

2. In yeast cells occurs a process which is similar to glycolysis - alcohol


fermentation. In course of this process through several stages from
pyruvate is produced:
A. Ethanol D. Glyceraldehyde
B. Lactate E. Pyruvate
C. Acetaldehyde

3. Glucokinase:
A. Has a high Km for glucose and C. Is widely distributed and
hence is important in the occurs in most mammalian
phosphorylation of glucose primarily tissues
after ingestion of a carbohydrate rich D. Is widely distributed in
meal Prokaryotes
B. None of these

4. The reaction catalysed by phosphofructokinase:


A. Is the rate-limiting reaction of the C. Uses fructose-1-
glycolytic pathway phosphate as substrate
B. Is activated by high D. Is inhibited by ADP
concentrations of ATP and citrate E. Is inhibited by fructose 2,
6-bisphosphate

5. Compared to the resting state, vigorously contracting muscle shows:


A. An increased conversion of C. A increased NADH/NAD+
pyruvate to lactate ratio
B. Decreased oxidation of pyruvate D. Decreased concentration
of CO2 and water of AMP

6. Pasteur effect is:


A. Inhibition of anaerobic glycolysis D. Activation of lactate
B. Oxygen is not involved synthesis
C. Inhibition of enzyme E. All of these
phosphofructokinase

7. Which one of the following would be expected in pyruvate kinase


deficiency?
A. Hemolytic anemia B. Increased levels of lactate
in the R.B.C
C. Decreased ratio of ADP to ATP E. Increased levels of
in R.B.C pyruvate
D. Increased phosphorylation of
Glucose to Glucose-6-phosphate

8. Which one of the following statements concerning glucose metabolism is


correct?
A. Glucose enters most cells by a C. Pyruvate kinase catalyses
mechanism in which Na+ and glucose an irreversible reaction
are co-transported D. An elevated level of
B. The conversion of Glucose to insulin leads to a decreased
lactate occurs only in the R.B.C level of fructose 2, 6-
bisphosphatein hepatocyte

9. Anaerobic oxidation of glucose to lactate is regulated by appropriate


enzymes. What enzyme is the major regulator of this process?
A. Phosphofructokinase D. Aldolase
B. Lactate dehydrogenase E. Enolase
C. Glucose-6-phosphate isomerase

10. 6-Posphofructokinase activity can be decreased by all of the following


EXCEPT:
A. AMP. D. Low pH.
B. ATP at high concentration. E. Decreased concentration
C. Citrate. of fructose 2, 6-bisphosphate.

11. During glycolysis, Fructose 1,6-biphosphate is decomposed by the


enzyme:
A. Aldolase D. Diphosphofructophosphat
B. Enolase ase
C. Fructokinase E. Hexokinase

12. The oxidation of lactic acid to pyruvic acid requires the following vitamin
derivative as the hydrogen carrier.
A. NAD+ D. FMN
B. Lithium pyrophosphate E. FAD
C. Coenyzme A

13. Erythrocytes require energy of ATP for their vital functions. What
process provides these cells with the necessary amount of ATP?
A. Anaerobic glycolysis D. Oxidation of fatty acids
B. Aerobic oxidation of glucose E. Citric acid cycle
C. Pentose phosphate pathway

14. Alcoholic fermentation occurs in yeast and several bacterial species. In


this process product of pyruvate decarboxylation is:
A. Acetaldehyde D. Phosphoenolpyruvate
B. Lactate E. Glucose
C. Ethanol

15. Which of the following enzymes is not involved in glycolysis?


A. Glucose oxidase D. Hexokinase
B. Enolase E. Glucokinase
C. Aldolose

16. The combination of subunits in lactate dehydrogenase molecule makes


it possible to create the next number of isoenzymes:
A. Five D. Six
B. Three E. Eight
C. Four

17. In aerobic glycolysis glucose is transformed to:


A. Pyruvic acid D. Citric acid
B. Ethylic alcohol E. Acetyl-CoA.
C. Lactic acid

18. All of the following statements apply to the digestive enzyme α–amylase
EXEPT:
A. It catalyses the hydrolysis of α1→4 C. The form of α–amylase in
glycosidic linkages exept those of human pancreas is the most
glucose unit that serve as branch important isoenzyme
points D. Glucose is the major product of
B. Glycogen and hydrated starch are α–amylase action on starch
normal substrates E. Cellulose, a plant carbohydrate,
is not hydrolysed by α–amylase
in man

19. Chose the reaction of glycolysis catalyzed by an enzyme


phosphofructokinase:
A. Production of fructose- D. Production of
1,6-bisphosphate dihyroxyacetone phosphate
B. Production of glucose- E. Production of
6-phosphate 1,3-bisphosphoglycerate
C. Production of fructose
1-phosphate

20. In glycolytic pathway all the substances are formed EXEPT:

A. Acetyl-CoA D. Adenosine diphosphate


B. Glyceraldehydes –3-phosphate E. Fructose-bis-phosphate
C. Phosphoenol pyruvate
21. Glycolysis is also referred to as:
A. Embden-Meyerhof-Parnas pathway D. Watson-Crick pathway
B. Lineweaver-Burk pathway E. Chargaff pathway
C. Krebs-Henseleit pathway

22. Which of the following is a substrate for aldolase activity in Glycolytic


pathway?
A. Fructose1,6-bisphosphate D. Fructose-6-phosphate
B. Glyceraldehyde-3-phosphate E. Glucose1, 6-
C. Glucose-6-phosphate bisphosphate

23. Which of the following enzyme-catalysed reactions has a product


containing a newly formed high energy phosphate bond?
A. 2-Phosphoglycerate to D. 3-Phosphoglycerate to 2-
phosphoenolpyruvate phosphoglycerate
B. The phosphorylation of glucose E. Dihydroxyacetone
C. Fructose 1,6-bisphosphate to phosphate to glyceraldehydes
glyceraldehydes-3-phosphate and phosphate
dihydroxyacetone phosphate

24. Glycolysis is regulated by:


A. Phosphofructokinase D. Aldolase
B. Enolase E. All of these
C. Glyceraldehydes-3-phosphate
dehydrogenase

25. Pyruvate kinase requires __ ions for maximum activity.


A. Mg2+ D. Br-
B. Mn2+ E. Cl-
C. Fe2+

26. Glucokinase can be characterized by one of the following statements:


A. It has a much higher Michaelis D. It is located in mitochondria
constant (Km) for glucose than does E. The reaction that is catalysed
hexokinase by this enzyme produces a high
B. It is the major regulatory enzyme of energy phosphate
glycolysis
C. ATP is an activator of this enzyme

27. One mole of glucose in glycolytic pathway in anaerobic conditions


produces:
A. Two moles of lactate D. Four moles of lactate
B. Five moles of lactate E. One mole of lactate
C. Three moles of lactate
28. Lactate dehydrogenase is an oligomeric molecule which contains how
many number of subunits:
A. Four D. Six
B. Two E. Eight
C. Three

29. The predominant products of starch hydrolysis by amylase are:


A. Maltose, maltotriose and limit D. Glucose, maltose and
dextrins maltotriose
B. Maltose, lactose and limit E. Lactose, sucrose and
dextrins maltose
C. Maltotriose, limit dextrins and
glucose

30. Glycolysis proceeds in:


A. Cytoplasm of every living cell D. Nuclei of liver cells
B. Cytoplasm of red blood cell only E. Lysosomal vesicles
C. Mitochondria of striated muscle cell

31. Excessive intake of ethanol increases the ratio:


A. NADH/NAD+ D. FAD/FADH2
+
B. NAD /NADH E. NADPH/NADP+
C. FADH2/ FAD

32. In glycolysis ATP is produced by the process of:


A. Substrate phosphorylation D. Oxidative phosphorylation
B. Photosynthetic phosphorylation
C. Transfer of phosphate group from
fructose-biphosphate upon ADP

33. Conversion of glucose to glucose-6- phosphate in human liver is


catalyzed by:
A. Glucokinase only D. Glucose-6-phosphate
B. Hexokinase only dehydrogenase
C. Hexokinase and glucokinase E. Glucophosphatase

34. As a result of anaerobic glycolysis glucose is converted to:


A. Lactic acid D. Succinic acid
B. Glycogen E. Acetoacetic acid
C. Citric acid

35. The following is an enzyme required for glycolysis:


A. Pyruvate kinase D. Glucose-6-phosphatose
B. Pyruvate dehydrogenase E. Glycerokinase
C. Pyruvate carboxylase
36. What is the net production of high energy phosphate bonds in anaerobic
glycolysis?
A. 2 moles D. 8 moles
B. 4 moles E. 10 moles
C. 6 moles

37. When O2 supply is inadequate, pyruvate is converted to:


A. Lactate D. Alanine
B. Phosphopyruvate E. Citrate
C. Acetyl CoA

38. UDP-Glucose is converted to UDP-Glucuronic acid by:


A. NAD+ D. NADP+
B. ATP E. FAD
C. GTP

39. Biosynthesis of the purine ring occurs owing to ribose-5-phosphate by


gradual joining of nitrogen and carbon atoms inside the heterocycle
structure and closing of the rings. The metabolic source of ribose-5-
phosphate is:
A. Pentose phosphate pathway. D. Gluconeogenesis.
B. Glycolysis. E. Glycogcnolysis.
C. Glycogenosis.

40. Which of the following compound is a positive allosteric modifier of the


enzyme pyruvate carboxylase?
A. Acetyl CoA D. ATP
B. Biotin E. AMP
C. Oxaloacetate

41. Galactose is phosphorylated by galactokinase to form:


A. Galactose-1-phosphate C. Galactose-1, 6 diphosphate
B. Galactose-6-phosphate D. All of these

42. Individuals who eat fresh fava beans are protected to a certain extent
from malaria. Which enzyme deficiency takes place under these
conditions?
A. Glucose-6-phosphate C. Ribulosephoshateisomerase
dehydrogenase D. Pyruvate dehydrogenase
B. Transaldolase E. Transketoase

43. In a patient pain along great nerve trunks is observed as well as increase
of pyruvate in blood. Insufficiency of what vitamin may cause these
symptoms?
A. Vitamin B1 C. Vitamin B6
B. Vitamin C D. Vitamin KE. Vitamin PP
44. The pentose phosphate pathway sometimes is referred to as:
A. Hexose monophosphate shunt D. Krebs-Henseleit pathway
B. Hexose bisphosphate shunt E. Embden-Meyerhof-Parnas
C. Chargaff pathway pathway

45. In the tricarboxylic acid cycle (TCA) are oxidized:


A. Acetyl-CoA D. Glucose
B. Pyruvate E. Fructose
C. Lactate

46. Two important byproducts of pentose phosphate pathway are:


A. NADPH and pentose D. Pentose and sedoheptulose
B. NADH and pentose sugars E. Glucose and NADH
C. Pentose and 4 membered sugars

47. Lipoic acid is a cofactor of the next enzyme complex:


A. Pyruvate dehydrogenase D. Cytochrome oxidase
B. Lactate dehydrogenase E. Transketolase
C. Succinate dehydrogenase

48. Which of the following metabolite integrates glucose and fatty acid
metabolism?
A. Acetyl CoA D. Lactate
B. Pyruvate E. Glucose
C. Citrate

49. Pyruvate dehydrogenase complex and α-ketoglutarate dehydrogenase


complex require the following for their oxidative decarboxylation:
A. CoASH, TPP,NAD+,FAD, Lipoate D. CoASH and TPP
B. CoASH and lipoic acid E. TPP
C. NAD+ and FAD

50. The following enzyme is required for the hexose monophosphate shunt
pathway:
A. Glucose-6-phosphate C. Phosphorylase
dehydrogenase D. Aldolase
B. Glucose-6-phosphatase E. Phosphofructokinase

51. UDP Glucuronic acid is required for the biosynthesis of


A. Glycogen D. Chondroitin sulphates
B. Lactose E. Pyruvate
C. Starch

55. Our body can get pentoses from:


A. HMP shunt B. Glycolytic pathway
C. Uromic acid pathway E. Gluconeogenesis
D. TCA cycle

56. Which from listed below pathways is responsible for the synthesis of
ribose-5-phosphate, a component of nucleic acids:
A. Pentose phosphate pathway D. Krebs cycle
B. Glycolysis E. Embden-Meyerhof-Parnas
C. Oxidative decarboxylation of pyruvate pathway

57. Transketolase (one of enzymes of pentose phosphate pathway) contains


as coenzyme:
A. TPP D. NAD
B. Tocoferol E. Folic acid
C. Pyridoxine

58. Which of the following substances inhibit pentose phosphate pathway?


A. NADHPH D. NAD+
B. ADP E. Mg2
C. FAD

59. Pyruvate dehydrogenase complex contains derivatives of the following


vitamins:
A. Lipoic acid D. Retinol
B. Pyridoxine E. Ubiquinone
C. Cyanocobalamine

60. Dehydrogenase enzymes of the hexose-monophosphate shunt are


A. NADP+ specific D. FMN specific
+
B. NAD specific E. Biotin specific
C. FAD specific

61. There are several pathways for glucose transformation and utilization,
one of them is pentose phosphate pathway, which actively proceeds in liver,
adrenal cortex, red blood cells. What is the main aim of this pathway?
A. NADPH2 generation and production C. FADH2 generation
of pentoses D. Synthesis of glycogen and
B. NADH2 and gluconioc acid fat
production E. Acetyl-CoA production

62. Oxidative decarboxylation of pyruvate requires


A. CoASH D. Pyridoxal phosphate
+
B. NADP E. Biotin
C. Cytichromes

63. The next product is formed during oxidative decarboxylation of


pyruvate?
A. Acetyl-CoA D. Lactic acid
B. Acyl-CoA E. Succinyl-CoA
C. Oxaloacetic acid

64. Which from listed below vitamins is involved in the oxidative


decarboxylation of pyruvate?
A. Lipoic acid D. Ascorbic acid
B. Tocoferol E. Folic acid
C. Pyridoxine

65. Before pyruvic acid enters the TCA cycle it must be converted to:
A. Acetyl CoA D. Citrate
B. Lactate E. Glucose
C. α-ketoglutarate

66. The oxidative phase of pentose phosphate pathway is very active in red
blood cells and hepatocytes. Which of the following is the end product of
this phase?
A. Ribulose-5-phoshate D. Glyceraldehyde-3-
B. 6-Phospho-gluconate phosphate
C. Pyruvate E. Fructose-6-phoshate

67. Which of the following enzymes catalyze reaction: Fructose+ATP →


Fructose-1-phosphate + ADP:
A. Fructokinase D. Glucokinase
B. Galactokinase E. Pyruvate kinase
C. Hexokinase

68. Which one of the following is a rate limiting enzyme of


gluconeogenesis?
A. Pyruvate carboxylase D. Pyruvate kinase
B. Hexokinase E. Phosphoglucomutase
C. Phsophofructokinase

69. Reactions of synthesis and degradation of glycogen in cells are


regulated by phosphorylation of the next key enzymes of glycogen
metabolism:
A. Glycogen synthase and D. Phosphoprotein
glycogen phosphorylase phosphatase and protein
B. Glycogen phosphorylase and kinase
lipase E. Adenilate cyclase and
C. Glycogen synthase and protein lipase
kinase
70. Phosphorolysis of carbohydrates plays a key role in a mobilization of
polysaccharides. Under the action of phosphorylase from glycogen is
produced by the next substance:
A. Glucose -1-phosphate D. Glucose
B. Glucose 1,6-bis-phosphate E. Fructose 6-phosphate
C. Glucose 6-phosphate

71. Glycogen synthesis takes place under the action of several enzymes.
Indicate, what enzyme provides the formation of 1,6-glycosidic bonds in
glycogen molecule?
A. Glycosyl 4,6-transglycosidase D. Glucokinase
B. Glycogen synthase E. Glucose 1-phosphate
C. Hexokinase uridil transferase

72. Insulin exhibits the next effect on glycogen metabolism:


A. Stimulation of glycogen D. Stimulation of glycogen
breakdown by activation of utilization in muscle cells
phosphorylase E. Suppression of glycogen
B. Inhibition of glycogenolysis by breakdown by inhibition of
suppression of phosphorylase debranching enzyme.
C. Stimulation of glycogenesis by
activation of glycogen synthase

73. Glycogen synthase is characterized by all of the following statements


EXEPT:
A. It is activated by phosphorylation D. It requires the primer strand of
B. Its substrate is uridine diphosphate glycogen
glucose E. It is found in association with
C. The enzyme exists in active and glycogen granules
inactive forms

74. The greatest quantity of the body glycogen can be found in which of the
following human tissue?
A. Liver D. Cardiac muscle
B. Kidney E. Brain
C. Sceletal muscles

75. The conversion of alanine to glucose is termed:


A. Gluconeogenesis D. Specific dynamic action
B. Glycolysis E. Glycogenesis
C. Oxidative decarboxylation

76. The increase in glucose concentration in blood under the action of


glucagone is connected with activation of the next enzyme:
A. Glycogen phosphorylase C. Glucokinase
B. Hexokinase D. Aldolase
E. Glycogen synthase

77. The blood sugar raising action of the hormones of suprarenal cortex is
due to:
A. Gluconeogene D. Due to
sis inhibition of glomerular filtration
B. Glycogenolysis E. Glycolys
C. Glucagon-like is
activity

78. Emotional hyperglycemia is induced by enhanced secretion of the


hormone:
A. Epinephrine D. Cortisol
B. Insulin E. Thyroxine
C. Progesteron

79. Activity of enzymes of glycogen metabolism is regulated in the process:


A.Covalent modification by D.By isomerization of reaction
phosphorylation-dephosphorylation products
B.By limited proteolysis E.By a noncompetitive inhibition
C.By dissociation into subunits

80. Amylo 1,6 glucosidase is referred as:


A. Debranching C. Glucantr
enzyme ansferase
B. Branching D. Phosph
enzyme orylase
E. Sythase

81. Protein avidin, a minor constituent of uncooked eggs, is a powerful


inhibitor of biotin enzymes. Which of the below listed metabolic
transformations would be blocked in case of the avidin addition to the cells
homogenates?
A. Pyruvate→Oxaloacetate. D. Glucose→ribosc 5-phosphate.
B. Glucose→pyruvate. E. Lactate→pyruvate.
C. Oxaloacetate→glucose

82. Post-translational covalent modification is an important factor in the


regulation of the enzymes' activity. Choose the mechanism of regulation of
glycogen phosphorylase and glycogen synthetase activities from the
following:
A. Phosphorylation-dephosphorylation D. Restricted proteolysis.
B. Methylation. E. ADP-ribosylation.
C. Adenylation.

83. In animals, an enzyme unique to gluconeogenesis is:


A. Fructose 1,6- D. Glyceral
bisphosphatase dehydes 3-phosphate
B. Enolase dehydrogenase
C. Phosphglycero E. Aldolase
mutase

84. The increase in glucose concentration in blood under the action of


glucagon is connected with activation of the next enzyme:
A. Glycogen phosphorylase D. Aldolase
B. Hexokinase E. Glycogen synthase
C. Glucokinase

85. The activation of liver phosphorylase is achieved by the next process:


A. Phosphorylation by ATP D. Glycosylation in Golgi
B. Limited proteolysis vesicles
C. Dephosphorylation by E. Phosphorylation by cAMP
proteinphosphatase

86. The carbohydrate reserved in human body is:


A. Glycogen D. Inulin
B. Starch E. Lactose
C. Glucose

87. Inactivation of glycogen synthase is dependent on the next reaction:


A. Dephosphorylation by protein D. Irreversible inhibition by
phosphatase glucose 6-phosphate
B. Phosphorylation by ATP E. Incorporation of enzyme
C. Allosteric inhibition by glucagone into lysosomal vesicles.

88. All the following about glycogenolysis are true except:


A. Glucose is the end product in liver C. It is stimulated by adrenaline
and muscle and nor adrenaline
B. It is the breakdown of glycogen to D. Von-Gierk's disease
glucose deficiency of glucose-6-
phosphatase

89. The active form of glycogen ___ is phosphorylated; the active form of
glycogen ___ is dephosphorylated.
A. Phosphorylase; synthase D. Hydrolase; semisynthase
B. Hydrolase; dehydrogenase E. Synthase; phosphorylase
C. Dehydrogenase; hydrolase

90. Which of the following is a precursor for glucose synthesis via


gluconeogenesis?
A. Glycerol B. Leucin
C. Cholesterol E. Cortisone
D. Glucagon

91. Lactate formed in muscles can be utilised through


A. Cori’s cycle D. Citric
B. Rapoport- acid cycle
Luebeling cycle E. Tricarbo
C. Glucose- xylic acid cycle
alanine cycle

92. Some hours after an intensive physical training a sportsman showed


activated gluconeogenesis. Which of the following is the basic substrate of
gluconeogenesis?
A. Lactate. D.α-Ketoglutarate.
B. Aspartate. E. Serine.
C. Glutamate.

93. The precursor to glycogen in the glycogen synthase reaction is:


A. UDP-glucose. D. UTP-glucose.
B. Gucose-l-phosphate. E. None of the
C. Glucose-6- phosphate. above

94. The characteristic enzymes of gluconeogensis are found in the cytosol,


except for:
A. Pyruvate carboxlyase, D. Frustocse-1,6-
which is in the mitrochondria. bisphosphatase, which is in
B. Fructose-1.6- the glycogen granule.
bisphosphatase, which is in the E. Pyruvate
mitochondria. carboxylase, which is in the
C. Glucose-6- glycogen granule.
phosphatase, which is in the
mitrochondria.

95. Each of the following enzymes is required for the conversion of glycerol
to glucose EXCEPT:
A. Phosphoenol pyruvate C. Glycerol-3-phosphate
carboxykinase dehydrogenase
B. Glucose-6-phosphatase D. Fructose 1.6 bisphosphtase
E. Triosophosphate isomerase

96. Which of the following supports gluconeogenesis?


A. Pyruvate + ATP + HCO3 = C. Acetyl CoA +
+
oxaloacetate + ADP + Pi + H oxaloacetate + H2O = citrate +
B. α-ketoglutarate + aspartate = CoA
glutamate + oxaloacetate D. Leucine degradation
E. Lysine degradation
97. Increase in blood glucose concentration under the action of glucagone
is caused by the activation of which of the following enzyme:
A. Glycogen phosphorylase D. Aldolase
B. Hexokinase E. Glycogen synthase
C. Glucokinase

98. The concentration of glucose in the blood plasma of a healthy man


varies within the following limits:
A. 3.3-5.5 mM/l. D.6.0-9.5 mM/1.
B. 2.0-4.0 mM/1. E. 1.0-2.0 mM/1.
C. 10.0-25.0 mM/1.

99. In diabetes mellitus and during starvation an increase of ketone bodies


in blood is observed, which are used as energetic material. Indicate the
substance from which they are synthesized.
A. Acetyl- Co A D. a-Ketoglutarate
B. Citrate E. Malate
C. Succinyl-Co A

100. Glucose is completely reabsorbed in renal tubules up to the next value


of blood glucose level:
A. 10 mmoles/l D. 80 mg/dl
B. 7,5 mmoles/l E. 150 mg/dl
C. 5,5 mmoles/l

101. What is not correct about the action of growth hormone?


A. It decreases liver qlycogen store D. It causes increased uptake of
B. It increases plasma glucose arnino acids by tissues
concentration E. It has ketogenic effect
C. It increases protein synthesis

102. Destruction of pancreatic islets of Langerhans results in the decrease


of production of:
A. Glucagon and insulin D. Insulin and adrenaline
B. Parathhormone and cortisone E. Callicrein and angiotensin
C. Thyroxyne and calcitonin

103. Cyclic AMP is formed from ATP by the enzyme adenylate cyclase which
is activated by the hormone:
A. Epinephrine D. Progesterone
B. Insulin E. Cortisol
C. Testosterone .

104. Increase in blood glucose concentration under the action of glucagone


is caused by activation of the following enzyme:
A. Glycogen phosphorylase D. Aldolase
B. Hexokinase E. Glycogen synthase
C. Glucokinase

105. Blood glucose level is decreased by:


A. Insulin D. Glucocorticoid hormones
B. Glucagone E. Testosterone
C. Epinephrine

106. Appearance of sugar and ketone bodies is revealed in the patient's


urine. Blood glucose concentration is 10.1 mM/l. What is a presumptive
diagnosis of the patient?
A. Diabetes mellilus. D. Pancreatitis.
B. Atherosclerosis. E. Myocardial infarction.
C. Toxic hepatitis.

107. Utilization of glucose occurs by means of sugar transport from the


extracellular matrix through the plasma membrane into the cell. What
hormone stimulates this process?
A. Insulin D. Aldosterone
B. Glucagon E. Adrenaline
C. Thyroxine

108. Adrenalin is water soluble hormone, its effect is mediated by secondary


messengers, which are:
A. cAMP D. Acetylcholine
B. NAD E. ATP
C. Cytochrome c

109. All of the following are disaccharides EXCEPT:


A.D-glucose C.Lactose
B.Maltose D.Sucrose

110. Glucose, fructose and galactose are classified as which type of sugar?
A. Monosaccharides C. Oligosaccharides
B. Disaccharides D. Polisaccharides

111. Glycogen, starch, and glycosaminoglycans are classified as:


A. Polysaccharides C. Disaccharides
B. Monosaccharides D. Oligosaccharides

112. Which of the following statements concerning glycosaminoglycans is


true?
A. They contain repeating acetylneuraminic acid
disaccharides C. They seldom contain
B. They contain branches of N- sulphate groups
D. They are most often positively E. They contain short
charged oligosaccharide chain

113. The most abundant glycosaminoglycan in the body is:


A. Chondroitin sulphate D. Dermatan sulphate
B. Keratan sulphate E. Heparin
C. Hyaluronic acid

114. Dextrans are:


A. Soluble polysaccharides of glucose galactose
B. Soluble polysaccharides of fructose D. Soluble polysaccharides of
C. Soluble polysaccharides of ribose

115. Sugars that contain aldehyde groups that are oxidized to carboxylic
acids are classified as:
A. Reducing sugars C. Polar sugars
B. Non-reducing sugars D. Non-polar sugars

116. Which of the following glycosaminoglycans is also known as cement


substance of tissues?
A. Hyaluronic acid C. Determatan sulphate
B. Chondroitin sulphate D. Heparin sulphate

EXAMPLES OF KROK 1 TESTS

117. An untrained person who has not been practicing physical exercises
for a long time complains of a muscle pain as a result of intensive manual
work. What is the probable reason of the pain syndrome?
A. Accumulation of lactate in D. Accumulation of
muscles creatinine in muscles
B. Decreasing of lipids level in E. Increase of ATP level in
muscles muscles
C. Increased disintegration of
muscle proteins

118. The high speed sprint causes a feeling of pain in skeletal muscles of
untrained people that occurs due to lactate accumulation. The activation of
what biochemical process is it resulting from?
A. Glycolysis D. Lipogenesis
B. Gluconeogenesis E. Glycogenesis
C. Pentose phosphate pathway
119. A 7-year-old girl manifests obvious signs of anemia. Laboratory tests
showed the deficiency of pyruvate kinase activity in erythrocytes. The
disorder of what biochemical process is a major factor in the development
of anemia?
A. Anaerobic glycolysis D. Oxidative phosphrylation
B. Deamination of amino acids E. Breaking up of peroxides
C. Tissue respiration

120. The intake of aspirin by a 3-year-old child with a fever caused marked
erythrocytes hemolysis. The inherited deficiency of what enzyme could be
the cause of the hemolytic anemia development?
A. Glucosо-6-phosphate dehydrogenase. D. Glycerol-phosphate
B. Glucose-6-phosphatase. dehydrogenase.
C. Glycogen phosphorylase. E. γ-Glutaminyl transferase.

121. A 2-year-old boy has the increase of liver and spleen sizes detected and
eye cataract present. The total sugar level in blood is increased, but glucose
tolerance is within the normal range. The inherited disturbance of the
metabolism of what substance is the cause of the indicated state?
A. Galactose. D. Maltose.
B. Fructose. E. Saccharose.
C. Glucose.

122. A newborn child had dyspepsia phenomena (diarrhea, vomiting)


detected after feeding with milk. After additional feeding with glucose the
morbid symptoms disappeared. The insufficient activity of what enzyme that
takes part in the carbohydrates breakdown causes the indicated disorders?
A. Lactase. D. Isomaltase.
B. Amylase. E. Maltase.
C. Saccharase.

123. A newborn child with the signs of cataract, growth and mental
retardation, who manifested vomiting and diarrhea, was brought to an
emergency clinic. A presumptive diagnosis of galactosemia was made. The
deficiency of what enzyme occurs in case of this disease?
A. Galactose-1-phosphate uridyl D.Hexokinase.
transferase. E. Glucose-6-phosphate
B. Glucokinase. dehydrogenase
C. UDP-galactose-4-epimerase.

124. A cataract and fatty degeneration of the liver develop in the conditions
of high galactose and low glucose level in blood. What disease do these
symptoms testify to?
A. Galactosemia. D. Steroid diabetes.
B. Diabetes mellitus. E. Fructosemia.
C. Lactosemia.
125. In a patient are manifested symptoms of intoxication with arsenic
compounds. What metabolic process is damaged taking into account that
arsenic containing substances inactivate lipoic acid?
A. Oxidative decarboxylation of pyruvate D. Coupling of oxidation and
B. Fatty acids biosynthesis phopsphorylation
C. Neutralization of superoxide anions E. Microsomal oxidation

126. In a weak apathic infant an enlarged liver was detected, which in


investigation of biopcia pieces showed an excess of glycogen. Blood
glucose concentration is under the normal value. What may be the cause of
this disease?
A. Lowered activity of glycogen D. Lowered activity of
phosphorylase in a liver glucokinase
B. Lowered activity of glycogen E. Deficiency of gene
synthase responsible for synthesis of
C. Lowered activity of glucose 6- glucose 1-phosphate uridyl
phosphate isomerase transferase

127. During biochemical investigation of blood in a patient was detected


hypoglycemia in fasting condition. Investigation of liver bioptates revealed
the failure of glycogen synthesis. What enzyme deficiency may cause such
status?
A. Glycogen synthase D. Fructose bis-phosphatase
B. Phosphorylase E. Pyruvate carboxylase
C. Aldolase

128. In an infant with point mutations in genes the absence of glucose 6-


phosphatase, hypoglycemia and hepatomegalia were revealed. What
disease is characterized by these symptoms?
A. Gierke disease D. Cori disease
B. Adison disease E. Mac Ardle disease
C. Parkinson disease

129. In a patient a lowering in ability to physical load was revealed, while in


skeletal muscles the glycogen content was increased. The decrease in
activity of what enzyme may cause this condition?
A. Glycogen phosphorylase D. Glycogen synthase
B. Phosphofructokinase E. Glucose 6-phosphatase
C. Glucose 6-phosphate
dehydrogenase

130. What biochemical process is stimulated in the liver and kidneys of a


patient exhausted by starvation?
A. Gluconeogenesis. C. Synthesis of bilirubin.
B. Synthesis of urea. D.Formation of hippuric acid.
E. Synthesis of uric acid.

131. Under Girke's glycogenosis the conversion of glucoso-6-phosphate


into glucose is disturbed, which results excessive glycogen accumulation in
liver and kidneys. The deficiency of what enzyme is the cause of the
disease?
A. Glucoso-6-phosphatase. D. Hexokinase.
B. Glycogen synthase. E. Aldolase.
C. Phosphorylase.

132. A child is sluggish and inert. His liver is enlarged. The liver biopsy
showed the excess of glycogen. The concentration of glucose in blood
plasma is below the normal range. What is the cause of the glucose level
decrease in blood?
A. Reduced (or absent) activity of D. Reduced (or absent) activity of
glycogen phosphorylase in the liver glucose-6-phosphatase.
B. Reduced (or absent) activity of E. Deficiency of the gene, which is
hexokinase. responsible for the synthesis of
C. High activity of glycogen synthase glucose-1-phosphate uridine
transferase.

133. The major process responsible for maintaining blood glucose in 4


hours after last meal is:
A. Glycogenolysis D. Gluconeogenesis
B. Glycolysis E. Glycogenesis
C. The pentose phosphate
pathway

134. The major process responsible for maintaining blood glucose in 40


hours after last meal is:
A. Gluconeogenesis D. The pentose
B. Glycolysis phosphate pathway
C. Glycogenolysis E. Glycogenesis

135. A 46-year-old woman complains of dryness in the oral cavity, thirst,


frequent urination, general weakness. Biochemical research of the patient's
blood showed hyperglycemia and hyperketonemia. Sugar and ketone
bodies are revealed in the urine. Diffuse changes in myocardium are
marked on the electrocardiogram. Make an assumptive diagnosis of the
illness.
A. Diabetes insipidus. C. Acute pancreatitis.
B. Alimentary hyperglycemia. D. Diabetes mellitus.
E. Ischemic cardiomyopathy.

136. A patient was admitted to a hospital in comatous state. The


accompanying mates explained that the patient loss his consciousness
during the training on the last stage of marathon distance. What coma type
can be recognized?
A. Hypoglycemic D. Hypothyroid
B. Hyperglycemic E. Hepatic
C. Hypovolemic

137. A patient addressed to physician with complaints on permanent thirst.


In laboratory investigation it was revealed hyperglycemia, polyuria and
increased content of 17-ketosteroids in urine. What disease is the most
probable?
A. Steroid diabetes D. Glycogenosis of the 1
B. Insulin dependent diabetes type
mellitus E. Myxoedema
C. Addison disease

138. A 38-year-old man is receiving treatment for schizophrenia in hospital.


Fhe initial levels of glucose, ketone bodies and urea in the blood are within
the normal range. Shock therapy put into practice by regular insulin
injections resulted in the development of the comatose state which
improved the clinical status of the patient. What is the most probable cause
of insulin coma?
A. Hypoglycemia. D. Ketonemia.
B. Dehydratation of tissues. E. Hyperglycemia.
C. Metabolic acidosis.

139. In a 57-year-old patient, suffering from diabetes mellitus, ketoacidosis


has been developed. Biochemical background of this status is decrease in
utilization of acetyl-CoA due to a deficiency of:
A. Oxaloacetate D. Aspartate
B. 2-oxoglutarate E. Succinate.
C. Glutamate

140. In patients’ blood glucose level is over the renal threshold, polyuria is
observed, as well as acidosis and ketonuria. What disease can be
suggested?
A. Diabetes mellitus D. Addison disease
B. Starvation E. Hyperthyreosis
C. Hypercorticism

141. In a patient a hyperglycemia, ketonuria, polyuria and glucosuria were


revealed. What disorders of acid-base equilibrium can be expected in this
conditions?
A. Metabolic acidosis D. Respiratory acidosis
B. Respiratory alkalosis E. Normal acid-base equilibrium
C. Metabolic alkalosis
142. A 58 years old woman, in a heavy state, cloudy consciousness, dry
skin, cyanosis, an odor of spoiled apples from mouth were revealed. Blood
glucose level – 15,1 mmol/l, in urine – 3,5 %. This state is caused by:
A. Hyperglycemic coma D. Uremic coma
B. Hypoglycemic coma Hypovolemic coma
C. Anaphylactic shock

143. By consumption of fresh milk in kid developed disorders of digestive


tract. Consumption of other sugar-containing food-stuffs did not cause
similar disorders. Genetically determined insufficiency of what enzyme can
be considered in this case?
A. Lactase D. Hexokinase
B. Phosphoglucomutase E. Glucose-6 phosphate
C. Glycogen synthetase isomerase

144. A 45-year-old woman does not have any symptoms of insulin


dependent diabetes mellitus but testing on an empty stomach showed the
increase of the blood glucose level (7.5 mM/l). What additional laboratory
test needs to be done to substantiate the diagnosis?
A. Determination of tolerance to glucose. D. Determination of tolerance to
B. Determination of ketone bodies glucose on an empty stomach.
concentration in the urine. E. Determination of glycosylated
C. Determination of rest nitrogen level in hemoglobin level.
the blood.

145. A condition called diabetic ketoacedosis is caused by a lack of insulin


leading to a build-up of ketoacids. Excessive ketone bodies are formed by
the biochemical imbalance in uncontrolled or poorly managed diabetes.
Which compounds are called ketone bodies?
A. Acetoacetate, β-hydroxybulyrate, and D. Acyl-CoA, malonyl-CoA
acetone E. Cholic and deoxycholic acid
B. Aspartate, pyruvate
C. α-ketoglutarate, malate, sussinate

146. Renal threshold, polyuria is observed, as well as acidosis and


ketonuria. What disease can be suggested?
A. Diabetes mellitus D. Addison disease
B. Starvation E. Hyperthyreosis
C. Hypercorticism

147. A worker of a chemical plant was brought to a hospital with signs of


poisoning. In the woman's hair a high level of arsenate that blocks the lipoic
acid was revealed. The disorder of what biochemical process is the most
probable cause of poisoning?
A. Oxidative decarboxylation of pyruvate. C. Reduction of methemoglobin.
B. Microsomal oxidation. D. Reduction of organic oxides.
E. Rendering superoxide radicals
harmless
.
148. During the investigation in some person was revealed blood glucose
concentration on a level 4,5 mMoles/l. It may be the next interpretation of
obtained result:
B. A person is healthy E. A person has
C. A person has diabetes diabetes insipidus
mellitus F. A person has
D. A person possess steroid diabetes
increased tolerance toward glucose

149. A 40-year-old woman diagnosed with diabetes mellitus is admitted to a


department of endocrinology. The patient complains of thirst and increased
hunger. What pathological components are exposed at laboratory research
of the patient's urine?
A. Glucose, ketone bodies C. Protein, creatine
B. Protein, amino acid D. Bilirubin, urobilin

150. A woman in the unconscious state was brought to an emergency clinic.


Laboratory research revealed that the blood glucose level makes 1.98 mM/1,
the level of hemoglobin is 82 g/1, the amount of erythrocytes is 2.1∙1012, SSE
(speed of erythrocytes settling) is 18 mm/hour and the amount of leucocytes
is 4.3∙109/l. Make a possible diagnosis.
A. Hypoglycemia. D.Somatotropin deficiency
B. Diabetes mellitus. E. Diabetes insipidus.
C. Galactosemia.

151. Under diabetes mellitus, the level of ketone bodies in blood


dramatically rises, which results in the development of metabolic acidosis.
What substance is the precursor of the ketone bodies synthesis?
A. Acetyl-CoA. D.Malonyl-CoA.
B. Succinyl-CoA. E. Mcthylmalonyl-CoA.
C. Propionyl-CoA

152. A patient manifests ketonuria. What disease is recognized by the


augmented concentration of ketone bodies in his urine?
A. Diabetes mellitus. D. Tuberculosis of the kidney.
B. Acute glomerular inflammation. E. Myocardial infarction.
C. Urolithiasis.

153. A patient suffering from diabetes mellitus fainted after the introduction
of insulin, and then cramps appeared. What level of sugar was determined
in the patient's blood by means of biochemical analysis?
A. 10.0 mM/1. C. 8.0 mM/1.
B. 3.3 mM/1. D. 5.5 mM/1.
E. 1.5 mM/1.

154. Glucagon is produced in which endocrine gland?


A. Langerhans islands, α cells D. Thymus
B. Parathyroid gland E. Medullar part of adrenals
C. Pitiutary gland

155. A 17-year-old male presents complaining of an inability to perform


strenuous exercise without bringing on painful muscle cramps and
weakness. He indicated that mild to moderate exercise resulted in no
problems. When he was administered an ischemic exercise test, his serum
lactate concentrations did not increase significantly. A deficiency in which
of the following enzymes is most likely the cause of the patient’s muscle
cramps?
A. Glycogen phosphorylase D. Glycogen synthase
B. Glucose-6-phosphatase E. Very long chain acyl-CoA
C. Carnitine palmitoyl transferase II dehydrogenase

156. A 27-year-old man has been rushed to the emergency room following
his sudden collapse and entry into a state of unconsciousness. Examination
of personal belongings revealed the patient is an insulin-dependent
diabetic. A rapid decline in which of the following humoral factors likely
triggered the sudden collapse of the patient?
A. Glucose D. Insulin
B. Glucagon E. Triglyceride
C. Fatty acids

CLINICAL CASES AND SITUATIONAL TASKS

157. What substances are used for the synthesis of glucose in the human
body during prolonged fasting or exhausting work? Explain your answer.

Answer: In such conditions, synthesis of glucose is possible from


noncarbohydrate compounds by gluconeogenesis. These metabolites are lactic
and pyruvic acids, glycogenic amino acids, glycerol.

158. Explain the negative impact of excessive intake of carbohydrates on


the human body. Which biochemical disorders it causes?

Answer: Excessive consumption of carbohydrates leads to the synthesis of lipids,


obesity, atherosclerosis, development of cardiovascular diseases.

159. Examination of a patient revealed increased activity of LDH1, LDH2 and


creatine. What human organ might be damaged?
Answer: Increased activity of LDH1, LDH2 and creatine in blood serum indicates
development of pathological process in heart muscle.

160. Diastase was not indicated in the urine of a patient during biochemical
investigations. Explain results of the test.

Answer: Deficiency of diastase in patients’ urine indicates acute renal failure.

161. A patient was transported to the hospital. Examination revealed the


next indexes: concentration of blood glucose – 10 mmol/l, concentration of
glucose in urine – 5.5%, concentration of ketone bodies in blood serum –
increased. What is the most likely diagnosis?

Answer: The above symptoms indicate hyperglycemic coma.

162. A female infant appeared normal at birth but developed signs of liver
disease and muscular weakness at 3 months. She had periods of
hypoglycemia, particularly on awakening. Examination revealed an enlarged
liver. Laboratory analyses following fasting revealed ketoacidosis, blood pH
7.25, and elevations in both alanine transaminase (ALT) and aspartate
transaminase (AST). Administration of glucagon following a carbohydrate
meal elicited a normal rise in blood glucose, but glucose levels did not rise
when glucagon was administered following an overnight fast. Liver biopsy
revealed an increase in the glycogen content (6 percent of wet weight).
Deficiency of what enzyme is a most likely for this patient? Explain your
answer.

Answer: Definitive diagnosis would await analysis of the glycogen structure and
enzyme activities, but the hepatomegaly, increased liver glycogen content, fasting
hypoglycemia, and muscle weakness are consistent with Cori disease, glycogen
storage disease type III. The increase in glycogen content results from an inability
to degrade glycogen beyond the limit dextrin of phosphorylase. A deficiency in the
debranching enzyme leaves glycogen with short outer branches.

163. Fill in the blanks:


Answer:

164. The glucose-alanine cycle is used primarily as a mechanism for skeletal


muscle to eliminate nitrogen while replenishing its energy supply.
a. Put requirements and production of ATP into their place in glucose-
alanine cycle presented in the picture below.
b. Explain biological significance of glucose-alanine cycle in humans.
Answer:

a.

b. Glucose oxidation produces pyruvate which can undergo transamination to


alanine. This reaction is catalyzed by alanine transaminase, ALT (ALT used to be
called serum glutamate-pyruvate transaminase, SGPT). Additionally, during
periods of fasting, skeletal muscle protein is degraded for the energy value of the
amino acid carbons and alanine is a major amino acid in protein. The alanine then
enters the blood stream and is transported to the liver. Within the liver alanine is
converted back to pyruvate which is then a source of carbon atoms for
gluconeogenesis. The newly formed glucose can then enter the blood for delivery
back to the muscle. The amino group transported from the muscle to the liver in
the form of alanine is converted to urea in the urea cycle and excreted.

165. Indicate enzymes catalyzing the following reactions of HMP:

Answer: 1 – glucose-6-phosphate dehydrogenase, 2 – lactonase, 3 – 6-


phosphogluconate dehydrogenase
SECTION ІV
STRUCTURE AND METABOLISM OF LIPIDS

1. Essential fatty acids cant by synthesized because mammals do not


possess the enzymes for their biosynthesis. Which of the following is an
essential fatty acid:
A. Linoleic acid D. Steraric
B. Palmitic acid E. Butyric
C. Oleic acid

2. Phosphoglycerides are complex lipids, represented by:


A. Phosphatidylcholine D. Thromboxane A
B. Sphingomyelin E. Ganglioside GM-2
C. Galactocerebroside

3. Sphingomyelin contains residues of the following substance:


A. Sphingosine D. Inositol
B. Glycerol E. Sulfuric acid
C. Galactose

4. Glycolipids contain the following constituent:


A. Glycerol D. Phosphoric acid
B. Sulfuric acid E. Choline
C. Sialic acid

5. Cholesterol is a starting material for biosynthesis of:


A. Bile acids D. Prostaglandins
B. N-acetyl-galactosamine E. Sphingomyelin
C. Lecithin

6. Gangliosides have the following physiological significance:


A. Provide hydrophilic and D. Catalyze the cleavage of
negative charge to the cell proteins in cell cytoplasm
surface. E. Are involved in activation of
B. Are used as energetic material amino acids.
C. Activate transcription
processes in cell nucleus

7. The following constituent is incorporated into gangliosides:


A. Ceramide D. Choline
B. Serine. E. Phosphoric acid
C. Glycerol

8. Phosphatidylinositol has the following functional significance:


A. Produces inositol phosphates as D. Regulates folding of DNA in
secondary messengers of hormones nuclear chromatin
B. Serves as an energetic material E. Induce phosphorylation of
C. Is a constituent of inner mitochondrial glucose
membrane

9. Ceramide is:
A. A derivative of sphingosine and D. Involving phosphate residue
fatty acid as a constituent
B. A derivative of glycerol and fatty E. A compound with macroergic
acid bond
C. Composed of fatty acid and
several monosaccharide residues

10. Fatty acids differ from one another due to differences in:
A. All of the above C. The degree of saturation
B. The number of carbon atoms D. The number of double bond

11. Arachidonic acid as essential nutrient is needed for normal growth and
development of animal and man. It is a precursor of biologically active
substances. Indicate what compound is synthesized from arachidonic acid:
A. Prostaglandine E1 D. Choline
B. Ethanolamine E. Noradrenalinе
C. Triiodothyronine

12. Triacylglycerols:
A. In the average individual, as would an equivalent weight of
represent sufficient energy to sustain glycogen.
life for several weeks. D. Are stored as hydrated
B. Would be expected to be molecules.
good emulsifying agents. E. Are generally negatively
C. Yield about the same charged molecules at
amount of ATP on complete oxidation physiological pH.

13. All of the following statements describe phosphoglycerides EXCEPT:


A. They are a major store of metabolic D. They are found in cell
energy membranes
B. They are both amphipathic and E. They contain two fatty acid
amphoteric moieties
C. They arise from glycerol-3-phosphate

14. Chose the lipid related compound which is primarily used as a source of
metabolic energy:
A. Ketone bodies D. Sphingomyelin
B. Phosphoglycerides E. Triglycerides
C. Steroids
15. Chose from listed below a hormone, which stimulates the formation of
glycogen and triacylglycerols:
A. Insulin D. Epinephrine
B. Norepinephrine E. Thyroxine
C. Glucagons

16. What bioactive compound (tissue hormone) is derived from 5-


hydroperoxy eicosatetraenoic acid (5-HPETE) and modulates a variety of
biologic functions?
A. Leukotriene D. Glucocorticoid
B. Prostaglandin E2 E. Sphingomyelin
C. Thromboxane

17. What compound of lipid nature may contain a carbohydrate moiety and
is present in most cell membranes?
A. Ganglioside GM2 D. Cholesterol
B. Sphingomyelin E. Leukotrienes
C. Phosphatydylserine

18. Lipids are the most valuable energetic material for an organism. What is
the main pathway of fatty acids metabolism in cell mitochondria?
A. –oxidation D. -oxidation
B. Decarboxylation E. -oxidation
C. Reduction

19. In digestion of dietary lipids there is a need of one of the digestive


secretions. What secretion listed below takes part in lipids emulsification?
A. Bile D. Saliva
B. Intestinal juice E. Gastric juice
C. Pancreatic juice

20. Phospholipids are important cell membrane components because:


A. They have both polar and non D. They combine covalently
polar portions with proteins
B. They have glycerol E. They consist of fatty acids
C. They can form bilayers in water

21. Dietary fats after absorption appear in the circulation as:


A. Chylomicron D. LDL
B. HDL E. Free fatty acids
C. VLDL

22. Which one of the following is not a phospholipids:


A. Lysolecithin C. Plasmalogen
B. Lecithin D. Phosphatidylinositol
E. Phosphatydylethanolamine

23. Lipids have the following properties:


A. All of these D. Structural component of
B. Insoluble in water and soluble cell membrane
in fat solvent E. Precursors in biosynthesis
C. High energy content of prostaglandins

24. Indicate which from listed below disorders of lipid metabolism occur in
fat tissue:
A. Obesity D. Retention hyperlipemia
B. Steatorrhea E. Fatty infiltration of liver
C. Ketosis
25. Hepatic liponenesis is stimulated by:
A. Insulin D. Epinephrine
B. cAMP E. Cortisol
C. Glucagon

26. Pancreatic lipase converts triacylglycerols into:


A. 2-Monoacylglycerol D. 3-Monoacylglycerol
B. 2, 3-Diacylglycerol E. 1, 3-Diacylglycerol
C. 1-Monoacylglycerol

27. Lipids are stored in the body mainly in the form of:
A. Triglycerides D. Fatty acids
B. Glycolipids E. Steroids
C. Phospholipids

28. Pancreatic lipase requires for its activity:


A. Co-lipase D. Amino acids
B. Bile salts E. All of these
C. Phospholipids

29. Waxes contain higher alcohols named as:


A. Cetyl D. Phytyl
B. Methyl E. Propionyl
C. Ethyl

30. β -Galactosidase is deficient in:


A. Krabbe’s disease D. Metachromatic
B. Fabry’s disease leukodystrophy
C. Gaucher’s disease E. Tay-sach’s disease

31. Bile acids are necessary for fat digestion. They are produced in the liver
from the next precursor:
A. Cholesterol B. Protoporphyrine IX
C. Corticosterol E. Arachidonic acid
D. Lecithin

32. Adipose tissue lacks:


A. Glycerol kinase D. Glycerol-3-phosphate
B. Hormone-sensitive lipase dehydrogenase
C. cAMP-dependent protein kinase

33. Which one of the following enzymes is NOT involved in the degradation
of dietary lipids during digestion?
A. Lipoprotein lipase; D. Phospholipase A2;
B. Pancreatic lipase; E. Cholesterol ester hydrolase
C. Gastric lipase;

34. Free fatty acids are transported in the blood:


A. Combined with albumin D. In unbound free salts
B. Combined with fatty acid E. Combined with globulin
binding protein
C. Combined with β -lipoprotein

35. Which one of the following substances is an intermediate in the


synthesis of both glycerol-containing phospholipids and triacylglycerol?
A. Phosphatidic acid D. CDP-Ethanolamine
B. Choline E. 3-Hydroxyburyrate
C. Acetoacetyl CoA;

36. The nitrogenous base in lecithin is:


A. Choline D. Betaine
B. Ethanolamine E. Alanine
C. Serine

37. The next substance is known as bile acid:


A. Taurocholate D. Mevalonate
B. Succinate E. Phosphatidic acid
C. Lignoceric acid

38. Lipid stores are mainly present in:


A. Adipose tissue D. Muscles
B. Liver E. Kidneys
C. Brain

39. Co-lipase is a:
A. Protein D. Phospholipid
B. Vitamin E. Amino acid
C. Bile salt
40. Number of carbon atoms in cholesterol is:
A. 27 D. 30
B. 17 E. 35
C. 19

41. Hexosaminidase A is deficient in:


A. Tay-Sachs disease D. Fabry’s disease
B. Gaucher’s disease
C. Niemann-Pick disease

42. The form in which most dietary lipids are packaged and exported from
the intestinal mucosa cells is as follows:
A. Chylomicrons D.2-monoacylglycerol
B. Mixed micelles E. Free fatty acids
C. Free triacylglycerol

43. Characteristic finding in Gaucher’s disease is:


A. Hepatosplenomegaly D. Deafness
B. Night blindness E. Nephropathy
C. Renal failure

44. The intermediates in fatty acid synthesis are linked to acyl carrier protein
(ACP), a component of fatty acid synthase. The prosthetic group of ACP is:
A.Phosphopantetheine D.Biotin
B.Methionine E.Cobalamin
C.Thiamine

45. Chylomicron remnants are catabolised in:


A. Liver D. Liver and intestine
B. Adipose tissue E. Kidneys
C. Intestine

46. Adipose tissue responds to low insulin/glucagon ratio by:


A. Stimulating hormone-sensitive D. Increasing the amount of
lipase. pyruvate kinase.
B. Dephosphorylating the E. Stimulating phenylalanine
interconvertible enzymes. hydroxylase
C. Stimulating the deposition of fat.

47. In adipose tissue, glycerol-3-phosphate required for the synthesis of


triglycerides comes mainly from:
A. Dihydroxyacetone phosphate C. Hydrolysis of phospholipids
formed in glycolysis D. Free glycerol
B. Hydrolysis of pre-existing E. Cholesterol
triglycerides
48. Glycerol released from adipose tissue by hydrolysis of triglycerides is
mainly:
A. Taken up by liver C. Reutilised in adipose tissue
B. Taken up by extrahepatic tissues D. Excreted from the body

49. Mental retardation occurs in:


A. All of these C. Gaucher’s disease
B. Tay-Sachs disease D. Niemann-Pick disease

50. Free glycerol cannot be used for triglyceride synthesis in:


A. Adipose tissue C. Kidney
B. Liver D. Intestine

51. The rate-limiting step of fatty acid synthesis is catalyzed by:


A. Acetyl-CoA carboxylase D. Malic enzyme
B. ATP-citrate lyase E. Malonyl transacylase
C. Acyl-CoA synthetase

52. Long chain fatty acids are first activated to acetyl-CoA in:
A. Cytosol D. Mitochondria
B. Microsomes E. Ribosome
C. Nucleus

53. β-Oxidation of fatty acids:


A. Occurs by a repeated sequence of C. Is controlled primarily by
four reactions allosteric effectors.
B. Generates ATP in reactions of D. Uses only even-chain, saturated
substrate level phosphorylatin. fatty acids as substrates.
E. Uses NADP.

54. One functional sub-unit of multi-enzyme complex for de novo synthesis


of fatty acids contains:
A. Two —SH groups D. Four —SH groups
B. One —SH group E. Five —SH groups
C. Three —SH groups

55. The enzyme acyl-CoA synthase catalyses the conversion of a fatty acid
of an active fatty acid in the presence of:
A. ATP D. GTP
B. ADP E. UDP
C. AMP

56. The high glucagon/insulin ratio seen in starvation:


A. All of the above
B. Stimulates -oxidation by D. . Promotes
inhibiting the production of malonyl mobilization of fatty acids from
CoA. adipose stores.
C. Leads to increased E. None of the above.
concentrations of ketone bodies in
the blood.

57. Long chain fatty acids penetrate the inner mitochondrial membrane:
A. As carnitine derivative D. Freely
B. As acyl-CoA derivative E. Requiring Ca dependent carrier
C. Requiring Na dependent carrier

58. De novo synthesis of fatty acids occurs in:


A. Cytosol D. All of these
B. Mitochondria E. None of these
C. Microsomes

59. Acyl Carrier Protein contains the vitamin:


A. Pantothenic acid D. Folic acid
B. Lipoic acid E. Ascorbic acid
C. Biotin

60. Which of the following is required as a reductant in fatty acid synthesis?


A. NADPH D. FMNH2
B. NADH E. FAD
C. FADH2

61. An enzyme required for the synthesis of ketone bodies as well as


cholesterol is:
A. HMG CoA synthetase D. HMG CoA lyase
B. Acetyl CoA carboxylase E. HMG CoA dehydrogenase
C. HMG CoA reductase

62. Ketone bodies are synthesized in:


A. Liver D. Brain
B. Adipose tissue E. Kidney
C. Muscles

63. Carnitine is required for the transport of:


A. Long chain fatty acids into C. Short chain fatty acids into
mitochondria mitochondria
B. Triglycerides into mitochondria D. Triglycerides out of liver
E. Glycerol into cytosol
64. What process of lipid metabolism requires a protein with covalently
bound prosthetic group derived from pantothenic acid?
A. Fatty acid biosynthesis D. Bile acid synthesis from
B. β-Oxidation of fatty acids cholesterol
C. Utilization of ketone bodies E. Utilization of LDL

65. Net ATP generation on complete oxidation of stearic acid is:


A. 146 D. 148
B. 131 E. 159
C. 129

66. Refsum’s disease results from a defect in the following pathway


EXCEPT:
A. Beta-oxidation of fatty acids D. Omega-oxidation of fatty acids
B. Alpha-oxidation of fatty acids E. Delta-oxidation of fatty acids
C. Gamma-oxidation of fatty acids

67. The removal of two- carbon units from a fatty acyl coenzyme A (fatty acyl
CoA) involves four sequential reactions. Which of the following best
describes the reaction sequence?
A. Dehydrogenation, hydratation, D. Hydrogenation, dehydration,
dehydrogenation, cleavage hydrogenation, cleavage
B. Oxidation, dehydration, oxidation, E. Reduction, hydration,
cleavage dehydrogenation, cleavage
C. Reduction, dehydration, reduction,
cleavage

68. The enzymes of β-oxidation are found in:


A. Mitochondria. D. Nucleus.
B. Cytosol. E. Microsomes.
C. Golgi apparatus.

69. During each cycle of β-oxidation:


A. Two carbon atoms are removed from D. Two carbon atoms are removed
the carboxyl end of the fatty acid from the methyl end of the fatty acid
B. One carbon atom is removed from Е. Three carbon atom is removed
the methyl end of the fatty acid from the methyl end of the fatty acid
C. One carbon atom is removed from
the carboxyl end of the fatty acid

70. Obesity generally reflects excess intake of energy and is often


associated with the development of:
A. Non-insulin dependent diabetes C. Hepatitis
mellitus D. Colon cancer
B. Nervousness
71. Net generation of energy on complete oxidation of palmitic acid is:
A. 129 ATP equivalents D. 148 ATP equivalents
B. 131 ATP equivalents E. 159 ATP equivalents
C. 146 ATP equivalents

72. NADPH required for fatty acid synthesis can come from:
A. Hexose monophosphate shunt D. Citric acid cycle
B. Oxidative decarboxylation of malate E. All of these
C. Extramitochondrial oxidation of
isocitrate

73. Citrate stimulates fatty acid synthesis by all of the following EXCEPT:
A. Participating in the production of C. Providing a mechanism to
ATP. transport acetyl CoA from the
B. Allosterically activating acetyl-CoA mitochondria to the cytosol.
carboxylase. D. Participating in a pathway that
ultimately produces CO2 and
NADPH in the cytosol.

74. What process of lipid metabolism requires a protein with covalently


bound prosthetic group derived from pantothenic acid?
A. Fatty acid biosynthesis D. Bile acid synthesis from
B. β-Oxidation of fatty acids cholesterol
C. Utilization of ketone bodies E. Utilization of LDL

75. De novo synthesis and oxidation of fatty acids differ in the following
respect:
A. Synthesis occurs in cytosol and C. NADH is required in synthesis
oxidation in mitochondria and FAD in oxidation
B. Synthesis is decreased and oxidation D. Malonyl CoA is formed during
increased by insulin oxidation but not during synthesis

76. De novo synthesis of fatty acids requires all of the following except:
A. NADPH D. ATP
B. NADH E. Biotin
C. Panthothenic acid

77. Carnitine is synthesized from:


A. Lysine and methionine D. Proline and hydroxyproline
B. Glycine and arginine E. Lysine and arginine
C. Aspartate and glutamate

78. Acetyl CoA carboxylase regulates fatty acid synthesis by which of the
following mechanism?
A. All of these C. Induction and repression
B. Covalent modification D. Allosteric regulation
E. None of these

79. β-Oxidation of fatty acids requires all the following coenzymes except:
A. NADP C. NAD
B. FAD D. CoA

80. Carboxylation of acetyl-CoA to malonyl-CoA takes place in the presence


of:
A. Biotin D. NADP+
B. FAD E. GTP
+
C. NAD

81. Malonyl-CoA reacts with the central:


A. -SH group D. -CH2OH group
B. -NH2 group E. –OH group
C. -COOH group

82. Propionyl CoA is formed on oxidation of:


A. Fatty acids with odd number of D. Fatty acids with even number of
carbon atoms carbon atoms
B. Polyunsaturated fatty acids E. None of these
C. Monounsaturated fatty acids

83. Which of the following can be used as a source of energy in extrahepatic


tissues?
A. Acetoacetate C. Both (A) and (B)
B. Acetone D. None of these

84. During the prolonged starvation in blood of a person an increase in


ketone bodies content occurs. It is caused by the next factors:
A. Production of acetyl-CoA. D. Enhancment of fatty acids
B. Decrease of free fatty acid level in biosynthesis in liver
blood plasma E. Decrease of triacylglycerols in
C. Mobilization of high density adipose tissue
lipoproteins

85. What process cannot occur in the absence of carnitine?


A. β-Oxidation of fatty acids D. Fatty acid biosynthesis
B. Cleavage of triacylglycerol E. Biosynthesis of prostaglandin
C. Utilization of ketone bodies

86. Propionyl CoA formed from the oxidation of fatty acids having an odd
number of carbon atoms is converted into:
A. D-Methylmalonyl CoA D. Butyryl CoA
B. Acetoacetyl CoA E. Acety lCoA
C. Acetyl CoA
87. What process of lipid metabolism requires a protein with covalently
bound prosthetic group derived from pantothenic acid?
A. Fatty acid biosynthesis D. Bile acid synthesis from
B. β-Oxidation of fatty acids cholesterol
C. Utilization of ketone bodies E. Utilization of LDL

88. The rate limiting step cholesterol biosynthesis is:


A. HMG CoA reductase D. Mevalonate kinase
B. Squalene synthetase E. Geranyl transferase
C. HMG CoA synthetase

89. An experimental animal has been given excessive amount of carbon-


labeled glucose for a week. What compound can the label be found in?
A. Palmitic acid D. Choline
B. Methionine E. Arachidonic acid
C. Vitamin A

90. The two final productsi in the β-oxidation of odd chain fatty acids are:
A. Acetyl CoA and propionyl-CoA D. Acetyl CoA and succinyl CoA.
B. Acetyl CoA and acetyl CoA E. Acetyl-CoA and buturyl-CoA
C. Acetyl CoA and malonyl CoA

91. In adipose tissue, glycerol-3-phosphate required for the synthesis of


triglycerides comes mainly from:
A. Dihydroxyacetone phosphate formed C. Free glycerol
in Glycolysis D. Hydrolysis of phospholipids
B. Hydrolysis of pre-existing
triglycerides

92. The enzymes of β-oxidation are found in:


A. Mitochondria. D. Nucleus.
B. Cytosol. E. Microsomes.
C. Golgi apparatus.

93. Cholesterol is a starting material for biosynthesis of:


A. Cholic acid D. Prostaglandins
B. N-acetyl-galactosamine E. Sphingomyeli
C. Lecithin

94. Indicate lipid compound which is predominant in low density


lipoproteins
A. Cholesterol D. Sphingomyelin
B. Phospholipids E. Glucocerebroside
C. Triglycerides
95. Lysolecithin is formed from lecithin by removal of:
A. Fatty acid from position 2 D. Choline
B. Fatty acid from position 1 E. Glycerol
C. Phosphorylcholine

96. Cholesterol is a starting material for biosynthesis of:


A. Cholic acid D. Prostaglandins
B. N-acetyl-galactosamine E. Sphingomyelin
C. Lecithin

97. The pathway of cholesterol synthesis begins from the next metabolite:
A. 2-Oxoglutaryl-CoA D. Acetyl-CoA
B. Succinyl-CoA E. Lauryl-CoA
C. Acetoacetyl-CoA

98. Lipo-oxygenase is required for the synthesis of:


A. Leukotrienes D. Prostacyclines
B. Prostaglandins E. All of these
C. Thromboxanes

99. Chose from listed below compounds the final product of cholesterol
metabolism in human body:
A. Chenodeoxycholic acid D. Cholecalciferol
B. Cortisol E. Ergrosterol
C. Prostaglandine E2

100. Triglycerides are transported from liver to extrahepatic tissues by:


A. VLDL D. LDL
B. Chylomicrons E. IDL
C. HDL

101. Lipoproteins contain the following constituent:


A. Lecithin D. Blood plasma proteins
B. Ca ions E. Bile acids
C. Albumin

102. A compound normally used to conjugate bile acids is:


A. Glycine C. Glucoronic acid
B. Serine D. Fatty acid

103. The rate limiting step in cholesterol biosynthesis is:


A. HMG CoA reductase D. Squalene synthetase
B. Mevalonate kinase E. Geranyl transferase
C. HMG CoA synthetase
104. Genetic deficiency of lipoprotein lipase causes hyperlipoproteinaemia
of the following type:
A. Type I D. Type V
B. Type IIa E. Type III
C. Type IIb

105. This lipoprotein removes cholesterol from the body:


A. HDL D. Chylomicrons
B. VLDL E. LDL
C. IDL

106. Indicate lipid compound which is predominant in low density


lipoproteins:
A. Cholesterol D. Sphingomyelin
B. Triglycerides E. Glucocerebroside
C. Phospholipids

107. Chylomicrons are present in fasting blood samples in


hyperlipoproteinaemia of the following types:
A. Types I and V D. Types IV and V
B. Types IIa and IIb E. Types V and VI
C. Types I and IIa

108. Increase in blood of this class of lipoproteins is beneficial to ward off


coronary heart disease:
A. LDL D. IDL
B. HDL E. Chylomicrones
C. VLDL

109. Note an intermediate metabolite which is on the cholesterol synthesis


pathway and is of multifunctional significance:
A. β-Hydroxy β-methyl glutaryl-CoA C. Acetoacetate
(HMG-CoA) D. Palmitoyl-CoA
B. Succinyl-CoA E. 2-Oxoglutaryl-CoA

110. Esterification of cholesterol occurs mainly in:


A. Adipose tissue D. Kidneys
B. Liver E. Brain
C. Muscles

111. The next compounds are intermediates in cholesterol synthesis


pathway EXCEPT:
A. 2-Oxoglutaryl-CoA D. Lanosterol
B. Mevalonic acid E. Squalene
C. Hydroxymethylglutaryl- CoA
112. Bile acids are derivatives of:
A. Cholesterol D. Phosphatidyl choline
B. Heme E. Long chain fatty acids
C. Sphingomyeline

113. The β-lipoprotein fraction increases in severe:


A. Diabetes Mellitus D. Muscular dystrophy
B. Uremia E. Pancreatatis
C. Nephritis

114. Cholesterol is the precursor for the biosynthesis of:


A. Bile acids D. Sphingmyelin
B. Prostaglandins E. Lecitine
C. Fatty acid

115. HDL is synthesized in:


A. Liver D. Liver and intestine
B. Kidneys E. Adipose tissue
C. Intestine

116. All the following correctly describe the intermediate 3-OH-3-methyl


glutaryl CoA except:
A. It is generated enzymatically in the D. It inhibits the first step in
mitochondrial matrix cholesterol synthesis
B. It is formed in the cytoplasm E. It is involved in the synthesis of
C. It is involved in the synthesis of ketone bodies
cholesterlol

117. Very low density lipoproteins are also known as:


A. Pre β--lipoproteins C. α-lipoproteins
B. β-lipoproteins D. None of these

118. Main metabolic end product of cholesterol:


A. Bile acid D. Glycine
B. 5-pregnenolone E. Palmytoyl-CoA
C. Coprosterol

119. A man 67 years old suffers from brain vessels atherosclerosis. After
investigation hyperlipidemia was detected. What class of lipoproteins in
blood plasma will be increased most of all in biochemical investigation?
A. LDL D. Chilomicrons
B. HDL E. VLDL
C. Non esterified fatty acids in complex
with albumin
120. Chylomicron, intermediate density lipoproteins (IDL), low density
lipoproteins (LDL) and very low density lipoproteins (VLDL) all are serum
lipoproteins. What is the correct ordering of these particles from the lowest
to the greatest density?
A. Chylomicron, VLDL, IDL, LDL D. Chylomicron, IDL, VLDL, LDL
B. LDL, IDL, VLDL, Chylomicron E. IDL, VLDL, LDL, Chylomicron
C. VLDL, IDL, LDL, Chylomicron

121. In a patient suffering from diabetes mellitus an increase in


concentration of VLDL and triacylglycerols was detected. Cholesterol and
HDL content in normal values.What type of lipid metabolism disorder can be
classified such changes of indicated data?
A. Hyperlipoproteinemia type IY D. Hypelipoproteinemia type III
B. Hyperlipoproteinemia type II E. Hyperlipoproteinemia type II b
C. Hyperlipoproteinemia type Y

122. Lipoprotein lipase:


A. Is stimulated by one of the C. Functions to mobilize stored
apoproteins present in VLDL. triacylglycerols from adipose tissue.
B. Is stimulated by cAMP-mediated D. Is an intracellular enzyme.
phosphorylation. E. Readily hydrolyzes three fatty
acids from a triacylglycerol.

EXAMPLES OF KROK 1 TESTS

123. In patients suffering from diabetes mellitus an increase in a content of


non esterified fatty acids (NEFA) in blood is observed. It may be caused by:
A. Increase in activity of D. Decrease in activity of
triacylglycerol lipase phosphatidylcholine-cholesterol-
B. Stimulation of ketone bodies acyltransferase in blood plasma
utilization E. Accumulation in cytosol of
C. Activation of synthesis of palmitoyl-CoA
apolipoproteins A1 , A2, A3

124. The essence of lipolysis, that is the mobilization of fatty acids from
neutral fats depots, is an enzymatic process of hydrolysis of triacylglycerols
to fatty acids and glycerol. Fatty acids that release during this process enter
blood circulation and are transported as the components of:
A. Serum albumins.. D. LDL
B. Globulins. E. Chylomicrons.
C. HDL.

125. Which one of the following statements about the absorption of lipids
from the intestine is correct?
A. Dietary triacylglycerol is partially D. Fatty acids that contain ten carbons o
hydrolyzed and absorbed as free fatty less are absorbed and enter the circulatio
acids and monoacyl glycerol primarily via the lymphatic system
B Release of fatty acids from E. Formation of chylomicrons does
triacylglycerol in the intestine is inhibited not require protein synthesis in the
by bile salts intestinal mucosa.
C. Dietary triacylglycerol must be
completely hydrotyzed to tree fatty acids
and glycerol before absorption

126. After consumption of lipids in the body than begins their digestion and
absorption in intestines. What products of lipid hydrolysis are absorbed in
the intestine?
A. Monoacylglycerol, fatty acids D. Monosacharides
B. Amino acids E. Lipoproteins
C. Polypeptides

127. After the consumption of animal food rich in fats, a patient feels
discomfort, and droplets of fats are found during laboratory investigation of
his feces. Bile acids are revealed in the urine. The cause of such state is the
deficiency of ___ in the digestive tract.
A. Bile acids. D. Triacylglycerols.
B. Fatty acids. E. Phospholipids.
C. Chylomicrons.

128. Fabry’s disease (one of sphingolipidoses) is an autosomal recessive


disease. Major symptoms of this disease: skin rash, kidney failure, pain in
lower extremities. It is caused by a deficiency of:
A. α-Galactosidase A D. Galactocerebrosidase
B. Hexosaminidase A and B E. Ceraminase
C. Gm1 Gangliosidase

129. A man 67 years old suffers from brain vessels atherosclerosis. After
investigation hyperlipidemia was detected. What class of lipoproteins in
blood plasma will be increased the most of all in biochemical investigation?
A. LDL D. Chylomicrons
B. HDL E. VLDL
C. Non esterified fatty acids in complex
with albumin

130. A 35-year-old man with pheochromocytoma has high levels of


epinephrine and norepinephrine registered in the blood. The concentration
of free fatty acids is increased by a factor of eleven. Which of the following
enzymes accelerates the lipolysis under the action of epinephrine?
A. Triacylglycerol lipase. C. Phospholi pasc Ar
B. Lipoprotein lipase. D. Phospholi pase C.
E. Cholesterol esterase.

131. Phosphoinositol is an important compound in cell regulatory signaling


system and is produced from phosphatidylinositol under the action of the
next enzyme:
A. Phospholipase C D. Phospholipase B
B. Neuraminidase E. Sphingomyelinase
C. Phospholipase A

132. In a course of laboratory investigation of blood, taken in fasting


conditions, it was detected the turbidity of serum, total lipids content – 20
g/l, cholesterol – 9 mM/l. After centrifugation on a surface of serum there
appears a white film, which suggests an increase in the amount of
chylomicrones in blood. What enzyme activity decrease may cause this
situation?
A. Lipoproteine lipase of blood D. Lipases of fat tissue
B. Pancreatic phospholipases E. Pancreatic lipase
C. Lecitine cholesterol acyl transferase

133. Arachidonic acid as essential nutrient is needed for normal growth and
development of animal and man. It is precursor of biologically active
substances. Indicate what compound is synthesized from arachidonic acid
A. Prostaglandine E1 D. Triiodothyronine
B. Noradrenalin E. Choline
C. Ethanolamine

134. In a human body the adipose tissue is the basic location of


triacylglycerols (TAG) deposit. At the same time their synthesis takes place
in hepatocytes. In the form of what molecular complex are TAG transported
from the liver into the adipose tissue?
A. VLDL. D. HDL.
B. Chylomicrons. E. Complexes with albumin.
C. LDL.

135. After the consumption of a diet rich in fats, a patient complains of


languor and nausea. Later signs of steatorrea appear. The level of blood
cholesterol makes 9.2 mM/1. The shortage of what substances causes this
state of the patient?
A. Bile acids. D. Phospholipids.
B. Triacylglycerols. E. Chylomicrons.
C. Fatty acids.

136. The insufficient secretion of what enzyme is the cause of incomplete


fats degradation in the digestive tract and appearance of great quantity of
neutral fats in feces?
A. Pancreatic lipase. B. Phospholi pase.
C. Entcrokinase. E. Pepsin.
D. Amylase.

137. Lipids are obvious energetic material for the body. What is the main
pathway of fatty acids metabolism in mitochondria?
A. β-Oxidation D. α -Oxidation
B. Decarboxylation E. γ-Oxidation
C. Reduction

138. In a patient suffering from diabetes mellitus in blood was detected


acetone. Note the process of its production in the body.
A. By condensation of two molecules C. In course of β-oxidation of
of acetyl-CoA fatty acids
B. In course of α-oxidation of fatty D. In course of γ-oxidation of
acids fatty acids
E. In tricarboxylic acid cycle.

139. Aerobic oxidation of substrates is typical of a cardiac muscle. Which


of the following is the major oxidation substrate of a cardiac muscle?
A. Fatty acids. D. Glucose.
B. Triacylglycerols. E. Amino acids.
C. Glycerol.

140. Carnitine is recommended to a sportsman as a preparation that


increases physical activity and improves achievements. What biochemical
process is mostly activated under the action of carnitine?
A. Transport of fatty acids into C. Lipids synthesis.
mitochondria. D. Tissue respiration.
B. Ketone bodies synthesis. E. Steroid hormones synthesis.

141. In diabetes mellitus and starvation there is an increase of ketone bodies


content in blood, which are utilized as energetic material by tissues. Note the
substance which is used in ketone bodies synthesis.
A. Acetyl-CoA D. α–Ketoglutarate
B. Citrate E. Malate
C. Succinyl-CoA

142. A 1-year-old child was brought to a clinic with signs of muscle


weakness. Through the inspection, the deficiency of carnitine in the
muscles was determined. The biochemical mechanism of the development
of this pathology consists in the disorder of the process of:
A. Transport of fatty acids into C. Substrate level of
mitochondria. phosphorylation.
2+
B. Regulation of the level of Ca in D. Utilization of lactate.
mitochondria. E. Synthesis of actin and myosin.
143. A patient with high rate of obesity was advised to use carnitine as a
food additive in order to enhance "fat burning". What is the role of carnitine
in the process of fat oxidation?
A. Transport of FFA (free fatty acids) C. It takes part in one of reactions
from cytosol to the mitochondria of FFA beta-oxidation
B. Transport of FFA from fat depots to D. FFA activation
the tissues E. Activation of intracellular lipolysis

144. Lipids are obvious energetic material for the body. What is the main
pathway of fatty acids metabolism in mitochondria?
A. β-Oxidation D. Reduction
B. Decarboxylation E. γ-Oxidation
C. α -Oxidation

145 A 1-year-old child was brought to a clinic with signs of muscle


weakness. Through the inspection, the deficiency of carnitine in the
muscles was determined. The biochemical mechanism of the development
of this pathology consists in the disorder of the process of:
A. Transport of fatty acids into C. Substrate level of
mitochondria. phosphorylation.
2+
B. Regulation of the level of Ca in D. Utilization of lactate.
mitochondria. E. Synthesis of actin and myosin.

146. Carnitine is recommended to a sportsman as a preparation that


increases physical activity and improves achievements. What biochemical
process is mostly activated under the action of carnitine?
A. Transport of fatty acids into C. Lipids synthesis.
mitochondria. D. Tissue respiration.
B. Ketone bodies synthesis. E. Steroid hormones synthesis.

147. Patients who suffer from severe diabetes and don't receive insulin have
metabolic acidosis. This is caused by increased concentration of the
following metabolites:
A. Ketone bodies D. Triacylglycerols
B. Fatty acids E. Cholesterol
C. Unsaturated fatty acids

148. In diabetes mellitus and starvation there is an increase of ketone


bodies content in blood, which are utilized as energetic material by tissues.
Note the substance which is used in ketone bodies synthesis.
A. Acetyl-CoA D. α–Ketoglutarate
B. Citrate E. Malate
C. Succinyl-CoA

149. Aerobic oxidation of substrates is typical of a cardiac muscle. Which


of the following is the major oxidation substrate of a cardiac muscle?
A. Fatty acids D. Glucose
B. Triacylglycerols. E. Amino acids.
C. Glycerol.

150. A patient suffers from arterial hypertension due to atherosclerotic


injury of blood vessels. The consumption of what dietary lipid needs to be
limited?
A. Cholesterol. C. Lecithine.
B. Oleic acid. D. Monooleateglycerol.
E. Phosphatidylserine.

151. Fats of phospholipids is disordered due to fat infiltration of the liver.


Indicate which of the presented substances can enhance the process of
methylation during phospholipids synthesis?
A. Methionine D. Glycerin
B . Ascorbic acid E. Citrate
C. Glucose

152. In a patient after investigation it was detected an increased content of


low density lipoproteins in blood serum. What disease can be expected in
this patient?
A. Atherosclerosis D. Acute pancreatitis
B. Pneumonia E. Kidney disease
C. Gastritis

153. A child 5 years old suffers from transient abdominal pains. Blood
serum is turbid in fasting conditions. Cholesterol content – 4,3 mmoles/l,
total lipids – 18 g/l. For precisement of diagnosis electrophoresis of blood
lipoproteins is administered. What classes of lipoproteins are expected to
be increased?
A. Chylomicrons D. LDL
B. HDL E. VLDL
C. IDL

154. In cases of complete or partial restriction of lipotropic factors in


humans develops a fat degeneration of liver. What substances can be
considered as lipotropic?
A. Choline D. Cholesterol
B. Pyridoxine E. Triacylglycerols
C. Fatty acids

155. A 44-year-old woman complains of common weakness, heart pain,


considerable increase of body weigt. Objectively: moon-like face, hirsutism,
AP- 165/100 mm Hg, height - 164 cm, weight - 103 kg; fat is mostly
accumulated in the region of neck, upper shoulder girdle, stomach. What is
the main pathogenetic mechanism of obesity?
A. Increased production of D. Decreased production of
glucocorticoids glucagon
B. Decreased production of thyroidal E. Increased production of
hormones mineralocorticoids
C. Increased production of insulin

156. The complaints and objective data permit to suppose an inflammatory


process in gall bladder, disorder of colloidal properties of bile, probability of
bile stones formation. What compound can cause their formation?
A. Cholesterol D. Phosphates
B. Oxalates E. Urates
C. Chlorides

157. In a worker of chemical cleaning the fatty liver dystrophy was


recognized. What substance biosynthesis disorder can lead to this
pathology?
A. Phosphatidyl choline D. Urea
B. Tristearylglycerol E. Folic acid
C. Phosphatidic acid

158. A patient with high blood cholesterol levels was treated with lovastatin.
This drug lowers blood cholesterol levels because it inhibits:
A. HMG CoA reductase in liver and C. Citrate lyase in liver
peripheral tissue D. VLDL excretion by the liver
B. Lipoprotein lipase in adipose tissue E. Absorption of dietary cholesterol

159. Laboratory investigation of the patient's blood plasma, which was


performed 4 hours after a consumption of a fat diet, displayed a marked
increase of plasma turbidity. The most credible cause of this phenomenon
is the increase of in the plasma.
A. Chylomicrons. D. Cholesterol.
B. HDL. E. Phospholi pids.
C. LDL.

159. Laboratory investigation of a patient revealed a high level of plasma


LDL, What disease can be diagnosed?
A. Atherosclerosis D. Pneumonia
B. Nephropathy. E. Gastritis.
C. Acute pancreatitis.

160. Arachidonic acid, an essential component of a human diet, acts as a


precursor of the vitally important physiologically active biomolecules.
Which substances are synthesized from arachidonic acid?
A. Prostaglandin E. D. Ethanolamine.
B. Choline. E. Triiodothyronine.
C. Noradrenaline.
161. In a worker of chemical cleaning the fatty liver dystrophy was
recognized. What substance biosynthesis disorder can lead to this
pathology?
A. Phosphatidyl choline D. Urea
B. Tristearylglycerol E. Folic acid
C. Phosphatidic acid

162. Clinical signs and laboratory testing of a patient allow making the
assumption of gall-bladder inflammation, colloid properties of bile disorder,
the occurrence of gall-stones. Which substances can underlie the formation
of gall-stones?
A. Oxalates. D. Chlorides.
B. Urates. E. Phosphates.
C. Cholesterol.

163. After laboratory investigation in a patient was detected an increased


content of low density lipoproteins in blood serum. What disease from listed
below can be expected in a patient?
A Atherosclerosis D. Acute pancreatitis
B. Pneumonia E. Kidney disease
C. Gastritis

164. In cases of complete or partial restriction of lipotropic factors


providement in humans develops a fet degeneration of liver. What
substances can be considered as lipotropic?
A. Choline D. Cholesterol
B. Fatty acids E. Pyridoxine
C. Triacylglycerols

165. An experimantal animal that was kept on protein-free diet developed


fatty liver infiltration, in particular as a result of deficiency of methylating
agents. This is caused by disturbed generation of the following metabolite:
A. Choline D. Acetoacetate
B. DOPA E. Linoleic acid
C. Cholesterol

166. A person with a low-density lipoprotein (LDL) receptor deficiency was


treated with lovastatin. As a consequence of the action of this drug, the
person should have:
A. Lower blood cholesterol levels D. Fewer LDL receptors in cell
B. Increased de novo cholesterol membranes
synthesis E. Higher blood triacylglycerol
C. Increased ACAT activity levels
167. A patient suffers from arterial hypertension due to atherosclerotic
injury of blood vessels. The consumption of what dietary lipid needs to be
limited?
A. Cholesterol. C. Phosphatidylserine.
B. Oleic acid. D. Monooleateglycerol.
E. Lecithine.

168. A 70 years old man is ill with vascular athrosclerosis of lower


extremities and coronary heart disease. Examination revealed disturbance
of lipidic blood composition. The main of factor of atherosclerosis
pathogenesis is the excess of the following lipoproteins:
A. LDL. D. IDL
B. VLDL. E. HDL
C. Cholesterol.

168. In metabolic transformations in human body appear active oxygen


intermediates, including hydrogen peroxide. This substance is reduced by
substrates as donor of hydrogen with the aid of the next enzyme:
A. Catalase D. Glutathion redsuctase
B. Cytochrome P-450 E. Superoxide dysmutase
C. Glutathion peroxidase

169. In a patient after the action of ionizing radiation an increased level of


malonic dialdehyde was detected in blood, indicating the activation of
peroxide oxidation of lipids. This may lead to injury of biological
membranes due to:
A. Degradation of phospholipids D. Breakdown of carbohydrate
B. Oxidation of cholesterol constituents
C. Changes in structure of transfer E. Activation of Na,K-ATPas
proteins

170. A teenage boy presents with moderate to severe epigastric pain.


Physical examination reveals extensive eruptive xanthomas and
hepatosplenomegaly. A blood sample reveals milky plasma. Which of the
following is the most likely lipoprotein to be elevated in this patient’s
plasma?
A. Chylomicrons D. IDL
B. Chylomicron remnants E. LDL
C. HDL

171. Laboratory results for a patient with uncontrolled Type I diabetes


mellitus reveal hyperglycemia (634 mg/dL) and hypertriglyceridemia (498
mg/dL). The most likely cause of the hypertriglyceridemia in this patient is
which of the following?
A. Decreased lipoprotein lipase B. Increased hepatic triglyceride
activity synthesis
C. Deficiency in apoprotein C-II E. Absence of hormone-sensitive
D. Deficiency in LDL receptors lipase

172. A 25-year-old female was referred to a lipid research center for


investigation of moderate hypertriglyceridemia because the plasma lipid
and lipoprotein profiles showed abnormalities. Both HDL and LDL were
more buoyant and showed elevations in TG content with the mass of TG
approximately the same as that of cholesterol. A deficiency in which of the
following is the most likely cause of this patient’s lipid abnormality?
A. Hepatic lipase D. Lecithin-cholesterol acyltransferase
B. Lipoprotein lipase (LCAT)
C. Apoprotein C-II E. Apoprotein B-100

CLINICAL CASES AND SITUATIONAL TASKS SITUATIONAL TASKS

173. A person limits himself to products that contain phospholipids for a


long period of time. Which metabolic disorders may this lead to?

Answer: When consumption of these products is limited, this may develop


disorders such as fatty liver, increased blood cholesterol, atherosclerosis,
disorders of the nervous system.

174. Analysis of the blood revealed a high content of cholesterol in -


lipoproteidnye faction. What are the causes and possible implications for
the body organism?

Answer: The reason for increase of -lipoproteins in blood may be a high-calorie


diet with a predominance of carbohydrates, animal fats. This increases the risk of
hypercholesterolemia, atherosclerosis, coronary heart disease, stroke and others

175. In person’s diet vegetable oil is absent. What may this lead to?

Answer: The consequence of the limited consumption of polyunsaturated fatty


acids may reduce the phospholipids, increased cholesterol levels and disturbance
of synthesis of prostaglandins and decreased activity of oxygen-reduction
processes.

176. The concentration of phospholipids in the blood serum is 0.7 g / liter.


What metabolic disorders can we observe?

Answer: Hypophospholipidemia. Normally, the contents of total phospholipids in


the blood serum of 1,52-3,62 g / L (1,98-4,71 mmol / l). The reasons could be
many disorders of liver, intestine, atherosclerosis and nutritional dystrophy.

177. Adult patient blood revealed a high concentrations of free fatty acids.
What are the causes of this metabolic disorders?
Answer: The increase in blood free fatty acids observed in diabetes, starvation, or
hyperproduction administration of adrenaline.

178. Why triglycerides are more efficient supply of energy then glycogen?

Answer: Triglycerides in their content have a lot of fatty acids. Fatty acids are the
most restored compounds and their energy is higher (9 kcal / g) than glycogen (4
kcal / g), this is built from the remnants of glucose. Unlike fats, glycogen is highly
hydrated. Thus, triglycerides contain 6 times more calories per 1 g than glycogen.

179. The patient is observed an allocation of undigested fat in the faeces.


What are the possible causes for this?

Answer: The steatorrhea, the causes of which may be insufficient revenue in the
intestine of bile acids, digestive disorders and absorption of lipids.

180. A Jewish couple of Eastern European descent presents to the clinic for
prenatal counseling after their only child died early in childhood. The family
could not remember the name of the disorder but said it was common in
their ancestry. Their first child was normal at birth, a slightly larger than
normal head circumference, an abnormal “eye finding,” and a severe
progressive neurologic disease with decreased motor skills and eventually
death. The autopsy is consistent with Tay-Sachs disease. What type of
inheritance is this disorder? What is the biochemical cause of the disorder?

Answer: Inheritance: Autosomal recessive; 1:30 carrier rate in Ashkenazi Jews.


Molecular basis of disorder: Lysosomal storage disorder with deficiency of
hexosaminidase A enzyme resulting in GM2 gangliosides accumulating
throughout the body. Tay-Sachs disease is a fatal genetic disorder where harmful
amounts of lipids called ganglioside GM2 accumulate in the nerve cells and brains
of those affected. Infants with this disorder appear normal for the first several
months of life, and then as the lipids distend the nerve cells and brain cells,
progressive deterioration occurs; the child becomes blind, deaf, and eventually
unable to swallow. Tay-Sachs disease occurs mainly in Jewish children of Eastern
European descent, and death from bronchopneumonia usually occurs by age 3 to
4 years. A reddish spot on the retina also develops, and symptoms first appear
around 6 months of age. It is a lysosomal storage disorder with insufficient activity
of the enzyme hexosaminidase A, which catalyzes the biodegradation of the
gangliosides. The diagnosis is made by the clinical suspicion and serum
hexosaminidase level. Currently there is no treatment available for this disease.

181. A couple is seen in your office for genetic counseling regarding Tay-
Sachs disease. They are very knowledgeable and request more information
about the specific enzyme that is defective in this disease. You explain that
Tay-Sachs results from the lack of which of ezymes?
Answer: Tay-Sachs disease is the result of the lack of the enzyme β-
Nacetylhexosaminidase.This enzyme hydrolyzes a terminal Nacetylgalactosamine
from the ganglioside GM2. This ganglioside is found in high concentrations in the
nervous system and is normally degraded in the lysosome by the sequential
removal of terminal sugars. The lack of β-N-acetylhexosaminidase results in the
accumulation of the partially degraded ganglioside in the lysosome leading to
significant swelling of the lysosome. The abnormally high level of lipid in the
lysosome of the neuron affects its function resulting in the disease.

182. A 48-year-old male presents to the clinic because of concerns about


heart disease. He reports that his father died from a heart attack at age 46,
and his older brother has also had a heart attack at age 46 but survived and
is on medications for elevated cholesterol. The patient reports chest pain
occasionally with ambulation around his house and is not able to climb
stairs without significant chest pain and shortness of breath. The physical
exam is normal, and the physician orders an electrocardiogram (ECG),
exercise stress test, and blood work. The patient’s cholesterol result comes
back as 350 mg/dL (normal 200). The physician prescribes medication,
which he states is directed at the ratelimiting step of cholesterol
biosynthesis. What is the rate-limiting step of cholesterol metabolism?
What is the class of medication prescribed?

Answer: Rate-limiting step: The enzyme hydroxymethylglutaryl-CoA reductase


(HMG-CoA reductase) catalyzes an early rate-limiting step in cholesterol
biosynthesis. Likely medication: HMG-CoA reductase inhibitor, otherwise known
as “statin” medications.
Hyperlipidemia is one of the most treatable risk factors of atherosclerotic
vascular disease. In particular, the level of the low-density lipoprotein (LDL)
correlates with the pathogenesis of atherosclerosis. Exercise, dietary adjustments,
and weight loss are the initial therapy of hyperlipidemia. If these are not sufficient,
then pharmacologic therapy is required. The exact LDL targets depend on the
patient’s risk of cardiovascular disease. For example, if an individual has had a
cardiovascular event previously (heart attack or stroke), the LDL target is 100
mg/dL; 1 to 2 risk factors without prior events = 130 mg/dL; and no risk factors =
160 mg/dL

183. A patient with hereditary type I hyperlipidemia presents with elevated


levels of chylomicrons and VLDL triglycerides in the blood. What is the main
function of the chylomicrons in circulation?

Answer: The liver and intestine are the main sources of circulating lipids.
Chylomicrons carry triacylglycerides and cholesterol esters from the intestine to
other target tissues. VLDLs carry lipids from the liver into circulation. Lipoproteins
are a mix of lipids and specific proteins and these complexes are classified based
on their lipid/protein ratio. Lipoprotein lipases degrade the triacylglycerides in the
chylomicrons and VLDLs with a concurrent release of apoproteins. This is a
gradual process which converts the VLDLs into IDLs and then LDLs.

184. Free cholesterol can affect cholesterol metabolism in the body by


inhibiting cholesterol biosynthesis. By which step free cholesterol can
inhibit its biosynthesis?

Answer: The major regulatory enzyme of cholesterol metabolism, β-hydroxy-β-


methylglutaryl-CoA reductase, is regulated by three distinctmechanisms. The first
is phosphorylation by a cAMP dependent protein kinase. Phosphorylation of β
-hydroxy- β -methylglutaryl-CoA reductase inactivates the enzyme. The other two
mechanisms involve the levels of cholesterol. The degradation of the enzyme is
controlled by cholesterol levels. The half-life of β–hydroxy-β-methylglutaryl-CoA
reductase is regulated by cholesterol levels with high concentrations of cholesterol
leading to a shorter half-life. The final regulatory mechanism involves control of
the expression of the β-hydroxy-β-methylglutaryl-CoA reductase gene. High levels
of cholesterol lead to a decrease in the mRNA levels coding for β-hydroxy-β-
methylglutaryl-CoA reductase.

185. A patient presents in your office with very high levels of serum
cholesterol. He states that he has tried to follow the diet and exercise
regimen you gave him last year. You decide that this patient would
benefitfrom a drug such as Lipitor (atorvastatin). Wich class of drugs is
effective in treating hypercholesterolemia because it has what effect?

Answer: The statin class of drugs – Lipitor (atorvastatin), Mevacor (lovastatin),


and Zocor (simvastatin) – is used to treat hypercholesterolemia. This class of
drugs lowers cholesterol levels by inhibiting the biosynthesis of cholesterol.
Specifically, these drugs inhibit the enzyme β-hydroxy-β-methylglutaryl-CoA
(HMG-CoA) reductase, which catalyzes the reaction that converts HMG-CoA to
mevalonate. This is the rate-limiting step of cholesterol biosynthesis. In addition to
the statin drugs which inhibit HMG-CoA reductase a number of other drugs are
used to lower cholesterol levels. The first are resins which are also referred to as
bile acid sequestrants such as cholestyramine. The resins work by binding to the
bile acids followed by excretion of the resin-bile complex. To make up for the loss
of the bile acids the body converts cholesterol into bile acids thus reducing the
cholesterol levels. Another type of drug used is the fibrates such as gemfibrozil.
These compounds work by lowering the levels of triglycerides and increasing the
levels of the “good” high-density lipoproteins (HDLs). Niacin is also effective in
lowering cholesterol levels when used in large doses (more than that required for
niacin as a vitamin). Niacin acts to lower levels of triglycerides and low-density
lipoproteins (LDLs) and increasing the levels of the “good” high-density
lipoproteins (HDLs). Drugs such as ezetimibe which inhibit the absorption of
cholesterol in the intestine are effective in lowering cholesterol levels. This drug is
often given in combination with a statin and this combination therapy is very
effective in lowering cholesterol levels.
186. Explain the mechanism of phospholipids breakdown, shown at the
scheme below:

Answer: Phospholipids are degraded by pancreatic phospholipase A 2, which


hydrolytically excises the fatty acid residue at C(2) to yield the corresponding
lysophosopholipids, which are also powerful detergents. Indeed, the phospholipid
lecithin (phosphatidylcholine) is secreted in the bile, presumably to aid in lipid
digestion. Pancreatic phospholipase A2, as does pancreatic lipase, preferentially
catalyzes reactions at interfaces. Phospholipase C gives a diacylglycerol,
phospholipase D gives a phosphatidate

187. Fill in the blanks:


O
2 CH3C CoA
Acetoacetyl-CoA
CoA CoA thiolase

O O
CH3C CH2 C CoA
Acetoacetyl-CoA
O
CH3C CoA
HMG-CoA synthase
CoASH

O OH O
-O C CH2 C CH2 C CoA
CH3
-Hydroxy--methylglutaryl-CoA
(HMG-CoA)

HMG-CoA lyase

O
? + CH3C CoA
-Hydroxybutyrate
Spontaneous? dehydrogenase
CO2 NAD+
NADH2

? ?

Answer:
O
2 CH3C CoA
Acetoacetyl-CoA
CoA CoA thiolase

O O
CH3C CH2 C CoA
Acetoacetyl-CoA
O
CH3C CoA
HMG-CoA synthase
CoASH
O OH O
-O C CH2 C CH2 C CoA
CH3
-Hydroxy--methylglutaryl-CoA
(HMG-CoA)

HMG-CoA lyase

O O O
CH3C CH2 C O- + CH3C CoA
Acetoacetate -Hydroxybutyrate
Spontaneous? dehydrogenase
CO2 NAD+
NADH2
O H O
CH3 C CH3 CH3 C CH2 C O-
Acetone OH
-Hydroxybutyrate
SECTION VI
STRUCTURE AND METABOLISM OF AMINO ACIDS

1. What is the number of amino acids involved in protein biosynthesis?


A. Twenty D. Fifteen
B. Twelve E. Thirty seven
C. Twenty five

2. Chose from listed below amino acids aliphatic one:


A. Isoleucine D. Proline
B. Lysine E. Phenylalanine
C. Histidine

3. Chose from listed below amino acids two aromatic ones:


A. Tyrosine D. Cysteine
B. Phenylalanine E. Alanine
C. Serine

4. Select from listed below amino acids non proteinogenous one:


A. β-Alanine D. Leucine
B. Aminoacetic acid E. Proline
C. Histidine

5. Note from listed below amino acids an acidic one:


A. Glutamate D. Alanine
B. Valine E. Lysine
C. Asparagine

6. In protein biosynthesis are involved:


A. L- amino acids only E. Both D- and L- amino
B. D-amino acids only acids equally well
C. D-amino acids predominantly
D. L-amino acids predominantly

7. A polypeptide is shown to have a high pI value (approx. at pH 8,9). What


from listed below amino acids is responsible for this property?
A. Arginine D. Tyrosine
B. Valine E. Cysteine
C. Serine

8. Denaturation of proteins is caused by:


A. Alteration of polypeptide chain folding C. Reduction and cleavage of
(conformation) disulphide bonds.
B. Hydrolytic cleavage of peptide bonds D. Formation of new peptide bonds
E. Dissotiation of carboxyl groups
9. Sequence of amino acids in polypeptide chain is defined as the next
structural level:
A. Primary D. Quaternary
B. Secondary E. Multimolecular complex
C. Tertiary

10. Note the correct value of normal protein concentration in human blood
plasma:
A. 65-85 g/l D. 85-100 g/l
B. 25-40 g/l E. 100-150 g/l
C. 45-60 g/l

11. Proteins of blood plasma are divided into albumin and globulins. What is
normal proportion of albumin to globulins (albumin/globulin coefficient)?
A. 2:1 D. 1:2
B. 1:1 E. 1:5
C. 5:1

12. Fibrinogen is an important protein of blood plasma. To what class of


proteins belongs fibrinogen:
A. Glycoprotein D. Chromoprotein
B. Albumin E. Nucleoprotein
C. Lipoprotein

13. In proteins amino acids are linked covalently by:


A. Peptide bonds D. Hydrophobic interactions
B. Hydrogen bonds E. Salt like bridges
C. Disulphide bonds

14. Protein is a biopolymer which can be defined as:


A. Linear chain of L-amino acids D. Linear chain of
B. Branched chain of nucleosides
mononucleotides E. Branched chain of D-
C. Linear chain of amino sugars amino acids

15. Domains are specific elements of the next structural level of protein
molecule:
A. Tertiary D. Quaternary
B. Secondary E. Multimolecular complex
C. Primary

16. Secondary structure of proteins includes the next structural element:


A. β-Pleated sheets D. Disulphide bridges
B. Triple superhelix
C. Coplanarity of peptide bond
E. Attachment of oligosaccharides
to a polypeptide chain

17. The net charge of protein molecule depends from the next factors:
A. Amino acid composition D. Quaternary structure of
B. pH value of solution protein
C. Conformation of protein E. Hydrophobic interactions
molecule between amino acid residues

18. Proteins can be precipitated by different agents. The precipitation with


ammonium sulphate is convenient due to:
A. Fractional precipitation of D. Irreversible precipitation of
protein mixtures and purification of protein from investigated
selected proteins material
B. Denaturation of precipitated E. Precipitation of amino
protein at room temperature acids accompanying proteins
C. Fragmentation of polypeptide
chain to oligopeptides

19. The concentration of protein in blood serum of a patient was on level of


40 g/l. Such situation can be classified as follows:
A. Hypoproteinemia D. Normal state
B. Dysproteinemia E. Paraproteinemia
C. Hyperproteinemia

20. Disulphide bridge in protein molecules is formed between the next


amino acids:
A. Cysteine-cysteine D. Proline-tryptophan
B. Lysine-aspartic acid E. Histidine-arginine
C. Tyrosine - histidine

22. Salt like bridges (ionic bonds) in protein molecule are formed between
the next amino acids:
A. Lysine-glutamic acid D. Aspartic acid-methionine
B. Leucine-alanine E. Serine-cysteine
C. Phenylalanine-tyrosine

23. Glutathion is a tripeptide possessing reducing properties. What amino


acid residue is responsible for reductive properties of glutathion?
A. Cysteine D. Valine
B. Glutamic acid E. Aspartic acid
C. Glycine

24. An amino acid that does not form an α-helix is:


A. Proline C. Tyrosine
B. Valine D. Tryptophan
E. Alanine

25. Which of the following is a dipeptide?


A. Anserine D. Glucagon
B. Glutathione E. β -Lipoprotein
C. Insulin

26. Which of the following is a tripeptide?


A. Glutathione D. Glucagon
B. Anserine E. Kallidin
C. Oxytocin

27. Each turn of α-helix contains the amino acid residues (number):
A. 3.6 D. 4.5
B. 3.0 E. 5.6
C. 4.2

28. At the lowest energy level α-helix of polypeptide chain is stabilised:


A. By hydrogen bonds formed between B. Disulphide bonds
the H of peptide N and the carbonyl O of C. Non polar bonds
the residue D. Ester bonds

29. In proteins the α-helix and β-pleated sheet are examples of:
A. Secondary structure C. Tertiary structure
B. Primary structure D. Quaternary structure

30. The a-helix of proteins is:


A. Stabilised by hydrogen bonds C. Made periodic by disulphide
between NH and CO groups of the bridges
main chain D. A non-periodic structure
B. A pleated structure

31. A disulphide bond can be formed between:


A. Two cysteine residues D. Two serine residues
B. Two methionine residues E. Two valine residues
C. A methionine and a cysteine residue

32. In a child, consuming meal of plant origin for a long time growth
retardation, anemia, liver and kidney impairment were observed. The cause
of such state is deficiency in diet of the next nutrients:
A. Essential amino acids D. Mineral macroelements
B. Lipids E. Carotene
C. Carbohydrates

33. Pyruvic acid can be obtained by transamination of alanine with:


A. α- ketoglutaric acid B. Acetoacetic acid
C. α- OH butyric acid E. Fumaric acid
D. Phosphoenol Pyruvic acid

34. The product of glutamate decarboxylation is:


A. Gamma-amino butyrate D. Oxaloacetate
B. Putrescine E. α-Ketoglutarate
C. Taurine

35. A vasodilating compound is produced by the decarboxylation of the


amino acid:
A. Histidine D. Arginine
B. Aspartic acid E. Glutamic acid
C. Glutamine

36. One of the products of amino acid deamination by amino acid oxidase
is:
A. Hydrogen peroxide D. Nitric oxide
B. Water E. Protons
C. Carbone dioxide

37. An important reaction for the synthesis of amino acid from carbohydrate
intermediates is transamination which requires the cofactor:
A. Pyridoxal phosphate D. Thiamin
B. Riboflavin E. Folic acid
C. Niacin

38. Which of the following enzymes catalyses reactions in the biosynthesis


of both catecholamines and indoleamines (serotonin)?
A. Aromatic amino acid C. Phenylethanolamine N-
decarboxylase methyltransferase
B. Dopamine β-hydroxylase D. Tryptophan hydroxylase
E. Tyrosine hydroxylase

39. The main sites for oxidative deamination are:


A. Liver and kidney D. Lung and spleen
B. Skin and pancreas E. Brain
C. Intestine and mammary gland

40. The amino acids involved in the synthesis of creatin are:


A. Arginine, glycine, active methionine D. Arginine, lysine, methionine
B. Arginine, alanine, glycine E. Glycine, lysine, alanine
C. Glycine, lysine, methionine

41. Note amino acids, which are participants of creatine biosynthesis:


A. Arginine. C. Methionine.
B. Lysine. D. Tryptophan.
E. Phenylalanine.

42. The amino acid that undergoes oxidative deamination at significant rate
is:
A. Glutamate D. Glutamine
B. Aspartate E. Serine
C. Alanine

43. Allosteric inhibitor of glutamate dehydrogenase is:


A. ATP D. GMP
B. ADP E. NAD+
C. AMP

44. In recognition of hepatitis the determination the following enzymes


activity in blood has diagnostic significance:
A. Amino transferases D. Aldolase
B. Amylase E. Creatin kinase
C. Lactate dehydrogenase

45. An unusually active amine, a mediator of inflammation and allergy,


appears via decarboxylation of histidine. Which of the following is it?
A. Histamine. D. γ-Aminobutyrate.
B. Serotonin. E. Tryptamine.
C. Dopamine.

46. Aminotransferases as a coenzyme contain:


A. Derivative of pyridoxine D. Coenzyme A
B. Thiamine E. Ubiquinone
C. α-Ketoglutarate

47. Which amino acid is a lipotropic factor?


A. Methionine D. Lysine
B. Lecuine E. Tyrosine
C. Tryptophan

48. Deamination of glutamate is catalysed by:


A. Glutamate dehydrogenase D. Pyruvate
B. Alanine aminotransferase carboxykinase
C. Glutamate decarboxylase E. Glutamate oxidase

49. Biogenic amine serotonine is produced by decarboxylation of the next


amino acid:
A. Tryptophane D. Arginine
B. Lysine\ E. Tyrosine
C. Histidine
50. Glutathione is a:
A. Tripeptide D. Pentapeptide
B. Dipeptide E. Amino acid
C. Polypeptide

51. The main site of urea synthesis in mammals is:


A. Liver D. Kidney
B. Skin E. Heart
C. Intestine

52. The enzymes of urea synthesis are found in:


A. Both mitochondria and cytosol D. Nucleus
B. Mitochondria only E. Peroxisomes
C. Cytosol only

53. The number of ATP required for urea synthesis is:


A. 0 D. 3
B. 1 E. 7
C. 2

54. Most of the ammonia released from L-α-amino acids reflects the coupled
action of transaminase and:
A. L-glutamate dehydrogenase C. Histidase
B. L-amino acid oxidase D. Serine dehydratase

55. In urea synthesis, the amino acid functioning solely as an enzyme


activator:
A. N-acetyl glutamate D. Arginine
B. Ornithine E. Urea
C. Citrulline

56. What amino acid from listed below participates in biosynthesis of heme?
A. Glycine D. Histidine
B. Aspartic acid E. Phenylalanine
C. Methionine

57. Albinos become tanned poorly, instead they get sunburns. The disorder
of what amino acid metabolism causes this phenomenon?
A. Tyrosine D. Glutamine.
B. Methionine. E. Histidine.
C. Tryptophan.

58. In a child in urine were detected phenylpyruvate and phenylacetate.


What enzyme insufficiency causes this phenomenon?
A. Phenylalanine –4-monooxygenase C. Fumarylacetoacetate hydrolase
B. Thyrosine-3-monooxygenase D. Cystathionyl-β-synthase
E. DOPA-decarboxylase

59. In mammalian tissues serine can be a biosynthetic precursor of:


A. Glycine D. Phenylalanine
B. Methionine E. Tyrosine
C. Tryptophan

60. The principal end product of protein metabolism, which is excreted in


the greatest quantity in human urine, is:
A.Urea D.Uric acid
B.Glutamin E.Allantoin
C.Ammonium and its salts

61. Indicate enzymes belonging to ornitine cycle of urea synthesis:


A. Carbamoylphosphate synthetase. D. Argininosuccinate
B. Ornitine decarboxylase. dehydrogenase.
C. Glutamatdehydrogenase. E. Enolase.

62. Which of denoted symptoms are characteristic to cystinosis


A. The increase of exertion of all C. Absence of cystine stones in
amino acids except cyclic; the most kidneys.
prominent is exretion of cystine and D. Hypoaminoaciduria.
cystein. E. Presence of phenylpyruvic
B. Normal blood cystine level. acid in blood

63. What chemical component in urine indicates on a hereditary metabolic


disease alkaptonuria?
A. Homogentisic acid D. Pyruvic acid
B. Phenylalanine E. Tyrosine
C. Phenylpyruvic acid

64. Amount of phenylacetic acid excreted in the urine in phenylketonuria is


A. 290–600 mg/dL D. 600–750 mg/dL
B. 200–280 mg/dL E. 750– 1000 mg/dL
C. 100–200 mg/dL

65. Tyrosinosis is due to defect in the enzyme:


A. Fumarylacetoacetate hydrolase D. Tyrosine hydroxylase
B. p-Hydroxyphenylpyruvate E. Phenylalanine–4-
hydroxylase monooxygenase
C. Tyrosine transaminase

66. An important finding in glycinuria is:


A. Excess excretion of oxalate in the C. Significantly increased serum
urine glycine level
B. Deficiency of enzyme glycinase
D. Defect in renal tubular reabsorption
of glycine

67. Increased urinary indole acetic acid is diagnostic of:


A. Hartnup disease D. Phenylketonuria
B. Maple syrup urine disease E. Alcaptonuria
C. Homocystinuia

68. An inborn error, maple syrup urine disease is due to deficiency of the
enzyme:
A. Isovaleryl-CoA-hydrogenase C. Adenosyl transferase
B. Phenylalnine hydroxylase D. -Ketoacid decarboxylase

69. An important feature of maple syrup urine disease is:


A. Blood levels of leucine, isoleucine C. Patient can not be treated by
and serine are increased dietary regulation
B. Without treatment death, of patient D. Excessive brain damage
may occur by the end of second year of
life

70. Ochronosis is an important finding of:


A. Alkaptonuria D. Richner Hanhart syndrome
B. Tyrosinosis E. Homocystinuia
C. Tyrosinemia

71. In albinism there is negative reaction to direct insolation, as a result


solar burns may appear. Metabolic disorders of what amino acid cause
these effects?
A. Tyrosine D. Glutamic acid
B. Tryptophan E. Histidine
C. Methionine

72. Transfer of the carbamoyl moiety of carbamoyl phosphate to ornithine is


catalysed by a liver mitochondrial enzyme:
A. Ornithine transcarbamoylase D. N-acetyl glutamate hydrolase
B. Carbamoyl phosphate synthetase I E. Carbamoyl phosphate
C. N-acetyl glutamate synthetase synthetase II

73. As a result of tryptophan hydroxylation in presence of tryptophan-5-


monoaxygenase is produced:
A. Serotonin D. Melanin
B. Histamine E. Adrenalin
C. Dopamine

74. A compound serving a link between citric acid cycle and urea cycle is:
A. Fumarate B. Citrate
C. Succinate E. Acetyl-S-CoA
D. Malate

75. In humann body is degraded approximately 70 g of amino acids daily.


The main end product of nitrogen metabolism in human is:
A. Urea D. Glutamine
B. Ammonia E. Creatinine
C. Uric acid

76. Choose characteristic features of phenylketonuria:


A. Deficiency of phenylalanine D. Deficiency of oxidase of
hydroxylase. homogentisic acid.
B. Excretion of enhanced E. Deficiency of tyrosinase
quantity of homogentisate with urine. activity.
C. Absence of pigments in the
skin, hair, cornea.

77. Tryptophan could be considered as a precursor of:


A. Melanotonin D. Epinephrine
B. Thyroid hormones E. Insuline
C. Melanin

78. What is the principal final nitrogen containing product of protein


catabolism in human body?
A. Urea B. Ammonia and ammonium ion
B. Glutamine C. Creatinin
E. Uric acid

79. The amino acid which detoxicated benzoic acid to form hippuric acid is:
A. Glycine D. Glutamic acid
B. Alanine E. Valine
C. Serine

80. Indicate amino acid which is a precursor of the thyroid hormones


thyroxine and triiodothyronine.
A. Tyrosine D. Glycine
B. Histidine E. Glutamate
C. Tryptophan

81. All of the following are required for synthesis of glutamine except:
A. Pyridoxal phosphate C. Glutamate
B. Ammonia D ATP

82. Cysteine can be synthesized from methionine and:


A. Serine C. Homocysteine
B. Homoserine D. Threonine
E. Histidine.

83. Hydroxylation of phenylalanine requires all of the following except:


A. NADH D. Molecular oxygen
B. Tetrahydrobiopterin E. Phenylalanine
C. Phenylalanine hydroxylase

84. Ammonia is transported from muscles to liver mainly in the form of:
A. Alanine D. Free ammonia
B. Glutamine E. Glutamate
C. Asparagine

85. ATP is required in following reactions of urea cycle:


A. Synthesis of carbamoyl phosphate C. Synthesis of argininosuccinate
and argininosuccinate and arginine
B. Synthesis of citrulline and D. Synthesis of carbamoyl
argininosuccinate phosphate and citrulline

86. Maple syrup urine diseases is an inborn error of metabolism of:


A. Branched chain amino acids D. Dicarboxylic amino acids
B. Aromatic amino acids E. Proline
C. Sulphur-containing amino acids

87. Cystinuria results from inability to:


A. Reabsorb cystine in renal tubules D. Metabolise cysteine
B. Convert cystine into cysteine E. Convert cysteine into methionine
C. Incorporate cysteine into proteins

88. The defective enzyme in histidinemia is:


A. Histidase D. Histidine oxidase
B. Histidine decarboxylase E. Histidine dehydrogenase
C. Histidine carboxylase

89. All the following statements about phenylketonuria are correct except:
A. Excreted urine darkens in the air D. It leads to decreased synthesis
B. Urinary excretion of phenylpyruvate of thyroid hormones,
and phenyllactate is increased catecholamines and melanin
C. It can be controlled by giving a low E. Phenylalanine cannot be
phenylalanine in a diet converted into tyrosine

90. In metabolic point of view, amino acids are classified as:


A. Glycogenic or Ketogenic D. Essential and nonessential
B. Cyclic and non cyclic E. All of these
C. Polar and nonpolar

91. The amino acid which synthesizes many hormones:


A. Tyrosine D. Histidine
B. Valine E. Glutamine
C. Alanine

92. In humans, NH3 is detoxified in liver as:


A. Urea D. Uronic acid
B. Uric acid E. Creatine
C. Creatinine

EXAMPLES OF KROK 1 TESTS

93. Examination of a patient suffering from cancer of urinary bladder


revealed high rate of serotonin and hydroxyanthranilic acid. It is caused by
excess of the following amino acid in the organism:
A. Tryptophan D. Methionine
B. Alanine E. Tyrosine
C. Histidine

94. High levels of serotonin and 3-oxianthranilate are revealed in the


blood of a patient suffering from urinary bladder cancer. By the
disturbance of the metabolism of what amino acid is it caused?
A. Tryptophan. D. Methionine.
B. Alanine. E. Tyrosine.
C. Histidine.

95. Glutamate decarboxylation results in formation of inhibitory transmitter


in CNS. Name it:
A. GABA D. Serotonin
B. Glutathione E. Asparagine
C. Histamine

96. Biochemical function of glutathion in an organism is connected with


reduction and detoxification of organic peroxides. During an interaction of
glutathion with hydroperoxides harmless organic alcohols are formed with
subsequent further oxidation. Indicate an amino acid composing glutathion.
A. Glutamate D. Isoleucine
B. Valine E. Tryptophan
C. Lysine

97. A patient diagnosed with carcinoid of bowels was admitted to the


hospital. Analysis revealed high production of serotonin. It is known that
this substance is formed of tryptophane aminooacid. What biochemical
mechanism underlies this process?
A. Decarboxylation D. Transamination
B. Desamination E. Formation of paired compounds
C. Microsomal oxydation
98. A patient with a cranial trauma manifests repeated epileptoid seizures.
The biosynthesis of what biogenic amine is disturbed in this clinical
situation?
A. GABA C. Serotonin
B. Histamine E. Dopamine
C. Adrenaline

99. 12 hours after an accute attack of retrosternal pain a patient presented a


jump of aspartate aminotransferase activity in blood serum. What pathology
is this deviation typical for?
A. Myocardium infarction D. Diabetes mellitus
B. Viral hepatitis E. Diabetes insipidus
C. Collagenosis

100. In experimental animals hold prolonged time on protein free diet, a fat
degeneration of liver has been developed. The possible cause may be
insufficiency of methylating agents. Indicate an amino acid, donor of methyl
groups
A. Methionine D. Cysteine
B. Phenylalanine E. Arginine
C. Lysine

101. The product of oxidase reactions is hydrogen peroxide, a very toxic


substance for cells. An important role in its reduction plays glutathion.
Indicate an amino acid present in glutathion structure.
A. Cysteine D. Aspartate
B. Serine E. Thyrosine
C. Alanine

102. Biogenic amines, namely histamine, serotonin, dopamine etc., are very
active substances that affect markedly various physiological functions of
the organism. What biochemical process is the principal pathway for
biogenic amines production in body tissues?
A. Decarboxylation of amino acids. D. Oxidation of amino acids.
B. Deamination of amino acids. E. Reductive animation.
C. Transamination of amino acids.

103. In diagnostics of an acute viral hepatitis estimation of the next


enzymatic activity in blood serum is the most valuable:
A. Alanyl aminotransferase D. Amylase
B. Glutathion peroxidase E. Alkaline phosphatase
C. Creatine kinase

104. In psychiatric practice, biogenic amines and their derivatives arc used
for the treatment of certain diseases of the central nervous system. Name
the substance of the mentioned below biochemical class which acts as an
inhibitory mediator.
A. GABA. D. Dopamine.
B. Histamine. E. Taurine.
C. Serotonin.

105. In human body are synthesized 10 amino acids only from 20 ones
needed for protein biosynthesis. What amino acid from listed below is
produced in human body?
A. Tyrosine D. Methionine
B. Histidine E. Phenylalanine
C. Lysine

106. In course of histidine catabolism a biogenic amine is formed that has


powerful vasodilatating effect. Name it:
A. Histamine D. Noradrenalin
B. Serotonin E. Dopamine
C. Dioxyphenylalanine

107. In clinical practice for parentheral nutrition a protein hydrolysate is


used. Amino acid constituents of such hydrolysate were separated with the
aid of paper chromatography into acidic and basic. Chose an acidic amino
acid from listed below:
A. Aspartate D. Glycine
B. Threonine E. Lysine
C. Serine

110. A 7-year-old child was admitted to an emergency clinic in the state of


allergic shock provoked by a wasp sting. High concentration of histamine
was determined in the patient's blood. Which biochemical reaction leads to
the production of this amine?
A. Decarboxylation. D Deamination.
B. Hydroxylation. E. Reduction.
C. Dehydration.

111. During hypersensitivity test a patient got subcutaneous injection of an


antigen which caused reddening of skin, edema, pain as a result of
histamine action. This biogenic amine is generated as a result of
transformation of the following histidine amino acid:
A. Decarboxylation D. Isomerization
B. Methylation E. Deaminization
C. Phosphorylation

112. According to clinical indications a patient was administered pyridoxal


phosphate. What processes is this medication intended to correct?
A. Transamination and decarboxylation of aminoacids
B. Oxidative decarboxylation of ketonic D. Synthesis of purine and
acids pyrimidine bases
C. Desamination of purine nucleotide E. Protein synthesis

113. A 46-year-old female has been suffering from progressive


myodystrophy Duchenne's disease) for a long time, the change of catalytic
activity of what flood enzyme proves to be a diagnostic test for the disease?
A. Creatine kinase. D. Glutamate dehydrogenase.
B. Lactate dehydrogenase. E. Adenylate kinase.
C. Pyruvate dehydrogenase.

114. Production of some toxic substances in large intestines occurs due to


decarboxylation of some amino acids. Indicate, what substance is produced
from ornithine?
A. Putrescin D. Cadaverine
B. Scatole E. Phenol
C. Indole

115. An individual who consumed 100 g of protein losses 13,5 g of nitrogen


in the urine, 2 g in the feces, and 0,5 g by other routes. This individual can
be suggested as:
A. A normal, healthy adult D. Recovering from major
B. A woman in her eight month of surgery
pregnancy E. Consuming a diet
C. A 6 year-old child consistently deficient in lysine

116. A newborn child rejects breast feeding, he is restless, his breathing is


unrhythmical, and the urine has a specific smell of beer ferment or maple
syrup. The innate defect of what enzyme causes this pathology?
A. Dehydrogenase of branched-chan α- C. Glycerol kinase.
keto acids. D. Aspartate aminotransferase.
B. Glucose-6-phosphate E. UDP-glucuronyltransferase
dehydrogenase

117. Under alcaptonuria, the excessive quantity of homogentisate was found


in the patient's urine (the urine darkens in the air). The innate defect of what
enzyme is apparent?
A. Homogentisate oxidase. D. Phenylalanine-4-
B. Alanine aminotransferase. monooxygenase.
C. Tyrosinase. E. Tyrosine aminotransferase.

118. 13 years old patient complains of general weakness, dizziness,


fatigue.Besides this mental underdevelopement is observed. Laboratory
investigations revealed high content of valine, isoleucine and leucine in
urine, which has a characteristic odor. What is the most probable cause of
this condition?
A. Maple syrup syndrome D. Porphyria
B. Cystinosis E. Hyperuricemia (gout)
C. Phenylketonuria
119. In case of skin combustion the injured area of skin gets red and painful,
the swelling of tissue also develops. What substance is responsible for
these effects?
A. Histamine D. Thiamine
B. Glutamine E. Asparaginate
C. Lysine

120. A patient suffering from collagenose manifests signs of connective


tissue destruction. The rise of blood concentration of what substances
confirms the presumptive diagnosis of the disease?
A. Oxyproline and oxylysine. D. Transaminases.
B. Creatine and creatinine. E. Urates.
C. Isoformsof LDH.

121. In a young child besides other clinical symptoms the sharp darkening
of urine after standing in open air was revealed. Blood and urine
examination detected the presence of homogentisic acid. What is the most
probable cause of disease?
A. Alkaptonuria D. Cystinuria
B. Porphyria E. Hemolytic anemia
C. Albinism

122. An infant shows the darkening of scleras, mucous membranes. The


excreted urine darkens in tin air, homogentistic acid is determined both in
the blood and urine. What is the diagnosis?
A. Alkaptonuria. D. Porphyria.
B. Albinism. E. Hemolytic anemia.
C. Cystinuria.

123. In two years old boy suffering from alkaptonuria urine became black
after standing. . This disease is hereditary disorder of:
A. Thyrosine metabolism D. Uric acid synthesis
B. Alanine metabolism E. Cystein metabolism
C. Urea synthesis

124. Laboratory analysis of the urine of a six-day infant displayed


excessive concentration of phenyl pyruvate and phenylacetate.
Metabolism of what amino acid is disturbed in the body of this child?
A. Phenylalanine. D. Histidine.
B. Tryptophan. E. Arginine.
C. Methionine.
125. In experimental animals helded prolonged time on protein free diet, a
fat degeneration of liver has been developed. The possible cause may be
insufficiency of methylating agents. Indicate an amino acid, donor of methyl
groups
A. Methionine D. Cysteine
B. Phenylalanine E. Arginine
C. Lysine

126. A 9-year-old boy was brought to a hospital with signs of mental and
physical retardation. A biochemical blood test revealed the increased level
of phenylalanine. The blockage of what, enzyme can result in such state of
the patient?
A. Phenylalanine-4-monooxygenase. D. Aspartate aminotransferase.
B. Homogentisate oxidase. E. Glutamate decarboxylase.
C. Glutamine transaminase.

127. Which of the following enzymes catalyses reactions in the biosynthesis


of both catecholamines and indoleamines (serotonin)?
A. Aromatic amino acid C. Phenylethanolamine N-
decarboxylase methyltransferase
B. Dopamine β-hydroxylase D. Tryptophan hydroxylase
E. Tyrosine hydroxylase

128. In a patient suffering from liver cirrhosis a decrease in urea


concentration in blood serum was detected. This may be caused by:
A. Disorder of urea synthesis in liver D. Deficiency of CO2 for urea
B. Absense of alanine aminotransferase synthesis
activity in hepatocytes E. Excess of ammonia blocking
C. Deficiency of ammonia for urea enzymes of urea synthesis
synthesis

129. In a child, consuming meal of plant origin for a prolong time, growth
retardation, anemia, liver and kidney impairment were observed. The cause
of such state is deficiency in diet of the next nutrients:
A. Essential amino acids D. Mineral macroelements
B. Lipids E. Carotene
C. Carbohydrates

130. Ammonia is a very poisonous chemical, especially for the nervous


system. What substance takes a particularly active part in the detoxification
of ammonia in the brain tissue?
A. Glutamic acid. D. Histidine.
B. Lysine. E. Alanine.
C. Proline.
131. Under the repeated action of ultraviolet rays, skin darkens because of
the synthesis of melanin which protects cells from damage. The principal
mechanism of this defence reaction is:
A. Activation of tyrosinase. D. Inhibition of homogentisate oxi-
B. Inhibition of tyrosinase. dase.
C. Activation of homogentisate oxidase. E. Inhibition of phenylalanine
hydroxylase.

132. A mother of a 5-year-old child has noticed that the child's urine is too
dark. The child does not have any complaints. Bile pigments are not present
in the urine. The diagnosis of alcaptonuria is set. The deficiency of what
enzyme is observed in this case?
A. Homohentisate oxidase. D. Oxyphenyl pyruvate oxidase.
B. Phenylalanine hydroxylase. E. Decarboxylase of
C. Tyrosinase. phenylpynivate.

133. A citrulline and a high level of ammonia are determined in the urine of a
newborn child. The formation of what substance is the most credible to be
disturbed?
A. Urea. D. Creatinine.
B. Uric acid. E. Creatine.
C. Ammonia.

134. A 13-year-old patient complains of general weakness, rapid fatigue.


There is retardation in his mental development. Laboratory investigation
revealed high concentrations of valine, isoleucine and leucine in his blood
and urine. The urine has a specific smell. What can the cause of such state
be?
A. Maple syrup disease. D. Histidinemia.
B. Addison's disease. E. Diffuse toxic goiter.
C. Tyrosinosis.

135. A 53-year-old man had Paget's disease diagnosed. The sharp increase
of oxyproline level was detected in the patient's daily urine output that first
and foremost testifies to the stimulation of the disintegration of:
A. Collagen. D.Elastin.
B. Keratin. E. Fibrinogen.
C. Albumin.

136. A ten-month-old child, whose parents are dark-haired, is fair-haired,


fair-complexioned, and blue-eyed. The neonate seemed to be healthy, but
during the last three months the cerebral circulation disorder and the
retardation of mental development appeared. The cause of such state is:
A. Phenylketonuria. D. Acute porphyria.
B. Galactosemia. E. Histidinemia
C. Glycogenosis.
137. The signs of skin depigmentation of a 19-year-old patient are caused by
the disorder of melanin synthesis. The disturbance of the metabolism of
what amino acid is it caused by?
A. Tyrosine. D. Proline.
B. Tryptophan. E. Lysine.
C. Histamine.

138. High levels of serotonin and 3-oxianthranilate arc revealed in the blood
of a patient suffering from urinary bladder cancer. By the disturbance of the
metabolism of what amino acid is it caused?
A. Tryptophan. D. Methionine.
B. Alanine. E. Tyrosine.
C. Histidine.

139. One of the forms of innate human pathology is accompanied by the


blockage of the conversion of phenylalanine into tyrosine. The biochemical
manifestation of the disease is the accumulation of certain organic acids in
the organism including:
A. Phenylpyruvate. D. Lactate.
B. Citrate. E. Glutamate.
C. Pyruvate.

140. A newborn child has dark coloring of scleras and mucous membranes.
The excreted urine darkens in the air. Laboratory tests of blood and urine
have revealed the occurrence of homogentisic acid. What can the cause of
this state be?
A. Alcaptonuria. D. Cystinuria.
B. Albinism. E. Histidinemia.
C. Galactosemia.

141. Affected by ultraviolet radiation, human skin darkens, which is a


protective reaction of the organism. What protective substance, namely
amino acid derivative, is synthesized in the cells under these conditions?
A. Melanin. D. Phenylalanine.
B. Arginine. E. Thyroxin.
C. Methionine.

142. In a child with functional disorders of central nervous system during


biochemical investigation were detected hyperammonemia. Preliminary
diagnosis – hereditary hyperammoniemia due to disorder of urea synthesis.
What enzymopathia can cause this disease?
A. Ornithine transcarbamoylase D. Glycyl transferase
B. Gluthation transferase E. Glucuronyl transferase
C. Sulfotransferase
CLINICAL CASES AND SITUATIONAL TASKS

143. Biuret reaction became negative after a while of proteins hydrolysis.


What changes in the protein it specifies?

Answer: The negative biuret reaction shows that there is no native protein and
indicates on the completion of hydrolysis.

143. The positive reaction with sulfosalicylic acid in urine of a patient was
observed. 0,253 % of protein was detected. What does it mean? What
consequences in the body can cause this condition?

Answer: The positive reaction with sulfosalicylic acid indicates on the presence of
a protein in urine, and it could be caused by kidney diseases. This state can
cause hypoproteinemia, the decrease of the oncotic preassure and development
of edema.

144. What is molecular weigh of glutathione, if the weigh percent of sulfur in it is


equal 10,42 %? What is the biological role of glutathione?

Answer. The molecular weigh of glutathione can be calculated according to


proportion:
100 g of glutathione contains 10,42 g of sulfur
хg ------------------------ 32 g of sulfur
х = 307 g of glutathione
Glutathione serves as a reductant; is conjugated to drugs to make them
more water soluble; is involved in amino acid transport across cell membranes
(the g-glutamyl cycle); is a substrate for the peptidoleukotrienes; serves as a
cofactor for some enzymatic reactions and as an aid in the rearrangement of
protein disulfide bonds.

145. In a patient 10 g of urine per day is excreted. Evaluate this result.

Answer: The increased content of urea in urine can be caused by negative


nitrogen balance. There is a nett loss of body protein. It reflects either a response
to trauma or infection, or an intake that is inadequate to meet the need to replace
tissue proteins that are turning over.

146. Laboratory analysis of the urine of a six-day infant displayed


excessive concentration of phenylpyruvate and phenylacetate.
Metabolism of what amino acid is disturbed in the body of this child?

Answer: Excessive concentration of phenylpyruvate and phenylacetate


indicates on the development of phenylketonuria, which is an autosomal
recessive metabolic genetic disorder characterized by a mutation in the gene for
the hepatic enzyme phenylalanine hydroxylase, rendering it nonfunctional. This
enzyme is necessary to metabolize the amino acid phenylalanine to the amino
acid tyrosine. When phenylalanine hydroxylase activity is reduced, phenylalanine
accumulates and is converted into phenylpyruvate (also known as phenylketone),
which is detected in the urine.
147. An 81/2-month-old infant was admitted to the hospital in a coma and a
temperature of 39.4°C (102.9°F). His pulse was elevated, his liver was
enlarged, and an electroencephalogram was grossly abnormal. Since the
infant could not retain milk given by gavage feeding, intravenous glucose
was administered. He improved rapidly and came out of the coma in 24
hours. Analysis of his urine showed abnormally high amounts of glutamine
and uracil, which suggested a high blood ammonium ion concentration. The
laboratory confirmed this. Considering the data, which enzyme may be
defective in this patient?

Answer: The patient exhibits signs of a defect in the urea cycle. The presence of
elevated uracil in addition to ammonia and glutamine points to an accumulation of
carbamoyl phosphate. If ornithine transcarbamoylase is deficient, carbamoyl
phosphate will accumulate in the mitochondria and leak into the cytosol, providing
the starting compound for the synthesis of uracil.

148. A newborn male infant was diagnosed as having phenylketonuria


(PKU), and immediately placed on diet low in phenylalanine (Phe); careful
compliance with the diet and frequent monitoring of the patient’s plasma
Phe level resulted in the level being maintained at the lower limit of the
normal range. The patient appeared to be developing normally until 4
months of age, when he developed truncal hypotonia and spasticity of the
limbs. Despite being on a low-phenylalanine diet, at 5 months the patient
had several grand mal (epileptic) seizures. After an abnormal Phe-loading
test, the patient’s urine was found to have a markedly elevated urinary
biopterin concentration. Which of enzymes is most likely deficient in this
patient?

Answer: The patient, despite being put on a low-Phe diet, exhibits neurologic
problems resulting from an inability to synthesize catecholamine and indoleamine
neurotransmitters. This is caused by a deficiency in dihydropteridine reductase
(DHPR). DHPR regenerates tetrahydrobiopterin (BH 4), which is oxidized to
dihydrobiopterin by phenylalanine hydroxylase, as well as tyrosine hydroxylase
and tryptophan hydroxylase (tryptophan 5-monooxygenase). If phenylalanine
hydroxylase were deficient, a diet low in Phe would alleviate the effects. Since the
urinary biopterin concentration is elevated, a deficiency in GTP cyclohydrolase I is
eliminated because that is an enzyme in the biosynthetic pathway of BH4. Phe
hydroxylase, Tyr hydroxylase, and Trp hydroxylase activities are low because of a
lack of BH4.
149. A 1-year-old girl is brought to her pediatrician’s office with concerns
about her development. The mother reports that the baby is not achieving
the normal milestones for a baby of her age. She also reports an unusual
odor to her urine and some areas of hypopigmentation on her skin and hair.
On exam, the girl is noted to have some muscle hypotonia and
microcephaly. The urine collected is found to have a “mousy” odor. What is
the most likely diagnosis? What is the biochemical basis of the
hypopigmented skin and hair?

Answer: Likely Diagnosis: Phenylketonuria (PKU). Biochemical basis of


hypopigmentation: phenylalanine is competitive inhibitor of tyrosinase (key
enzyme in melanin synthesis) Elevated phenylalanine can be caused by a variety
of different enzyme deficiencies resulting in impaired conversion of phenylalanine
to tyrosine. The most common deficiency is in phenylalanine hydroxylase
(autosomal recessive) resulting in the classic picture of PKU. Two other enzyme
deficiencies leading to PKU include dihydropteridine reductase and 6-pyruvoyl-
tetrahydropterin synthase, an enzyme in the biosynthetic pathway of
tetrahydrobiopterin. With PKU, the baby appears normal at birth but then fails to
reach normal developmental milestones. If unrecognized, the child will develop
profound mental retardation and impairment of cerebral function. A mousy odor of
the skin, hair, and urine can often be detected clinically. Treatment consists of
dietary modifications with limitation of phenylalanine intake and supplementation
of tyrosine. The diagnosis of PKU and initiation of diet modification needs to be
implemented prior to 3 weeks of age to prevent mental retardation and the other
classic signs of PKU.

150. A 1-year-old girl presents at your clinic the day after you saw the 3-
month-old boy. The symptoms are the same so you order a test on
phenylalanine hydroxylase to confirm your diagnosis of phenylketonuria. To
your surprise the phenylalanine hydroxylase activity is well within the
normal range. Which of enzymes might you check next to support your
diagnosis?

Answer: The correct response is dihydropteridine reductase. This


enzymereduces dihydrobiopterin to tetrahydrobiopterin the obligate electron donor
for phenylalanine hydroxylase. Tyrosinase is the first enzyme on the pathway to
melanin. Dopamine hydroxylase and tyrosine transaminase are enzymes on other
tyrosine metabolic tracts. Homogentisic acid oxidase is an enzyme on the
pathway of tyrosine to fumarate and acetoacetate.

151. Skin color is the aggregate result of the expression of a number of


genes modified by ethnic origin and genetic inheritance. What can cause the
hypopigmentation?

Answer: Excess phenylalanine inhibits tyrosinase the first step toward melanin
production, thus resulting in hypopigmentation. Excess melanin leads to
hyperpigmentation. Melatonin is a hormone involved in the sleep cycle. Excessive
stimulation of tyrosinase would lead to more melanin and therefore
hyperpigmentation. Para-hydroxyphenylpyruvate means less transamination and
perhaps more tyrosine converted to melanin and hyperpigmentation.

152. Name types of deamination od amino acids:


+ 2H
R CH COOH R CH2 COOH + NH3
NH2

1
+ H2 O
R CH COOH R CH COOH + NH3
NH2 OH

2
R CH2 CH COOH R CH CH COOH + NH3
NH2

3
+ ½ O2
R CH COOH R C COOH + NH3
NH2 O

4
Answer: 1. - reductive deamination, 2 - hydrolytic deamination, 3 - reductive,
4 – oxidative.

153. Complete the chemical equation of the reachtion:


HOOC CH2 CH2 CH COOH
Glutamate NH2
NAD
Glutamate dehydrogenase
NADH2

?
H2O

Spontaneously
NH3

?
Answer:
HOOC CH2 CH2 CH COOH
Glutamate NH2
NAD
Glutamate dehydrogenase
NADH2

HOOC CH2 CH2 CH COOH


Iminoglutarate NH
O O C H22O
NH
+
NH3 C OPO32
H2NSpontaneously NH
CH2
CH2 NH3
CH2
carbamoyl
phosphate Pi CH2
CH2
HOOC CH2 CH2 CH COOH
citrulline
CH2
+ 1
-ketoglutarate O
HC NH3
HC NH3+ COO

154. Name enzymes COO


of urea

cycle
Urea:Cycle COO CH2

ATP HC NH2
ornithine AMP + PPi 2
O H2O COO
4 COO
H2N C NH2
H2N NH2+ CH2 aspartate
C H
urea HC N C NH2+
NH 3 COO NH
CH2
COO CH2
CH2 CH2 arginino-
HC
arginine succinate
CH2 CH2
CH
HC NH3+  HC NH3+
COO
COO 
COO
fumarate
Answer: 1. Ornithine transcarbamylase, 2. Argininosuccinate Synthase, 3.
Argininosuccinase lyase, 4. Arginase.

155. Name the enzyme, which catalises this reaction:

Answer: Phenylalanine hydroxylase


SECTION VII
PRINCIPLES OF MOLECULAR BIOLOGY AND MOLECULAR GENETICS

1. Indicate nitrogenous base which is a specific component of


ribonucleoproteins
A. Uracil D. Thymine
B. Adenine E. Cytosine
C. Guanine

2. What proteins are obvious constituents of nucleoproteins?


A. Histones and protamins D. Prolamin and glutelin
B. Albumin and globulins E. Cytochromes b5 and c
C. Dynein and kinesin

3. How is called elementary monomer of polymer chain of nucleic acids?


A. Mononucleotide D. Nitrogenous base
B. Nucleoside E. Ribosyl-3-phosphate
C. Amino acid

4. Note the complementary to cytosine nitrogenous base in DNA double helix


according to Watson and Crick model
A. Guanine D. Thymine
B. Adenine E. Xanthine
C. Uracil

5. One step of DNA double helix in B-form contains the next number of base
pairs:
A. 10 D. 7
B. 15 E. 20
C. 13

6. How many hydrogen bonds support a complementary base pair A-T?


A. 2 D. 5
B. 1 E. 7
C. 3

7. What type of chemical bonds joins together mononucleotides in RNA


chain?
A. Phosphodiester bonds D. Coordinative
B. Hydrogen E. Hydrophobic interactions
C. Ionic

8. The base sequence of a segment of DNA is pCpApGpTpTpApGpC.


Indicate a complementary sequence.
A. pGpTpCpApApTpCpG C. pCpGpApTpTpGpApC
B. pGpCpTpApApCpTpG D. pTpApGpCpCpApGpT
E. pCpApGpTpTpApGpC

9. If the cytosine content of double-helical DNA is 20 mole% of the total


bases, the adenine content would be:
A. 30 mole% D. 40 mole%
B. 10 mole% E. 50 mole%
C. 20 mole%

10. Which scientists first gave experimental evidence that DNA is the
genetic material?
A. Avery, MacLeod, and McCarty C. Beadle and Tatum, who used a
who repeated the transformation mutational and biochemical
experiments of Griffith, and analysis of the bread mold
chemically characterized the Neurospora to establish a direct
transforming principle. link between genes and enzymes.
B. Garrod, who postulated that D. Meselson and Stahl who
Alcaptonuria, or black urine showed that DNA is replicated
disease, was due to a defective semiconservatively.
enzyme. E. Watson and Crick who gave a
model for the structure of DNA

11. The core of nucleosome is composed from the next types of histones
A. H2a,H2b,H3,H4 D. H2a,H2b,H2c,H2d
B. H1,H2a, H2b, H3 E. H1,H2,H3,H4
C. H1,H3,H4

12. A sample of some substance was submitted to hydrolysis; in


hydrolysate were detected organic bases of purine and pyrimidine classes,
ribose and phosphates, amino acids. What substance was most probably
taken for investigation?
A. Ribonucleoprotein D. Deoxyrybonucleoprotein
B. Chromoproterin E. Glycoprotein
C. Phosphoproterin

13. To which of the following does thymine form hydrogen bonds in DNA?
A. Adenine D. Guanine
B. Thymine E. Uracil
C. Cytosine

14. Which of these organelles contains DNA?


A. Mitochondria D. Rough endoplasmic reticulum
B. Golgi apparatus E. Smooth endoplasmic reticulum
C. Lysosomes

15. In a double stranded molecule of DNA, the ratio of purines: pyrimidines


is:
A. Always 1:1 D. Genetically determined
B. Variable E. Determined by the number of
C. Determined by the base purines in the sense strand of the
sequence in RNA DNА

16. In which of the following molecules would you find an anticodon?


A. Transfer RNA D. Small nuclear RNA
B. Messenger RNA E. Heterogenous RNA
C. Ribosomal RNA

17. How many hydrogen bonds support a complementary base pair G-C?
A. 3 D. 1
B. 7 E. 2
C. 5

18. Nucleic acids absorb ultraviolet light in 250-270 nm regions. This


property is determined by the next chemical structures:
A. Nitrogenous bases D. Phosphoric acid
B. Hydrogen bonds E. Phosphodiester bonds
C. Ribose

19. Which of the following nucleotide bases is not found in iRNA?


A. Thymine D. Guanine
B. Adenine E. Cytosine
C. Uracil

20. Which of the following molecules does not form part of DNA?
A. Amino acid D. Deoxyribose
B. Purine E. Phosphate
C. Pyrimidine

21. Which of the following statements is true of double-helical DNA?


A. The 3’ hydroxyl groups of D. The step of B-helix contain
each chain are at opposite ends of 12 base pairs
the molecule E. The duplex structure is
B. The planes of the bases lie stabilized by hydrophobic
parallel to the helix axis interactions between bases
C. The chains have a backbone
of linked glycosides

22. With what mRNA codon would the tRNA in the diagram be able to form a
codon-anticodon base pairing interaction?
A. 3'-AUG-5' D. 3'-UAC-5'
B. 3'-GUA-5' E. 3'-UAG-5'
C. 3'-CAU-5'
23. Radiolabelled H3-thymidine was introduced to cell culture medium. In
which cell organelles will be detected H3-label thereafter?
A. Nucleus D. Lysosomes
B. Ribosomes E. Endoplasmic reticulum
C. Golgi apparatus

24. Which of the following statements concerning characteristics of


histones is true?
A. They are acidic proteins D. They are large proteins with
located in the nucleus mol weight more then 100 KDa
B. They are covalently linked to E. They have no post-
single-stranded DNA translationally modified amino
C. Evolutionary they are highly acids
conserved proteins

25. Nitrosamines belong to deaminating mutagens. From what nitrogene


base does uracyl appears as a result of their action?
A. Cytosine D. Thymine
B. Adenine E. Methyluracil
C. Guanine

26. Which of the following is an accurate statement concerning the


differences between DNA and RNA?
A. RNA lacks the base C. RNA has the sugar
thymine (which is found in DNA) deoxyribose, but DNA has the
and has uracil instead sugar ribose.
B. RNA is usually double- D. RNA contains three
stranded, but DNA is usually different nucleotides, but DNA
single-stranded. contains four different nucleotides.

27. Orotic acid is an intermediate in biosynthesis of the following


nitrogenous base:
A. Cytosine D. Guanine
B. Hypoxanthine E. Xanthine
C. Adenine

28. The activity of xanthine oxidase is dependent from the next coenzyme:
A. FAD D. CoA-SH
B. NAD E. NADP
C. FMN

29. Which of the following contribute N to both purine and pyrimidine:


A. Aspartic acid. B. Glutamic acid.
C. Glycine. E. Ammonia.
D. Arginine.

30. Formation of thymidine nucleotides, which are used for the biosynthesis
of DNA, begins from dUDP, which on the first stage is hydrolised to dUMP,
and thereafter methylated. What compound serves as the donor of methyl
groups?
A. Methylenetetrahydrofolate D. Methionine
B. Choline E. Carnitine
C. Lecithin

31. In reaction of transformation of ribose to deoxyribose in course of


deoxyribonucleotide production for DNA biosynthesis participates a low
molecular weight protein thioredoxine. It contains two SH groups, which in
course of reaction are oxidized. What coenzyme is used in restoration of
reduced form of thioredoxine?
A. NADP H2 D. NAD H2
B. Glutathion E.AMP
C. Coenzyme Q

32. Biosynthesis of what mononucleotide is arrested under the influence of


5-fluorouracil?
A.dTMP D.dIMP
B.dUMP E.dAMP
C.dCMP

33. The smallest unit of DNA capable of coding for the synthesis of a
polypeptide is:
A. Cistron D. Replicon
B. Operon E. Terminator
C. Repressor gene

34. mRNA is complementary to the nucleotide sequence of


A. Coding strand D. Template strand
B. Ribosomal RNA E. snRNA
C. tRNA

35. All pribnow boxes are variants of the sequence:


A. 5′–TATAAT –3′ D. 5′–TCCTAG –3′
B. 5′–GAGCCA –3′ E. 5′–UCCUAG –3′
C. 5′–UAACAA –3′

36. From nitrates, nitrites and nitrosamines in the body is produced nitrose
acid, which cause oxidative deamination of nitrogeneous bases of
nucleotides. This may lead to a point mutation by change of cytosine to one
of the next base:
A. Uracil D. Guanine
B. Thymine E. Inosine
C. Adenine

37. 5’-Terminus of mRNA molecule is capped with;


A. 7-Methylguanosine triphophate D. Adenosine diphosphate
B. Guanosine triphosphate E. Adenosine monophosphate
C. Adenosine triphosphate

38. Which of the following statements best describe the action of introns?
A. They are excised upon processing C. They are spacer sequences
of heterogeneous nuclear RNA to D. They are added to mRNA during
messenger RNA the plicing reaction.
B. They are retained upon processing
of ribosomal RNA

39. Okazaki fragments are formed during the synthesis of:


A. DNA D. rRNA
B. mRNA E. Proteine
C. tRNA

40. Reverse transcriptase is capable of synthesis:


A. RNA → DNA D. DNA → DNA
B. DNA → RNA E. Protein → DNA
C. RNA → RNA

41. Synthesis of DNA is also known as:


A. Replication C. Transcription
B. Duplication D. Translation

42. In treatment of infection diseases are used antibiotics streptomycine,


neomycine, kanamycine. What step of protein synthesis in bacterial cell
they inhibit?
A. Translation D. mRNA processing
B. Replication E. Splicing
C. Transcription

43. What enzyme is used for synthesis of genes from template RNA or DNA
in gene engineering? (This enzyme was discovered in some RNA containing
viruses).
A. Revertase D. Topoisomerase I
B. Exonuclease E. Helicase
C. Endonuclease
44. Genetic information is stored in DNA, which does not participate directly
in protein synthesis in the cell. What process provides the transformation of
genetic information into amino acid sequence of polypeptide chain?
A. Translation D. Replication
B. Transcription E. Splicing
C. Translocation

45. In post-translation modification of nascent protein chain are involved the


next proteins:
A. Chaperons 60 K D. Cytochrome c
B. Cathepsins E. Ubiquitin
C. Caspases

46. What amino acid is coded by the triplet of bases AUG?


A. Methionine D. Cysteine
B. Serine E. Valine
C. Tyrosine

47. Which of the following toxins inhibits eukaryotic protein synthesis


through the depurination of a single adenine residue in 28 s ribosomal
RNA?
A. Diphtheria toxin D. Puromycin
B. Ricin E. Cycloheximide
C. Sarcin

48. The genetic code refers to which one of the following?


A. The nucleotide sequences that C. The amino acid sequence of
correspond to common amino cellular proteins
acids D. The ratios of Mendelian
B. The number of chromosomes in inheritance
the diploid cells of the species E. The hierarchy of DNA, RNA and
protein

49. The translation of mRNA into the amino acid sequence of a polypeptide
in prokaryotes is terminated at the end of the message by one of the three
stop codons in the mRNA chain. The stop codon is recognized by:
A. A specific protein D. A specific ribosomal RNA
B. A specific uncharged tRNA E. A specific ribosomal subunit
C. A specific aminoacyl-tRNA

50. The inherited information is saved in DNA, though directly in the


synthesis of protein in a cell it does not participate. What process provides
the realization of the inherited information in a polypeptide chain?
A. Translation D. Replication
B. Transcription E. Transformation
C. Translocation
51. Glycosylation of proteins after completion of their synthesis in
ribosomes proceeds in the next cell compartment:
A. Golgi vesicles D. Proteasomes
B. Mitochondria E. Ribosomes
C. Lysosomes

52. Redundancy of the genetic code means that:


A. A given base triplet can code for C. The third base in codon is not
more then one amino acid important in coding
B. There is no punctuation in the code D. A given amino acid can be coded
sequences for by more then one base triplet
E. Codons are not ambiguous

53. Restrictases are enzymes of bacterial origin, which are used in


recombinant DNA technology. They belong to the next class of enzymes:
A. Hydrolases D. Liases
B. Oxido-reductases E. Isomerases
C. Transferases,

54. The next technique is used for multiple amplification of distinct and
selected segment of DNA:
A. Polymerase chain reaction D. Northern blot analysis
(PCR) E. Restriction fragment length
B. DNA fingerprint analysis polymorphism (RFLP) analysis
C. Southern blot analysis

55. For the formation of the transport form of amino acid during the protein
synthesis in ribosomes. is required.
A. tRNA. D. mRNA.
B. Revertase. E. Ribosome
C. GTP.

56. In case of poisoning by amanitine, a death-cup mushroom toxin, RNA-


polymerase B (II) is blocked. This leads to the blockage of:
A. Processing of mRNA. D.Synthesis of primers.
B. Synthesis of tRNA. E. Synthesis of mRNA.
C. Reverse transcription.

57. Degeneration of the genetic code is the ability of more than one triplet to
encode a single amino acid. Which amino acid is encoded by only one
triplet?
A. Methionine. D. Leucine.
B. Serine. E. Lysine.
C. Alanine.
58. A human genome contains about 30000 genes, and the amount of
variants of antibodies reaches millions. What mechanism is used for the
formation of new genes that are responsible for the synthesis of such
amount of antibodies?
A. Recombination of genes. D. Reparation of DNA.
B. Amplification of genes. E. Formation of Okazaki fragments.
C. Replication of DNA.
59. Choose from the following a potent inhibitor of transcription:
A. Actinomycin D D. Doxorubicin
B. Steptomycin E. Actinorodin
C. Kanamycin

60. Indicate a specific feature of transcription in prokaryotes:


A. All of the above it is transcribed completely (i.e.,
B. mRNA transcript is used transcription and translation are
directly for translation without coupled).
modification. D. Prokaryote mRNAs are
C. Since prokaryotes lack a polycistronic, they contain amino acid
nucleus, mRNA also is coding information for more than one
translated on ribosomes before gene.

61. What is not typical for transcription of eukaryotes?


A. mRNA transcript is used C. Transcription and translation are
directly for translation without not coupled (mRNA must first be
modification exported to the cytoplasm before
B. mRNA transcript is not translation occurs).
mature (pre-mRNA); must be D. Eukaryote mRNAs are
processed. monocistronic, they contain amino
acid sequences for just one gene.

62. Indicate function of snRNA (small nuclear RNA):


A. It forms complexes with B. It forms complexes called ribosomes
proteins used in eukaryotic RNA with protein, the structure on which
processing (e.g., exon splicing and mRNA is translated.
intron removal). C. It binds to 3’ UTR target mRNAs and
result in silencing.
D. It encodes the amino acid sequence of
a polypeptide.
63. The genetic code is brought by the?
A. mRNA D. siRNA
B. DNA E. rRNA
C. snRNA

64. Active translation always occurs on:


A. Polysomes D. Inner membrane of
B. Lysosomes mitochondria
C. Liposomes

65. Escherichia coli 70S ribosome model consists from:


A. 50S subunit and 30S subunit C. 50S subunit and 40S subunit
B. 50S subunit and 20S subunit D. 40S subunit and 30S subunit

66. The lac operon is an example of:


A. Transcriptional control C. Replicational control
B. Post-transcriptional control D. Transcriptional control

67. In the lac operon, RNA polymerase:


A. Binds to the promoter D. Is synthesized by the regulatory
B. Binds to the operator gene
C. Binds to the regulatory gene E. Binds to the regulator protein

68. The promoter region of a bacterial operon:


A. Is a binding site for RNA C. Codes for inducer substances
polymerase D. Codes for corepressor substances
B. Codes for repressor proteins E. Is binding site for inducers

69. The regulatory gene of bacterial operon:


A. codes for repressor proteins D. is a binding site for RNA
B. codes for repressor proteins polymerase
C. acts as an on-off swith for E. is a binding site for inducers
the structural genes

70. The sugar lactose induces synthesis of the enzyme lactase. What
happens when E. coli cells run out of lactose?
A. Repressor protein binds to C. RNA polymerase attaches to the
the operator. promoter.
B. Repressor protein binds to D. RNA polymerase attaches to the
the promoter. repressor.

72. In the lactose operon system in E. coli, the repressor is:


A. A protein C. Bound to the promoter sequence
B. A product of a structural gene D. Lactose
E. A short length of DNA

73. Which of the following statements concerning the regulatory gene


associated with the lac operon is correct?
A. mRNA is transcribed from the R D. Lactose inhibits the
gene whether lactose is present or translation of R gene mRNA.
not. E. Lactose binds to the
B. mRNA is only transcribed from the promoter of the lac operon.
R gene when lactose is present.
C. mRNA is only transcribed from the
R gene when lactose is not present.

74. According to the Jacob-Monod model of gene regulation, inducer


substances in bacterial cells probably:
A. Combine with repressor proteins, D. Combine with promoter
inactivating them. regions, activating RNA
B. Combine with operator regions. polymerase.
C. Combine with structural genes, E. Combine with nucleoli,
stimulating them to synthesize triggering production of more
messenger RNA. ribosomes.

75. In the tryptophan operon of E. coli the end product of biochemical


pathway, tryptophan, binds to the repressor protein which then binds to
the:
A. Operator to inhibit transcription D. Operator to accelerate
B. Promoter to accelerate transcription
transcription E. Repressor gene to
C. Promoter to inhibit transcription accelerate transcription

76. Nucleosomes:
A. both D and E are correct D. disappear during
B. are composed of DNA and transcription
histones E. play a role in coiling and
C. are small particles found in only the uncoiling the chromosomes
nuclei of plant cells

77. An antibody molecule is made up of four polypeptide chains joined


together by disulfide bonds. Which one of the following statements
concerning these polypeptide chains is correct?
A. All four chains have variable regions D. Only the two heavy chains have
B. Only the two light chains have variable variable regions
regions E. All of four chains are of equal
C. Only the two heavy chains have length
constant regions
78. Which one of the following statements concerning antibodies is
correct?
A. Antigens are bound by the variable D. The heavy but not the light,
regions of the antibody molecules. chains of the antibody molecule
B. Antibodies are usually proteins but have variable and constant
sometimes they are polysaccharides. regions.
C. Each antibody has two chains. E. The tail region is important in
antigen specificity.

79. Fragments of DNA containing one or several genes and a control


element that can move from one part of the genome to another are called:
A. Transposons D. Plasmids.
B. Exons. E. Promoters
C. Introns.

80. Genes and\or exons can be duplicated by all of the following exept:
A. mRNA processing in the nucleus. E. Breakage and fusion during
B. Reverse transcription. meiosis.
C. Transposition.
D. Unequal crossing over during meiosis.

81. Which statement about antibodies is false?


A. Human blood contains high levels of D. Antibodies are found in
circulating antibodies against all positive vertebrate animals.
antigens. E. Each antibody combines with a
B. Two sites on each antibody molecule specific antigen.
can bind antigen.
C. Antibodies are globulin proteins.

82. The enormous variety of antibodies produced by an individual is due to:


A. Excision of genetic material during as a template for antibody
fatal development and exon removal formation.
during mRNA processing. D. Induction of cell division by
B. Specific rearrangement of genes in cross-linking of the antibody
developing lymphocytes induced by molecules.
antigens in the nucleus. E. Mutations within the genome.
C. Synthesis of antibodies upon
exposure to an antigen using the antigen

83. Restriction endonucleases are useful in recombinant DNA technique


because they:
A. Cut DNA at specific sites. C. Restore the bonds in the DNA
B. Restrict the number of nucleotides backbone.
that can be removed at one time. D. Synthesize cDNA from RNA.
E. Can be used to locate genes for
mapping.

84. Which one of the following statements is false?


A. Molecules of mRNA are C. The nucleolus is a specialized
synthesized on the ribosomes from region of a chromosome where tRNA
nucleotides brought by tRNA. is synthesized.
B. In base substitution mutations only D. Some amino acids are specified by
a single nucleotide of a gene is several “synonymous” codons.
altered.

EXAMPLES OF KROK 1 TESTS

85. In modern biochemical investigations for diagnostics of inherited diseases,


detection of certain viruses (for example HIV), authentification of individual (gene
dactylography in forensic medicine) so called "DNA-diagnostics" is imployed.
What method is used in these investigations?
A. Polymerase chain reaction D. Rentgen-structural analysis
B. Chromatography E. Electron microscopy
C. Electrophoresis

86. From nitrates, nitrites and nitrosamines in organism is formed nitrous


acid which causes oxidative deamination of nitrogene bases of nucleotides.
This induce a point mutation by replacement of cytosine to...
A. Uracil D. Guanine
B. Thymine E. Inosine
C. Adenine

87. 5’-bromouracil inhibits the biosynthesis of one from the listed below
compounds. Chose this compound
A. DNA D. rRNA
B. Protein E. Histones
C. mRNA

88. A physician prescribed allopurinol to a patient suffering from gout. What


pharmacological property of allopurinol provides a therapeutic effect in this
case?
A. Competitive inhibition of xanthine D. Decrease of pyrimidine nucleotides
oxydase reutilization.
B. Acceleration of pyrimidine E. Acceleration of nucleic acids
nucleotides catabolism. biosynthesis
C. Increace of nitrogen-containing
substances excretion.

89. In a child a physical and mental underdevelopment is observed. In urine


is excreted large quantity of orotic acid. This hereditary disease is a result
of the next metabolic disorder:
A. Pyrimidine nucleotides synthesis C. Purine nucleotides synthesis
B. Pyrimidine nucleotides breakdown D. Purine nucleotides breakdown
E. Ornithine cycle of urea production

90. In one month old child an enhanced content of orotic acid in urine is
detected, a child has diminished weight gain. What treatment must be
undertaken in order to correct metabolic disorders?
A. Injections of uridine D. Injections of thymidine
B. Injections of adenosine E. Injections of histidine
C. Injections of guanosine

91. The decrease of uric acid concentration and the accumulation of


xanthine and hvpoxanthine were found in the blood of a 12-year-old boy.
The genetic defect of the synthesis of what enzyme does it testifies to?
A. Xanthine oxydase D. Ornithine carbamoyl transferase
B. Arginase E. Glycerol kinase
C. Urease

92. RNA of AIDS virus invaded leukocyte and caused production of viral
DNA in a cell with the aid of the enzyme revertase. This is based on the next
process:
A. Reversed transcription D. Convariant replication
B. Operone activation E. Reversed translation
C. Operone repression

93. Into human body were incorporated mercury ions. This led to the
increase in rate of transcription of the gene, responsible for detoxification of
heavy metals. What protein gene amplification is in the background of this
process?
A. Metallothioneine D. Transferrin
B. Ceruloplasmin E. Ferritin
C. Interferone

94. Oncology patient was administered an antitumor drug – metothrexate.


After some period tumor cells lost sensitivity to this drug. What gene
amplification caused this effect?
A. Dihydrofolate reductase D. Ribonucleitide reductase
B. Glutathion reductase E. Methemoglobin reductase
C. Thioredoxine reductase

95. For identification of father of a child is used the next analytical


technique:
A. Polymerase chain reaction (PCR) D. DNA fingerprint analysis
B. Northern blot analysis E. Southern blot analysis
C. Restriction fragmrnt length
polymorphism analysis
96. In oncology patients prolong application of antitumor drugs induce
appearance of resistance of target cells to this drugs. What process is
responsible for this effect?
A. Gene mutation D. Gene expression
B. Gene recombination E. Gene amplification
C. Gene modification

97. In a patient was recognized endemic goiter. What type of post-


translational modification of thyroglobuline is damaged in a patient?
A. Iodination D. Acetylation
B. Phosphorylation E. Glycosylation
C. Methylation

98. A 50-year-old patient is diagnosed with gout and there is hyperuricemia


in his blood. The metabolism of what substances is disturbed?
A. Purines D. Carbohydrates
B. Fats E. Pyrimidines.
C. Amino acids

99. The samples of blood of a child and of a supposed father were directed
for affiliation to medical forensic examination. Which chemical components
need to be identified in the explored samples of blood?
A. DNA D. mRNA
B. tRNA E. mnRNA
C. rRNA

100. Parents of a 5-year-old child consulted a doctor. Examination of the


child discovered retardation in mental development and growth, as well as
abasement of the child's agility. The basal metabolism is lowered. What
disease does the child suffer from?
A. Lesch-Nyhan syndrome. D. Hyperparathyroidism.
B. Cretinism. E. Endemic goiter.
C. Phenylketonuria.

101. A 65-year-old man, suffering from gout, complains of pains in the area
of kidneys. Ultrasonic inspection revealed the presence of stones inside the
kidneys. Which biochemical process is the main cause of kidney stones
formation?
A. Degradation of purine nucleotides. D. Degradation of heme.
B. Catabolism of proteins. E. Reduction of cysteine.
C. Ornithine cycle.

102. In a man 45 years old, suffering from gout, increased concentration of


uric acid in blood was observed. For treatment allopurinol was
administered, which is a competitive inhibitor of the following enzyme:
A. Xanthine oxidase B. Adenosine deaminase
C. Hypoxanthine phosphoribosyl D. Guanine deaminase
transferase E. Adenin phosphoribosyl transferase

103. An 8-year-old boy suffers from Lesch-Nyhan's disease. The increased


concentration of uric acid was determined in his blood. Which biochemical
process disorder is the cause of this inherited disease?
A. Degradation of purine nucleotides D. Degradation of pyrimidine
B. Synthesis of purine nucleotides. nucleotides.
C. Synthesis of pyrimidine E. Synthesis of deoxyribonucleotides
nucleotides.

104. A 72 years old woman complains on pains in joints, restriction of


movement in joints. The joints are swollen, looking as enlarged knots. In
blood and urine an increased concentration of uric acid is detected. What
disease is characterized by these symptoms?
A.Gout D.Thyrosinosis
B.Pellagra E.Liver cirrhosis
C.Alkaptonuria

105. If a double-stranded DNA molecule undergoes two rounds of


replication in an in vitro system that contains all of the necessary enzymes
and nucleoside triphosphates that have been labeled with 32P, which of the
following best describes the distribution of radioactivity in the four resulting
DNA molecules?
A. Two of the molecules contain C. Exactly one of the molecules
radioactivity in both strands. contains no radioactivity.
B. Exactly one of the molecules D. Three of the molecules contain
contains radioactivity in only one radioactivity in both strands.
strand. E. All four molecules contain
radioactivity in only one strand.

CLINICAL CASES AND SITUATIONAL TASKS

106. Antibiotic resistance develops because these drugs are overused in


medical practice and in livestock feeds. Suggest the mechanism by which
this extensive use promotes antibiotic resistance.

Answer: When antibiotics are used in large quantities, the bacterial cells that
possess resistance genes (acquired through mutations or through intermicrobial
DNA transfer mechanisms) survive and even flourish. Because of antibiotic use,
which acts as a selection pressure, resistant organisms become the dominant
cells in their ecological niche.

107. Briefly outline the basic principles of PCR. Calculate the degree of
amplification attained by 15 PCR cycles.
Answer: PCR begins by adding polymerase, primers to a heated sample of the
target DNA. As the mixture cools, the primers attach to their complementary
sequence on either side of the target sequence. Each strand then serves as a
template for DNA replication. On the end of this process, referred to as a cycle,
the copies of the target sequence have been doubled. The process can be
repeated indefinitely, synthesizing an extraordinary number of copies. After 15
replications 215 copies have been produced.

108. List and describe properties of the genetic code.

Answer: The genetic code is degenerate (several codons have the same
meaning), specific (each codon specifies only one amino acid), universal (with a
few exceptions each codon always specifies the same amino acid). In addition
genetic code is nonoverlapping and without punctuations (i.e., mRNA is read as
continuous coding sequence).

109. A 48-year-old man has had a lengthy history of skin cancer. In the past
6 years he has had over 30 neoplasms removed from sun-exposed areas
and has been diagnosed with xeroderma pigmentosum. Which disorder best
describes the enzymatic defect in patients with xeroderma pigmentosum?
Explain your answer.

Answer: Xeroderma pigmentosum is a genetic disease in which the ability to


remove pyrimidine dimers caused by exposure to UV light is impaired. The
mechanism used to remove these pyrimidine dimmers (also used to repair DNA
that has formed adducts with carcinogenic compounds) is excision repair. The
enzymes used in this repair mechanism cleave the affected strand on either side
of damaged nucleotides. The oligonucleotide containing the damaged nucleotides
is removed and the gap is filled in by DNA polymerase and DNA ligase.

110. A 32-year-old female is being treated with methotrexate for a recently


diagnosed choriocarcinoma of the ovary, and presents with complaints of
oral mucosal ulcers. The patient recalls being advised not to take folate-
containing vitamins during therapy. An uncomplicated surgical exploration
was performed 5 weeks ago with removal of the affected ovary. The patient
has been taking methotrexate for 2 weeks and has never had any of the
above symptoms before. On examination, patient was afebrile and appeared
ill. Several mucosal ulcers were seen in her mouth. The patient also had
some upper abdominal tenderness. Her platelet count is decreased at
60,000/mm3 (normal 150,000 to 450,000/mm 3). What is the most likely
etiology of her symptoms? What is the biochemical explanation of her
symptoms? Explain the mechanism of methotrexate action and its
cytotoxicity.

Answer: Likely cause of her symptoms: Side effects of methotrexate


(antimetabolite chemotherapy) affecting rapidly dividing cells such as oral
mucosa. Biochemical explanation of her symptoms: Related to effects of
methotrexate on cell cycle of all cells (particularly rapidly dividing cells). Folate
antagonists inhibit dihydrofolate reductase (tetrahydrofolate needed for purine
synthesis). Mechanism of action: Chemotherapeutic agents are used to treat
various types of cancers. Although some are specific for cancer cells, most
chemotherapeutic agents are toxic for both normal and cancer cells. Methotrexate
acts as a folate antagonist, affecting DNA synthesis. Because cancer cells divide
faster than normal cells, a higher proportion of these neoplastic cells will die.
Nevertheless, normal cells that also are rapidly dividing, such as the
gastrointestinal mucosa, the oral mucosa, and the bone marrow cells, may be
affected. The patient was advised to avoid folate during therapy, since folate
would be an “antidote,” and would allow the cancer cells to escape cell kill.

111. A 46-year-old male presents to the emergency department with severe


right toe pain. The patient was in usual state of health until early in the
morning when he woke up with severe pain in his right big toe. The patient
denies any trauma to the toe and no previous history of such pain in other
joints. On examination, he was found to have a temperature of 38.2°C
(100.8°F) and in moderate distress secondary to the pain in his right toe. The
right big toe was swollen, warm, red, and exquisitely tender. The remainder
of the examination was normal. Synovial fluid was obtained and revealed
rod- or needle-shaped crystals that were negatively birefringent under
polarizing microscopy, consistent with gout. What is the likely diagnosis?
How would you make a definite diagnosis? What is the pathophysiology of
this disorder?

Answer: Diagnosis: Gouty arthritis. Confirming diagnosis: Demonstration of the


presence of the monosodium urate crystals within the synovial leukocytes or in
material derived from tophi under polarizing microscopy. Pathophysiology:
Increased conversion of purine bases to uric acid or a decreased excretion of uric
acid by the kidney. Elevated levels of the insoluble uric acid result in precipitation
of urate crystals in the joints.

112. Inherited defects in components of purine catabolism and salvage are


associated with various conditions and syndromes. The enzyme
hypoxanthine-guanine phosphoribosyltransferase (HGPRT) is a key enzyme
in the purine salvage pathway. It is responsible for reforming IMP and GMP
from hypoxanthine and guanine, respectively. In this manner purine bases
are salvaged back into the purine nucleotide pool. What genetic defects that
lead to the loss of HGPRT activity are the primary cause for the following
conditions?

Answer: Lesch-Nyhan syndrome results from an inherited deficiency in HGPRT.


This syndrome is associated with mental retardation and self-destructive behavior,
which may be associated with inadequate production of purine nucleotides
through the salvage pathway in certain neuronal cells. In addition, Lesch-Nyhan
patients have gout resulting from the inability to salvage purine bases, which leads
to increased levels of uric acid. However, most patients with gout do not have a
defect in HGPRT but have hyperuricemia resulting from a number of factors,
including diet.

113. Fill in the blanks.


DNA (strand 1) TGT ___ ___
DNA (strand 2) __A C__ ___
mRNA (from strand2) U _ _ _ C A _ _ _
tRNA anticodons ___ ___ GCA

Answer :
DNA (strand 1) TGT GCA CGT
DNA (strand 2) ACA CGT GCA
mRNA (from strand2) UGU GCA CGU
tRNA anticodons ACA CGU GCA

114. Put the numbers of the enzymes on their place in the picture. Using
arrows indicate the direction of replication and direction of synthesis of
leading and lagging strands.

1 – DNA polymerase
2 – DNA ligase
3 – SSB proteins
4 – topoisomerase
5 – Okazaki fragment
6 – RNA primer
7 - RNA primase
8 – helicase

31
4 1

Answer: 8
1

6
1
5
7 2
115. Many translation-inhibiting antibiotics work selectively on prokaryotic
organisms because the process of translation is sufficiently different in
them than it is in humans. That is why translation is a clinically important
target of a variety of such drugs. Put antibiotics, inhibitors of translation
chloramphenicol, erythromycin, tetracycline, streptomycin into the boxes
that indicate their mechanism of action.

Answer:
116. Three possible ways in which DNA can replicate are illustrated. The two
original strands of DNA are shown in dark; newly synthesized DNA is light.
Choose from the above a semiconservative model of DNA replication, name
the rest models.

1 2 3

Original
double helix

First round
of replication

Second
round of
replication

Answer: 1 – conservative, 2 – semiconservative, 3 – dispersive.


SECTION VIII
MOLECULAR MECHANISMS OF HORMONE ACTION ON TARGET CELLS.
BIOCHEMISTRY OF HORMONAL REGULATION

l. Indicate a substance which serves as a secondary messenger and


increase Ca2+ ion concentration.
A. Inositol-1,4,5-triphosphate D. Inositol-6-phosphate
B. Inositol-3,6-bisphosphate E. Free inositol
C. Phosphatidyl-inositol-4,5-bisphosphate

2. A hormone secreted from anterior pituitary is:


A. Growth hormone D. Epinephrine
B. Vasopressin E. Norepinephrine
C. Oxytocin

3. A hormone secreted from posterior pituitary is:


A. Vasopressin D. Adrenocorticotropic hormone
B. Thyrotropic hormone E. Growth hormone
C. Prolactin

4. The number of amino acids in human growth hormone is


A. 191 D. 291
B. 19 E. 391
C. 151

8. Acromegaly results due to excessive release of:


A. Growth hormone D. Thyroxine
B. Insulin E. Oxytocin
C. Glucagon

5. All of the following are known to be part of a signal transduction cascade


EXCEPT:
A. Phosphorylation of fibronectin D. Elevation of intracellular
B. Dissociation of the components of a [Ca2+]
heterotrimeric G-protein E. Activation of cGMP
C. Enzymatic breakdown of phosphodiesterase
phosphatidyl inositol bisphosphate (PIP2
)

6. Ca2+ ions constitute one of the most ancient evolutionary second


messengers. They are activators of glycogenolisis in case of reacting with:
A. Calmodulin D. Kinase of myosin light
B. Calcitonin chains
C. Calciferol E. Phosphorylase C
7. Secondary messengers diacylglycerol and inositol triphosphate are
produced from subsequent phospholipid of plasma membrane due to the
activity of the following enzyme:
A. Phospholipase C D. Phospholipase D
B. Phospholipase A1 E. Phosphodiesterase
C. Phospholipase A2

8. All of the following are known to be part of a signal transduction cascade


EXCEPT:
A. Phosphorylation of fibronectin D. Elevation of intracellular
B. Dissociation of the components of a [Ca2+]
heterotrimeric G-protein E. Activation of cGMP
C. Enzymatic breakdown of phosphodiesterase
phosphatidyl inositol bisphosphate
(PIP2 )

9. The receptors of which of the following hormones are not associated


with G-protein?
A. Aldosterone D. Vasopressin
B. TSH E. Glucagon
С. Epinephrine

10. Adrenalin is water soluble hormone, its effect is mediated by secondary


messengers, which are:
A. cAMP D. Acetylcholine
B. NAD E. Cytochrome c
C. ATP

11. There are intracellular receptors for which of the following hormones?
A. Thyroxine D. Insulin
B. Folllicle stimulating hormone (FSH) E. ACTH
C. Oxytocin

12. cAMP activates which of the following enzymes?


A. Protein kinase A D. Phosphodiesterase
B. Tyrosine kinase E. Adenylyl cyclase
C. Phospholipase С

13. Which of the following is not a steroid hormone?


A. ACTH D. Testosterone
B. Aldosterone E. Estrogen
C. Cortisol

14. A patient appealed to the doctor with complaints about tremor and
hypokinesia. The biochemical analysis of blood showed the reduced
amount of dopamine. Name its methabolite-precursor.
A. Tyrosine D. Phenylalanine
B. Dioxyphenylalanine E. Phenylpyruvate
C. Tyramine

15. Thyroxine labeled with 131I is administered to a patient for the purpose
of imaging the thyroid gland. The radioactive half-life of the isotope is 8
days. The biological half-life (the time required for half of the compound to
be eliminated from the body) is 2 days. The time at which 3/4 of the original
radioactivity will no longer be detectable in the body is closest to
A. 3.2 days D. 4.8 days
B. 2.0 days E. 16.0 days
C. 4.0 days

16. Endemic goiter is known to be widespread in certain geochemical areas.


The deficiency of what chemical element causes this disease?
A. Iodine D. Copper
B. Iron E. Cobalt
C. Zinc

17. ACTH induces rise in:


A. Cyclic AMP C. Calcium
B. Cyclic GMP D. Magnesiu

18. Tetraiodothyronine is produced in the next gland:


A. Follicles of thyroid gland D. Parathyroid glands
B. Cortex of adrenals E. Langerhans islands
C. Medulla of adrenals

19. Chose a substance from listed below which inhibits production of


thyroxine:
A. 5-fluorouracil D. Thiouracil
B. Dihydrouracil E. TSH-releasing hormone
C. 5-hydroxyuracil

20. Calcitonin is hormone of polypeptide nature, which is produced in:


A. Thyroid gland D .Cortical part of adrenals
B. Incretory pancreas E. Hypothalamus
C Parathyroid gland

21. Physiological effect of calcitonin is:


A. Lowering of calcium level in blood D. Regulation of water balance
B. Decrease of blood glucose level E. Stimulation of smooth muscle
C. Stimulation of lipogenesis contraction
22. During operation on a thyroid gland parathyroid glands were removed
by mistake. The patient got titanic cramps. The metabolism of which
chemical element was disturbed?
A. Calcium D. Iron
B. Magnesium E. Sodium
C. Potssium

23. Physiological effect of parathyroid hormone (PTH) is as follows:


A. Increase of calcium level in blood D. Decrease of calcium level in
B. Stimulation of protein synthesis blood\
C. Lowering of blood glucose level E. Lipid mobilization

24. The number of amino acids in the hormoneoxytocin is:


A. 9 D. 18
B. 7 E. 20
C. 14

25. Parathyroid hormone (PTH):


A. Is a polypeptide E. Exhibits properties of
B. Is a derivative of amino acid tryptophan polysaccharide
C. Is substance of steroid structure
D. Is derivative of N-acetyl-galactosamine

26. Utilization of glucose occurs by means of sugar transport from the


extracellular matrix through the plasma membrane membrane into the cell.
What hormone stimulates this process?
A. Insulin D. Aldosterone
B. Glucagon E. Adrenaline
C. Thyroxine

27. In patient S. blood glucose level is 10 mmoles/l, polyuria, glucosuria and


ketonuria are observed. What pathological state can be suggested?
A. Diabetes mellitus D. Addison disease
B. Starvation E. Hyperthyreosis
C. Hypercorticism

28. Glucagon is produced in the next endocrine gland:


A. Langerhans islands, α cells D. Thymus
B. Parathyroid gland E. Medullar part of adrenals
C. Pitiutary gland

29. Adrenalin is a hormone, which is produced in:


A. Medullar part of adrenals D. Hypophysis
B. Langerhans islands, β cells E. Thymus
C. Cortical part of adrenals
30. Destruction of pancreatic islets of Langerhans results in the decrease of
production of:
A. Glucagon and insulin D. Insulin and adrenaline
B. Parathhormone and cortisone E. Callicrein and angiotensin
C. Thyroxyne and calcitonin

31. The next enzyme is involved in the production of leukotriens from


arachidonic acid:
A. Cyclooxygenase D. Lipoxygenase
B. Monooxygenase E. Catalase
C. Dioxygenase

32. TSH stimulates the synthesis of:


A. Thyroxine D. Insulin
B. Adrenocorticoids E. Glucagon
C. Epinephrine

33. Cyclooxygenase catalyzes the synthesis of which of the following


compounds?
A. Prostaglandines D. Porphobilinogen
B. Cyclic AMP E. Cyclic GMP
C. Leucotriens

34. The activity of cyclooxygenase can be suppressed by some medical


preparations. What preparation exhibits irreversible inhibitory action upon
this enzyme?
A. Acetylsalicylic acid D. Oligomycine
B. Insulin E. Aminalone
C. Allopurinol

35. Leucotriens are humoral factors, produced by leukocytes using as a


precursor:
A. Arachidonic acid D. Homogentisic acid
B. Nicotinic acid E. Ascorbic acid
C. Pantothenic acid

36. Thyroid hormones are synthesized by the iodination of the amino acid:
A. Glycine D. Tyrosine
B. Phenylalanine E. Valine
C. Alanine

37. Glucagon:
A. Stimulates muscle glycogenolysis D. Increases gluconeogenesis
B. Increases protein synthesis in liver
C. Inhibits lipolysis in adipocytes
38. What hormone stimulates the reabsorption of water in kidney tubules?
A. Vasopressin D. Aldosterone
B. Parathyroid hormone (PTH) E. Atrial natriuretic peptide (ANP)
C. Calcitonin

39. The primary action of steroid hormones is at the level of:


A. Transcription D. mRNA degradation
B. RNA export from the nucleus E. Gene rearrangement
C. Pre-mRNA splicing

40. The principal site of peptide neurohormone biosynthesis is the:


A. Rough endoplasmic reticulum D. Postsynaptic density
B. Nucleus E. Synaptic vesicle
C. Dendrite

41. A hormone response element is best defined by which one of the


following statements?
A. It is the DNA sequence to which D. It is the region of a
a specific hormone-receptor complex steroid hormone receptor to
binds which the hormone binds
B. It is transmembrane protein to E. It is plasma protein that
which steroid hormones bind carries a specific lipophilic
C. It is a DNA sequence to which hormone through the
steroid hormones bind bloodstream

42. What endocrine gland produces hormones regulating mineral elements


turnover?
A. Posterior pituitary D. Ovary
B. Suprarenal glands E. Anterior pituitary
C. Pancreatic gland

43. Aldosteron regulates the next physiological process:


A. Reabsorption of sodium ions in D. Stimulation of lipolysis and
exchange to potassium ions in renal oxidation of fatty acids
tubules E. Reabsorption of glucose in
B. Stimulation of gluconeogenesis convoluted tubules
C. Stimulation of sodium excretion with
urine

44. The next substance is a precursor of progesterone:


A. Cholesterol D. Phenylalanine
B. Cholic acid E. Glutathion
C. Cholecalciferol

45. G-proteins act as:


A. Hormone carriers C. Second messengers
B. Hormone receptors D. Signal transducers

46. During the operation on a thyroid gland parathyroid glands were


removed by mistake. The patient got tetanic cramps. The metabolism of
which chemical element was disturbed?
A. Calcium D. Iron.
B. Magnesium. E. Sodium
C. Potassium.

47. In the human organism the some amino acid are transformed into
hormones. In what compound tryptophan transforms in?
A. Serotonin D. Corticosteron
B. Histamine E. -alanine
C. GABA

48. After a brain hemorrhage that led to the damage of hypotalamic nuclei,
diabetes insipidus of a 67-year-old patient developed. What became the
reason of polyuria in this case?
A. Decrease of water reabsorbtion D. Hyperglycemia.
B. Decrease of potassium ions E. Hypoglycemia.
reabsorbtion.
C. Acceleration of glomerular filtration.

49. Graves disease is caused by dysfunction of the following endocrine


gland:
A. Hyperthyreosis D. Hypothyreosis
B. Adrenocortex hyperfunction E. Hyperproduction of
C. Adrenocortex hypofunction ACTH

50. Cushing’s disease, which is characterized by obesity, hypertension and


elevated blood glucose level, is caused by disorder in production and
secretion of the next hormones:
A. ACTH and glucocorticoids C. ACTH and
overproduction glucocorticoids insufficiency
B. Insulin insufficiency D. Thyroxine insufficiency
E. Estriol overproduction

51. Melanocyte stimulating hormone is secreted by:


A. Intermediate lobe of pituitary gland D. Pineal gland
B. Anterior lobe of pituitary gland E. Thyroid gland
C. Posterior lobe of pituitary gland

52. Addison’s disease is a severe disorder of sodium-potassium turnover


due to failure in production of the following hormone:
A. Aldosterone B. Thyroxine
C. Triiodothyronine E. Progesterone
D. Testosterone

53. Cretinism is caused by the next inborn endocrine disorder:


A. Inborn and persistent D. Inborn hypersecretion of
hypofunction of thyroid gland glucocorticoids
B. Hyperproduction of ACTH by E. Underdevelopment of
tumor of adenohypophysis thymus
C. Inborn hypofunction of adrenal
cortex

54. Myxoedema is manifested in adults due to the next endocrine disorder:


A. Severe hypoparathyroidism D. Underproduction of
B. Severe hypothyroidism corticosteroids
C. Insufficiency of ACTH secretion E. Hyperproduction of
corticosteroids

55. Some hormone induce uncoupling of respiration and oxidative


phosphorylation in mitochondria and lower the efficiency of ATP
production. What is this hormone?
A. Thyroxine D. Oxytocine
B. Adrenalin E. Testosterone
C. ACTH

56. Note from listed below hormones a pricipal representative of


mineralocorticoids.
A. Aldosterone D. Dihydrocorticosterone
B. Corticosterone E. Estradiol
C. Hydrocortisone

57. The next compounds are the end products of steroid hormones
catabolism, excreted with urine.
A. 17-ketosteroids, conjugated with C. Deoxycorticosterone acetate
sulfuric acid D. Hydroxymandelic acid
B. Progesterone conjugated with sulfuric E. Taurodeoxycholic acid
acid.

58. Chose from listed hormones a one with the most potent anti-
inflammatory activity:
A. Cortisol D. Megesterol
B. Aldosterone E. Progesterone
C. Testosterone propionate

59. The next substance is a precursor of progesterone:


A. Cholesterol C. Cholic acid
B.Cholecalciferol D.Phenylalanine
E.Glutathion

60. In human body some amino acids are transformed into hormones. What
compound is produced from tryptophan?
A. Serotonin D. Corticosteron
B. Histamine E. -alanine
C. GABA

61. How does glucocorticoids influence on the carbohydrate methabolism in


a liver?
A. Stimulate gluconeogenesis E. Stimulate activity of
B. Stimulate glycogenesis from glucose glycogenphosphorylase
C. Stimulate glycogen hydrolysis
D. Stimulate glycogen phosphorolysis

62. Cyclic AMP is formed from ATP by the enzyme adenylate cyclase which
is activated by the hormone:
A. Insulin D. Progesterone
B. Epinephrine E. Aldosterone
C. Testosterone

EXAMPLES OF KROK 1 TESTS


63. The formation of a secondary mediator is obligatory in membrane-
intracellular mechanism of hormone action. Point out the substance that is
unable to be a secondary mediator:
A. Glycerol D. cAMP
B. Diacylglycerol E. Ca2+
C. Inositol-3,4,5-triphosphate

64. On some diseases it is observed aldosteronism with hypertension and


edema due to sodium retention in the organism. What organ of the internal
secretion is affected on aldosteronism?
A. Adrenal glands D. Pancreas
B. Testicle E. Hypophysis
C. Ovaries

65. A typical symptom of cholera is body water loss and sodium ions loss.
The biochemical mechanism of unfavorable action of cholera toxin consists
in:
A. Activation of adenilate cyclase activity D. Stimulation of rennin secretion
of enterocytes by the cells of kidneys
B. Activation of synthesis of atrial glomerular arteriolae
natriuretic factor E. Activated oxidation of
C. Decrease of synthesis of antidiuretic aldosterone in the cells of
hormone in hypothalamus adrenal glands
66. A women with low arterial pressure after the parenteral introduction of a
certain hormone showed the essential rise of arterial pressure as well as
blood levels of glucose and lipids. What hormone was administered to the
patient?
A. Adrenaline D. Progesterone
B. Insulin E. Estradiol
C. Glucagon

67. Biologically active substances, especially hormones, are products of


hydrolysis and modification of certain proteins. From which of the listed
below proteins do lipotropin, cortcotropin, melanotropin and endorphins
appear in hypophisis?
A. Proopiomelanocortin (POMC) D. Neuroglobulin
B. Neuroalbumins E. Thyreoglobulin
C. Neurostromin

68. A 52 year-old patient with bronchial asthma was treated with


glucocorticoids. Fever reaction appeared as a result of postinjective
abscess. The patient had subfebrile temperature, which didn’t correspond to
latitude and severity of inflammatory process. Why did patient have low
fever reaction?
A. Inhibited endogen pyrogens production D. Violation of heat-producing
B. Violation of heat loss through lungs mechanisms
C. Inflammatory barrier formation in E. Thermoregulation center
injection place inhibition

69. Aspirin has antiinflammatory effect due to inhibition of the


cyclooxygenase activity. Level of what biological active acids will
decrease?
A. Prostaglandins D. Biogenic amines
B. Leucotriens E. Iodinethyronyns
C. Catecholamines

70. Increased production of thyroidal hormones T 3 and T4, weight loss,


tachycardia, psychic excitement and so on present on thyrotoxicosis. How
do thyroidal hormones effect energy metabolism in the mitochondrion of
cells?
A. Disconnect oxidation and oxidated D. Stops respiratory chain
phosphorylation E. Activates oxidated
B. Activates phosphorylation of substance phosphorylation
C. Stops phosphorylation of substance

71. Products of some proteins hydrolysis and modification are the


biologically active substances called hormones. Lipotropin, corticotropin,
melanotropin and endorphins are synthesized in the hypophysis of the
following protein:
A. Proopiomelanocortin (POMC) D. Neuroglobulin
B. Neuroalbumin E. Thyreoglobulin
C. Neurostromin

72. Utilization of arachidonic acid via cyclooxigenase pathway results in


formation of some bioactive substances. Name them:
A. Prostaglandins D. Somatomedins
B. Thyroxine E. Insulin-like growth factors
C. Biogenic amins

73. A 45 y.o. woman suffers from Cushing's syndrome - steroid diabetes.


Biochemical examination revealed: hyperglycemia, hypochloremia. Which of
the under-mentioned processes is the first to be activated?
A. Gluconeogenesis D. Glucose transport to the cell
B. Glycogenolysis E. Glycolysis
C. Glucose reabsorption

74. The patient with complaints of permanent thirst applied to the doctor.
Hyperglycemia, polyuria and increased concentration of 17-ketosteroids in
the urine were revealed. What disease is the most likely?
A. Steroid diabetes D. Type I glycogenosis
B. Insulin-dependent diabetes mellitus E. Addison's disease
C. Myxoedema

75. Pregnant women have a requirement in the promoted amount of


cholecalciferol; one of its metabolite is a powerful synergist of
parathormone, which stimulates the process of bone resorbtion and output
of calcium and phosphates in blood. name this metabolit?
A. 1,25-Dihydroxycholecalciferol D. Ergocalciferol
B. 1-hydroxycholecalciferol E. 25-Hydroxycalciferol
C. Cholecalciferol

76. A woman 47 years old complains for persistent feeling of thirst, rapid
fatigue, loss of weight. Daily diuresis is 3-4 l. Blood glucose level is 4,8
mmoles/l, in urine there is no glucose. In this case it is reasonable to
investigate blood content of:
A. Vasopressine D. Cortisole
B. Estrogens E. Thyroxine
C. Aldosterone

77. A 40-year-old woman suffers from Cushing's disease, so-called steroid


diabetes. Hyperglycemia and hypochlorinemia are biochemically exposed.
Which of the following biochemical processes is activated in the first place?
A. Gluconeogenesis. B. Glycogenolysis.
C. Reabsorbtion of glucose. E. Glycolysis.
D. Transport of glucose into cells. .

78. For analgesia, a certain substance which imitates the physiological


properties of morphine but is synthesized inside the human brain can be
used. Name this substance.
A. Endorphine. D. Calcitonin.
B. Oxytocin. E. Somatoliberin.
C. Vasopressin.

79. Laboratory testing of the patient's blood plasma showed K+ level that
is equal to 7.0 mM/1. What is the possible cause of such state?
A. Decrease of aldosterone level. E. Increase of sexual hormones
B. Increase of aldosterone level. level.
C. Decrease of thyroid hormones level.
D. Increase of thyroid hormones level.

80. A patient suffering from rheumatism was administered glucocorticoid


therapy. What changes in carbohydrate metabolism in liver can be
expected?
A. Stimulation of gluconeogenesis E. Increase of glycogen
B. Stimulation of glycogenesis phosphorylase activity
C. Stimulation of glycogen hydrolysis
D. Stimulation of glycogen phosphorolysis

81. A 35-year-old addicted to alcohol patient manifests sharp muscle and


cardiac weakness against the background of treatment by diuretics. The
patient has vomiting, diarrhea, depression, A/P - 100/60. The cause of such
state is the increased excretion through the urine of:
A. Potassium. D. Calcium.
B. Sodium. E. Phosphates.
C. Chlorine.

82. A 23-year-old patient complains of a headache, change of appearance


(increase in feet and wrists size, face features distortion). His voice grew
harsh, the memory worsened. The disease set in three years ago without
apparent causes. The analysis of the urine is without special changes. A
possible cause of this status can be:
A. Hyperproduction of somatotropin. D. Deficiency of aldosterone.
B. Deficiency of glucagon. E. Hyperproduction of
C. Deficiency of thyroxine. corticosteroids.

83. A 10-year-old boy was brought to a hospital for the inspection of the
cause of growth retardation. He had grown only by three centimeters in the
last two years. What hormone's deficiency is the cause of such state?
A. Somatotropin. B. Corticotropin.
C. Gonadotropin. E. Parathormone.
D. Thyrotropin.

84. A 35-year-old addicted to alcohol patient manifests sharp muscle and


cardiac weakness against the background of treatment by diuretics. The
patient has vomiting, diarrhea, depression, A/P - 100/60. The cause of such
state is the increased excretion through the urine of:
A. Potassium. D. Calcium.
B. Sodium. E. Phosphates.
C. Chlorine.

85. A 23-year-old patient complains of a headache, change of appearance


(increase in feet and wrists size, face features distortion). His voice grew
harsh, the memory worsened. The disease set in three years ago without
apparent causes. The analysis of the urine is without special changes. A
possible cause of this status can be:
A. Hyperproduction of somatotropin. E. Hyperproduction of corticoste-
B. Deficiency of glucagon. roids.
C. Deficiency of thyroxine.
D. Deficiency of aldosterone.

86. A patient complains of body weight loss, excessive irritability,


insignificant increase of temperature, exophtalmia. Hyperglycemia and the
rise of nitrogen-containing substances in blood serum were detected. Which
is the most credible diagnosis in this case?
A. Diffuse toxic goiter D. Tuberculosis of adrenal
B. Neurosis glands.
C. Bronzed disease E. Myxedema

87. Some compounds increase the permeability of internal membranes of


mitochondria for Н+, that results in disconnection of processes of
respirations with phosphorylation and stopping of ATP synthesis. Name this
compound:
A. Thyroxine D. Insulin
B. Vasopressin E. Oxytocin
C. Adrenalin

88. In blood of a patient a hypercalcemia, hypophosphatemia, in urine –


hyperphosphaturia is observed. What is a possible cause of this state?
A .Enhanced secretion of parathyroid D. Suppressed calcitonine
hormone secretion
B. Suppression of parathyroid hormone E. Enhanced thyroxine secretion
synthesis
C. Enhanced secretion of calcitonine
89. Arachidonic acid as essential nutrient is needed for normal growth and
developement. It is precursor of biologically active substances. Indicate
what compounds are synthesized from arachidonic acid
A . Prostaglandine E1 D. Triiodothyronine
B. Noradrenalin E. Choline
C. Ethanolamine

90. In 13 years old girl a hypotension and polyuria is observed. Preliminary


diagnosis – diabetes insipidus. It is caused by deficiency of:
A. Vasopressine D. Cortisol
B. Aldosterone E. Oxytocine
C. Adrenalin

91. Laboratory testing of the patient's blood plasma showed K+ level that
is equal to 7.0 mM/1. What is the possible cause of such state?
A. Decrease of aldosterone level
B. Increase of aldosterone level E. Increase of sexual hormones
C. Decrease of thyroid hormones level level
D. Increase of thyroid hormones level

92. Prostaglandins comprise a family of oxygenated lipid signaling


molecules derived from polyunsaturated fatty acids such as arachidonic
acid. They are involved in regulating a number of cellular processes. Some
of the prostaglandins act to increase vasodilation and levels of cAMP in
cells, whereas others increase vaso- and bronchoconstriction and smooth
muscle contraction. In the conversion of arachidonic acid to prostaglandins,
the oxygenation step is accomplished by the enzyme that synthesizes which
of the following compounds?
A. Prostaglandin H2 D. Prostaglandin D2
B. Prostaglandin E2 E. Prostaglandin I2
C. Prostaglandin F2α

93. Signaling via prostanoids begins by interaction of the prostanoid with its
receptor. The receptor involved is usually located in which part of the cell?
A. Plasma membrane of a cell near C. Endoplasmic reticulum of the cell
the cell making the prostanoid making the prostanoid
B. Nucleus of a cell in a different D. Lysosomes of a cell circulating in
organ from the cell making the the blood
prostanoid E. Golgi of a cell circulating in the
blood

CLINICAL CASES AND SITUATIONAL TASKS


94. A 40-year-old woman suffers from Cushing's disease, so-called steroid
diabetes. Hyperglycemia and hypochlorinemia are biochemically exposed.
Which biochemical process will be activated in the first place?

Answer. Cushing's syndrome is a hormone disorder caused by high levels of


cortisol in the blood. Cortisol stimulates gluconeogenesis, in particular, in the liver:
This pathway results in the synthesis of glucose from non-hexose substrates such
as amino acids and glycerol from triglyceride breakdown. Enhancing the
expression of enzymes involved in gluconeogenesis is probably the best-known
metabolic function of glucocorticoids.

95. A 33-year old patient suffers from Grave’s disease. Which of biochemical
processes will be disrupted?

Answer. Thyroid hormones increase the permeability of internal membranes of


mitochondria for Н+, that results in disconnection of processes of respirations with
phosphorylation and stopping of ATP synthesis.

96. A patient with low arterial pressure after the parenteral introduction of a
certain hormone showed the essential rise of arterial pressure as well as
blood levels of glucose and lipids. What hormone was administered to the
patient and what is the mechanism of its action?

Answer. Administration of epinephrine leads to the splitting of glycogen and fats


and the sugar and fatty acids appear in blood in large number, which are used by
tissues for the producing of warmth and energy. At the same time the epinephrine
activates the action of the cardiovascular, respiratory and muscle systems, lead
the mobilization of the reserve possibilities of the organism.

97. In 13 years old girl a hypotension and polyuria is observed. Preliminary


diagnosis – diabetes insipidus. Which hormone deficiency can cause this
disease?

Answer. There are several different types of DI, each with a different cause. The
most common type in humans is central diabetes insipidus, caused by a
deficiency of vasopressin, also known as antidiuretic hormone (ADH). The second
common type of diabetes insipidus is nephrogenic diabetes insipidus, which is
caused by an insensitivity of the kidneys to ADH. It can also be an iatrogenic
artifact of drug use.

98. A 39-year-old female presents to the clinic for a routine health


maintenance exam. The patient reports she is feeling nervous and anxious
all the time with frequent palpitations. On further questioning she reports
having diarrhea and has been losing weight. She has also noticed a change
in hair and fingernail growth and frequently feels warm while others are cold
or comfortable. She denies any history of depression or anxiety disorder
and is not taking any medications. On examination, her heart rate is 110
beats per minute. She has a slight tremor and has increased reflexes in all
extremities. A nontender thyroid enlargement is appreciated in the thyroid
region. Her TSH level is low at 0.1 mIU/mL. The patient is told that she has
an autoimmune antibody process.What is the most likely diagnosis? What
is the biochemical mechanism for this disorder?

Answer. This 39-year-old woman has symptoms of hyperthyroidism, thyroid


excess.This causes a tachycardia, tremor, nervousness, thin skin, weight loss
through the hypermetabolic state, and hyperreflexia. If unchecked, the high levels
of thyroid hormone can sometimes even cause adrenergic crisis (so called thyroid
storm), which has a high rate of mortality. Normally, the thyroid hormone
(thyroxine) is under tight control. The pituitary release of thyroid stimulating
hormone is stimulated by insufficient thyroxine, and suppressed by excess
thyroxine. In Graves disease, the most common cause of hyperthyroidism an
autoimmune immunoglobulin is produced that stimulates the TSH receptor of the
pituitary. This is confirmed by either assaying for the thyroidstimulating
immunoglobulin, or a radionuclide scan revealing diffuse increased uptake
throughout the thyroid gland.

99. The thyroid hormones T3 and T4 are synthesized in the follicular cells of
the thyroid gland. From which of the following essential amino acids are the
thyroid hormones synthesized?

Answer. Thyroid hormones are synthesized from tyrosyl residues on thyroglobulin


in the colloidal space of the thyroid follicular cells. Although tyrosine can be
obtained from the diet, it can also be synthesized from the essential amino acid
phenylalanine by the action of phenylalanine hydroxylase.

100. A 26-year-old male presents complaining of heat intolerance, heavy


sweating, tremulousness, and feeling “jittery inside.” Physical examination
reveals reddened conjunctiva and warm and moist palms, but the thyroid
gland was not visibly enlarged. Which of the following tests would be most
helpful to obtain an accurate diagnosis?

Answer. This patient appears to have a hyperthyroid condition even though the
thyroid does not appear to be enlarged. Thyroid function tests would be most
helpful to determine if this is the case. The free thyroxine level is a direct measure
of the amount of free T4, the biologically active T4, in the serum. Elevation of the
free T4 indicates a hyperthyroid condition.
SECTION IX
BIOCHEMISTRY OF THE NERVOUS TISSUE

1. Which compound is the main source of energy for the brain tissue?
А. Glucose С. Ketone bodies
В. Lactic acid D. Fatty acids
Е. Amino acids

2. The tyrosine metabolism results in synthsesis of such mediator as:


A. Dopamine D. Histamin
B.Serotonin E. γ –amunobutyric acid
C. Enkephalin

3. For the compensatory treatment of nervous system alterations (dimentia)


with simultaneous involvmentd of skin (dermatitis) and gastrointestinal tract
(diarhea) a antipellagric vitamin was administered to the patient:
А. PP D. B6
B. C E. K
C. A

4. The peculiarity of the amino acid constituents of the brain is the large
amount of monoaminodicarboxylic acids, including:
А. Glutamate, aspartate D. Serine, histidine
В. Arginin, lysine Е. Valine, tyrosine
С. Glutamine, asparagine

5. The neurotransmitters of the group of biogeneous amines include:


А. Norepinephrine, serotonin Е. Acetylcholine, choline
В. Glycine, proline
С. Glutamate, aspartate

6. The amino acids, containing “excitatory amino acids” include:


А. Glutamate, aspartate D. Histidine, lysine
В. GABA, glycine Е.Glutamine, asparagine
С. Proline, serine

7. The neurotransmitters, containing “depressive amino acids” include:


А. GABA, glycine D. Proline, lysine
В. Glutamate, glutamine Е. Histidine, tyrosine
С. Aspartate, asparagine

8. The main represantatives of opioid neuropeptides are met-enkephaline


and leu-enkephaline which chemical structure is typical for:
А. Pentapeptides D. Octapeptides
В. Tetrapeptides Е. Decapeptides
С. Hexapeptides
9. Which tissue is the most sensitive to the lack of oxygen?
A. Nervous D. Epithelial
B. Muscular E. Bone
C. Connective

10. Which metabolite is the main energy supplier for the brain?
A. Glucose D. Glycerol
B. Fatty acids E. Glycogen
C. Amino acids

11. Which types of phospholipids differently from other tissues are


neurospecific?
A. Gangliosides D. Phosphatidylserine
B. Satured fatty acids E. Unsatured fatty acids
C. Phosphatidylcholine

12. Whch of the mentioned below amno acids are the most commonly
present in the nervous tissue?
A. Glutamate and aspartate D. Valine, isoleucine
B. Glycine,proline E. Phenylalanine, threonine
C. Histamine, tyrosine

13. Which amino acid is used for ammonia detoxication in the nervous
tissue?
A. Glutamate D. Valine
B. Leucine E. Glutamine
C. Proline

14. Which depressing mediator is synthesized in nervous tissue from the


glutamate?
A. γ-aminobutyric acid D. Glutamine
B. Taurine E. Enkephaline
C.Glycine

15. The source for catecholamine biosynthesis is:


A. Tyrosine D. Glutamine
B. Tryptophan E. Methionine
C. Glutamate

16. Which enzyme hydrolizes the mediator in cholinergic sinapses?


A. Acethylcholinestherase D. Ceruloplasmin
B. Monoaminoxidase E. Xanthinoxidase
C. Histaminase

17. Which mediator is used as antistress remedue?


A. γ-aminobutyric acid D. Histamine
B. Glutamine E. Acethylcholine
C. Norepinephrine

18. Which of the following neuropeptides exerts morphin-like properties?


A. Enkephaine D. Substance P
B. Vasointestinal peptide E. Neurotensine
C. Cholecistokinine

19. Which changes of the mediators balance are induced by stress?


A. The enhancement of C. Inhibition of cathecholamine
cathecholamines synthesis and synthesis
secretion D. Inhibition of endorphine synthesis
B. Enhancement of acetylcholine E. Inhibition of glycine synthesis
synthesis

20. Which enzyme catalises the oxidative desamination of


cathecholamines?
A. Monoaminoxidase D. Acethylcholinesterase
B. Glutamate dehydrogenase E. Glutaminase
C. Cholinesterase

21. Which monooxidase inhibitors are commonly used in clinical practice as


psychopharmacologic drugs, f.i. in treatment of the depressive conditions, schzizofrenia:
А. Iprinosil, pirasidol D. Riboflavin, retinol
В. Methionine, cysteine Е. Epinephrine, serotonine
С. Heparin, carnosine

22. Inhibition of the release of what mediator into the synaptic cleft occurs
in tetanic toxin infection?
А. Glicine D. Acetylcholine
В. Norepinephrine Е. Glutamate
С. GABA

23. GABA is a depressing mediator in CNS. The inhibition of GABA-involved nervous


impulse transfer between the neuronal cells is caused by the increased permeability of
biomembrane for the ions of:
А. СІ D. Mg++
В. К+ Е. Na
С. Са++

24. What compound may be used by the CNS cells after extensive physical
exercises and prolonged starvation?
А. Acetoacetate D. Glutathione
В.Glycerol Е. Purins
С. Amino acids
25. Which neurotransmitter of CNS is synthesized from alpha-ketoglutarate in the course
of reaction of the following sequence – transamination and decarboxylation
А. GABA D. Dophamine
В. Histamine E. Serotonine
С. Tryptophan

26. What is the neurochemical background for the use of antidepressants in


the treatment of depression?
A. The increase of norepinephrine C. The increase of GABA
concentration in the brain synapses concentration in brain synapses
B. The increase of the acethylcholine D. The inhibition of cholinesterase
concentration in brain synapses activity in brain synapses
E. Stimulation of monooxidase activity

27. Which methabolic pathway is the most important for the energy supply
of the brain?
A. Aerobic oxidation of glucose D. Fatty acids oxidation
B. Anaerobic oxidation of glucose E. Ketone bodies oxidation
C. Gluconeogenesis

28. In which brain substance is the respiratory metabolism the most active?
A. Brain cortex D. Parasympathic part of the nervous
B. White matter system
C. Sympathic part of the nervous E. Synapsis
system

29. Acetylcholine is a neurotransmitter with the following mode of action


upon target cell:
A. Binding with specific C. Opening of chloride anion
cytoplasm receptor protein inside specific channels in cell membrane
the cell D. Hyperpolarization of cell
B. Binding with cell surface membrane
receptor and opening of sodium
channels

30. After appearance in synaptic cleft acetylcholine is rapidly inactivated by


the following process:
A. Decarboxylation D. Reduction with NADH2
B. Oxidative deamination E. Oxidation with peroxidase
C. Hydrolytic cleavage

31. Cholinesterase is a serine dependent enzyme extremely sensitive to the


following compound:
A. Allopurinol C. Arsenic acid salts
B. Diisopropylfluorophosphate D. Cyanides
E. Mercury chloride

32. Note the class of enzymes to which acetycholine esterase belongs:


A. Hydrolase D. Lyase
B. Oxidoreductase E. Isomerase
C. Transferase F. Ligase

33. A well known neurotransmitters epinephrine and norepinephrine are


produced from the following precursor amino acid:
A. Phenylalanine D. Glutamate
B. Histidine E. Tryptophane
C. Lysine

34. Dopamine as a specific neurotransmitter is produced from a precursor


amino acid:
A. Tyrosine D. Arginine
B. Tryptophane E. Histidine
C. Methionine

35. Parkinson’s disease is caused by a dysfunction in the following


neurotransmitter system:
A. Dopaminergic D. Serotoninergic
B. Cholinergic E. Met-enkephalinergic
C. Catecholaminergic

36. The following amino acid is a source of serotonine:


A. Tryptophane D. Tyrosine
B. Histidine E. Proline
C. Serine

37. Chose a neurotransmitter which is produced from proopiomelanocortine


after its limited proteolysis in pituitary gland:
A. ACTH D. Oxitocine
B. α -MSH E. Leu-enkephaline
C. β-Lipotropine

38. Voltage gated channels have the following functional significance:


A. Propagation of action potential C. Provide hyperpolarization of
along the axon neuronal membrane
B. Release of neurotransmitter into
synaptic cleft

39. Chose a neurotransmitter substance which is produced after


decarboxylation of glutamate:
A. Pyroglutamate C. Serotonin
B. Histamine D. Aminobutyrate, gamma-
E. Dopamine

40. Ligand gated channels are integral membrane proteins with the
following functional significance:
A. Propagation of action C. Break in the propagation of
potential along the axon action potential
B. Provide a synaptic
transmission

41. Biogenic amines with a neurotransmitter activity are rapidly inactivated


in the synaptic cleft by the action of the following enzyme:
A. Monoamine oxidase D. Superoxide dismutase
B. Peroxidase E. Glucuronyl transferase
C. FAD-dependent oxygenase

42. The energetic metabolism of brain tissue depends mainly from the
following metabolic pathways:
A. Aerobic glucose oxidation C. Ketone bodies utilization
B. Oxidation of nonesterified D. Gluconeogenesis
fatty acids E. Glycogenolysis

43. Myelin sheets of nerve fibers are composed from the sphingomyelin,
which contains the next substance as a constituent:
A. Glycerol D. Neuraminic acid
B. Phosphocholine E. Glucuronic acid
C. Galactose

44. Gangliosides are important components of neuronal plasma membrane.


They contain the following constituents EXCEPT:
A. Acetylneuraminic acid D. Lignoceric acid
B. Ceramide E. Galactose
C. Phosphoric acid

45. Cerebrosides are glycolipids of brain tissue and the following sugar is
incorporated in their structure:
A. Lactose D. Acetylneuraminic acid
B. Galactose E. Muramic acid
C. Mannosamine

46. In a brain of patients with schizophrenia the increased number of the


following neurotransmitter receptors is registered:
A. Dopamine specific D. Cholinoreceptors
B. Serotonin specific E. GABA-receptors
C. Adrenoreceptors
47. Disorders in myelinization of nerve fibers leads to severe neurologic
disorders and mental retardation. Such disorders are typical for inborn
errors in metabolism of:
A. Neutral lipids D. Glycerophosphatides
B. Cholesterol E. Lipoproteins
C. Sphingolipids

48. Cerebrosides contain all the following except:


A. Sulphate C. Sphingosine
B. Galactose D. Fatty acid

49. Cholinesterase is a serine dependent enzyme extremely sensitive to the


next compound:
A. Diisopropylfluorophosphate D. Cyanides
B. Allopurinol E. Mercury chloride
C. Arsenic acid salts

50. Fabry’s disease (one of sphingolipidoses) is an autosomal recessive


disease. Major symptoms of this disease: skin rash, kidney failure, pain in
lower extremities. It is caused by a deficiency of:
F. Hexosaminidase A and B I. α-Galactosidase A
G. Gm1 Gangliosidase J. Ceraminase
H. Galactocerebrosidase

51. A well known neurotransmitters epinephrine and norepinephrine are


produced from the next precursor amino acid:
A. Phenylalanine D. Glutamate
B. Histidine E. Tryptophane
C. Lysine

52. Cerebonic acid is present in:


A. Cerebrosides C Esterified cholestrol
B.Triglycerides D Sphingomyelin

53. Parkinson’s disease is caused by a dysfunction in the following


neurotransmitter system:
A. Cholinergic D. Serotoninergic
B. Catecholaminergic E. Met-enkephalinergic
C. Dopaminergic

54. Gangliosides are complex glycosphingolipids found in:


A. Brain D. Muscle
B. Liver E. Fat
C. Kidney
55. Chose a neurotransmitter which is produced from proopiomelanocortine
after its limited proteolysis in pituitary gland:
A. Leu-enkephaline D. β-Lipotropine
B. ACTH E. Oxitocine
C. α -MSH

56. Voltage gated channels have the next functional significance:


A. Provide hyperpolarization of C. Release of neurotransmitter
neuronal membrane into synaptic cleft
B. Propagation of action
potential along the axon

57. Chose a neurotransmitter substance which is produced after


decarboxylation of glutamate:
A. Aminobutyrate, gamma- D. Serotonin
B. Pyroglutamate E. Dopamine
C. Histamine

58. Ligand gated channels are integral membrane proteins with the
following functional significance:
A. Propagation of action C. Break in the propagation of
potential along the axon action potential
B. Provide a synaptic
transmission

59. Biogenic amines with a neurotransmitter activity are rapidly inactivated


in the synaptic cleft by the action of the next enzyme:
A. Monoamine oxidase D. Superoxide dismutase
B. Peroxidase E. Glucuronyl transferase
C. FAD-dependent oxygenase

60. NH3 is detoxified in brain mainly as:


A. Glutamine D. Asparagine
B. Uric acid E. Urea
C. Creatinine

61. Myelin sheets of nerve fibers are composed from the sphingomyelin,
which contain the next substance as a constituent:
A. Glycerol D. Neuraminic acid
B. Phosphocholine E. Glucuronic acid
C. Galactose

62. Which of the following is not a neurotransmitter?


A. Cyclic AMP D. Dopamine
B. Acetylcholine E. Epinephrine
C. Noradrenaline
63. Cerebrosides are glycolipids of brain tissue and in their structure is
incorporated the next sugar:
A. Galactose D. Acetylneuraminic acid
B. Lactose E. Muramic acid
C. Mannosamine

64. In a brain of patients with schizophrenia the increase in number of the


next neurotransmitter receptors is registered:
A. Dopamine specific D. Colinoreceptors
B. Serotonin specific E. GABA-receptors
C. Adrenoreceptors

65. Disorders in myelinization of nerve fibers leads to severe neurologic


disorders and mental underdevelopment. Such disorders are typical for
inborn errors in metabolism of:
A. Sphingolipids D. Glycerophosphatides
B. Neutral lipids E. Lipoproteins
C. Cholesterol

66. Tetanus toxin induces tonic straining of skeletal and vessel muscles
due to inhibition of secretion by nerve endings such neurotransmitter as:
A. Glycine D. Acetylcholine
B. GABA E. Glutamate
C. Norepinephrine

EXAMPLES OF KROK 1 TESTS

67. A patient, manifesting the memory decline after craniocerebral injury


came to the neurologist. By alteration of what biochemical process is this
complaint mediated?
A. Insufficient production of mediators D. Insufficient ammonia detoxification
B. Insufficient synthesis of glutamine E. Lactate accumulation
C. The decreased synthesis of GABA

68. A patient, suffering from epilepsia, presented seizures. The physician


administered him an amino acid, performing important functions in brain, in
particular participating in ammonia detoxification. It is also the source of the
depressing mediator production. What amino acid did the physician
administer to the patient?
A. Glutamate D. Tryptophan
B. Tyrosine E. Arginine
C. Methionine
69. Toxicity of ammonia is mediated by its ability to alter the tricarboxylic
acids cycle in brain mitochondria. What amino acid does ammonia exclude
from the tricarboxylic acids cycle?
А. Alpha-ketoglutarate D. Malate
В. Succinate Е. Isocitrate
С. Citrate

70. Examination of the 30-year old patient evaluated the pathologic changes
of the liver and brain. The content of cuprum in blood was acutely
decreased and in urine increased. The preliminary diagnosis was
recognized as Wilson’s disease. The activity of what enzyme will make
possible the confirmation of the diagnosis?
А. Ceruloplasmin D. Xanthinoxidase
В. Fumarase E. Leucine aminopeptidase
С.Carboangidrase

71. Which peptide neurotransmitter, produced in hypophysis through the


breakdown of highmolecular protein propiomelacortin, is bonded with
opiate receptor?
А. Met-enkephaline D. Neurotensin
В. Oxitocin Е. Vasopresin
С. ACTH

72. The examination of the patient evaluated the liver alteration,


accompanied by hyperammoniemia, hyperacidaemia, aminoaciduria and
frequent faintings. Which acid is effective for the ammonia bonding in the
brain?
А. Glutamate D. Fumarate
В. Іsocitrate Е. Oxoloacetate
С. Succinate

73. GABA is the main depressive mediator in the brain. Which drugs interact
with alosteric centers of GABA-receptor complexes and potentiate the
depressive action of GABA?
А. Benzodiazepins D. Enkephalines
В. MAO inhibitors Е. Antidepressants
С. Endorphines

74. The drug L-DOPA administered to the patient suffering from Parkinson’s
disease caused dizziness and memory decline. To decrease the side effects
of L-DOPA the physician administered lower doses of this drug and:
А. Activator of DOPA decarboxilase D. Activator of monoaminoxidase
В. Inhibitor of DOPA decarboxilase Е. Activator of diaminoxidase
С. Inhibitor of monoaminoxidase
75. Phosphororganic compounds, such as pesticides, tabun, zarin, zoman
are known to be potent poisons, causing acute excitement of the nervous
sytem. By what biochemical process is this mediated?
А. Irreversible inhibition of D. Enhanced biosynthesis of
acetylcholinesterase acetylcholine
В. Enhanced biosynthesis of Е. Decreased biosynthesis of
acetylcholinesterase acetylcholinesterase
С. Reversible inhibition of
acetylcholinesterase

76. The experimental animal with the purpose of psychic stimulation was
injected iproniazid. Iproniazid inhibits the enzyme, catalizing the
desactivation of biogenous amines. Name this enzyme:
А. Monoaminoxidase D. Aspartate aminotransferase
В. Decarboxylase Е. Amino acid oxidase
С. Diaminoxidase

77. The experimental aminal with the purpose of psychic stimulation was
injected iproniazid, catalizing the deactivation of biogenous amines. Name
coenzyme, being the constituent of this enzyme.
А. FAD D. КоАSH
В. TDP Е. NADP
С. PALP

78. The nervous-paralytical poison zarin was used in the field of fighting,
causing large human losses. What is the mechanism of action of this
poison?
А. Irreversible inhibition of D. Decreased dopamine content in
acetylcholinesterase brain synapses
В. Increased concentration of Е. Depolarisation of postsynaptic
catecholamines in the synaptic cleft membrane
С. Activation of tyrosine hydroxylase

79. The patient suffering from psychic disease refuses to consume food
within 2 weeks. Due to what compounds is his brains supplied with energy?
А. Ketone bodies D. Lactate
В. Cholesterol Е. Acetyl-KoA
С. Fatty acids

80. The feeling of fear in humans is accompanied by the synthesis of


dioxyphenylalanine in the limbic system of the brain. What compound is the
precursor of dioxyphenylalanine synthesis?
А. Tyrosine D. Lysine
В. Glutamine Е. 5-oxytriptophan
С. Triptophan
81. ɑ-Ketoglutarate enters the cycle of tricarboxylic acids in brain, avoiding
oxidative decarboxylation due to the activation of the metabolitical shunt
under the conditions of hypoxia. Which biogeneous amin plays crucial role
in this process?
А. GABA D. Dopamine
В. Tryptamin Е. Histamine
С. Serotonin

82. During narcosis the consumtion of the rich in energy compounds by


brain is acutely decreased. The content of which macroergic compound was
increased in nervous cells because of this process?
А. Kreatine phosphate D. Pyrophosphate
В. ADP Е. UDP-glucose
С. Acethyl-KoA

83. The brain activity increases the ammonia content in the nervous tissue.
What amino acid will play crucial role in ammonia bonding in this tissue?
А. Glutamate D. Isoleucine
В. Lysine Е. GABA
С. Arginine

84. A baby a fter birth manifested clonico-tonic seizures caused by food


intake, touch, different sounds. It was determined that this disease is
mediated by the lack of one of the pyridine dependent enzymes in brain
tissue. Name this enzyme.
А. Glutamate decarboxylase D. Aminotransferase
В. Cistathion synthase Е. 5-oxytryptophan decarboxylase
С. Cineruninase

85. Melatonin in human organism regulates the processes of sexual


maturation, sleep, functions of the gastrointestinal tract, cardio-vascular
system, has antistressory and anti-seizure effects. What compound
activates the synthesis of melatonin?
А. Norepinephrine D. Methionine
В. Serotonin Е. GABA
С. Dopamine

86. Ammonia toxicity for the CNS is mediated by inhibition of the


tricarboxylic acids cycle, decrease of tissue respiration, oxidative
phosphorilation, ketonaemia. Name the main pathway of ammonia
detoxification in human brain?
А. Glutamine production D. Ammonia salts production
В. Urea synthesis Е. Nitrogenous base
С. Uric acid synthesis
87. Which of the mentioned below tissues is the most sensitive to hypoxia?
А. Nervous D. Muscular
В. Epithelial Е. Bone
С. Fatty

88. What is the principle of colorimetric determination of cholinesterase


activity in blood serum?
А.Production of choline and acetic D. The production of intensive blue
acid, removing рН of the solution, coloration resulting from interaction
determined on the base of the with the Folin reagent
coloration of the phenol red indicator Е. The oxidation of
В. The colour reaction with picrine paraphenylendiamin resulting in the
acid in the alkaline medium production of coloured compound
С. The production of the scarlet colour
with iron nitrate

89. The normal value of the cholinestherase activity in blood serum is:
А. 45 – 95 µmol/l D. 98 – 115 µmol/l
В. 12 – 30 µmol/l Е. 105 – 145 µmol/l
С. 10 – 15 µmol/l

90.Tetanic toxin induces tonic strain of the skeletal muscles and vessels, inhibiting
the secretion of mediator from the nervous ending. Name this mediator.
А. Acethylcholine D. Glycine
В. Norepinephrine Е. Glutamate
С. GABA

91. In Parkinson’s disease the alteration in the dophaminergic transmission


occurs, that is why dopamine precursor L-DOPA is used for the treatment of
this disease. To decrease the side effects of L-DOPA this drug is
administered together with:
А. Inhibitor of aromatic amino acids С. Monoaminoxidase inhibitor
decarboxylase D. Monoaminoxidase activator
В. Activator of aromatic aminoacids Е. Blockers of dopamine receptors
decarboxylase

92. The number of what receptors are increased in the brain of patients,
suffering from schizophrenia:
А. Dopamine D. Choline
В. Serotonin Е. GABA
С. Epinephrine

93.The distortion of the process of myelinisation of the nervous fibers cause


severe neurologic symptoms and intellectual retardation. These clinical
signs are typical for the hereditary disturbances of metabolism of:
А. Sphingolipids В. Cholesterol
С. Neutral lipids Е. Lipoproteins
D. Glycerophospholipids

94.Monoaminoxidase inhibitors are widely used as psychopharmacological


drugs. They influence the content of all mentioned above neurotransmitters
in the brain, EXCEPT:
А. Acethylcholine D. Serotonin
В. Norepinephrine Е. Epinephrine
С. Dopamine

95. GABA is the depressing mediator of CNS. Choose the amino acid which
is the precursor for GABA synthesis:
А. Glutamate D. Tryptophan
В. Tyrosine Е. Glutamine
С. Aspartate

96. The breakdown of highmolecular protein propiomelanocortin in


hypophysis produces all mentioned above neuronal and hormonal
compounds, EXCEPT:
А. Neurotensin D. Lipotropin
В.Endorphine Е. Enkephaline
С. ACTH

97.Which statement about opioid peptides is not true?


А. Activate the adenilate cyclase D. Interacting with opioid peptides
system in neurons exhibit analgesing and euphoric
В. Are produced from the precursor effects like morphin and its analogues
proteins through the limited Е. Include enkephalines, endorphines,
proteolysis neoendorphines and dinorphinse
С. Are inactivated through the
proteolytical breakdown

98. The drugs interacting with alosteric centres of GABA-receptor


complexes and enhancing the depressive effect of GABA are:
А. Benzodiazepines
В. MAO inhibitors
С. Morphin and its analogues
D. Tricyclic antidepressants
Е. Neuropeptides
99. GABA is inactivated by:
А. Deamination D. Transamination
В. Reduction Е. Decarboxylation
С. Oxidation

100. The insufficience of some vitamins in the human organism causes


peripheral neuropathy and other neurologic symptoms. What process
inducing nervous system disturbances is being altered by the lack of
vitamin В6?
А. Transamination and D. Synthesis of amino acids and
decarboxylation of aminoacids nucleotides
В.Antioxidant defense Е. Dehydration of the tissue
С. Oxidative decarboxylation of alpha- respiration substrates
ketoacids

101.The bonding of GABA with the postsynoptic membrane receptors


cause:
А. Increased membrane permeability D. Increased membrane permeability
for Сl+ for Na+
В. Increased membrane permeability Е. Activation of adenilate cyclase
for К+ system
С. Increased membrane permeability
for Са+

102. The monoaminodicarboxylic acids, being the peculiarity of the amino


acid constituents of the brain, include:
А. Glutamate, aspartate D. Serine, histidine
В. Arginine, lysine Е. Valine, tyrosine
С. Glutamine, asparagine

103. The neurotransmitters of the group of biogenous amines include:


А. Epinephrine, serotonine D. Endorphines, enkephalines
В. Glycine, proline Е. Acethylcholine, choline
С. Glutamate, aspartate

104. The neurotransmitters containing “excitatory amino acids” include:


А. Glutamate, aspartate D. Histidine, lysine
В. GABA, glycine Е. Glutamine, asparagine
С. Proline, serine

105. The neurotransmitters called “depressing amino acids” include:


А. GABA, glycine D. Proline, lysine
В. Glutamate, glutamine Е. Histidine, tyrosine
С. Aspartate, asparagine
106. The main represantatives of the opioid peptides are met-enkephaline
and leu-enkephaline which due to their chemical structure belong to:
А. Pentapeptides D. Octapeptides
В. Tetrapeptides Е. Decapeptides
С. Hexapeptides

107. Which CNS neurotransmitter is synthesized from -ketoglutarate in the


following sequence – deamination and decarboxylation?
А. GABA D. Dopamine
В. Histamine Е. Serotonine
С. Tryptamine

108. Monoaminoxidase inhibitors are widely used as psychopharmacologic


drugs in practical medicine, f.i. for the treatment of depressive conditions
the following drug is used:
А. Iproniazid, pirazolidol D. Riboflavin, retinol
В. Methionine, cystein Е. Epinephrine,serotonine
С. Heparin, carnosine

CLINICAL CASES AND SITUATIONAL TASKS

1. The deficiency of the thiamine in the human organism results in many


neurologic symptoms: the lost of reflexes, increased excitability, dizziness.
Explain, why the thiamine deficiency has such negative influence on the
function of brain?

Answer: Glucose is the main source of energy for the brain.The main reaction in
glucose breakdown is oxidative decarboxylation of the pyruvate with the
participation of thiamine diphosphate. The lack of the latter inhibits the
degradation of pyruvate to СО2, Н2О and ATP synthesis.

2. It is known, that glycogen, being the energetic store of the organism, is


accumulated in liver and muscles, but never in brain, although the nervous
tissue consumes a lot of glucose. Why glycogen is not accumulated in
brain?

Answer: Glycogen is accumulated in brain tissue in the amount up to 0.1%. Its


increased content would induce the increase of the brain weight and intracranial
pressure, brain atrophy and death.

3. Glutamate, transported with blood to the brain tissue, is converted there


to glutamine. Concentration of glutamine in blood going from brain is
increased, Name the source of the supplementary amount of glutamine and
biochemical significance and mechanism of these transformations.
Answer: Nervous system is the most sensitive to the toxic effect of ammonia. In
brain tissue free ammonia is produced mainly as a course of hydrolytical
deamination of AMP. Glutamate plays a crucial role in ammonia excretion,
partially entering from blood and partially it is produced in the brain tissue from -
ketoglutarate as a result of transamination and reductive amination. The latter is
the most important mechanism of the ammonia detoxification in the brain.
Glutamate under the influence of highactive glutamine synthetase bonds one
more molecule of ammonia and is converted to glutamine, excreted from brain to
blood.

4. The mother of a 16-year-old female calls the clinic because of concerns


about her daughter’s eating habits. The mother states the she will not eat
anything and is obsessed with exercise and losing weight. She also states
that her daughter has been more withdrawn from friends and family. After
discussion with the mother, the patient comes in for a physical examination.
The patient is 5 ft 1 in tall and weighs 85 lb. She is in no acute distress but
appears to have a depressed affect. The patient states she is worried that
her friends will think she is fat if she eats more. She denies any binge
eating. Her physical examination is normal, other than dry skin and thin fine
hair on extremities. Laboratory tests reveal that she is anemic and has a low
albumin and magnesium level. She has normal liver and thyroid tests. What
is the most likely diagnosis? What potential medical problems may develop
in a patient with this disorder? How can this disorder affect her menstrual
cycles?
Answer: Diagnosis is anorexia nervosa. This is differentiated from bulimia
because she denies binge eating with associated guilty feelings. Medical
complications: Dry skin, lanugo, bradycardia, hypotension, dependent edema,
hypothermia, anemia, osteoporosis, infertility, cardiac failure, and even death.
Menstrual complications: Amenorrhea secondary to depression of the
hypothalamic-pituitary axis. Infertility will result secondary to anovulation. Anorexia
nervosa is a disease affecting primarily young women who have distorted body
images. Although their weight is less than 30 percent under ideal body weight,
they see themselves as overweight. Anorectics often use diuretic and laxative
agents to accomplish their weight loss. Patients with bulimia, who usually induce
emesis, may be at normal weight or even above ideal body weight; in contrast,
anorectics are almost always under ideal body weight. Often, affected individuals
become amenorrheic, have fine lanugo hair, and become hypothermic. Therapy
must be multifaceted and include family and individual counseling, behavioral
modification, and possibly medication. Severe cases may be fatal.
SECTION X
BIOCHEMISTRY OF THE MUSCULAR TISSUE

1. Anaerobic glycolysis occur under conditions of relative oxygen


insufficiency. This biochemichal process is accompanied by the pH
decrease in tissues resulting from the accumulation of:
А. Lactate D. Succinate
В. Citrate Е. Fumarate
С. Oxaloacetate

2. The fastest mechanism of ATP production for the urgent muscular


contraction is:
А. ATP generation from creatine
phosphate
В. Aerobic glycolysis
С. Anaerobic glycolysis

D. Glycogenolysis in muscles
Е. Triglicerides oxidation
3. In muscular dystrophies the production and excretion of creatinine is
inhibited. But what index is increased in urine under these conditions?
А. Creatine D. Indicanes
В. Urates Е. Bilirubin
С. Porphyrins

4. Organ-specific enzymes are evaluated for the diagnosis of some


diseases. What enzymes are determined to diagnose muscular dystrophies?
А. Aspartate aminotransferase, С. Alanine aminotransferase, urease
kreatine kinase D. Hexokinase, phosphorylase
В. Arginase, pyruvate decarboxylase Е. Fumarase, superoxide dismutase

5. The huge amount of nitrogen, derived from the amino acids, is excreted
from the human organism with urea in the form of creatinine. Which amino
acids participate in biosynthesis of creatine?
А. Glycine, arginine, methionine D.Tryptophan, lysine, alanine
В. Glycine, phenylalanine, cysteine E. Leucine, histidine, glutamine
С. Valine, serine, tyrosine

6. Indicate the right sequence of the biochemical information transfer in the


mechanism of muscular contraction?
А. Са → Troponin → Tropomyosin → D. Са → Tropomyosin → Troponin →
Actin → Myosin Actin → Myosin
В. Са → Troponin → Tropomyosin → Е. Са → Tropomyosin → Troponin →
Myosin → Actin Myosin → Actin
С. Са → Actin → Myosin → Troponin
→ Tropomyosin

7. The sportsman ran a marathon distance, resulting in a production of


lactate in his muscles. The biggest amount of this lactate:
А. Gets to blood, is caught by liver D. Is used for the glucose synthesis in
where it is converted to glucose the muscular tissue
В. Gets to mitochondrias and is Е. Is accumulated in the organism,
oxidized to pyruvate causing acidosis
С. Is excreted with urea

8. What reaction in skeletal muscles causes ATP production for the urgent
muscular contraction?
А. Creatine phosphokinase D. Glucokinase
В. Phosphofructokinase Е. Hexokinase
С. Adenylate kinase
9. A 30 year old patient manifested a decreased tolerance to physical
exercises, whereas the content of glycogen in his muscles increased. The
decrease of what enzyme caused this condition?
А. Glycogen phosphorylase D. Fumarase
В. Phosphofructokinase Е. Succinate dehydrogenase
С. Glycogen synthase

10. The main biochemical regulator of the muscular contraction and relax is
the change of the ions of:
А. Са++ D. СІ-
В. К+ Е. Na+
++
С. Mg

11. In the course of studies it was evaluated that muscular contraction


needs energy in the form of:
А. ATP D. ADP
В. GTP Е. AMP
С. CTP

12. Within short-termed muscular work with maximal activity the following
processes are the source of ATP:
А. Anaerobic breakdown of glycogen Е. Fatty acids oxidation
В. Resynthesis from creatine
phosphate
С. Anaerobic glycolysis
D. Adenilate kinase reaction

13. The fibrillar proteins include:


A. F-actin D. Tropomyosin
B. Myosin E. a- and b - actinin
C. Troponin

14. During water extraction of muscular and nervous tissue the following
compounds are released into the water solution:
A. Anserine D. Creatine
B. GABA E. Cholesterol
C. Carnosine

15. In which organs is creatine phosphate synthesized?


А. Kidneys and liver D.Pancreas
В. Adrenal glands Е. Spleen
С. Lungs

16. Which statements on the white muscular fibers are right?


А. They are characterized by С. Fatigue faster compared to the red
anaerobic carbohydrates catabolism fibers
В. Contain more glycogen than the D. Contract faster than the red fibers
red fibers Е. Contain more hemoglobinum than
the red fibers

17. Chose from listed proteins a ONE which is NOT a component of thin
filaments of sarcomere.
A. Myosine D. Troponine
B. Actin E. Nebuline
C. Tropomyosine

18. Thick filaments of sarcomere are composed from the next protein:
A. Titin D. Myosine type II
B. Tropomyosine E. α-Actinin
C. Myosine type I

19. The next statement about myosine II is correct.


A. The molecule contains one C. The molecule contains two
heavy chain and two light chains long polypeptide chains folded as
B. The molecule contains two β-pleated sheets
heavy chains and two pairs of light D. It is a globular protein
chains possessing ATP-ase activity
E. It binds calcium ions tightly

20. Which of the following does not change length during muscle
contraction?
A. The H zone D. The sarcomere
B. The A band E. The myofibril
C. The I band

21. Calcium ions binds with the highest affinity with the next myofibrillar
protein:
A. Troponin C D. Troponine I
B. Actin E. Myosine
C. Tropomyosine

22. Concerning cardiac and skeletal muscle, which statement is most


accurate?
A. Cardiac muscle contains structures skeletal myofibril nuclei are
known as intercalated discs centrally placed.
B. Cardiac muscle fibres are not D. Skeletal muscle fibres exhibit more
striated whereas skeletal muscle branching than cardiac muscle
fibres are striated. fibres
C. Cardiac Myofibril nuclei are E. None of the above
eccentrally placed whereas
23. Creatine is synthesized from the next amino acid precursor:
A. Arginine D. Glutamate
B. Alanine E. Lysine
C. Histidine

24. Smooth muscle is not cross striated because:


A. Myofibrils are not in register with C. Myofibrils are in register with each
each other other
B. Myosin and actin in the myofibril D. It has gap junctions
E. It is surrounded by a basal lamina.

25. Creatine is reversibly phosphorylated by the following macroergic


compound:
A. ATP D. NADP
B. Phosphoenolpyruvate E. UTP
C. Carbamoyl phosphate

26. The end product of creatine catabolism, which is excreted with urine is
the next compound:
A. Creatinine D. Urea
B. Cretine phosphate E. β-Alanine
C. Uric acid

27. Chose the correct statement about Cori cycle:


A. Exchange with lactate-and D. Transfer of ketone bodies
glucose between muscles and liver from liver to muscles
B. Exchange with glutamate- E. Production of glucose from
and glutamine between muscles glycogen in liver and its transport
and liver to muscles
C. Exchange with lactate-and
glycogen between muscles and
liver

28. What amino acid derivative excreted with urine may serve as indicator of
muscle tissue damage?
A. Methylhistidine D. Citrulline
B. Indolilacetate E. Isovaleric acid
C. Mandelic acid

29. For early recognition of heart muscle damage in myocardial infarction


the highest validity has increased activity of the following enzyme in blood
plasma:
A. Creatine kinase D. Aspartate aminotransferase
B. Lactatedehydrogenase E. Amylase
C. Alanine aminotransferase
30. Duchenne’s myodystrophy is hereditary disease caused by mutation of
specific gene in X chromosome. Deficiency of what protein as gene product
is involved in the mechanism of disease manifestation?
A. Dytrophin D. Troponin
B. Myosine E. Nebulin
C. Actine

31. People after prolong hypodynamia after intensive physical exercises


manifest severe pain in muscles. What is the most probable cause of such a
phenomenon?
A. Hyperproduction and D. Accumulation of creatinine in
retention of lactate in muscles muscles
B. Decrease of lipids in E. Increased proteolysis in
myocytes muscles
C. Increase of ADP in myocytes

32. Tetanus toxin induces tonic straining of skeletal and vessel muscles due
to inhibition of secretion by nerve endings such neurotransmitter as:
A. GABA D. Glycine
B. Norepinephrine E. Glutamate
C. Acetylcholine

33. What motor proteins analogous to myosin provide intracellular transport


of subcellular particles?
A. Dynein and kinesin D. Titin and nebulin
B. Actinin E. Tubulin and actin
C. Vimentin and integrin

34. For early diagnosis of muscular dystrophy the most informative is an


increase of the activity of certain enzyme in blood plasma. Note it:
A. Creatine kinase C. Aspartate aminotransferase
Lactate dehydrogenase D.E. Alfa-amylase
B. Alanine aminotransferase

35. The isozyme CK-MB is specifically increased in the blood of patients


who had:
A. Recent myocardial infarction B.C. Infective hepatitis
Skeletal muscle disease D. Myxoedema

36. The fastest regeneration of ATP from ADP for immediate muscular
contraction is achieved by the following process:
A. Transfer of phosphate group C. Glycogenolysis
from creatine phosphate to ADP D. Anaerobic glycolysis
B. Aerobic glycolysis E. Oxidation of fatty acids
37. Tetany can occur in:
A. Hypocalcaemia C. Alkalosis
B. Hypercalcaemia D. Hypocalcaemia and alkalosis

38. Thick filaments which are involved in intracellular transport and particles
migration are composed from the next protein:
A. Actinin D. Keratine
B. Collagen E. Filamin
C. Tubulin

39. Creatin phosphate is synthesized in the next cell compartment:


A. Mitochondria D. Nucleus
B. Endoplasmic reticulum E. Cell cytoplasm
C. Lysosomes

40. Which of the following statements regarding the connective tissue of a


skeletal muscle is correct?
A. The endomysium consists of fine C. The epimysium seperates each
reticular fibres surrounding the muscle fascicle from the others
muscle fibre. D. The perimysium envelopes the
B. The perimysium surrounds the whole skeletal muscle.
individual muscle fibres within a E. None of the above
fascicle

41. When skeletal muscle contracts an arrangement of the alternating light


and dark bands traversing each skeletal muscle cell changes. Which of the
following statements is not correct?
A. The space occupied by the H zone C. The light I bands will shorten
will not change D. The z lines come closer together
B. The dark A bands will remain a E. The I band consists solely of thin
constant length actin filaments

42. Chose the substance which triggers directly the contraction of myofibril
of skeletal muscle:
A. Ca ions D. cAMP
B. Creatine E. ATP
C. Creatine phosphate

43. Which of the following statements is false?


A. Smooth muscle is responsible for low force resulting in contraction of
the movement of the skeleton. the whole muscle
B. Endomysium surrounding smooth D. There are no tendons in smooth
muscle is very rich in reticular fibres muscle
C. Smooth muscle specialises for The nuclei of smooth muscle cells are
continuous contractions of relatively heterochromatic
44. In what type of muscles in contraction process is involved calmodulin ?
A. Smooth muscles C. Skeletal muscles
B. Cardiac muscle

45. What amino acid derivative excreted with urine may serve as indicator of
muscle tissue damage?
A. Methylhistidine D. Citrulline
Indolilacetate E. Isovaleric acid
C. Mandelic acid

46. The basic unit of contraction is the:


A. Sarcomer D. Actin
B. Dynein E. Z-Lines
C. Myosin

47. The next statement about myosine II is correct:


A. The molecule contains one C. The molecule contains two
heavy chain and two light chains long polypeptide chains folded as
B. The molecule contains two β-pleated sheets
heavy chains and two pairs of light D. It is a globular protein
chains possessing ATP-ase activity
E. It binds calcium ions tightly

48. After intensive physical exercises, what helps to repay the body's
oxygen debt?
A. Breathing D. Thinking
B. Stretching E. Vomiting
C. Eating

49. The isozyme CK-MB is specifically increased in the blood of patients


who had:
A. Recent myocardial infarction C. Infective hepatitis
B. Skeletal muscle disease D. Myxoedema

EXAMPLES OF KROK 1 TESTS

50. The intensive muscular work induced the significant decrease of the
buffer capacity of the blood of the worker. Which metabolite induced this
condition?
А. Lactate D. 3-Phosphoglycerate
В. 1,3-Diphosphoglycerate Е. Pyruvate
С. а-Ketoglutarate

51. A patient suffering from progressive muscular dystrophy was


administered the biochemical examination of urea. The presence of the
large amount of what compound in urea can confirm the muscular
pathology in this patient?
А. Creatine D. Urea
В.Hippuric acid Е. Creatinine
С. Porphyrins

52. Hepatitis, myocardial infarction induce an acute increase of alanine- and


aspartate aminotransferase in patient’s blood? What is the cause of the
increase of these enzymes?
A.The damage of cell membrane and C. Increased rate of amino acids
passage of the enzymes to blood breakdown in tissues
B. Increased rate of amino acid D. Lack of pyridoxin
synthesis in tissues E. Increase of the enzyme activity by
the hormones

53. Skeletal muscles and myocardium use different blood compounds as the
substrates of oxidation. Which of the following compounds is utilized in
myocardium but is not used by skeletal muscles?
А.Lactate D. Fatty acids
В. Glucose Е. Ketone bodies
С. Glycogen

54. Muscular tissue is capable to contract. This is provided by the presence


of:
А. Actin and myosin D. Thrombin
В. Fibrinogen Е. Hemoglobin
С. Creatinine

55. A 40 year old man ran 10 km within 60 minutes. Which changes of the
energetic metabolism will occur in his muscles?
A.The rate of fatty acids oxidation will C. Gluconeogenesis will increase
increase D. Glycogenolysis will increase
B. Proteolysis will increase E. Glycolysis will increase

56. McArdle's disease is characterized by the following clinical signs:


excessive glycogen accumulation in muscles, progressive myopathy,
manifested by painful seizures after muscular work, myoglobulinuria. At the
same time the lactate content in blood is not changed or decreased. The
lack of what enzyme of glycogen metabolism causes this disease?
А. Glycogen phosphorylase D. Kinase of phosphorylase
В. Adenilate cyclase Е. Protein kinase
С. Glycogen synthase

57. Normal value of this protein marker in health makes up 80 ng/l. In


myocardial infarction its concentration is acutely increased within first 2
hours and then gets significantly decreased due to excretion with urea.
Name this protein.
А. Myoglobin D. Hemoglobin
В. Tropomyosin Е. Myosin
С. Actin

58. The deficiency of myoadenilate desaminase, the enzyme catalizing


irreversible desamination of AMP to IMP induces fast fatigue during
physical exercises as well as delayed onset muscle soreness and muscular
pain. This myopathy is caused by:
А. The disturbance of purins D. The disturbance of glycogen
metabolism metabolism
В. Accessive accumulation of fatty Е. The changes of the number of
acids mitochondrias, their size and internal
С. Insufficient carnitine content structure

59. The oxidation of noncarbohydrate compounds (mainly fatty acids)


provides 65-70% of energetic needs of the myocard. Which of the free fatty
acids is the most easily oxidized in heart muscle?
А.Oleic acid D.Arachidonic acid
В.Palmitic acid Е.Linolic acid
С.Stearic acid

60. Which of the following clinical signs are not typical for the muscular
tissue pathology?
А.Glucosuria D.Decrease of cAMP
В.Myoglobulinuria Е.Significant decrease of the content
С.Creatinuria of carnosine and anserine

61. Myofibrillar proteins are the proteins providing muscular contractions.


Which of the following proteins exert ATP-ase activity?
А.Myosin D. Troponin I
В.Actin Е. Troponin С
С.Troponin T

62. The changes of the cytoplasm concentration of calcium ions are the
main biochemical regulator of the muscular contraction and relax. What
component of the troponin system is activated by the increased calcium
concentration?
А.Troponin C D.Troponine T
В.Myosin Е.Troponin I
С. Actin

63. In long-term rhythmic muscular work ATP in muscles is synthesized by


means of oxidation of:
А.Lipids В.Lactate
С.Glucose Е.Glycogen
D.Aminoacids

64. The contraction of the smooth muscles when the calcium concentration
increases up to 10µm is significantly different from the analogic process in
the barred muscles and is initiated by means of bonding of calcium to:
А.Calmodulin D.Troponin С
В.Actin Е. Troponin І
С.Myosin

65. The excessive intake of vitamin A is accompanied by increased


membrane permeability or membrane destruction with the release of the
acid proteases and acid phosphatase from the lysosomes, The excretion of
what muscular cells metabolite is indicative for their damage?
А.Creatine D.Creatinine
В.Lactate Е.Glucose
С.Pyruvate

66. What effect of magnium ions on the muscular tissue causes the wide
use of the magnium-containing drugs in clinical practice?
А.Decreases calcium concentration D. Enhances the nervous impulses
В.Activates troponin complex transfer in the synapses
С.Increases calcium concentration Е.Increases ATP and phosphate
concentration

67. The muscular sarcoplasm contains proteins, which belong to different


heterogeneal fractions: myogenous fraction contains enzymes of glycolysis
and tissue respiration enzymes, myoalbuminum and myoglobinum, the main
function of which are:
А.Oxygen deposition D.Calmodulin activation
В. Initiation of muscular contractions Е.Troponin C activation
С.Initiation of muscular relaxing

68. After short distance run untrained people develop delayed onset muscle
soreness, resulting from lactate accumulation in muscles. The enhanced activity of
what biochemical process in the organism may be associated with this condition?
А.Glycolysis D.Gluconeogenesis
В.Glycogenesis Е.Lipogenesis
С.Penthosophosphate pathway

69. The large quantity of the MB-form of creatine kinase isof orms evaluated
in the blood of patient with destructive changes of the muscular tissue.
What is the most evident diagnosis?
А.Myocardial infarction D.Polymyositis
В. Muscular atrophy С.Duchenne disease
С.Muscular dystrophy
70. A 1 year child got to clinic with the signs of muscular involvement. The
examination evaluated carnitine deficiency in muscles. The disturbance of
what process is the biochemical background for this pathology?
А.Тhe fatty acids transport to C. Substrate phosphorylation
mitochondrias D.Lactate utilization
2+
В.Regulation of Ca level in Е.Actin and myosin synthesis
mitochondrias

71. A 46-year-old female has been suffering from progressive myodystrophy


Duchenne's disease) for a long time, the change of catalytic activity of what
flood enzyme proves to be a diagnostic test for the disease?
A. Creatine kinase. D. Glutamate dehydrogenase.
B. Lactate dehydrogenase. E. Adenylate kinase.
C. Pyruvate dehydrogenase.

CLINICAL CASES AND SITUATIONAL TASKS

72. A 51-year-old male presents to the emergency center with chest pain. He
states that he has had chest discomfort or pressure intermittently over the
last year especially with increased activity. He describes the chest pain as a
pressure behind his breastbone that spreads to the left side of his neck.
Unlike previous episodes, he was lying down, watching television. The
chest pain lasted approximately 15 minutes then subsided on its own. He
also noticed that he was nauseated and sweating during the pain episode.
He has no medical problems that he is aware of and has not been to a
physician for several years. On examination, he is in no acute distress with
normal vital signs. His lungs were clear to auscultation bilaterally, and his
heart had a regular rate and rhythm with no murmurs. An electrocardiogram
(ECG) revealed ST segment elevation and peaked T waves in leads II, III, and
aVF. Serum troponin I and T levels are elevated. What is the most likely
diagnosis? What biochemical shuttle may be active to produce more
adenosine triphosphate (ATP) per glucose molecule?
Answer: Likely diagnosis is – acute myocardial infarction.Biochemical shuttle:
The malate-aspartate shuttle is primarily seen in the heart, liver, and kidney. This
shuttle requires cytosolic and mitochondrial forms of malate dehydrogenase and
glutamate-oxaloacetate transaminase and two antiporters, the malate-α-
ketoglutarate antiporter and the glutamate-aspartate antiporter, which are both
localized in the mitochondrial inner membrane. In this shuttle cytosolic
nicotinamide adenine dinucleotide (NADH) is oxidized to regenerate cytosolic
NAD+ by reducing oxaloacetate to malate by cytosolic malate dehydrogenase.
SECTION XI
BIOCHEMISTRY OF NUTRITION

1. Note substance, which activates pepsinogen to pepsin:


A. Hydrochloric acid D. Bile acids
B. Trypsin E. Adenosine triphosphate
C. Enterokinase

2. Chose the enzyme which plays an important role in production of


hydrochloric acid by parietal cells of gastric mucosa glands:
A. Carbonic anhydrase D. Cytochrome oxidase
B. Catalase E. Peroxidase
C. Pyruvate dehydrogenase

3. Which of the following is not a function of the pancreas?


A. Secretes alkaline juice which D. Secretes lipase which acts on bile-
neutralized chyme while chyme is emulsified fats to convert them to fatty
stored in the stomach acids and glycerol
B. Secretes insulin as well as E. Secretes amylase which acts on
glucagon into pancreatic duct starches to convert them to maltose
C. Secretes both endocrine and
execrine substances

4. Zymogens of proteolytic enzymes are activated by the next process:


A. Limited proteolysis side chain
B. Hydroxylation of lysine E. Phosphorylation of serine residues
C. Carboxylation of glutamic acid in protein molecule
D. Decarboxylacion of aspartic acid

5. The specificity of trypsin is cleavage of peptide bonds formed by the next


amino acid residues:
A. Carboxyl group of arginine D. Carboxyl group of glutamic acid
B. Carboxyl group of aliphatic amino E. Carboxyl group of phenylalanine
acids like valine
C. Amino groups of proline

6. A decrease in production and secretion of trypsin is observed in an


inflammatory changes in pancreatic gland. Digestion and absorption of what
substances will be impaired in this situation?
A. Proteins D. Disaccharides
B .Polysaccharides E. Lipids
C. Nucleic acids

7. Which one of the following statements is correct?


A. Trypsin can act as an activator for D. Pepsin is an endopeptidase
all zymogens of pancreatic secreted by pancreas
proteinases E. Chymotrypsin acts on peptide
B. Trypsin is an exopeptidase that bonds formed by the amino acid
liberates free amino acids glycine
C. Pepsinogen is activated by
enteropeptidase

8. The nutritional validity of proteins depends from the next factors:


A. Essential amino acids content D. Glycine and alanine content
B. Solubility in water medium E. Histidine content
C. Molecular weight of protein

9. The bile salts are:


A. Detergents for breaking up large fat C. Stimulants to pancreatic secretion
globules to small ones of enzymes
B. Enzymes for digesting food in the D. The major ingredients of gallstones
small intestines E. Reabsorbed primarily by the
gallbladder

10. An individual who consumes 100 g of protein loses 13,5 g of nitrogen in


the urine, 2 g in the feces, and 0,5 g by other routes. This individual most
evidently is:
A. A 6-year-old child D. Consuming a diet deficient in lysine
B. A 74-old woman E. A normal, healthy adult
C. A 33-old men after trauma

11. What substance is produced from tryptophan in case of protein


putrefaction in digestive tube?
A. Indole D. Putrescine
B. Cresol E. Serotonine
C. Phenol

12. The level of protein putrefaction in bowels can be evaluated by


determination of the next compound in urine:
A. Indicane D. Indoxyle
B. Scatole E. Scatoxyle
C. Indole

14. Lipids of food stuffs in digestive tract are cleaved and absorbed in
intestines. What products of lipid hydrolysis are absorbed in intestines?
A. Fatty acids D. Monosaccharides
B. Amino acids E. Polypeptides
C. Lipoproteins
15. In duodenum the digestion of carbohydrates occurs due to action of
pancreatic enzymes. What enzymes from listed below can hydrolyse 1-4
glycosidic bonds?
A. -Amylase C. Carboxypeptidase
B. Lipase D. Trypsin
E. Elastase

16. In dietary deficiency or insufficient production of endogeneous


lipotropic factors in humans is developing fat liver degeneration. What
substances from listed below can be considered as lipotropic factor?
A. Choline D. Cholegterol
B. Pyridoxine E. Fatty acids
C. Triacylglycerols

17. Selenium is an ultra trace element and is incorporated in structure of


enzyme:
A. Gluthation peroxydase D. Myeloperoxydase
B. Lactate dehydrogenase E. Pyruvate kinase
C. Catalase

18. What action does pancreatic juice have on carbohydrate digestion?


A. All of the above D. Converts lactose to glucose and
B. Converts sucrose to glucose and galactose
fructose E. Converts maltose to glucose
C. Converts polysaccharides and
starches to maltose

19. Bile acids are derivatives of:


A. Cholesterol D. Phosphatidyl choline
B. Heme E. Long chain fatty acids
C. Sphingomyeline

20. In digestion of dietary lipids there is need in one of the digestive secrets.
What secrete from listed below take part in lipids emulsification?
A. Bile D. Saliva
B. Intestinal juice E. Gastric juice
C. Pancreatic juice

21. In the course of the investigation of secretory gastric function a


hypochlorhydria was recognized. Activity of what enzyme will be decreased
in this condition?
A. Pepsin D. Amylase
B. Hexokinase E. Lipase
C. Dipeptidase
22. In the piece of gastric mucosa, excised from the patient with gastric
ulcer disease, Helicobacter pylori was detected. What enzyme can be
detected simultaneously in the tissue specimen?
A. Urease D. Aspartate AT
B. Dehydrogenase E. Carboxypeptidase
C. Alanine AT

25. Gastrin, which is secreted by stomach cells, functions to:


A. Increase the secretions of the D. Initiate the cephalic phase of
gastric glands digestion
B. Inhibit the production of E. Inhibit the secretory activities of
hydrochloric acid liver and small intestine
C. Change pepsin into pepsinogen

26. Which sequence illustrates the order of the steps from food to
biochemically useful energy?
A. Digestion, acetyl-CoA production, D. Digestion, acetyl-CoA production,
citric acid cycle, ATP production ATP production, citric add cycle
B. Citric acid cycle, digestion, acetyl- E. Digestion, citric acid cycle, ATP
CoA production, ATP production production. acetyl-CoA production
C. Digestion, citric acid cycle, acetyl-
CoA production, ATP production

27. Which of the following substances are produced from hydrolysis of


dietary fats in the small intestine?
A. Fatty acids D. Monosacchandes
B. Amino adds E. Triglycerides
C. Bile salts

28. In comparing the potential energy of lipids and carbohydrates on a


weight basis, it is correct to say that lipids provide:
A. Considerably more energy than D. Almost the same amount of energy
carbohydrates. as carbohydrates.
B. Considerably less energy than E. Slightly more energy than
carbohydrates. carbohydrates.
C. Slightly less energy than
carbohydrates.

29. The chemical digestion of __?__ begins in stomach through the action of
the enzyme __?__
A. Protein; pepsin. D. Fat; lipaae
B. Fat; bile E. Both C and D
C. Carbohydrate; salivary amylase

30. Secretin produced by the mucosa of the duodenum is stimulated by:


A. Acidity of the chyme B. Distention of the stomach
C. Large quantities of bile E. None of the above
D. A fatty chyme

31. Amino acids, monosaccharrides, glycorol and fatty acids are:


A. End products of digestion D. Enzymes
B. All hormones E. Transported in the lymph to all the
C. Products of carbohydrate lymph nodes in the body
metabolism

32. Physiological daily requirement of protein in healthy adults has the next
value:
A. 1 g per kg body weight B.0,3 g per D.5 g per kg body weight
kg body weight E.15 g per kg body weight
C. 0,1 g per kg body weight

33. Trypsinogen is produced in exocrine part of pancreatic gland and


excreted to duodenum, where it is activated by the next factor:
A. Enteropeptidase D. Gastrin
B. Chymotrypsinogen E. Cholecystopancreozymine
C. Secretin

34. Only one type of digestive juice contains carbohydrate, protein and fat-
digesting enzymes. Which one is it?
A. Pancreatic juice D. Intestinal juice
B. Saliva E. Gastric juice
C. Bile

35. Essential fatty acids include the next items:


A. Palmitic acid D. Palmitoleic acid
B. Arachidonic acid E. Succinic acid
C. Oleic acid

36. The action of bile in lipid digestion is to:


A. Emulsify fats by a detergent action D. Chemically reduce cholesterol
B. Chemically degrade triglycerides to esters to cholesterol
fatty glycerol E. A and B
C. Chemically convert triglycerides to
diglycerides

37. The cause of endemic goiter is deficiency of trace element


A. Iodine D. Iron
B. Zinc E. Maganese
C. Copper

38. Which of these is false about "gastric juice?''


A. None of the above
B. Contains the intrinsic factor for D. Becomes neutralized in the
absorption of vitamin B12 duodenum by the action of sodium
C. Is controlled in part by a hormone bicarbonate from the pancreas
called gastrin E. Is highly acidic because of its HCL
content

39. What substance is produced from tryptophan in case of protein


putrefaction in digestive tube?
A. Putrescine D. Indole
B.Cresol E.Serotonine
C.Phenol

40. What process occurs during emulgation of lipids?


A. Breakage of lipid fraction with C. Oxidation
the increase of its surface D. Reduction
B. Phosphorolysis

41. Which compound is the emulgator of lipids in the duodenum?


A. Bile acids D. Enteral juice
B. Chloride acid E. Lipase
C. Sodium bicarbonate

42. The deficiency of what enzyme is the most frequent cause of the
incomplete digestion of lipids in the digestive tract and increased content of
the neutral fat in the faeces?
A. Pancreatic lipase D. Enterokinase
B. Enteral lipase E. Gastric lipase
C. Phospholipase

43. The investigation of the gastric juice obtained from the 6 month child
evaluated the increased lipase activity. What is optimal pH for this enzyme?
A. 5,5 D. 3,2
B. 9,5 E. 7,8
C. 1,5

44. In the course of lipids digestion the products of their hydrolysis are
accumulated in blood. Which of the following compounds are increased in
blood plasm during lipids absorption?
A. Chylomicrons D. HDL
B. IDL E. LDL
C. VLDL

45. Which enzyme breakdowns intracellular lipids?


A. Trigliceride lipase D. Lipase
B. Lipoprotein lipase E. Sphyngolipase
C. Phospholipase
EXAMPLES OF KROK 1 TESTS

46. In a patient in the course of the investigation of gastric juice lactic acid
was detected. The latter was detected by:
A. Uffelman reaction D. Resorcinol test
B. Benzidine test E. Fehling test
C. Urease test

47. In a patient in the course of the clinical and laboratory investigation in


gastric juice a compound was detected, suggesting a malignant tumor in
stomach. This compound may be:
A. Lactic acid D. Rennin
B. Pepsin E. Castle intrinsic factor
C. Pepsinogen

48. In a child, consuming a meal of plant origin exclusively after some


period a growth retardation, anemia and kidney impairment were observed.
The cause of this state is deficiency in diet of the next nutrients:
A. Essential amino acids D. Mineral macroelements
B. Carbohydrates E. Carotene
C. Lipids

49. In a patient an insufficiency of enzyme production in gastric mucosa was


recognized. The investigation of what enzyme will be not informative in
gastric diseases in adults?
A. Rennin D. Gastricsin
B. Pepsin A E. Pepsin B
C. Uropepsin

50. An inborn baby after feeding with breast milk manifested dyspepsia,
vomiting. These symptoms disappeared after feeding with glucose solution.
The lack of what enzyme, participating in carbohydrates digestion, causes
these disturbances?
A. Lactase D. Amilase
B. Maltase E. Isomaltase
C. Sucrase

51. The child after consumption of fresh milk developed disorders of the
digestive tract whereas consumption of the other products, containing
carbohydrates, does not cause these symptoms. The genetically mediated
lack of what enzyme may be the reason of these disorders?
A. Lactase D. Hexokinase
B. Phosphoglucomutase E. Glucose-6-phosphate-isomerase
C. Glycogensynthase
А. FA dentist with the aim of plague prophylaxis
52. D. Fe administered a 2 year child
a medication, containing:
В.J Е. Mn
С. Br

53. A patient, living in the mountain region, has an enlarged thyroid gland.
This is most evidently caused by the deficiency of the following trace
element in food:
А. J D. Fe
В. Br Е. Mn
С. F

54. In a patient an enhanced level of total bilirubin in blood was detected, in


urine bilirubin glucuronides were detected, fecal stercobilin content was
decreased. What vitamins insufficiency can be suggested in this case?
A.B1 , B2 . D.H, N (lipoic acid)
B.PP, C E. B15 (pangamic acid).
C.K

55. In dietary deficiency or insufficient production of endogeneous


lipotropic factors in humans is developing fat liver degeneration. What
substances from listed below can be considered as lipotropic factor?
A. Choline D.Cholesterol
B.Pyridoxine E. Fatty acids
C.Triacylglycerols

56. A sick child manifested the enlargement of abdomen, curvature of lower


limbs, skul enlargement, general weakness. What nutrient insufficiency can
cause the developement of such manifestations?
A. Vitamin D D.Iron
B. Carbohydrates E.Lipids
C.Vitamin C

57. A patient complains for a loss of weight, general weakness, bleeding of


gums, loosening and hasitation and fall out of teeth. What vitamin deficiency
is observed in a patient?
A. Ascorbic acid D. Cyanocobalamine
B.Pyridoxine E.Tocoferol
C.Phylloquinone

58. In treatment of gastric ulcer besides conventional drugs it is


recommended to drink juice from fresh cabbage. What substance in this
nutritional product favors healing of ulcer?
A. Vitamin U D. Vitamin B15
B. Pyridoxine E. Vitamin P (bioflavonoids)
C. Vitamin H
59. In the piece of gastric mucosa, excised from the patient with gastric
ulcer disease, Helicobacter pylori was detected. What enzyme can be
detected simultaneously in the tissue specimen?
A. Urease D. Aspartate AT
B. Dehydrogenase E. Carboxypeptidase
C. Alanine AT

CLINICAL CASES AND SITUATIONAL TASKS

60. The investigation of the pancreatic gland secret showed the deficiency
of the pancreatic inhibitor of trypsin. What is the danger of this condition?

Answer: All pancreatic enzymes, hydrolyzing proteins, are synthesized in the


form of inactive zymogens, preserved in granules, surrounded by protein-lipid
membrane. Pancreatic secret contains pancreatic inhibitor of trypsin, which
inactivates that small amount of trypsine which may appear in it. The lack of this
protein will cause the activation of the proteolytical enzymes yet in pancreas what
will result in the damage of the gland tissue and vessels.

61. A patient was hospitalized with the complaints for diarrhea after bread
consumption. Objectively weight loss, dehydration of the organism,
adynaemia, sleep disorders were evaluated. Comprogram showed the
domination of fatty acids and soap. The evaluation of blood serum showed
the decreased content of calcium and phosphorus, decreased activity of the
alkaline phosphatase, hypoglycaemia in the fasted state and plain glucose
chart after sugar loading, acute decrease of the lipids content as well as free
and bounded cholesterol. What disease may be suspected?

Answer: Based on the patient’s complaints, objective signs and results of the
laboratory investigations celiacia may be diagnosed. This disease is characterized
by the altered intestinal absorbtion due to the intolerance to grain protein –
glyadine.

62. The clinical and laboratory examination of the patient evaluated the
presence of the lactic acid in his gastric juice. What does it indicate? What
should be recommended to the patient?

Answer: The presence of the lactic acid in gastric juice is the sign of the
enhancement of the processes of the lactate fermentation in the stomach and
proves the low concentration of the hydrochloric acid. Besides this, lactic acid is
the product of malignant cells metabolism, that is why this patient should be
advised the consultation of oncologist.

63. A patient manifests complaints for the pain in stomach, loss of apetite,
general weakness. The laboratory investigation of urine evaluated the
increased content of uropepsin. What do these changes indicate?
Answer: The content of uropepsin in urea is determined with the purpose of the
evaluation of the secretory activity of the gastric mucosa. The increase of this
index is detected in peptic ulcer disease, mainly in the involvement of the
duodenum as well as in the treatment with corticosteroids.

64. A 3-year-old boy with failure to thrive and possible hepatic failure. He
presents with hypoglycemia and recurrent episodes of nausea and vomiting
after ingestion of foods high in fructose. What is the most likely diagnosis?
What biochemical basis of this disorder?

Answer: Diagnosis: Fructose intolerance.Biochemical basis of disorder: Because


of a genetic disorder, the hepatic aldolase B enzyme is defective, and functions
normally in glycolysis but not in fructose metabolism. Glucose production is
inhibited by elevated fructose 1-phosphate. When fructose is ingested, severe
hypoglycemia results. Treatment: Avoid dietary fructose.
Individuals with a deficiency in aldolase B have the condition known as
fructose intolerance. As with most enzyme deficiencies, this is an autosomal
recessive disease; it does not cause difficulty as long as the patient does not
consume any foods with fructose or sucrose. Frequently, children with fructose
intolerance avoid candy and fruit, which should raise some eyebrows! Likewise,
they usually do not have many dental caries. However, if chronically exposed to
fructose-containing foods, infants and small children may have poor weight gain
and abdominal cramping or vomiting.

65. A 38-year-old female complains of alternating constipation and diarrhea


associated with times of stress and abdominal cramping and bloating
relieved with bowel movements. She is prescribed a cellulose dietary
supplement. What is the most likely diagnosis? What biochemical basis of
this disorder?

Answer: Diagnosis: Irritable bowel syndrome. Biochemical mechanism: Cellulose-


containing foods are not digestible but swell up by absorbing water and correlate
with larger softer stools. The increase in dietary fiber also increases the intestinal
transit time and decreases the intracolic pressure, thereby decreasing the
symptoms of irritable bowel.
Irritable bowel syndrome affects many individuals in Western countries, and it
manifests as abdominal cramping and bloating in the absence of disease. It is
thought to be caused by increased spasms of the intestines. Constipation with or
without episodes of diarrhea may be seen. Weight loss, fever, vomiting, bloody
stools, or anemia would be worrisome and should not be attributed to irritable
bowel syndrome. Typically, affected patients are anxious and may be under
stress. After ruling out other disease processes, a trial of fiber-containing foods,
stress reduction, and avoidance of aggravating foods are effective therapies.
Patients should be advised to avoid laxative use. Rarely antispasmodic or
antiperistaltic agents can be used. Notably, increased fiber in the diet may also
decrease the absorption of fats and may lower the risk of colon cancer.

66. During your medical school training, you spend some time in a pediatric
clinic in a third-world country. One of the first patients you encounter is an
8-monthold girl brought to the clinic because of excessive exhaustion and
fatigue. On further questioning of the mother, she reports that she was
previously breastfeeding but had to stop to return to work. To feed all of her
other children, she has had to dilute her formula with water to make the
formula last longer for the entire family. After your physical exam is
performed, you diagnose the infant with severe malnutrition and aid the
mother with resources to increase food intake for their household. What is
this syndrome called (with deficiencies in both calories and protein)? What
is the difference between this syndrome and kwashiorkor syndrome? What
physical findings might differentiate the two syndromes?

Answer: Diagnosis: Marasmus. Physical findings of kwashiorkor and not


marasmus: Subcutaneous fat, distended abdomen, hepatomegaly, and fatty liver.
Protein-energy malnutrition is caused by inadequate food intake or diseases that
interfere with food absorption or digestion. The two major types of malnutrition are
marasmus and kwashiorkor. In marasmus, a child usually between the ages of 1
to 3 years has inadequate caloric intake leading to loss of subcutaneous fat, loose
wrinkled skin, and either flat or distended abdomen resulting from atropic
abdominal wall muscles. Often, children are susceptible as they go from breast
milk to solid food. The affected child usually has the appearance of an “old
person’s face.” In kwashiorkor, the main issue is lack of protein, leading to edema,
sparse hair, enlarged liver, and a distended abdomen. The edema of the face and
legs is different from that of marasmus. The therapy for both of these diseases is
caloric replacement.

67. A 22-year-old soldier collapses from dehydration during maneuvers in


the desert and is sent to a military hospital. Prior to enlisting, a physician
observed a high level of glucose in his urine during an examination. At first,
he was not allowed to enlist because he was suspected of being a diabetic.
Further tests, however, determined that his insulin level was normal. A
glucose tolerance test exhibited a normal pattern. Laboratory tests following
his dehydration episode repeat the previous findings, but further testing of
the urine reveals that only D-glucose is elevated. Other sugars were not
elevated. This patient’s elevated urinary glucose and his dehydration
episode are caused by a deficiency in?

Answer: Because the patient’s liver enzymes are normal and her symptoms
seem to correlate with her intake of fruit juices, most likely her problem stems from
an inability to absorb fructose. Since removal of cow’s milk from her diet did not
eliminate the problem, a lactase deficiency can be ruled out. GLUT 5 is the
primary transporter of fructose in the intestine and a deficiency in this transporter
would lead to an inability to absorb fructose in the gut, making it a substrate for
bacterial metabolism that produces various gases, including hydrogen, as well as
organic acids.

68. A 24-year-old African-American female presents with complaints of


intestinal bloating, gas, cramps, and diarrhea following a meal including
dairy products. A lactose-tolerance test confirms your suspicion that she
had a deficiency of lactase in her intestine. Which of dairy products could
you recommend that would be least likely to cause her difficulties in the
future?

Answer: The microorganisms that convert milk to yogurt (Streptococcus salivarius


thermophilus and Lactobacillus delbrueckii bulgaricus) metabolize most of the
lactose in the milk, thus removing the source of this patient’s intestinal
disquietude. Yogurt is also a good source of dietary calcium.
SECTION XII
FUNCTIONAL ROLE OF WATER SOLUBLE AND FAT SOLUBLE VITAMINS IN
METABOLISM AND PROVIDEMENT OF CELL FUNCTIONS

1. Vitamins are:
A. Accessory food factors C. Produced in endocrine glands
B. Generally synthesized in the body D. Proteins in nature

2. A severe deficiency in nicotinic acid is the cause of the next disease:


A. Pellagra D. Scurvy
B. Rickets E. Hemorrhagic diathesis
C. Polyneuritis

3. Select the metabolic process with which is most likely associated vitamin
C:
A. Biosynthesis of collagen D. Biosynthesis of
B. Intestinal calcium absorption glycogen
C. Biosynthesis of DNA E. Oxidative
decarboxylation of α-ketoacids

4. Select the metabolic process with which thiamine is mostly associated:


A. Decarboxylation of α- D. Oxidation of fatty acids
ketoacids E. Biosynthesis of
B. Biosynthesis of collagen prothrombin
C. Biosynthesis of amino acids

5. Beri-beri is a disease caused by deficiency of the next nutritional factor:


A. Thiamine D. Nicotinic acid
B. Carotene E. Tocoferol
C. Linolenic acid

6. Biotin as a cofactor participates in the next metabolic reactions:


A. Incorporation of CO2 into D. Transfer of phosphate
molecules of metabolites groups (kinase reaction)
(carboxylation) E. Production of H2O2
B. Decarboxylation of pyruvate (oxidase reaction)
C. Oxidation of fatty acids

7. Chose from the following vitamins one which is considered as antianemic


factor.
A. Folic acid D. Ascorbic acid
B. Thiamine E. Pyridoxine
C. Pantothenic acid

8. Biochemical functions of water soluble vitamins are realized due to their


transformation to coenzymes. What coenzyme is formed by vitamin PP?
A. NAD (nicotinamide adenine C. Pyridoxalphosphate
dinucleotide) D. FMN
B. FAD (flavin adenine (flavinmononucleotide)
dinucleotide) E. Thiamine pyrophosphate

9. Which of the following symptoms would be seen in a patient with a severe


deficiency of thiamine?
A. A decreased level of C. A low level of cell
transketolase activity in red blood transaminase activity
cells D. Xerophthalmia
B. An increased clotting time of E. A decrease in blood level
blood of pyruvate and lactate

10. A prolong deficiency in cobalamine supply leads to development of the


next disease:
A. Pernicious anemia D. Scurvy
B. Hemolytic anemia E. Rickets
C. Hemorrhagic diathesis

11. Thiamine pyrophosphate is a cofactor of which of the following


enzymes?
A. Pyruvate dehydrogenase C. Citrate synthase
complex D. Arginase
B. Aminotransferase E. Succinate dehydrogenase

12. Pantothenic acid is a precursor of the next coenzyme:


A. Coenzyme A D. Coenzyme Q
B. FAD E. SAM (S-adenosylmethionine)
C. NADP

13. Para-aminobenzoic acid is believed to be an inhibitor in biosynthesis of


the next vitamin in: bacteria:
A. Folic acid D. Cobalamin
B. Biotin E. Pyridoxine
C. Pantothenic acid

14. Both Wernicke’s disease and beriberi can be reversed by administrating:


A. Thiamin D. Biotin
B. Retinol E. Vitamin B12
C. Pyridoxine

15. The Vitamin B1 deficiency causes:


A. Beri-beri D. Pellagra
B. Ricket E. Osteoporosis
C. Nyctalopia
16. Concentration of pyruvic acid and lactic acid in blood is increased due
to deficiency of the vitamin:
A. Thiamin D. Pantothenic acid
B. Riboflavin E. Biotin
C. Niacin

17. Vitamin B1 coenzyme (TPP) is involved in:


A. Oxidative decarboxylation C. Transamination
B. Hydroxylation D. Carboxylation

18. Increased glucose consumption increase the dietary requirement for:


A. Thiamin D. Biotin
B. Pyridoxine E. Riboflavin
C. Niacin

19. Riboflavin is a coenzyme in the reaction catalysed by the enzyme:


A. Acyl CoA synthetase D. Enoyl CoA dehydrogenase
B. Acyl CoA dehydrogenase E. Enoyl CoA synthetase
C. -Hydroxy acyl CoA

20. The daily requirement of riboflavin for adult in mg is:


A. 2.0–3.5 D. 4.0–8.0
B. 0–1.0 E. 9.0-10.5
C. 1.2–1.7

21. The pellagra preventive factor is:


A. Niacin D. Pyridoxine
B. Riboflavin E. Biotin
C. Pantothenic acid

22. Niacin is synthesized in the body from:


A. Tryptophan D. Aspartate
B. Tyrosine E. Valine
C. Glutamate

23. The enzymes with which nicotinamide act as coenzyme are:


A. Dehydrogenases D. Carboxylases
B. Transaminases E. Oxidases
C. Decarboxylases

24. Pantothenic acid is a constituent of the coenzyme involved in:


A. Acetylation D. Oxidation
B. Decarboxylation E. Carboxylation
C. Dehydrogenation

25. The precursor of CoA is:


A. Pantothenate D. Thiamin
B. Riboflavin E. Biotin
C. Pyridoxamine

26. Pyridoxal phosphate is central to:


A. Transamination D. Carboxylation
B. Deamination E. Oxidation
C. Amidation

27. The vitamin required as coenzyme for the action of transaminases is:
A. Pyridoxal phosphate D. Riboflavin
B. Niacin E. Ascorbic acid
C. Pantothenic acid

28. Vitamin B6 deficiency may occur during therapy with:


A. Isoniazid C. Sulpha drugs
B. Terramycin D. Aspirin

29: Biotin is a coenzyme of the enzyme:


A. Carboxylase D. Decarboxylase
B. Hydroxylase E. Deaminase
C. Dehydrogenase

30. The coenzyme required for conversion of pyruvate to oxaloacetate is:


A. Biotin D. TPP
B. FAD E. FMN
+
C. NAD

31. Consumption of raw eggs can cause deficiency of:


A. Biotin C. Riboflavin
B. Pantothenic acid D. Thiamin

32. The cofactor or its derivative required for the conversion of acetyl CoA
to malonyl-CoA is:
A. FAD D. Biotin
B. ACP E. Pantothenic acid
+
C. NAD

33. A cofactor required in oxidative decarboxylation of pyruvate is:


A. Biotin C. Pantothenic acid
B. Lipoate D. Para aminobenzoic acid

34. The central structure of B12 referred to as corrin ring system consists of:
A. Cobalt D. Iron
B. Manganese E. Sodium
C. Magnesium
35. Vitamin B12 has a complex ring structure (corrin ring) consisting of four:
A. Pyrrole rings C. Pyrimidine rings
B. Purine rings D. Pteridine rings

36. A deficiency of vitamin B12 causes:


A. Perniciuos anemia D. Rickets
B. Beri-Beri E. Pellagra
C. Scurvy

37. Folic acid or folate consists of the:


A. Base pteridine, p-amino benzoic D. Base purine, p-hydroxy
acid and glutamate benzoic acid and glutamate
B. Base pteridine, p-amino benzoic E. Base pirimidine, p-amino
acid and asparate benzoic acid and asparate
C. Base purine, p-amino benzoic acid
and glutamate

38. Folate as a coenzyme is involved in the transfer and utilization of:


A. Single carbon moiety C. Hydroxyl group
B. Amino group D. Amido group

39 Folate deficiency causes:


A. Microcytic anemia C. Iron deficiency anemia
B. Hemolytic anemia D. Megaloblastic anemia

40. Coenzyme A contains a nitrogenous base which is:


A. Adenine C. Choline
B. Guanine D. Ethanolamine

41. Chemically, lipoic acid is:


A. Sulphur containing fatty acid D. Amino acid
B. Saturated fatty acid E. Nucleic acid
C. Unsaturated fatty acid

42. Deficiency of vitamin C causes:


A. Scurvy C. Pernicious anaemia
B. Pellagra D. Beriberi

43. Retinol is produced in human body from the next precursor:


A. β-Carotene D. Ergosterol
B. Xanthophyll E. Tryptophan
C. 7-Dehydrocholeterol

44. The precursor of cholecalciferol in human is the following substance:


A. 7-dehydrocholesterol B. Carotene
C. Heme, released after D. Ergosterol
degradation of hemoglobin E. Phylloquinone

45. Vitamin K has the next physiological significance:


A. Provides the synthesis of C. Support the maturation of
prothrombine and other coagulating mucosal epithelium
factors. D. Regulates the excretion of
B. Stimulates absorption of bile in the liver.
calcium in intestines E. It is a cofactor of
decarboxylases

46. Vitamin K is a cofactor of carboxylase, which produce the next amino


acid derivative:
A. γ-Carboxyglutamate D. γ-Carboxyaspartic acid
B. Carboxyproline E. ε-Hydroxylysine
C. N-Carboxyhistidine

47. Deficiency of ergocalciferol causes development of the next disease:


A. Rickets D. Pellagra
B. Xerophthalmia E. Pernicious anemia
C. Scurvy

48. The appearance of osteoporosis in adults may be caused by deficiency


of the next vitamin:
A. Ergocalciferol D. Ubiquinon
B. Tocoferol E. Pantothenic acid
C. Phylloquinon

49. Vitamin A or retinal is a:


A. Polyisoprenoid compound C. Benzoquinone
containing cyclohexenyl ring derivative
B. Steroid D. 6-Hydroxychromane

50. -Carotene, precursor of vitamin A, is oxidatively cleaved by:


A. -Carotene dioxygenase C. Hydroxylase
B. Oxygenase D. Transferase

51. The molecule of vitamin A1 contains:


A. -Ionone ring C. -Carotene ring
B. Benzene ring D. Purine ring

52. One of the manifestation of vitamin A deficiency is:


A. Night blindness C. Loss of hair
B. Painful joints D. Thickening of long bones

53. Deficiency of Vitamin A causes:


A. Xeropthalmia
B. Hypoprothrombinemia
C. Megaloblastic anemia
D. Pernicious anemia
E. Beriberi
54. Retinal is a component of:
A. Rhodopsin D. Iodopsin
B. Cardiolipin E. Nucleoprotein
C. Glycoproteins

55. The most potent Vitamin D metabolite is:


A. 1,25-Dihydroxycholecalciferol D. 7-Dehydrocholesterol
B. 25-Hydroxycholecalciferol E. 1- Dehydrocholesterol
C. 24, 25-Dihydroxycholecalciferol

56. Deficiency of vitamin D causes:


A. Ricket and osteomalacia D. Skin cancer
B. Tuberculosis of bone E. Beriberi
C. Hypthyroidism

57. Calcitriol synthesis involves:


A. Both liver and kidney D. Muscle
B. Intestine E. Brain
C. Adipose tissue

58. The most important natural antioxidant is:


A. Vitamin E D. Vitamin K
B. Vitamin D E. Vitamin A
C. Vitamin B12

59. Vitamin K is involved in posttranslational modification of the blood


clotting factor by acting as cofactor for the enzyme:
A. Carboxylase D. Oxidase
B. Decarboxylase E. Dehydrogenase
C. Hydroxylase

60. Vitamin K is a cofactor for:


A. Gamma carboxylation of C. Formation of -amino butyrate
glutamic acid residue D. Synthesis of tryptophan
B. -Oxidation of fatty acid

EXAMPLES OF KROK 1 TESTS

61. The content of vitamin PP is very low in milk and eggs, never the less
these products have antipellagric action. It is caused by high content of
precursor of this vitamin in mentioned products, namely:
A. Tryptophan D. Adenine
B. Riboflavin E. GDP
C. Thiamine
62. In an ill child the enlargement of abdomen, curvature of lower limbs, skul
enlargement, general weakness is observed. What nutrient insufficiency can
lead to the developement of such manifestations?
A. Vitamin D D. Iron
B. Carbohydrates E. Lipids
C. Vitamin C

63. A patient complains for a loss of weight, general weakness, bleeding of


gums, loosening and hasitation and fall out of teeth. What vitamin deficiency
is observed in a patient?
A. Ascorbic acid D. Phylloquinone
B. Cyanocobalamine E. Tocoferol
C. Pyridoxine

64. A patient complains for a loss of possibility to discriminate things in


twilight, yet in daylight he see normally. The cause of this can be:
A. A hypovitaminosis D. PP hypovitaminosis
B. A hypervitaminosis E. D hypervitaminosis
C. Glucose –6-phosphate
defeciency

65. In a patient with frequent intraorgan and mucosal bleeding in urine were
detected proline and lysine. Deficiency of what vitamin causes a damage of
their hydroxylation?
A. Vitamin C D. Vitamin D
B. Vitamin A E. Vitamin E
C. Vitamin K

66. In a patient were detected symptoms of skin lesions (dermatitis),


digestive disorders (diarrhea), nerve system disfunction (dementia). The
cause of named disorders can be:
A. Hypovitaminosis PP D. Hypovitaminosis B1
B. Hypervitaminosis D E. Hypovitaminosis B6
C. Hypervitaminosis A

67. A patient complains for loss of apetite, fall down of hair, general body
exhaustion, conjunctivitis. From an anamnesis it was recognized, that
patient consumed fish oil. Excess of what vitamin can be suspected in this
case?
A. Vitamin A D. Vitamin H
B. Vitamin E E. Vitamin D
C. Vitamin C

68. After several months in polar expedition in a person appeared the next
symptoms: bleeding of gums, multiple tooth decay (caries), loss of hair,
skin hemorrhages, headache and general weakness. What disease can be
suggested?
A. Scurvy D. Pernicious anemia
B. Polyneuritis E. Pellagra
C. Beri-beri

69. In a patient with symptoms of acidosis (pH lowering in blood ) in urine


was detected significant quantity of methylmalonic acid. This is caused by
insuficiency of the next vitamin:
A. B12 D. C
B. B2 E. D
C. B5

70. Hydroxyproline is an essential amino acid in the collagen structure.


Which of the following vitamins takes part in the formation of this amino
acid by the proline hydroxylation pathway?
A. C. D.B 5
B. D. E.B1
C. B 3
71. Vitamin therapy is appointed to a pregnant with a few involuntary
abortions in anamnesis. What vitamin contributes to the bearing of
pregnancy?
A. α-Tocopherol. D. Pyridoxine
B. Ascorbic acid. E. Rutin.
C. Cobalamine

72. A child manifests epileptic seizures caused by vitamin B6 deficiency.


This is conditioned by the decrease of the 7-aminobutyrate level in the
nervous tissue which acts as an inhibiting neurotransmitter. The
activity of which enzyme is decreased in this case?
A. Pyridoxal kinase. D.Glutamate decarboxylase.
B. Alanine aminotransferase. E. Glutamate synthetase
C. Glutamate dehydrogenase.

73. An ophthalmologist found that an outpatient has an increase in the time


of sight adaptation for darkness. What kind of vitamin deficiency can be the
cause of the symptom?
A. A. D. K.
B. E. E. D.
С. С

74. A patient suffers from dermatitis, diarrhea and dementia. The absence of
which vitamin is the cause of such clinical state?
A. Nicotinamide. D. D.Biotin.
B. Ascorbic acid. E. Rutin.
C. Folic acid.
75. A patient was diagnosed with megaloblastic anemia. The lack of which
substance in the human organism g cause this disease?
A. Copper. D. Cholecalciferol.
B. Glycine. E. Magnesium.
C. Cobalamine.

76. A patient with frequent bleedings inside the internal organs and mucous
membranes was found having proline and lysine in the collagen fibers
composition. What vitamin absence results in the disturbance of these
amino acids hydroxylation?
A. C. D. A.
B. Е. E. D.
C. К.

77. According to the clinical signs, pyridoxal phosphate was prescribed to a


patient. For the correction of what biochemical processes is this drug
recommended? -
A. Transamination and decarboxylation of C. Deamination of amino acids.
amino acids. D. Synthesis of purines and
B. Oxidative decarboxylation of pyrimidines.
ketoacids. E. Protein synthesis.

78. A patient was diagnosed with dermatitis as a result of prolonged


consumption of uncooked eggs. What vitamin deficiency developed in this
case?
A. Biotin. D.Para-amino benzoic acid.
B. Folic acid. E. Vitamin C.
C. Pantothenic acid.

79. There is an increase in the pyruvate level in the patient's blood and
urine. What kind of avitaminosis developed in this case?
A. B1 avitaminosis. D. B2 avitaminosis.
B. Е avitaminosis. E. B12 avitaminosis.
C. B3 avitaminosis.

80. Under different pathological states the level of active forms of oxygen
rises, which results in the destruction of cellular membranes. In order to
prevent the damage of membranes, antioxidants are used. The most
powerful natural antioxidant is:
A. a-Tocoferol. D.Fatty acids.
B. Glucose. E. Glycerol.
C. Vitamin A.

81. Most participants of the round-the-world voyage of Magellan perished


from avitaminosis that was manifested by general weakness, hypodermic
hemorrhages, loss of teeth, bleeding of gums. Name the disease which
develops as a result of this avitaminosis.
A. Scurvy. C. Rachitis. D.Polyneuritis.
B. Pellagra. E. Birmer's anemia.

82. During the patronage a doctor revealed that a child had symmetric
roughness of skin on his cheeks, diarrhea, disturbance of nervous activity.
The deficiency of what food factors caused the appearance of such
symptoms?
A. Nicotine acid, tryptophan. D.Methionine, lipoic acid.
B. Lysine, ascorbic acid. E. Phenylalanine, pangamic acid.
C. Threonine, panthothenic acid.

83. The Institute of gerontology recommends the aged people to take a


complex of vitamins that contains vitamin E. What is the basic biological
action of vitamin E?
A. Antioxydant. D. Antineuritic.
B. Antihemorrhagic. E. Antidermatitic.
C. Antiscurving.

84. A physician recommended a patient, who suffered from ulcer of


duodenum, to drink daily juices made of cabbage and potatoe. What
substances contributing to the prophylaxis and cicatrization of ulcers do
these vegetables contain?
A. Vitamin U. D. Vitamin B6.
B. Panthothenic acid. E. Vitamin K.
C. Vitamin C.

85. The treatment of a child, who suffers from rickets, with vitamin D 3 proved
to be unsuccessful. Which is the most likely cause of treatment
inefficiency?
A. Disturbance of hydroxylation of vitamin D. Increased consumption of
D3 vitamin D3 by microorganisms of
B. Insufficiency of lipids in food. intestines.
C. Disturbance of insertion of vitamin D3 E. Disturbance of vitamin D3
into the molecule of enzyme. transport by the proteins of blood.

86. After prolonged treatment of a patient with antibiotics, the suppression


of intestinal microorganisms occurred. What kind of hypovitaminosis can
resuit from this treatment?
A. B6. D. P.
B. C. E. D.
C. A.
87. Malignant hyperchromc anemia, or Birmer's disease, is a pathological
state caused by the deficiency of vitamin B 12. What chemical element is a
constituent of the structure of this vitamin?
A. Cobalt. D. Iron.
B. Molybdenum. E. Magnesium.
C. Zinc.

88. After the resection of a 2/3 part of the stomach, the amount of
erythrocytes in the blood of a patient has decreased, their volume has
multiplied, and the level of hemoglobin has decreased. The deficiency of
what vitamin causes such changes of blood contents?
A. PP. D. B12
B. C. E. D.
С. P.

89. The simultaneous disorder of the human reproductive function and the
dystrophy of skeletal muscles are caused by the deficiency of:
A. Vitamin E. D. Vitamin D.
B. Vitamin A. E. Vitamin B6.
C. Vitamin K.

90. For diagnostics of certain illnesses the determination of blood


transaminases activity is required. Which vitamin is a component of the
cofactors of the enzymes?
A. B6. D. B8.
B. B12. E. B5.
C. B3.

91. Pyridoxal phosphate was prescribed to a patient according to the clinical


indication. For the correction of what biochemical pathway is this drug
recommended?
A. Transamination and decarboxylation of D. Synthesis of purine and
amino acids. pirymidine bases.
B. Oxidative decarboxylation of E. Protein synthesis.
ketoacids.
C. Deamination of purine nucleotides.

CLINICAL CASES AND SITUATIONAL TASKS

92. What is the active form of thiamine and how it is formed?

Aswer: Thiamine is mainly the transport form of the vitamin, while the active
forms are phosphorylated thiamine derivatives. There are five known natural
thiamine phosphate derivatives: thiamine monophosphate (ThMP), thiamine
diphosphate (ThDP), also sometimes called thiamine pyrophosphate (TPP),
thiamine triphosphate (ThTP), and the recently discovered adenosine thiamine
triphosphate (AThTP), and adenosine thiamine diphosphate (AThDP).
The ThDP, also known as thiamine pyrophosphate (TPP) or cocarboxylase, is
catalyzed by an enzyme called thiamine diphosphokinase according to the
reaction thiamine + ATP → ThDP + AMP (EC 2.7.6.2). ThDP is a coenzyme for
several enzymes that catalyze the transfer of two-carbon units and in particular
the dehydrogenation (decarboxylation and subsequent conjugation with coenzyme
A) of 2-oxoacids (alpha-keto acids). The enzymes transketolase, pyruvate
dehydrogenase (PDH), and 2-oxoglutarate dehydrogenase (OGDH) are all
important in carbohydrate metabolism. The cytosolic enzyme transketolase is a
key player in the pentose phosphate pathway, a major route for the biosynthesis
of the pentose sugars deoxyribose and ribose. The mitochondrial PDH and OGDH
are part of biochemical pathways that result in the generation of adenosine
triphosphate (ATP), which is a major form of energy for the cell. PDH links
glycolysis to the citric acid cycle, while the reaction catalyzed by OGDH is a rate-
limiting step in the citric acid cycle. In the nervous system, PDH is also involved in
the production of acetylcholine, a neurotransmitter, and for myelin synthesis.

93. A 47-year-old female is brought to the emergency department with


complaints of malaise, nausea and vomiting, and fatigue. The patient
reveals a long history of alcohol abuse for the last 10 years requiring drinks
daily especially in the morning as an “eye opener.” She has been to rehab
on several occasions for alcoholism but has not been able to stop drinking.
She is currently homeless and jobless. She denies cough, fever, chills,
upper respiratory symptoms, sick contacts, recent travel, hematemesis, or
abdominal pain. She reports feeling hungry and has not eaten very well in a
long time. On physical exam she appears malnourished but in no distress.
Her physical exam is normal. Her blood count reveals a normal white blood
cell count but does show an anemia with large red blood cells. Her amylase,
lipase, and liver function tests were normal. What is the most likely cause of
her anemia? What is the molecular basis for the large erythrocytes?

Answer: Cause of anemia of this 47-year-old alcoholic female is folic acid


deficiency. Abnormal proliferation of erythroid precursors in the bone marrow,
since folate deficiency encumbers the maturation of these cells by inhibition of
deoxyribonucleic acid (DNA) synthesis. Folate (folic acid) is an essential vitamin
which, in its active form of tetrahydrofolate (THF), transfers 1-carbon groups to
intermediates in metabolism. Folate plays an important role in DNA synthesis. It is
required for the de novo synthesis of purines and for the conversion of
deoxyuridine 5-monophosphate (dUMP) to deoxythymidine 5-monophosphate
(dTMP). Additionally, folate derivatives participate in the biosynthesis of choline,
serine, glycine, and methionine. However, in situations of folate deficiency,
symptoms are not observed from the lack of these products as adequate levels of
choline and amino acids are obtained from the diet. Folate deficiency results in
megaloblastic anemia. Megaloblastic anemia is characterized by macrocytic
erythrocytes produced by abnormal proliferation of erythroid precursors in the
bone marrow. Folate deficiency encumbers the maturation of these cells by
inhibition of DNA synthesis. Without an adequate supply of folate, DNA synthesis
is limited by decreased purine and dTMP levels. Folate deficiency perturbs DNA
metabolism and methylation reactions. In cases of chronic alcoholism, folate
deficiency may result from poor nutrition or from poor absorption of folate
secondary to a conjugase deficiency. Once folate deficiency occurs, abnormal
megaloblastic replication of epithelial mucosa can occur, which further impairs
folate uptake.

94. Because of the close interrelationship between the vitamins, patients


with deficiencies of either folate or vitamin B 12 exhibit similar symptoms.
Which of the following tests would best help distinguish between a folate
and vitamin B12 deficiency?
Answer: Vitamin B12 is a cofactor in two biochemical reactions, the conversion of
homocysteine to methionine by the enzyme methionine synthase and the
conversion of L-methylmalonyl-CoA to succinyl- CoA by methylmalonyl-CoA
mutase. N5-methyl THF is a methyl donor in the methionine synthase reaction. A
folate deficiency would result in decreased methionine synthase activity and
decreases in methionine and cystathionine concentrations, while homocysteine
levels would be increased. A vitamin B 12 deficiency would also yield these same
results, but in addition methylmalonate levels would increase as a consequence of
a decrease in the activity of methylmalonyl-CoA mutase activity.

95. A muscular 25-year-old male presents with dermatitis and an inflamed


tongue. A history reveals that he has been consuming raw eggs as part of
his training regimen for the past 6 months. Select the vitamin that is most
likely to be deficient.

Answer: Raw eggs contain a protein, avidin, which binds biotin strongly. Because
native avidin is resistant to hydrolysis by digestive proteases,
when it binds biotin it prevents its absorption. Avidin that has been denatured by
cooking will be broken down during the digestive process. A biotin deficiency
manifests itself as an erythematous, scaly skin eruption and can also cause hair
loss and conjunctivitis. A biotin deficiency can also occur following prolonged total
parenteral nutrition if biotin is not supplemented.

96. A 30-year-old male goes to his dentist complaining of loosening teeth.


Examination also reveals his gums are swollen, purple, and spongy. The
dentist also notes that the patient’s fingers have multiple splinter
hemorrhages near the distal ends of the nail and that a wound on the
patient’s forearm has failed to heal properly. Select the vitamin that is most
likely to be deficient.

Answer: The patient exhibits the classic symptoms of scurvy, a deficiency in


vitamin C. In addition to being an important biological antioxidant, ascorbic acid is
required for the hydroxylation of proline and lysine residues of procollagen in the
synthesis of collagen. A deficiency leads to defects in collagen synthesis, which
adversely affects the intercellular cement substances in connective tissue, bones,
and dentin.

97. A 59-year-old male is brought to the emergency department by the EMS


after a family member found him extremely confused and disoriented, with
an unsteady gait and strange irregular eye movements. The patient has
been known in the past to be a heavy drinker. He has no known medical
problems and denies any other drug usage. On examination, he is afebrile
with a pulse of 110 beats per minute and a normal blood pressure. He is
extremely disoriented and agitated. Horizontal rapid eye movement on
lateral gaze is noted bilaterally. His gait is very unsteady. The remainder of
his examination is normal. The urine drug screen was negative and he had a
positive blood alcohol level. The emergency room physician administers
thiamine. What is the most likely diagnosis? What is importance of thiamine
in biochemical reactions?

Answer: Most likely diagnosis in this case is Wernicke-Korsakoff syndrome


(thiamine deficiency) often associated with chronic alcoholics. Thiamine deficiency
is uncommon except in alcoholics as a result of nutritional deficiencies and
malabsorption. The classic clinical triad of dementia, ataxia (difficulty with
walking), and eye findings may be seen, but more commonly, only forgetfulness is
noted. Sometimes, thiamine deficiency can lead to vague symptoms such as leg
numbness or tingling. Because thiamine is water soluble, it can be added to
intravenous fluids and administered in that way. Other manifestations include beri
beri, which is cardiac involvement leading to a high cardiac output, and
vasodilation. Affected patients often feel warm and flushed, and they can have
heart failure.

98. A full-term female infant failed to gain weight and showed metabolic
acidosis in the neonatal period. A physical examination at 6 months showed
failure to thrive, hypotonia, small muscle mass, severe head lag, and a
persistent acidosis (pH 7.0 to 7.2). Blood lactate, pyruvate, and alanine were
greatly elevated. Treatment with thiamine did not alleviate the lactic
acidosis. Which of the following enzymes is most likely deficient in this
patient?

Answer: The increased concentrations of pyruvate, lactate, and alanine indicate


that there is a block in the pathway leading from pyruvate toward the TCA cycle. A
deficiency in pyruvate dehydrogenase would lead to a buildup of pyruvate.
Pyruvate has three fates other than conversion to acetyl-CoA by pyruvate
dehydrogenase: conversion to oxaloacetate by pyruvate carboxylase, reduction to
lactate by lactate dehydrogenase, and transamination to the amino acid alanine.
Thus, because pyruvate builds up, an increase in lactate and alanine would be
expected if pyruvate dehydrogenase was deficient.
99. A 3-month-old male infant developed seizures and progressively
worsened, showing hypotonia, psychomotor retardation, and poor head
control. He had lactic acidosis and an elevated plasma pyruvate level, both
more than seven times the normal amount. Pyruvate carboxylase activity
was measured using extracts of fibroblasts and was found to be less than 1
percent of the normal level. Oral administration of which of the following
amino acids would you recommend as the best therapy for this patient?

Answer: A deficiency in pyruvate carboxylase results in a diminution of


oxaloacetate, the C4 acid that acts as the acceptor for an acetyl group from acetyl-
CoA. In order for the TCA cycle to continue efficiently, C4 acids must be
replenished. Amino acids whose carbon skeletons feed into the TCA cycle and
increase the C4 pool will accomplish this. Glutamine, which is converted to α-
ketoglutarate, will lead to an increase in all of the C 4 acids (succinate, fumarate,
malate, and oxaloacetate). Alanine and serine are converted to pyruvate, which as
a result of the deficiency in pyruvate carboxylase will not increase the C4 pool.
Lysine and leucine are ketogenic amino acids and thus also do not increase the
C4 pool.

100. A deficiency in thiamine (vitamin B1) would most likely lead to which
clinical manifestations?

Answer: In addition to being an important cofactor for the enzymes involved in the
oxidative decarboxylation of pyruvate, α-ketoglutarate, and branched-chain α-
ketoacids, thiamine is also a cofactor for the enzyme transketolase, the enzyme
that transfers a glycoaldehyde group from a ketose sugar to an aldose sugar in
the pentose phosphate pathway. One of the diagnostic tools in determining a
thiamine deficiency is determination of the activity of red blood cell transketolase
in the presence and absence of added thiamine. A thiamine deficiency would be
expected to increase blood lactate concentrations. A deficiency of biotin would
lead to decreased carboxylase activity, whereas an increased methylmalonate
concentration would be observed with a deficiency in vitamin B 12. A deficiency in
vitamin K would lead to an increase in prothrombin time.
SECTION XIII
BIOCHEMISTRY AND PATHOBIOCHEMISTRY OF BLOOD

1. Fibrinogen is transformed into fibrin monomer by the next biochemical


process:
A. Limited proteolytic cleavage D. Carboxylation of glutamic
B. Phosphorylation with involvement acid residues
of ATP and proteinkinase E. Acetylation with involvement
C. Dephosphorylation by protein of acetyl-CoA.
phosphatase

2. For extrinsic pathway of blood coagulation the next factor is necessary:


A. Proconvertin (VII) E. Plasma thromboplastine
B. Hageman factor (XII) antecedent (XI)
C. Christmas factor (IX)
D. Kallicreine

3. Light chains of immunoglobulins are of following types:


A. Kappa and lambda D. Lambda and delta
B. Alpha and kappa E. Alpha and beta
C. Alpha and gamma

4. Prothrombin is activated and transformed to thrombin by an active form


of the next intrinsic factor:
A. Plasma thromboplastin D. Hageman factor
antecedent (X) (XII)
B. Convertin (factor VII) E. Christmas factor
C. Antihemophilic globulin A (IX)
(VIII)

5. Hemophilia A is caused by deficiency of the next blood coagulation


factor:
A. Antihemophilic globulin A D. Tissue thromboplastin
B. Proconvertin E. Christmas factor
C. Proaccelerin

6. Immunoglobulins are classified on the basis of:


A. Type of heavy chains D. Molecular weight
B. Type of light chains
C. Types of light and heavy chains

7. Post translational modification of coagulation factors and formation of γ-


carboxyglutamic acid has the next functional significance:
A. Increases the affinity to Ca ions C. Induces formation of
B. Decreases the affinity to Ca ions additional ionic bonds in
peptide chain
D. Induces appearance of Mg ion E. Provides cross linking of
ligands in protein molecule fibrin filaments

8. The molecular weight of heavy chains of immunoglobulins is:


A. 50,000–70,000 Da D. 70,000–1,00,000 Da
B. 20,000–25,000 Da E. 1,00,000-5,00,000 Da
C. 25,000–50,000 Da

9. Thrombin belongs to the next class of enzymes:


A. Hydrolases D. Lyases
B. Oxido-reductases E. Isomerases
C. Transferases F. Ligases

10. Plasminogen is activated and transformed to the active form plasmin by


the following enzyme:
A. Urokinase D. FSF (fibrin stabilization
B. Thrombin factor)
C. Thrombokinase E. Ca ions

11. The most abundant immunoglobulin in plasma is:


A. IgG D. IgD
B. IgA E. IgE
C. IgM

12. Allergic reactions are mediated by:


A. IgE D. IgD
B. IgA E. IgM
C. IgG

13. C1 component of classical complement pathway is made up of:


A. Complements 1q and 1r D. Complements 1q, 1r and
B. Complements 1q and 1s 1s
C. Complements 1r and 1s

14. Which from listed below immunoglobulins is secretoty and inhibits


adsorption and growth of bacteria on mucosa surfaces?
A.IgA D. IgE
B. IgM E. IgG
C. IgD

15. IgG cleaved by papain into:


A. Two Fab and one Fc fragments D. One Fab and two Fc
B. Two light and two heavy chains fragments
C. Two pairs of one light and one
heavy chain each
16. Severe combined immunodeficiency (SCID) is caused by congenital
defect of the next enzyme:
A. Adenosine deaminase D. Adenosine phosphorylase
B. Alanyl aminotransferase E. AMP nucleotidase
C. Transketolase

17. Chose the symbol of heavy chain type in immunoglobulin M:


A. μ D. λ
B. β E. α
C. γ

18. Immunoglobulin M molecule possesses the next quaternary structure:


A. It is pentameric molecule D. It is tetrameric molecule
B. It is dimeric molecule E. It is monomeric molecule
C. It is trimeric molecule

19. The antibody class which can pass through the placenta to protect the
fetus is
A. Immunoglobulin G (IgG) D.Immunoglobulin D (IgD)
B. Immunoglobulin M (IgM) E.Immunoglobulin E (IgE)
C. Immunoglobulin A (IgA)

20. The minimum number of polypeptide chains in an immunoglobulin is:


A. Four D. Six
B. Two E. Seven
C. Five

21. Cross linking of fibrin monomers in the filament and formation of tight
clot is provided by the next factor:
A. FSF (fibrin stabilization C. Thromboplastin
factor) D. Convertin
B. Plasmin E. Accelerin

22. Chose from listed below factors one of the extrinsic pathway of blood
coagulation:
A. Proconvertin D. Hageman factor
B. Prothrombine E. Proaccelerin
C. Antihemophilic globulin A

23. Chose from presented vitamins one with antihemorrhagic activity:


A. Philloquinone D. Ergocalciferol
B. Retinol E. Pangamic acid
C. Tocopherol

24. Vitamin K serves as a cofactor in the next enzymatic reaction:


A. Carboxylation of glutamic acid side D. Phosphorylation of serine or
chain threonine
B. Decarboxylation of glutamic or aspartic E. Amidation of glutamic acid side
acid side chain chain
C. Limited proteolytic cleavage of
zymogens

25. The molecular weight of light chains of immunoglobulins is:


A. 20,000–25,000 D. 50,000–75,000 Da
B. 10,000–15,000 Da E. 75,000-100,000 Da
C. 25,000–50,000 Da

26. Secretory component is present in:


A. IgA D. IgD
B. IgG E. IgE
C. IgM

27. The components of complement system are activated by:


A. Phosphorylation C. Glycosylation
B. Microsomal hydroxylation D. Proteloysis

28. Chose the symbol of light chain type of immunoglobulins:


A. λ D. γ
B. α E. μ
C. β

29. Hemophilia B is caused by deficiency of the next blood coagulation


factor:
A. Christmas factor (factor IX) D. Tissue thromboplastin
B. Proaccelerin E. Proconvertin
C. Antihemophilic globulin A

30. Chose the correct value of normal protein concentration in human blood
plasma:
A.65-85 g/l D.85-100 g/l
B.45-60 g/l E.100-150 g/l
C.25-40 g/l

31. Proteins of blood plasma are divided into albumin and globulins. What is
quantitative proportion of albumin to globulins (albumin/globulin
coefficient)?
A.1.5:2 D.1:1
B.2:1 E.1:5
C.5:1

32. The three primary types of plasma proteins are:


A. Albumins, globulins, fibrinogen D. Serum, fibrin, fibrinogen
B. Heme, iron, globin E. None of the above
C. Antibodies, metallo-proteins,
lipoproteins

33. The predominant constituent of blood rest nitrogen is:


A. Urea D. Phenylalanine
B. Creatinine E. Glutathion
C. Bilirubin

34. One of the components of blood residual nitrogen fraction is bilirubin,


which is produced from:
A. Heme D. Creatinine
B. Cholesterol E. Tryptophan
C. Levulinic acid

35. Uric acid is a final product of catabolism of the next component of


nucleic acids
A. Guanine D. Ribose
B.Phosphate E. Orotat
C.Thymine

36. Myoglobin belongs to:


A. Hemoproteins D. Transferrins
B. Albumins E. Phosphoproteins
C. Glycoproteins

37. Hemoglobin belongs to?


A. Chromoproteins D. Lipoprotein
B. Nukleoproteins E. Glycoproteins
C. Phosphoproteins

38. The structure of heme in hemoglobin is:


A. Protoporphyrin IX, attached to the Fe2+ D. Porphyrin coupled with Fe
3+
B. Four pyrrol rings, attached to Fe E. Protoporphyrin X, attached to
2+
C. Four pyrrol rings, attached to Fe and Fe 3+

39. Protoporphyrin IX belongs to one of the following:


A. Hemoglobins D. Properdins
B. Transferrins E. Cryoglobulines
C. Ceruloplasmins

40. Methylene blue promotes oxidation of hemoglobin. Give the name of the
obtained compound:
A. Methemoglobin C. Hemine
B. Hematin D. Carboxyhemoglobin
E. Carbhemoglobin

41. Thalassemia is a genetic disorder of synthesis of one of the chains of


hemoglobin. Inhibition of β - chain synthesis leads to formation of β-
thalassemia. Which of the following will be the best feature of the disease?
A. Increased concentration of hemoglobin C. Hemolytic anemia.
F D. Sickle erythrocyte shape
B. Decreased concentration of E. Reduction of hemoglobin A2
hemoglobin F

42. In complex proteins their prosthetic group is associated with protein


moiety. Choose from the list one amino acid responsible for the formation of
bonds between these two parts:
A. Histidine D. Tyrosine
B. Serine E. Lysine
C. Alanine

43. Choose from the following a major end product of protein metabolism
that is excreted in the largest quantity of urine:
A. Ammonia and ammonium salts D. Allantoin
B. Glutamine E. Urea
S. Uric acid

44. What from the following belongs to nitrogen-free organic compounds?


A. Vitamin C C. Glucagon
B. ATP D. Glutamine

45. What physical and chemical properties of blood explain the presence of
electrolytes?
A. Osmotic pressure C. Erythrocyte sedimentation rate
B. Oncotic pressure D. Viscosity of blood

46. In erythrocytes carbonic acid is formed from CO 2 and H2O. What enzyme
provides a synthesis of carbonic acid in erythrocytes and its degradation in
the capillaries?
A. Anhydrase D. Alkaline phosphatase
B. Amylase E. Lipase
C. Elastase

47. Hemoglobin differs from similar proteins by specific physical and


chemical properties. Indicate physical and chemical properties that
distinguish hemoglobin from myoglobin?
A. Molecular weight C. Electrophoretic activity
B. Solubility D. Spectral properties
48. As a complex protein, hemoglobin consists of protein and non-protein
moieties. Indicate the components of hemoglobin.
A. 4 Hem groups, 2 α - and 2 β - D. 4 Hem groups and 4 α
polypeptide chains -polypeptide chains
B. Hem, 1 α - and 3 β - polypeptide E. Hem, 2 α - and 2 β -
chains polypeptide chains
C. 4 Hem groups and 4 β - polypeptide
chains

EXAMPLES OF KROK 1 TESTS

49. An examination of a patient revealed hyperglycemia, ketonuria, polyuria,


and glycosuria. What type of disorder of acid-base balance is observed in
this case?
A. Metabolic acidosis C. Metabolic alkalosis
B. Respiratory alkalosis D. Respiratory acidosis

50. Patient complains of pain behind the sternum, sweating and increased
heart rate. Which of the following enzymes should be determined in his
blood to confirm the diagnosis of myocardial infarction?
A. AST, creatine kinase, LDH1 D. ALT, aldolase, LDH4
B. α-fetoprotein, aldolase, creatine kinase E. Amylase, alkaline phosphatase,
C. Acid phosphatase, LDH5, LDH4 ALT

51. Patient was transported to the clinic with inherited hemophilia A, which
is manifested in prolonged bleeding. The cause of hemophilia A might be a
deficiency of:
A. Antihemophilic globulin A C. Fibrinogen
B. Prothrombin D. Antihemophilic globulin B

52. In the blood serum of a patient a marked decrease of albumins and


fibrinogens levels were detected. Which organelles’ activity of liver
hepatocytes is reduced?
A. Granular endoplasmic reticulum C. Mitochondria
B. Lysosomes D. Golgi complex

53. Ionized copper was observed in the urine of a patient, as well as its delay
in several organs and tissues. Biosynthesis of what protein is abolished?
A. Ceruloplasmin C. Cryoglobulin
B. Transferrin D. Haptoglobin

54. Examination of a child revealed staphylococcus infection. What changes


in protein composition in blood are the most typical for this condition?
A. Identification of C - reactive protein C. Decreasing of the amount of
B. Decreasing of the amount of albumins ceruloplasmin
D. Increased albumin content

55. Laboratory analysis revealed low blood pH value, low concentration of


hydrogen carbonate (alkaline reserve of blood), increased content of lactic
and pyruvic acids in blood and urine. What type of the acid-alkaline balance
is disturbed?
A. Metabolic acidosis D. Respiratory alkalosis
B. Respiratory acidosis E. Respiratory alkalosis
C. Metabolic alkalosis

56. Which of the following immunoglobulins is a secretory component of


saliva and inhibits the adsorption of bacteria on tooth enamel?
A. IgA D. IgE
B. IgM E. IgG
C. IgD

57. Examination of a patient revealed a nephrotic syndrome. What changes


in protein fractions of blood are expected?
A. Decreased concentration of albumines D. Decreased concentration of γ-
B. Decreased concentration of α1- globulins
globulins E. Increased concentration of α2-
C. Increased concentration of β-globulins globulins

58. As a result of cyanide poisoning blockade of tissue enzymes


(cytochromes) occurs. What type of hypoxia might be observed in these
conditions?
A. Hemic D. Respiratory
B. Hypoxic E. Tissue
C. Circulatory

59. In a child during the first three months after birth a severe form of
hypoxia was indicated. The reason for this might be a disorder of the
replacement of fetal hemoglobin to:
A. Hemoglobin A
B. Mmethemoglobin
C. Hemoglobin S
D. Hemoglobin M

60. Cyanides are strong poisons for the human body. What compound for
binding them?
A. Methemoglobin C. Cyanmethemoglobin
B. Oxyhemoglobin D. Carbhemoglobin

61. Disorders of primary structure of the protein lead to changes in its


physicochemical and functional properties. Which of the above methods
might be used for identification of abnormal hemoglobins?
A. Chromatography D. Ultracentrifugation
B. Hydrolysis E. X-ray analysis
C. Electrophoresis

62. In a patient with glomerulonephritis a nitrogenemia is observed. What


substance makes the greatest contribution to the rest nitrogen?
A. Urea D. Uric acid
B. Amino acids E. Ammonia salts
C. Creatinin

63. According to blood analysis of the patient the rest nitrogen consists 48
mmols/l, urea – 15.3 mmols/l. What organ disease may cause such results of
laboratory investigation?
A. Kidneys D. Spleen
B.Liver E. Intestines
C.Stomach

64. Ambulance delivered a patient to the hospital with a preliminary


diagnosis “acute pancreatitis”. What enzyme activity must be estimated in
blood and urine in order to support this diagnosis?
F. Alpha-amylase I. Gamma-amylase
G. AlAT (GPT) J. Lactate dehydrogenase
H. AsAT (GOT)

65. In diagnostics of myocardial infarction the next isoform of lactate


dehydrogenase in blood has diagnostic significance:
A.H4 (iso 1) D.HM3 (iso 4)
B.H3M (iso 2) E.M4 (iso 5)
C.H2M2 (iso 3)

66. In a patient with symptoms of enhanced blood coagulability


(thromboses, thrombophlebitis) heparin was injected, never the less
coagulation was not inhibited. What protein factor deficiency of
anticoagulant system may exists in a patient?
A. Antithrombin III C .I –inhibitor of proteinases
B.  2-Macroglobulin D. Antithromboplastine
E. Anticonvertin

67. Which of the following drugs would be best to use on a patient who has
just had a heart attack?
A. Heparin D. Warfarin
B. Tissue plasminogen activator E. Thrombin
C. Dicoumarol

68. Patient with the symptoms of the increased blood coagulation


(thromboses, thrombophlebitis) was treated parenterally with an
anticoagulant – heparin. However the speed blood coagulation did not
decreased. The deficit of what protein factor of the anticoagulation system
of blood is observed?
A. Antithrombin III D. Antithromboplastin
B. α2-macroglobulin E. Anticonvertin
C. α1-inhibitor of proteinases

69. In the patients blood there was detected certainly high activity of
protrombin that is the threat of vessels thrombosis. What preparation
should be used in this case?
A. Heparin D. Sodium oxalate
B. Potassium oxalate E. Ethylene diamine tetra acetate
C. Sodium citrate

70. During the test on AIDS there were got two positive results of
imunoenzyme analysis (IEA). What method might be used for the exception
of pseudopositive result got with IEA?
A. PCR D. Immunofluorescence
B. Radioimmune analysis E. Molecular hybridization
C. Luminescence analysis

71. In fecal masses of newborn child consuming a natural feeding high


content of IgA was detected. This condition depends from:
A. High content of IgA in mother’s milk D. Decreased synthesis of IgC
B. Increased synthesis of IgA E. Increased synthesis of IgA and
C. Decreased synthesis of IgM IgM

72. Patient was transported to the clinic with suspicion on the myocardial
infarction. For the prophylaxis of thrombogenesis he was prescribed a
preparation of fibrinolysine (plazmin), which catalyze transformation of?
A. Fibrine into peptides D. Proconvertin into convertin
B. Fibrinogen into fibrin E. Plasminogen into plasmin
C. Protrombin into trombin

73. Streptokinase as a medical preparation is used for revascularization of


occluded by thrombus blood vessels. It possesses the following biological
activity:
A. Activates plasminogen D. Dissolves fibrin filaments by
B. Arrests the polymerization of fibrin proteolysis
monomers E Inhibits the activity of thrombin
C. Inhibits cross linking of fibrin filaments

74. In blood serum of a patient a marked increase in activity of trypsine,


alpha-amylase and lipase was detected. What disease can be considered?
A. Acute pancreatitis C. Chronic hepatitis
B. Cholestasis D. Malignant tumors
E. Insecticide poisoning

75. Blood plasma proteins of a healthy person were resolved by


electrophoresis at pH 8,6 into several fractions. What fraction possesses the
greatest electrophoretic mobility in indicated conditions?
A. γ–Globulin D. Albumin
B. α–Globulin E. Fibrinogen
C. β–Globulin

76. The appearance of Bence-Jonce protein can be defined as follows:


A. Paraproteinemia C.Hyperproteinemia
B.Hypoproteinemia D. Dysproteinemia

77. In acute phase of pneumonia an increase in proportion of the next


protein fraction in blood serum can be observed:
A. αGlobulin D. γGlobulin
B. Albumin E. Immunoglobulin D
C. βGlobulin

78. 39 years old man was admitted into hospital with symptoms of nephrotic
syndrome. What changes in blood proteins caused tissue swelling?
A.Decrease in albumin content D.Decrease in-globulin content
B.Decrease in 1- globulin content E. Increase in 2-globulins
C.Increase in -globulins content content.

79. In diagnostics of an acute viral hepatitis estimation of the next enzymatic


activity in blood serum is the most valuable:
A. Alanyl aminotransferase D. Amylase
B. Glutathion peroxidase E. Alkaline phosphatase
C. Creatine kinase

80. In rickets the next enzyme activity is highly elevated in blood serum:
A. Alkaline phosphatase D. Amylase
B. Aldolase E. Lactate dehydrogenase
C. Alanyl aminotransferase

81. In prostatic carcinoma marked increase in activity of the next enzyme in


blood serum is observed:
A. Acid phosphatase D. Lactate dehydrogenase
B. Aldolase E. Alkaline phosphatase
C. Alanyl aminotransferase

82. In mumps the next enzyme activity is highly elevated in blood serum:
A.Amylase D.Lactate dehydrogenase
B.Aldolase E.Alkaline phosphatase
C.Alanyl aminotransferase
CLINICAL CASES AND SITUATIONAL TASKS

83. HbA consists of 2 α - and 2 β- polypeptide chains; HbF – of 2 α - and 2 γ -


polypeptide chains. That makes HbF much more related to oxygen than
HbA. Explain biological significance of this condition?

Answer: HbF binds 2,3 - diphosphoglyciric acid weaker than that of HbA,
because of its higher affinity to O 2. This helps fetus to obtain oxygen from
maternal blood.

84. Explain how to differentiate kidney retention and productive azotemia by


the content of rest nitrogen in blood serum and total nitrogen in urine?

Answer: Increased concentration of rest nitrogen accompanied by elevated


nitrogen content in urine leads to productional azotemia. Low concentration of
urine nitrogen explains kidney disease.

85. Specify the direction of movement of hemoglobin molecules under the


conditions of electrophoresis at pH 4.8.

Answer: In acidic medium (pH 4.8) hemoglobin exists in the form of the cation,
because the excess of protons in the environment slows dissociation of carboxyl
groups of amino acids of globin. During electrophoresis hemoglobin moves from
the starting line to cathode, a negatively charged electrode.

86. Examination of the blood of cancer patient revealed that concentration


of the rest nitrogen was about 60 mmol/l, renal function was not impaired.
What causes this condition? What kind of azotemia is observed?

Answer: These indexes indicate increased breakdown of tissue proteins with


increasing of toxic processes. Such state is typical for productive azotemia.

87. A 25-year-old Mediterranean pregnant female has a history of


asymptomatic mild hypochromic, microcytic anemia, elevated hemoglobin
A2 and F on electrophoresis. Her brother had severe hemolytic disease that
required transfusions and ultimately caused his premature death at age 10.
She is diagnosed with β-thalassemia minor. What is the molecular
genetics behind this disorder? What was the likely test and
what is the biochemical basis?

Answer: Molecular genetics: Impaired production of β-globin peptide chain.


Numerous mutations have been identified in the production of ribonucleic acid
(RNA) including in the promoter region and splice junctions. Likely test:
Oligonucleotide probe. After chorionic villus sampling is performed, a radioactive
probe can be used and hybridized with specific genetic mutations in the fetus’
deoxyribonucleic acid (DNA), allowing for prompt detection and prenatal
diagnosis.

88. An electrophoretic analysis of patient’s hemoglobin indicates that there


is a decrease in the relative amount of the β-chain with respect to the α-
chain, both the β- and the α-chains migrate at the same position as normal
chains. Most likely his anemia is caused by the following condition?

Answer: Since the β-chain is decreased with respect to the α-chain, it is most
likely that there is a mutation that decreases the expression of the β-chain gene,
in which a mutation in the promoter region could result. A point mutation in the β-
chain leading to an amino acid substitution could lead to changes in
electrophoretic mobility but would not alter the levels of expression. A frameshift
mutation in the β-chain would result in decreased β-chain on the
electrophoregram.

89. The blood clotting cascade in humans is represented in the picture


below. Using this scheme answer the following questions:

a. Put correct coagulation factors and their active forms instead of the
question marks in the picture;
b. Antithrombin is a glycoprotein produced by the liver and is responsible
for inactivation of several enzymes of the coagulation system. Show the
main targets of anthitrombin;
c. Blood clotting or coagulation is the body's major defense mechanism
against blood loss. Such abnormalities as haemophilia A, B, C are the
defects in the following clotting factors (show your answer in the picture
using arrows);
d. Explain the role of protein C in blood coagulation.
Answer:
d. Several substances, known as anticoagulants, are in use to inhibit the
blood clotting. The blood contains a potent anticoagulant namely protein C
which is activated by thrombin. Active protein C hydrolyses and inactivates
clotting factors V and VIII.
SECTION XIV
FUNCTIONAL AND CLINICAL BIOCHEMISTRY OF LIVER TISSUE.
BIOTRANSFORMATION OF XENOBIOTICS AND ENDOGENOUS TOXIC
COMPOUNDS

1. Protoporphyrine cycle of heme is broken by the enzyme heme oxygenase


with oxidation of one methene bridge, which is released as the next product:
A. Carbone monoxide D. Formic acid
B. Formaldehyde E. Methane
C. Carbone dioxide

2. Biliverdin is transformed into bilirubin by the next chemical process:


A. Reduction with involvement C. Hydroxylation by
of NADPH monooxygenase
B. Oxidation with biliverdine D. Oxidative deamination
oxidase E. Decarboxylation of propionic
acid residues

3. Daily urinary urobilinogen excretion in adult men is:


A. 0–4 mg D. 13–20 mg
B. 5–8 mg E. 25-45 mg
C. 9–12 mg

4. Bile pigments are not present in urine in:


A. Hepatic jaundice D. Rotor’s syndrome
B. Haemolytic jaundice E. Neonatal jaundice
C. Obstructive jaundice

5. Bilirubin UDP-glucuronyl transferase is absent from liver in:


A. Crigler-Najjar syndrome, type I D. Rotor’s syndrome
B. Gilbert’s disease E. Dubin-Johnson syndrome
C. Crigler-Najjar syndrome, type II

6. Lipoproteins of the next class are synthesized and released to blood as


primary product by liver cells:
A. LDL D. HDL
B. VLDL E. Chylomicrons
C. IDL

7. In liver cholesterol is metabolized in all processes EXEPT:


A. Release to blood and D. Excretion as bile component
thereafter excretion by kidneys E. Incorporation into lipoprteins
B. Biosynthesis of bile acids and trasport to peripheral tissues
C. Incorporation into plasma
membranes of liver cells
8. An experimantal animal that was kept on protein-free diet developed fatty
liver infiltration, in particular as a result of deficiency of methylating agents.
This is caused by disturbed generation of the following metabolite:
A. Choline D. Acetoacetate
B. DOPA E. Linoleic acid
C. Cholesterol

9. Breakdown of 1gm hemoglobin produces:


A. 35 mg of bilirubin D. 70 mg of bilirubin
B. 20 mg of bilirubin E. 100 mg of bilirubin
C. 50 mg of bilirubin

10. Each hemoglobin molecule contains:


A. Four iron atoms D. Three iron atoms
B. One iron atom E. Six iron atoms
C. Two iron atoms

11. Fatty of phospholipids is disordered due to fat infiltration of the liver.


Indicate which of the presented substances can enhance the process of
methylation during phospholipids synthesis?
A. Methionine D. Glycerin
B. Ascorbic acid E. Citrate
C. Glucose

12. Free bilirubin (indirect bilirubin) is transformed in liver cells into


conjugated (direct) bilirubin by conjugation with glucuronic acid with the aid
of enzyme glucuronyl transferase. As substrate this enzyme uses the
following compound:
A. UDP-glucuronate D. CDP- glucuronate
B. Free glucuronate E. UMP-glucuronate
C. CDP-glucose

13. In the normal resting state of human most of the blood glucose burnt as
fuel is consumed by:
A. Liver D. Muscles
B. Brain E. Kidneys
C. Adipose tissue

14. What digestive process is altered in case of obstruction of bile duct and
arrest of bile excretion to intestines?
A. Hydrolysis of triglycerides D. Hydrolysis of complex sugars
B. Absorption of carbohydrates and polysaccharides
C. Hydrolysis of proteins E. Absorption of amino acids.

15. During breakdown of hem, the methenyl bridge between the following
two pyrrole rings is broken:
A. I and II D. IV and I
B. II and III E. III and I
C. III and IV

16. Pre- hepatic jaundice occurs because of:


A. Increased hemolysis D. Resus conflict
B. Liver damage E. None of these
C. Biliary obstruction

17. In dietary deficiency or insufficient production of endogeneous


lipotropic factors in humans is developing fat degeneration of liver. What
substances from listed below can be considered as lipotropic factor?
A. Choline D. Triacylglycerols
B. Fatty acids E. Cholesterol
C. Pyridoxine

18. Excretion of conjugated bilirubin from liver cells into biliary canaliculi is
defective in:
A. Dubin-Johnson syndrome D. Lucey-Driscoll syndrome
B. Gilbert’s disease E. Rotor’s syndrome
C. Crigler-Najjar syndrome

19. To obese patient with risk of liver fat degeneration is recommended diet
enriched with lipotropic factors. What nutritional component is the most
important in diet?
A. Methionine D. Glycine
B. Cholesterol E. Glucose
C. Vitamin C

20. The greater amount of nitrogen is excreted from the organism in form of
urea. Inhibition of urea synthesis and accumulation of ammonia in blood
and tissues are induced by the decreased activity of the following liver
enzyme:
A. Carbamoyl phosphate synthetase D. Amylase
B. Aspartate aminotransferase E. Pepsin
C. Urease

21. Reactions of methylation take place in biosynthesis of catecholamins,


creatine, lecithin, as well as in detoxification of some xenobiotics. As donor
of methyl groups serves the next substance:
A. Methionine D.Choline
B. Leucine E. Adenosine
C.Glutathion
22. Reduced NADP H2 is necessary for biosynthesis of fatty acids,
cholesterol, for detoxification of ammonia and xenobiotics. In what
metabolic pathway is generated NADPH2?
A. Hexosomonophosphate shunt D. Glycolysis
B.Oxidation of fatty acids E. Tricarboxylic acid (TCA) cycle
C. Gluconeogenesis

23. The process of microsomal oxidation has the next physiological


significance:
A. Degradation of bile acids D. Utilization of unusual
B. Oxidative deamination of amino carbohydrates
acids E. Oxidation of fatty acids
C. Biosynthesis of steroid hormones

24. Utilization and detoxification of ethanol in liver is performed by the next


pathway:
A. Oxidation by C. Conjugation with sulfuric acid
dehydrogenases D. Acetylation
B. Conjugation with glucuronic E. Reduction with participance
acid of glutathione

25. In oxidation of ethanol by microsomal oxidizing system besides


acetaldehyde is generated the next additional harmful product:
A. Hydrogen peroxide D. Malone dialdehyde
B. Superoxide anion E. Carbone monoxide
C. Hypochlorite anion

26. Development of resistance toward drugs and medicinals (drug tolerance)


appears due to the following process:
A. Increase in activity of D. Decrease in activity of
microsomal oxidation microsomal oxidation
B. Decrease in activity of E. Activation of peroxide
mitochomdrial oxidation oxidation
C. Increase in activity of
mitochondrial oxidation

27. Monooxygenase systems of liver cell endoplasmic reticulum contain


cytochrome P 450, NADPH2 cytochrome P 450 reductase, FAD and FMN
dependent enzymes. All these enzymes catalyze the following type of
chemical reaction:
A. Hydroxylation D. Methylation
B. Acylation E. Conjugation
C. Dehydrogenation

28. What substance serves as donor of hydrogen in reactions of microsomal


oxidation?
A. NADP H2 D. FAD
B. Ascorbic acid E. FMN
C. NAD H2

29. Glucuronic acid which is used for conjugation reaction in detoxification


of xenobiotics exists in the next active form:
A. UDP-glucuronate D. CDP-glucuronate
B. Glucuronyl pyrophosphate E. Glucuronyl-CoA
C. AMP-glucuronate

30. The aim of conjugation stage in detoxification of xenobiotics is as


follows:
A. To make substance more C. To transform substance to
soluble in water mrdium insoluble form and deposit it in
B. To make substance fat vesicles
soluble D. To induce ingestion of
substance by macrophages

31. Introduction of one oxygen atom into organic compounds is catalyzed


by enzymes, which are defined as:
A. Peroxidases D. Monooxygenases
B. Catalases E. Oxidases
C. Dioxygenases

32. Harmful effect of alkaloid cocaine on liver cells is caused by its


bioactivation and formation of nitroxide derivative, which is a strong
oxidant. The transformation is initiated by the following chemical reaction:
A. N-dealkylation D. Hydrolysis
B. Deacylation E. N-hydroxylation
C. O-dealkylation

33. Besides cytochrome P-450 microsomal oxidation system in liver cell


exists the second monooxygenase system, which is provided by the next
enzyme:
A. Glutathion peroxidase D. Monoamine oxidase
B. Catalase E. Xanthine oxidase
C. FAD containing oxygenase

34. Sulfate is used in conjugation step of detoxification as the following


active form:
A. Phosphoadenosyl phosphosulfate D. Guanosine sulfate
B. UDP-sulfate E. Thionyl chloride
C. Sulfuryl-CoA

35. Detoxification function of liver in patients is evaluated with the following


test:
A. Quick test (hippuric acid C. Bilirubin concentration in
synthesis) blood
B. Glucose tolerance test D. LDL content in blood serum

36. During the investigation of detoxification function of liver a patient was


given 4 g of sodium benzoate per os. From urine, collected during four
subsequent hours, it was obtained 1 g of hippuric acid. What indicates this
result?
A.Normal value of detoxification C. Enhanced detoxification function of
function of liver liver
В. Insufficient detoxification function
of liver

37. What coenzyme is necessary for NADPH-cytochrome P450 reductase


activity?
A. NADP D. FMN
B. ATP E. FAD and FMN both
C. FAD

38.Cytochrome P450 belongs to the next class of enzymes:


A.Oxidoreductases D. Lyases
B. Transferases E. Isomerases
C. Hydrolases

39. Flavin containing monooxygenase belongs to the next class of enzymes:


A. Oxidoreductases D. Lyases
B. Transferases E. Isomerases
C. Hydrolases

40. Chose the correct statement about characteristic properties of flavin


containing monooxygenase of liver tissue.
A. This enzyme exhibits a narrow D. It forms a hydroperoxide derivative
stereochemical substrate specificity of FAD (peroxyflavin) which oxidizes
B. Its activity is modulated by substrate
calmodulin E. It generates hydrogen peroxide,
C. It is constitutive enzyme, activity which oxidizes substrate
not dependent from presence of
inducers.

EXAMPLES OF KROK 1 TESTS

41. A child is languid, apathetic. Liver is enlarged and liver biopsy revealed
a significant excess of glycogen. Glucose concentration in the blood stream
is below normal. What is the cause of low glucose concentration?
A. Low (absent) activity of glycogene phosphorylase in liver
B. Low (absent) activity of hexokinase phosphatase
C. High activity of glycogen E. Deficit of a gene that is responsible
synthetase for synthesis of glucose 1-
D. Low (absent) activity of glucose 6- phosphaturidine transferase

42. In neonates after birth during 5-6 days develops jaundice. The cause of
this disorder is insufficient activity of the following enzyme:
A. UDP-glucuronyl transferase D. Heme oxygenase
B. Porphobilinogen synthase E. Biliverdin reductase
C. Aminolevulinate synthease

43. In blood of a patient was found an increased content of total bilirubin, in


urea were detected bilirubin diglucuronides, absence of stercobilin in feces
(acholic stool). What vitamins deficiency can be developed i9n this special
case?
A. D, K, A D. Biothin and lipoic acid
B. B1, B2, B6. (vitamin N)
C. PP, C, U E. P, B15 (pangamic acid).

44. In a patient was recognized congenital liver disease, which is


accompanied with high bilirubinemia due to increase in free
(nonconjugated) bilirubin. In liver biopsia was detected trace activity of
glucuronyl transferase. What disease can be recognized?
A. Crigler-Najjar syndrome D. Dubin-Johnson syndrome
B. Gilbert syndrome E. Wilson disease
C. Physiological jaundice

45. After a serious viral infection a 3-year-old child has repeated vomiting,
loss of consciousness, convulsions. Examination revealed
hyperammoniemia. What may have caused changes of biochemical blood
indices of this child?
A. Disorder of ammonia neutralization neutralization
in ornithinic cycle D. Increased purtefaction of proteins
B. Activated processes of aminoacids in intestines
decarboxylation E. Inhibited activity of transamination
C. Disorder of biogenic amines enzymes

46. A patient suffering from rheumatism was administered glucocorticoids


therapy. What changes in carbohydrate metabolism in liver can be
expected?
A. Stimulation of gluconeogenesis D. Stimulation of glycogen
B. Stimulation of glycogenesis phosphorolysis
C. Stimulation of glycogen hydrolysis E. Increase of glycogen
phosphorylase activity

47. A patient has yellow skin colour, dark urine, dark-yellow feces. What
substance will have strengthened concentration in the blood serum?
A. Unconjugated bilirubin D. Verdoglobin
B. Conjugated bilirubin E. Biliverdin
C. Mesobilirubin

48. A 46 year old woman suffering from chololithiasis developed jaundice.


Her urine became dark-yellow and feces became colourless. Blood serum
will have the highest concentration of the following substance:
A. Conjugated bilirubin D. Mesobilirubin
B. Unconjugated bilirubin E. Urobilinogen
C. Biliverdin

49. A 4 y.o. boy has had recently serious viral hepatitis. Now there are such
clinical presentations as vomiting, loss of consciousness, convulsions.
Blood analysis revealed hyperammoniemia. Disturbunce of which
biochemical process caused such pathological condition of the patient?
A. Disturbed neutralization of proteins in bowels
ammonia in liver D. Activation of aminoacid
B. Disturbed neutralization of decarboxylation
biogenic amines E. Inhibition of transamination
C. Increased putrefaction of enzyms

50. A newborn child was found to have reduced intensity of sucking,


frequent vomiting, hypotonia. Urine and blood exhibit increased
concentration of citrulline. What metabolic process is disturbed?
A. Ornithinic cycle D. Cori cycle
B. Tricarboxylic acid cycle E. Glyconeogenesis
C. Glycolysis

51. In a patient with jaundice it was detected a block in transformation of


bilirubin to bilirubin diglucuronide. The concentration of indirect bilirubin in
blood was markedly increased. What pathology can be suggested in a
patient?
A. Prehepatic jaundice D. Tumor of pancreas and
B. Hepatic jaundice occlusion of bile duct
C. Posthepatic jaundice E. Addison’s disease

52. In 35 years old patient a jaundice was revealed, as well as fever,


bradycardia. Laboratory investigation revealed increase in direct and
indirect bilirubin, appearance of direct bilirubin and bile acids in urine,
missing of stercobilinogen in feces. What type of jaundice can be expected?
A. Post-hepatic C. Jaundice of neonates type
B. Hepatic D. Gilbert disease

53. Investigation of patient indicated on inflammatory processes in gall


bladder, alteration of colloidal stability and high risk of bile stones
formation. What substance from listed below favors the formation of bile
stones?
A. Cholesterol D. Oxalates
B. Urates E. Lecithine
C. Phosphates

54. In 14 years old patient a hereditary liver pathology was expected. It was
detected high content of direct bilirubin in blood, deposition of melanin in
liver due to alteration of bilirubin excretion to bile by liver cells. This state is
characteristic to the next disease:
A. Dubin-Johnson syndrome D. Physiological jaundice
B. Crigler-Najjar syndrome E. Wilson disease
C. Gilbert disease

55. A man is resting after intensive physical effort. Which from different
pathways of glucose metabolism is the most active at this time?
A. Gluconeogenesis from lactate D. Breakdown of glycogen to glucose
B. Glycolysis E. Gluconeogenesis from amino acids
C. Glycogenolysis

56. In a patient suffering from liver cirrhosis concentration of albumin in


blood plasma is 15 g/l (normal value 32-55 g/l), prothrombine test – 40
seconds (normal value – 12-20 sec.). To what functional disorder in liver
indicate these changes?
A. Inhibition of protein synthesis D. Production of bile
B. Detoxification function E. Secretion of lipoproteins to
C. Excretory function blood

57. In a patient with toxic damage of liver was revealed low concentration of
urea in blood plasma. The cause of this may be the next:
A. Alteration of ornithine cycle of urea D. Deficiency of CO2 and restriction of
synthesis urea synthesis
B. Absence of alanil E. Excess of ammonia, which inhibits
aminotransgferase in liver cells enzymes of urea synthesis
C. Deficiency in blood ammonia
concentration

CLINICAL CASES AND SITUATIONAL TASKS

58. A 45-year-old male with history of hepatitis C and now cirrhosis of the
liver is brought to the emergency center by family members for acute mental
status changes. The family reports that the patient has been very
disoriented and confused over the last few days and has been nauseated
and vomiting blood. The family first noticed disturbances in his sleep
pattern followed by alterations in his personality and mood. On examination,
he is disoriented with evidence of icteric sclera. His abdomen is distended
with a fluid wave appreciated. He has asterixis and hyperreflexia on
neurologic exam. His urine drug screen and ethyl alcohol (EtOH) screen are
both negative. A blood ammonia level was noted to be elevated, and all
other tests have been normal. What is the most likely cause of the patient’s
symptoms? What is asterixis? What was the likely precipitating factor of the
patient’s symptoms?

Answer: Diagnosis: Hepatic encephalopathy likely secondary to elevated


ammonia levels. Asterixis: Nonspecific to hepatic encephalopathy. Nonrhythmic
asymmetric tremor with loss of voluntary control of extremities while in a sustained
position. It is also known as “liver flap.” Precipitating factor: Increased nitrogen
load from upper gastrointestinal bleed.
Cirrhosis is a chronic condition of the liver with diffuse parenchymal injury and
regeneration leading to distortion of the liver architecture and increased resistance
of blood flow through the liver. The patient usually manifests malaise, lethargy,
palmar erythema, ascites, jaundice, and hepatic encephalopathy in the late
stages. Toxins accumulating in the blood stream affect the patient’s mental status.
The most common etiologies of cirrhosis are toxins such as alcohol, viral
infections such as hepatitis B or C infection, or metabolic diseases in children
(Wilson disease, hemochromatosis, or α1-antitrypsin deficiency). Treatment
depends on the exact etiology, although the common therapy includes avoidance
of liver toxins, salt restriction, and possibly procedures to reduce the portal
pressure.

59. A 20-year-old female was brought to the emergency department after


being found on the dormitory room floor nauseated, vomiting, and
complaining of abdominal pain. Her friends were concerned when she did
not show up for a biochemistry final at the local university. The patient had
been under a lot of stress with finals, a recent breakup with a boyfriend, and
trying to find a job. In the dormitory room, one of her friends noticed an
empty bottle of Tylenol (acetaminophen) near the bed with numerous pills
lying on the ground near their friend. On arrival to the emergency
department, the patient was found to be in moderate distress and vomiting.
The patient was quickly assessed, and laboratory work was obtained.
Patient had a hypokalemia noted on electrolytes and elevated liver enzymes.
Her white blood cell count was normal. Her urine drug screen was negative,
and her acetaminophen blood level was above 200 μg/mL. The emergency
department physician prescribes oral N-acetylcysteine to help prevent
toxicity from the acetaminophen. What is the pathophysiology of the liver
toxicity?What is the biochemical mechanism whereby the N-acetylcysteine
helps in this condition?

Answer: Acetaminophen is metabolized via the cytochrome P450 enzymes into a


deleterious product N-acetyl benzoquinoneimine, an unstable intermediate, which
causes arylated derivatives of protein, lipid, ribonucleic acid (RNA), and
deoxyribonucleic acid (DNA), causing destruction of these compounds. Because
the liver has high levels of cytochrome P450 enzymes, it is the major organ
affected by acetaminophen overdose.
Biochemical mechanism of N-acetylcysteine: As glutathione is used to
conjugate the acetaminophen toxic metabolite, the antidote Nacetylcysteine helps
to facilitate glutathione synthesis by increasing the concentrations of one of the
reactants of the first synthetic step.
The patient described has all the initial signs of a deliberate overdose of
acetaminophen. Normally acetaminophen is cleared by conjugation with either
glucuronic acid or sulfate followed by excretion. Metabolism also takes place,
producing an active intermediate capable of binding tissue macromolecules.
These conjugative and metabolic pathways involve a number of enzymes that
may themselves be compromised to such an extent that the threshold for the
concentration that constitutes an overdose is substantially lowered. More typically,
overdose concentrations are the result of deliberate ingestion, as in this clinical
case, or accidental ingestion, often involving either a child who finds a bottle of
acetaminophen and consumes its contents or a disoriented elderly person who
loses track of how many tablets have been consumed. Usually, the
acetaminophen serum level is drawn and plotted on a nomogram to determine the
possibility of hepatic damage. Hepatocyte necrosis with clinical manifestations of
nausea and vomiting, diarrhea, abdominal pain, and shock may ensue. Few
survivors of an overdose have long-term hepatic disease. The initial therapy is
gastric lavage, activated charcoal, supportive care, and administration of N-
acetylcysteine.

60. A 45-year-old female presents with hypercholesterolinemia, ultrasound


evidence of gallstones, and recurrent symptoms of gallbladder disease.
What factors would you need to consider to assess the need for
cholecystectomy? What are gallstones made of?

Answer: Surgical candidates: Frequent and severe attacks, previous


complications from gallstones, presence of underlying condition predisposing the
patient to increased risk of gallbladder disease. Components of gallstones:
Cholesterol, calcium bilirubinate, and bile salts.
This individual fits the “classic” patient with gallbladder disease: female
middle-aged, overweight. The gallbladder acts to store bile salts produced by the
liver. The gallbladder is stimulated to contract when food enters the small
intestine; the bile salts then travel through the bile duct to the ampulla of Vater into
the duodenum. The bile salts act to emulsify fats, helping with the digestion of fat.
Gallstones form when the solutes in the gallbladder precipitate. The two main
types of stones are cholesterol stones and pigmented stones. Cholesterol stones
are usually yellow-green in appearance and account for approximately 80 percent
of gallstones. Pigmented stones are usually made of bilirubin and appear dark in
color. Patients may have pain from the gallstones, usually after a fatty meal. The
pain is typically epigastric or right upper quadrant and perhaps radiating to the
right shoulder. If the gallbladder becomes inflamed or infected, cholecystitis can
result. The stones can also travel through the bile duct and obstruct biliary flow
leading to jaundice (yellow color or the skin), or irritate the pancreas and cause
pancreatitis.

61. A 26-year-old female at 35 weeks gestation with generalized pruritus


without a rash and slightly elevated liver transaminases and bilirubin. What
is the patient’s likely diagnosis? What are treatment options? What is the
cause of the patient’s generalized itching?

Answer: Diagnosis: Cholestasis of pregnancy. Cholestasis of pregnancy is a


condition in which the normal flow of bile from the gallbladder is impeded, leading
to accumulation of bile salts in the body. Generalized itching and, possibly,
jaundice may result. It is speculated that the hormones such as estrogen and
progesterone, which are elevated in pregnancy, cause a slowing of the gallbladder
function, leading to this disorder. Uncomplicated cholestasis is usually diagnosed
clinically by generalized itching in a pregnant woman, usually in the third trimester
without a rash. Elevated serum levels of bile salts can help to confirm the
diagnosis. Elevated bilirubin levels or liver transaminase enzymes may also be
seen. The usual treatment includes antihistamine medications for the itching.
Some experts recommend ursodeoxycholic acid, a naturally occurring bile acid
that seems to improve liver function and may reduce the serum bile acid
concentration. More severe cases may require bile salt binders such as
cholestyramine or corticosteroids. Treatment options: Oral antihistamines,
cholestyramine, ursodeoxycholic acid. Etiology of generalized itching: Increased
serum bile salts and accumulation of bile salts in the dermis of the skin.

62. A 18-year-old male with sickle cell anemia develops severe right upper-
abdominal pain radiating to his lower right chest and his right flank 36 hours
prior to admission to the ER. Twelve hours following the onset of pain, he
began to vomit intractably. In the past year he has had several episodes of
mild back and lower extremity pain that he attributed to mild sickle cell
crises. He reported that the present pain was not like his usual crisis pain.
He also reports that his urine is the color of iced tea and his stool now has a
light clay color. On examination, his temperature is slightly elevated, and
heart rate is rapid. He is exquisitely tender to pressure over his right upper
abdomen. The sclerae of his eyes are slightly yellowish in color. What is the
most likely cause of this patient’s symptoms?

Answer: Although a cholesterol-rich gallstone cannot be completely ruled out with


the given information, because the patient has experienced several mild sickle cell
crises that are accompanied by increased red blood cell destruction, his
symptoms are consistent with a gallstone caused by precipitation of calcium salt
of bilirubin. Large quantities of bilirubin can overwhelm the ability of the liver to
convert it to the more soluble diglucuronide conjugate. As a consequence, the
more insoluble unconjugated form enters the bile and is easily precipitated in the
presence of calcium ion. If a large stone forms, it can obstruct the bile duct and
result in the symptoms exhibited by the patient.
63. A 45-year-old male with history of hepatitis C and now cirrhosis of the
liver is brought to the emergency center by family members for acute mental
status changes. The family reports that the patient has been very
disoriented and confused over the last few days and has been nauseated
and vomiting blood. The family first noticed disturbances in his sleep
pattern followed by alterations in his personality and mood. On examination,
he is disoriented with evidence of icteric sclera. His abdomen is distended
with a fluid wave appreciated. He has asterixis and hyperreflexia on
neurologic exam. His urine drug screen and ethyl alcohol (EtOH) screen are
both negative. A blood ammonia level was noted to be elevated, and all
other tests have been normal. What is the most likely cause of the patient’s
symptoms? What was the likely precipitating factor of the patient’s
symptoms?

Answer: Diagnosis: Hepatic encephalopathy likely secondary to elevated


ammonia levels. Precipitating factor: Increased nitrogen load from upper
gastrointestinal bleed. Cirrhosis is a chronic condition of the liver with diffuse
parenchymal injury and regeneration leading to distortion of the liver architecture
and increased resistance of blood flow through the liver. The patient usually
manifests malaise, lethargy, palmar erythema, ascites, jaundice, and hepatic
encephalopathy in the late stages. Toxins accumulating in the blood stream affect
the patient’s mental status. The most common etiologies of cirrhosis are toxins
such as alcohol, viral infections such as hepatitis B or C infection, or metabolic
diseases in children (Wilson disease, hemochromatosis, or α1-antitrypsin
deficiency). Treatment ndepends on the exact etiology, although the common
therapy includes avoidance of liver toxins, salt restriction, and possibly procedures
to reduce the portal pressure.
SECTION XV
WATER AND MINERAL METABOLISM

1. Excretion of sodium with urine is stimulated by the following hormone:


A. Atrium natriuretic peptide (ANP) D. Aldosteron
B. Vasopressin E. Norepinephrin
C. Oxytocin

2. Addison disease is caused by aldosterone insufficiency, leading to:


A. Retention of potassium in the body D. Retention of water and tissue
B. Excessive loss of potassium from the swelling
body E. Disorder in calcium
C. Retention of Ca ions in blood metabolism

3. The predominant mineral of bones is:


A. Calcium hydroxyapatite D. Calcium sulphate
B. Calcium silicate E. Calcium phosphate
C. Calcium carbonate

4. The softening of bones in rickets is caused by deficiency of:


A. Cholecalciferol (D3) D. Parathyroid hormone
B. Pantothenic acid E. Pituitrine P
C. Nicotiamide

5. The cause of endemic goiter is deficiency of trace element:


A. Iodine D. Copper
B. Iron E. Manganese
C. Zinc

6. Copper as trace element is essential component of the next enzyme:


A. Cytochrome oxidase D. Xantine oxidase
B. Amylase E. Carbonic anhydrase
C. Trypsin

7. Zinc is a cofactor of the next enzyme:


A. Carbonic anhydrase D. Aminotransferase
B. Pepsin E. Galactosidase
C. Monooxygenase

8. Selenium is an ultra trace element and is incorporated in structure of


enzyme:
A. Glutathione peroxidase D. Myeloperoxidase
B. Lactate dehydrogenase E. Pyruvate kinase
C. Catalase

9. Trace element molybdenum is a cofactor of the enzyme:


A. Xantine oxidase D. Myeloperoxidase
B. Tyrosinase E. Cytochrome c
C. Tryptophane hydroxylase

10. Steroid hormones, regulating mineral metabolism, are the next:


A. Aldosteron D. Cortisol
B. Glucocorticoids E. Norepinephrin
C. Progesteron

11. Trace element cobalt is an element of the structure of vitamin:


A. Vitamin B12 D. Vitamin B6
B. Biotine E. Pantothenic acid
C. Vitamin B1

12. This is the accepted norm of daily requirement of water in adult humans:
A. 1,5-2 l D. 3-4 l
B. 0,25-0,5 l E. 5 l
C. 0,5-0,75 l

13. Retention of water in blood plasma and prevention of tissue swelling


depends on the oncotic pressure of plasma, which depends on the content
of the next compounds:
A. Albumins D. Lipids of lipoproteins
B. Immunoglobulins E. Inorganic ions
C. Fibrinogen

14. The asymmetry of sodium and potassium ions distribution across


plasma membrane of the cell is supported by the next membrane
constituent:
A. K,Na-dependent ATP-ase D. Alkaline phosphatase
B. Sodium selective channel E. Cholesterol content
C. Lecithine content of the membrane

15. Concentration of sodium ion in blood plasma of healthy adult is:


A. 105 mMoles/l D. 0,9 g/dl
B. 25 mMoles/l E. 1,5 g/l
C. 300 mMoles/l

16. Under hypoosmolar dehydratation could be:


A. Dryness of the skin and mucosal C. Muscle hypotonia
structures D. Collapse
B. Headache E. Blood closeness

17. Extracellular hyperhydratation can be induced by:


А. Oncotic pressure decreasing, caused С. Increasing of hydrostatic
by hypoproteinemia after proteins loosing pressure of the blood
through kidneys D. Alterations in a heart work
В. Decreasing of sodium level Е. Increasing of sodium level

18. Find a compound form the list, which does not contain iron:
А. Histamine D. Cytochromes
В. Muscle’s myoglobin Е. Hemoglobin of RBCs
С. Ferritin and homosiderrin

19. Which enzyme from the list below contains the copper?
А. Cytochromeoxidase D. Aminoxidase
В. Alkaline phosphatase Е. Tyrosinase
С. Alcoholdehidrogenase

20. In highlander the thyroid gland is bigger than normal. It is associate with
failing in nutrition og the next microelement:
А. Iodine D. Iron
В. Brome Е. Mangan
С. Copper

21. Please, specify the function, which is not characteristic for calcium in
human organism:
А. Calcium ions are cofactors for many D. Calcium ions play a role in
enzymes skeletal muscles contraction
В. Calcium ions are the second Е. Calcium salts are responsible
messenger in cell signaling for hard structure of bones
С. Calcium ions play a role in hemostasis

22. Concentration of calcium ion in urine of healthy adult is:


А. 2,5-7,5 mmol D. 55-105 mmol
В. 5,5-10,5 mmol Е. 0,25-0,75 mmol
С. 25-75 mmol

23. Specify the microelement which is the of T 3 and T4:


А. Iodine D. Iron
В. Fluorine Е. Selenium
С. Bromine

24. Specify the microelement with antioxidant function:


А. Selenium D. Iron
В. Fluor Е. Iodine
С. Brome

25. Which electrolyte has the major role in the volume of the extracellular
water homeostasis?
А. Sodium D. Iron.
В. Fluor Е. Iodine
С. Brome

26. Find from the list below the symptom of zinc deficiency:
А. Taste disorders D. Alterations in the hair growth
В. Affecting of the immune system Е. Dermatitis
С. Olfaction disorders

27. In cell the content of … is higher, than other microelements:


А. K+ D. Fe3+
В. Na+ Е. Ca2+
С. I+

28. Specify the mechanism of aldosterone action on the electrolytes


metabolism:
А. Decreasing of the Na+ excretion D. Decreasing of the K+ excretion
В. Increasing of the Na+ excretion E. Increasing of the K+ excretion
С. Decreasing of the water excretion

29. The penetration of the irritable cell membrane for potassium ions has
been increased during an experiment. What changes of membrane electric
status can occur?
A. Hyperpolarization D. Local response
B. Depolaization E. No changes
C. Action potential

30. Deposition of calcium in bones is controlled by the next hormone:


A. Calcitonin D. Calmodulin
B. Thyroid hormone triiodothyronin E. Calcitriol
C. PTH (parathyroid hormone)

31. The cause of endemic goiter is deficiency of trace element:


A. Iodine D. Copper
B. Iron E. Manganese
C. Zinc

32. Copper as trace element is essential component of the next enzyme:


A. Cytochrome oxidase D. Xantine oxidase
B. Amylase E. Carbonic anhydrase
C. Trypsin

EXAMPLES OF KROK 1 TESTS


33. After prolong vomiting and diarrhea in a patient is observed tachycardia,
skin dryness, muscular hypotonia and apathia. Such state can be caused
by:
A. Hypomagnesemia D. Hypocupremia
B. Hypopotassemia E. Hypercupremia
C. Hyperpotassemia

34. What disorders of water and mineral metabolism can be expected in a


worker, having a job in hot conditions?
A. Hypoosmolar dehydration D. Extracellular hyperhydration
B. Hyperosmolar dehydration E. Isotonic hyperhydration
C. Cellular hyperhydration

35. In a patient after thyroid gland surgery cramps and convulsions


appeared, hyperreflexia and hyperexcitability were observed. Preliminary
diagnosis is hypocalcaemia, which may be caused by:
A. Hypoparathyreosis, induced by gland
extirpation of parathyroid glands C. Overdose of vitamin E
B. Hyperparathyreosis due to tumor D. Overdose of vitamin D
developement from cells of parathyroid E. Insulin insufficiency

36. In a patient a development of calcium deposits in soft tissues, stones in


urinary ducts are observed. Preliminary diagnosis is hypercalcemia, which
may be caused by:
A. Hyperparathyreosis due to tumor of parathyroid glands during
development from cells of parathyroid thyroid gland surgery
glands D. Insulin insufficiency
B. Vitamin D deficiency E. Insulin overproduction
C. Hypoparathyreosis, induced by injury

37. Dietary intake of a 30 year old nursing woman contains 1000 mg of


calcium, 1300 mg of phosphorus and 20 mg of iron per day. It is necessary
to change content of these mineral substances in the following way:
A To increase phosphorus content D. To increase iron content
B. To increase calcium content E. To reduce iron content
C. To reduce fluorine content

38. A 35 y.o. patient who often consumes alcohol was treated with diuretics.
There appeared serious muscle and heart weakness, vomiting, AP- 100/60
mm Hg, depression. This condition is caused by intensified excretion with
urine of:
A. Potassium D. Calcium
B. Sodium E. Phosphates
C. Chlorine
39. A 50-year-old patient complains about general weakness, appetite loss
and cardiac arrhythmia. The patient presents with muscle hypotonia, flaccid
paralyses, weakened peristaltic activity of the bowels. Such condition might
be caused by:
A. Hypokaliemia D. Hypophosphatemia
B. Hypoproteinemia E. Hyponatremia
C. Hyperkaliemia

CLINICAL CASES AND SITUATIONAL TASKS

40. The elderly people (over 60 yr.) of both sexes are at risk for
osteoporosis. However, it more predominantly occurs in the post-
menopausal women. Explain why.

Answer: The ability to produce calcitriol from vitamin D is decreased with age,
particularly in the postmenopausal women. lmmobilized or sedentary individuals
tend to decrease bone mass while those on regular exercise tend to increase
bone mass. Deficiency of sex hormones (in women) has been implicated in the
development of osteoporosis.

41. A patient with the symptoms of irritability, muscular weakness,


tachycardia, cardiomegaly and cardiac arrest was delivered to the hospital.
What disorders of mineral metabolism can be expected?

Answer: Such symptoms are observed in hypokalemia. Decrease in the


concentration of serum potassium is observed due to overactivity of adrenal
cortex (Cushing's syndrome), prolonged cortisone therapy, intravenous
administration of K+-free fluids, treatment of diabetic coma with insulin, prolonged
diarrhea and vomiting.

42. Iron deficiency anemia is the most prevalent nutritional disorder


worldover, including the well developed countries (e.g. USA). Several
factors may contribute to iron deficiency anemia. These include inadequate
intake or defective absorption of iron, chronic blood loss, repeated
pregnancies and hookworm infections. Explain why strict vegetarians are
more prone for iron deficiency anemia.

Answer: This is due to the presence of inhibitors of iron absorption in the


vegetarian foods, besides the relatively low content of iron.

43. Wilson's disease (hepatolenticular degeneration) is a rare disorder of


abnormal copper metabolism. Characterize the main manifestations of this
disorder.
Answer: Copper is deposited in abnormal amounts in liver and lenticular nucleus
of brain. This may lead to hepatic cirrhosis and brain necrosis. Low levels of
copper and ceruloplasmin in plasma with increased excretion of copper in urine.
Copper deposition in kidney causes renal damage. This leads to increased
excretion of amino acids, glucose, peptides and hemoglobin in urine.
SECTION XVI
FUNCTIONAL ROLE OF KIDNEYS IN URINOGENESIS. NORMAL AND
PATHOLOGICAL CONSTITUENTS OF URINE

1. What disorders of water and mineral metabolism can be expected in a


worker, having a job in hot conditions?
A. Hypoosmolar dehydration D. Extracellular hyperhydration
B. Hyperosmolar dehydration E. Isotonic hyperhydration
C. Cellular hyperhydration

2. Excretion of sodium with urine is stimulated by the following hormone:


A. Atrium natriuretic peptide (ANP) D. Aldosteron
B. Vasopressin E. Norepinephrin
C. Oxytocin

3. Addison disease is caused by aldosterone insufficiency, leading to:


A. Retention of potassium in the C. Retention of Ca ions in blood
body D. Retention of water and tissue
B. Excessive loss of potassium from swelling
the body E. Disorder in calcium metabolism

4. The predominant mineral of bones is:


A. Calcium hydroxyapatite D. Calcium sulphate
B. Calcium silicate E. Calcium phosphate
C. Calcium carbonate

5. The softening of bones in rickets is caused by deficiency of:


A. Cholecalciferol (D3) D. Parathyroid hormone
B. Pantothenic acid E. Pituitrine P
C. Nicotiamide

6. The cause of endemic goiter is deficiency of trace element:


A. Iodine D. Copper
B. Iron E. Manganese
C. Zinc

7. Copper as trace element is essential component of the next enzyme:


A. Cytochrome oxidase D. Xantine oxidase
B. Amylase E. Carbonic anhydrase
C. Trypsin

8. Zinc is a cofactor of the next enzyme:


A. Carbonic anhydrase D. Aminotransferase
B. Pepsin E. Galactosidase
C. Monooxygenase
9. Selenium is an ultra trace element and is incorporated in structure of
enzyme:
A. Glutathione peroxidase D. Myeloperoxidase
B. Lactate dehydrogenase E. Pyruvate kinase
C. Catalase

10. Trace element molybdenum is a cofactor of the enzyme:


A. Xantine oxidase D. Myeloperoxidase
B. Tyrosinase E. Cytochrome c
C. Tryptophane hydroxylase

11. Steroid hormones, regulating mineral metabolism, are the next:


A. Aldosteron D. Cortisol
B. Glucocorticoids E. Norepinephrin
C. Progesteron

12. Trace element cobalt is an element of the structure of vitamin:


A. Vitamin B12 D. Vitamin B6
B. Biotine E. Pantothenic acid
C. Vitamin B1

13. There is accepted the next norm of daily requirement in water in adult
humans:
A. 1,5-2 l D. 3-4 l
B. 0,25-0,5 l E. 5 l
C. 0,5-0,75 l

14. Retention of water in blood plasma and prevention of tissue swelling


depends from the oncotic pressure of plasma, which depends from the
content of the next compounds:
A. Albumins D. Lipids of lipoproteins
B. Immunoglobulins E. Inorganic ions
C. Fibrinogen

15. The asymmetry of sodium and potassium ions distribution across


plasma membrane of the cell is supported by the next membrane
constituent:
A. K,Na-dependent ATP-ase membrane
B. Sodium selective channel D. Alkaline phosphatase
C. Lecithine content of the E. Cholesterol content

16. Concentration of sodium ion in blood plasma of healthy adult is:


A. 105 mMoles/l D. 0,9 g/dl
B. 25 mMoles/l E. 1,5 g/l
C. 300 mMoles/l
17. Under hypoosmolar dehydratation could be:
A. Dryness of the skin and C. Muscle hypotonia
mucosal structures D. Collapse
B. Headache E. Blood closeness

18. Extracellular hyperhydratation can be induced by:


А. Oncotic pressure decreasing, С. Increasing of hydrostatic pressure
caused by hypoproteinemia after of the blood
proteins loosing through kidneys D. Alterations in a heart work
В. Decreasing of sodium level Е. Increasing of sodium level

19. Find a compound form the list, which does not contain iron:
А. Histamine D. Cytochromes
В. Muscle’s myoglobin Е. Hemoglobin of RBCs
С. Ferritin and homosiderrin

20. Which enzyme from the list below contains the copper?
А. Cytochromeoxidase D. Aminoxidase
В. Alkaline phosphatase Е. Tyrosinase
С. Alcoholdehidrogenase

21. In highlander the thyroid gland is bigger as normal. It is associate with


failing in nutrition next microelement:
А. Iodine С. Copper
В. Brome Е. Mangan

22. Please, specify the function, which is not characteristic for calcium in
human organism:
А. Calcium ions are cofactors for hemostasis
many enzymes D. Calcium ions play a role in skeletal
В. Calcium ions are the second muscles contraction
messenger in cell signaling Е. Calcium salts are responsible for
С. Calcium ions play a role in hard structure of bones

23. Concentration of calcium ion in urine of healthy adult is:


А. 2,5-7,5 mmol D. 55-105 mmol
В. 5,5-10,5 mmol Е. 0,25-0,75 mmol
С. 25-75 mmol

24. Specify the microelement which is the part of T 3 and T4:


А. Iodine D. Iron
В. Fluor Е. Selenium
С. Brome

25. Specify the microelement with antioxidant function:


А. Selenium В. Fluor
С. Brome Е. Iodine
D. Iron

26. Which electrolyte has the major role in the volume of the extracellular
water homeostasis?
А. Sodium D. Iron.
В. Fluor Е. Iodine
С. Brome

27. Find from the list below the symptom of zinc deficiency:
А. Taste disorders D. Alterations in the hair growth
В. Affecting of the immune system Е. Dermatitis
С. Olfaction disorders

28. In cell the content of ___ is higher, than other microelements:


А. K+ D. Fe3+
В. Na+ Е. Ca2+
С. I+

29. Specify the mechanism of aldosterone action on the electrolytes


metabolism:
А. Decreasing of the Na+ excretion D. Decreasing of the K+ excretion
В. Increasing of the Na+ excretion E. Increasing of the K+ excretion
С. Decreasing of the water excretion

30. The penetration of the irritable cell membrane for potassium ions has
been increased during an experiment. What changes of membrane electric
status can occur?
A. Hyperpolarization D. Local response
B. Depolaization E. No changes
C. Action potential

31. Deposition of calcium in bones is controlled by the next hormone:


A. Calcitonin D. Calmodulin
B. Thyroid hormone triiodothyronin E. Calcitriol
C. PTH (parathyroid hormone)

32. The cause of endemic goiter is deficiency of trace element:


A. Iodine D. Copper
B. Iron E. Manganese
C. Zinc

33. Copper as trace element is essential component of the next enzyme:


A. Cytochrome oxidase D. Xantine oxidase
B. Amylase E. Carbonic anhydrase
C. Trypsin
34. In kidney diseases a decrease of glomerular filtration is observed, which
leads to decrease of elimination of final metabolic products from the body.
What substances are mostly retained in the body?
A. Creatinine and urea D. Ammonia and polyamines
B. Uric acid and indicane E. Glucose and glucosamine
C. Amino acids

35. A part of food protein is not digested in intestinal tract and is


decomposed by bacterial enzymes in large intestine. This is defined as
putrifaction. of protein. Detection of what substance in urine may serve as
indicator of the intensity of putrifaction processes?
A. Animal indican D. Creatine and creatinine
B. Protein E. Urates
C. Urea

36. The concentration of glucose in normal human urine is:


A. 0,00 g/l D. 10 mMole/l
B. 0,1 g/l E. 2 mMole/l
C. 30 mg%

37. In patients with ill kidneys even in normally balanced diet


osteodystrophic changes frequently are developing. It can be caused by
disorder in biosynthesis of:
A. 1,25- C. Cholecalciferole
dihydroxycholecalciferole D. Ergocalciferole
B. 7-dehydrocholesterol E. 25-hydroxycholecalciferole

38. In acidosis the excretion of ammonium ion with urine is:


A. Increased C. Without changes
B. Decreased

39. In 4 monthes child a “syndrome of blue clothes” is recognized, which is


accomoanied by periodical feaver, enhanced exitability, growth retardation.
Blood nitrogen is increased, in the urine an excess of animal indicane is
detected. What amino acid absorption is impaired?
A.Tryptophan D.Lysine
B.Thyrosine E.Histidine
C.Phenylalanine

40. After consumption of great quantity of watermelon the urine medium


(pH) became:
A. Alkaline D. Strong acidic
B. Neutral E. Strong alkaline
C. Weak acidic
41. Glucose is the threshold substance, the value of glucose threshold is
the next:
A. 180 mg% % D.20 mMole/l
B.5 mMole/l E.1 g/l
C. 135 mg

42. What hormone control water reabsorption in kidneys?


A. Vasopressin D. Testosterone
B. Parathyroid hormone (PTH) E. Insulin
C. Calcitonin

43. Excretion of sodium ions with urine is cotrolled by:


A. Aldosteron D. PTH
B. Oxytocine E. Melatonine
C.Adrenalin

44. The principal buffer system of urine is the next:


A. Phosphate D. Ammonium ions
B. Bicarbonate E. Proteins
C. Citric acid

45. The end product of purine metabolism,excreted with urine is:


A. Uric acid D. Arginine
B. Ammonium ion E. Creatinine
C. Urea

EXAMPLES OF KROK 1 TESTS

46. Dietary intake of a 30 year old nursing woman contains 1000 mg of


calcium, 1300 mg of phosphorus and 20 mg of iron per day. It is necessary
to change content of these mineral substances in the following way:
A To increase phosphorus content D. To increase iron content
B. To increase calcium content E. To reduce iron content
C. To reduce fluorine content

47. In a patient after thyroid gland surgery crumps and convulsions


appeared, hyperreflexia and hyperexcitability were observed. Preliminary
diagnosis is hypocalcemia, which may be caused by:
A. Hypoparathyreosis, induced by parathyroid gland
extirpation of parathyroid glands C. Overdose of vitamin E
B. Hyperparathyreosis due to tumor D. Overdose of vitamin D
developement from cells of E. Insulin insufficiency

48. After prolong vomiting and diarrhea in a patient is observed tachycardia,


skin dryness, muscular hypotonia, apathia. Such state can be caused by:
A. Hypomagnesemia B. Hypopotassemia
C. Hyperpotassemia E. Hypercupremia
D. Hypocupremia

49. In a patient a development of calcium deposits in soft tissues, stones in


urinary ducts are observed. Preliminary diagnosis is hypercalcemia, which
may be caused by:
A. Hyperparathyreosis due to tumor thyroid gland surgery
development from cells of parathyroid C. Vitamin D deficiency
glands D. Insulin insufficiency
B. Hypoparathyreosis, induced by E. Insulin overproduction
injury of parathyroid glands during

50. A 50-year-old patient complains about general weakness, appetite loss


and cardiac arrhythmia. The patient presents with muscle hypotonia, flaccid
paralyses, weakened peristaltic activity of the bowels. Such condition might
be caused by:
A. Hypokaliemia D. Hyperkaliemia
B. Hypophosphatemia E. Hyponatremia
C. Hypoproteinemia

51. A 35 y.o. patient who often consumes alcohol was treated with diuretics.
There appeared serious muscle and heart weakness, vomiting, diarrhea, AP-
100/60 mm Hg, depression. This condition is caused by intensified excretion
with urine of:
A. Potassium D. Calcium
B. Sodium E. Phosphates
C. Chlorine

52. Patient N. was admitted to clinic with complaints for pains in renal areas.
The most informative biochemical indexes for evaluation of kidney
functional state is determination of:
A. Creatine in blood serum and in blood plasma
urine D. Creatine in red blood cells and in
B. Creatinine in blood serum and in plasma
urine E .Creatine and creatinine in blood
C. Creatinine in red blood cells and in plasma.

53. In urine of patient B. were detected phenylpyruvic, phenyl-lactic and


phenylacetic acids. What enzyme deficiency may cause this phenomenon?
A. Phenylalanine hydroxylase D. Fumaryl acetoacetate
B. DOPA decarboxylase hydrolase
C. Thyrosine-3-monooxygenase E. Cystathionine –β-synthase

54. Urine of a patient in reaction with iron chloride gives purple-red


coloration, by exposure on air is quiqely darkening. Patient complains for
pains in junctions. What substance metabolism is impaired in this case?
A. Homogentisic acid D. Pangamic acid
B. Para-aminobenzoic acid E. Fumarylacetoacetic acid
C. δ–Aminolevulinic acid

55. In 15 years old boy, suffering from alkaptonuria, urine after standing
changes to a black color. This disease is hereditary disorder of:
A. Tyrosine metabolism D. Uric acid biosynthesis
B. Alanine metabolism E. Cysteine metabolism
C. Urea synthesis

56. In urine of patient M. a high content of argininosuccinate is detected.


What enzyme deficiency exists in a body?
A. Argininosuccinate lyase synthetase
B. Arginase E. Tryptophan-5-
C. Argininosuccinate synthetase monooxygenase
D. Carbamoyl phosphate

57. In a patient argininemia and argininuria is observed. Urea content in


blood and in urine is decreased. What enzyme deficiency has a place?
A. Glutamate dehydrogenase D. Argininosuccinate synthetase
B. Arginase E. Tryptophan-5-
C. Ornithine carbamoyl monooxygenase
transferase

58. 65 years old man , suffering from gout, complains for pains in kidney
area. In course of ultrasound diagnostics renal stones were detected..
Increased concentration of what substance is the most probable cause of
stones formation in this case?
A. Bilirubin D. Cystine
B. Uric acid E. Cholesterol
C. Urea

59. In patient S. blood glucose level is over the renal threshold, polyuria is
observed, as well as acidosis and ketonuria. What pathological state can be
suggested?
A. Diabetes mellitus D. Addison disease
B. Hypercorticism E. Hyperthyreosis
C. Starvation

60. In 13 years old girl a hypotension and polyuria is observed. Preliminary


diagnosis – diabetes insipidus. It is caused by deficiency of:
A. Vasopressine D. Adrenalin
B. Cortisol E. Oxytocine
C. Aldosterone

61. In diabetes mellitus and during starvation an increase of ketone bodies


in blood and in urine is observed. Chose the substance from which ketone
bodies are produced:
A. Acetyl-CoA D. ΑKetoglutarate
B. Citrate E. Malate
C. Succinyl CoA

62. Excretion of ammonium cation with urine is increased in the next condition:
A. Metabolic acidosis D. Hypoproteinemia
B. Respiratory alkalosis E. Obesity
C. Hyperlipidemia

63. In patient T. erythropoietic porphyria was recognized (Gunter”s disease):


urine and teeth are of redish shadow due to uroporphyrin
accumulation.Insufficiency of what enzyme from listed below may be
observed?
A.Uroporphirinogen synthase D.Tryptophan pyrrolase
B.Coproporphyrinogen oxidase E.Heme synthase
C.Uroporphyrinogen-decarboxylase

64. In boy P. suffering from mental underdevelopement the consumption of


milk induces vomiting and diarrhea. In blood is detected glucose-1-
phosphate on a background of low concentration of glucose and very high
contentratyion of reducing sugars. In urine is detected galactose. Indicated
symptoms are connected with deficiency of:
A.Galactose-1-phosphate uridyl C.Lactase
transferase D.UDP-glucose-4 epimerase
B.UDP-glucose pyrophosphorylase E.Galactokinase

65. In a patient a Konovalov-Wilson disease was recognized, at which


excretion of copper with urine is observed, as well as deposition of copper
in organs and tissues. What protein biosynthesis is damaged as the most
probable cause of this disease?
A. Ceruloplasmin D.Properdin
B. Transferrine A. Cryoglobuline
C.Haptoglobin

66. Physician paid no attention to analysis, which showed a tenfold increase


of diastase activity in urine. Patient has an emergency of pancreas autolysis
due to activation of the following enzyme:
A.Amylase D.Lipase
B.Pepsin A.Nuclease
C.Trypsin

67. A patient addressed to the physician with complaints for pains in small
joints as well as in temporomandibular joint. Joints are enlarged, looking as
thick nodes. In blood serum and in urine an increase of urates content is
detected. Metabolism of what substances is impaired?
A.Purine bases D.Pyrimidines
B.Amino acids E.Glycerol
C.Glucose

68. After consumption of fatty meal patient feels, after some time
steatorrhea appears. Cholesterol concentration in blood is 9,2 mmoles/l, test
for bile acids in urine is positive. A cause of this state may be the deficiency
of:
A.Bile acids D.Phospholipids
B.Fatty acids E.Chylomicrons
C.Triglycerides

69. A 58 years old woman, in a heavy state, cloudy conciousness, dry skin,
eyeballs are, cyanosis, an odor of spoiled apples in expired air. Blood
glucose level 15,1 mmol/l , in urine – 3,5 %. This state is caused by:
A.Hyperglycemic coma D.Anaphylactic shock
B.Uremic coma E.Hypovolemic coma
C.Hypoglycemic coma

70. A 13 years old patient. Complains for general weakness, fatigue. Mental
underdevelopement is observed. In course of investigation a high
concentration of valine, isoleucine, leucine is detected in blood and in urine.
Urine has specific odor. What may be the cause of that pathology?
A.Maple syrup disease D.Thyrosinosis
B.Basedov disease E.Histidinemia
C.Addison disease

71. In patient S. blood glucose level is over the renal threshold, polyuria is
observed, as well as acidosis and ketonuria. What pathological state can be
suggested?
A.Diabetes mellitus D.Addison disease
B.Hypercorticism E.Hyperthyreosis
C.Starvation

72. In patients with ill kidneys even in normally balanced diet


osteodystrophic changes frequently are developing. It can be caused by
disorder of biosynthesis of:
A.1,25-dihydroxycholecalciferole D.Ergocalciferole
B.7-dehydrocholesterol E.25-hydroxycholecalciferole
C.Cholecalciferole

73. A patient complains for polyuria (5 liters daily) and thurst. Blood glucose
level is 5,1 mmoles/l, urine specific density is equal 1,010. Glucose and
ketone bodies are absent. What may be the cause of this state?
A.Diabetes insipidus D.Diabetes mellitus
B.Mixedema E.Thyrotoxicosis
C.Steroid diabetes

74. 47 years old woman complains for persistent feeling of thurst, rapid
fatigue, loss of conciousness. Daily diuresis is 3-4 l. Blood glucose level is
4,8 mmoles/l, in urine there is no glucose.In this case it is resonable to
investigate blood content of:
A. Vasopressine D. Cortisole
B. Estrogens E. Thyroxine
C. Aldosterone

75. In a suckling child the darkening of sclera, mucous sheets, ear bowels is
observed, the urine also is developing a dark color after exposure on air. In
blood and urine a homogentisic acid is detected. What pathology can be
suggested?
A.Alcaptonuria D.Cystinuria
B.Porphyria A.Hemolytic anemia
C.Albinism

CLINICAL CASES AND SITUATIONAL TASKS

76. A 22-year-old soldier collapses from dehydration during maneuvers in


the desert and is sent to a military hospital. Prior to enlisting, a physician
observed a high level of glucose in his urine during an examination. At first,
he was not allowed to enlist because he was suspected of being a diabetic.
Further tests, however, determined that his insulin level was normal. A
glucose tolerance test exhibited a normal pattern. Laboratory tests following
his dehydration episode repeat the previous findings, but further testing of
the urine reveals that only D-glucose is elevated. Other sugars were not
elevated. Deficiency in which transporter caused the dehydration of this
patient’s?

Answer: The patient has normal levels of blood insulin and exhibits a normal
glucose tolerance test. This indicates that glucose absorption from the intestine is
normal as is clearance of glucose from the blood. The presence of glucose in the
urine is most likely a kidney problem. Because the defect seems to involve only D-
glucose and no other sugar, this points to a transporter with high specificity. The
kidney has the GLUT 2, SGLT1, and SGLT2 transporters. GLUT 2 and SGLT1 are
present in other tissues, and a defect in these would be expected to result in more
serious sequelae. SGLT2 is a sodiumdependent glucose transporter specific to
the kidney that has a high specificity for glucose. The glucose is present in the
urine because of a failure to reabsorb it as a consequence of a defect in SGLT2.
This leads to a loss of water also, because it is reabsorbed with glucose.
SECTION XVII
BIOCHEMICAL CONSTITUENTS OF CONNECTIVE TISSUE

1. What biochemical process is not typical for connective tissue of elderly


persons?
A. Reduction of collagen D. Reduction of
B. Reduction of water glycosaminoglycans
C. Increasing of collagen E. Reduction of hyaluronic acid

2. What amino acids are rare in elastin - a protein component of elastin


fibers?
A. Nonpolar D. Acidic
B. Polar E. Basic

3. Name hormones that inhibit synthesis of proteoglycans and collagen in


connective tissue:
A. Glucocorticoids D. Insulin
B. Somatomedins E. Androgens
C. Somototropin

4. Increased content of oxyproline in blood and urine with lesions of joints


and bones is observed due to increased catabolism of:
A. Collagen. D. Proteoglycans.
B. Hyaluronic acid. E. Elastin.
C. Glycosaminoglycans.

5. What amino acids, which comprise the structure of collagen, are


genetically encoded?
A. Hydroxyproline, hydroxylisine D. Glutamic acid, glutamine
B. Serine, glycine E. Phenylalanine, tryptophan
C. Alanine, valine

6. Influence of hypovitaminosis C on the structure of collagen fibers caused


by decreased activity of enzymes:
A. Lysyl hydroxylase, proline hydroxylase D. Procollagen peptidase
B. Lysyl oxidase, lysyl hydroxylase E. Collagenase.
C. Glycosil transferase

7. Hydroxyproline is an important amino acid within the collagen


biosynthesis. Indicate vitamin which is participated in the formation of this
amino acid by hydroxylation of proline?
A. C D. B2
B. D E. B6
C. B1
8. Defection of the structure of collagen fibers occurs during deficiency of
vitamin C due to the fact that this vitamin is a cofactor of:
A. Lysine hydroxylase, proline C. Glycosil transferase
hydroxylase D. Procollagen peptidase
B. Lysine hydroxylase and collagenase E. Collagenase

9. Indicate the most common amino acid residues in collagen.


A. Oxyproline, oxylysine, glycine, proline D. Tryptophan, oxylysine,
B. Tryptophan, cysteine, glycine, cysteine, valine
methionine E. Asparagine, glutamine, lysine
C. Lysine, arginine, cysteine, tryptophan

10. Gram-positive bacteria produce a protective capsule of hyaluronic acid,


which increases their pathogenicity. What enzyme can "destroy" the
protective capsule?
A. Hyaluronidase D. Alpha glycosidase
B. Carboxypeptidase E. Collagenase
S. Aminopeptidase

11. What class of glycosaminoglycans, due to the large number of carboxyl


groups, binds large amounts of water and supports tissue turgor?

A. Hyaluronic acid. D. Heparin.


B. Dermatan sulfate. E. Chondroitin sulphate
C. Keratan sulfate.

12. Insolubility of collagen connective tissue and its metabolic resistance to


various agents is caused by amino acid composition and special structure
of the protein. Indicate amino acids that quantitatively dominate in the
structure of collagen?
A. Glycine, proline, alanine D. Cysteine, threonine, glutamine.
B. Methionine, serine, threonine. E. Arginine, histidin,
C. Phenylalanine, tyrosine, tryptophan. phenylalanine.

13. Protein procollagen is synthesized by fibroblasts of connective tissue.


Thereafter glycosylation occurs. What carbohydrates bind to procollagen?
A. Galactose, glucose D. Arabinose, sucrose
B. Fructose, mannose E. Ribulose, xylulose
C. Ribose, deoxyribose

14. Collagen and elastin are fibrillar elements of connective tissue. Specify
the amino acid, which is a component only of the collagen and its
determination is used to diagnose several disorders of connective tissue:
A. Hydroxyproline D. Lysine
B. Proline E. Hydroxylysine
C. Glycine
15. After wound healing a scar was formed. What is the main component of
this type of connective tissue?
A. Collagen D. Chondroitin sulfate
B. Elastin E. Keratan sulfate
C. Hyaluronic acid

16. What factors trigger the biosynthesis of glycosaminoglycans?


A. Vitamin A, manganese ions D. Vitamin C and copper ions
V. Vitamin D, iron ions E. Vitamin E, zinc ions
S. Vitamin D, calcium ions

TESTS FOR KROK 1

17. In a 63 years old women suffering from rheumatism, increased


concentration of oxyproline in blood and urine was detected. What is the
main reason of this state?
A. Degradation of collagen C. Renal impairment
B. Activation of prolyl hydroxylase D. Activation of cathepsins

18. A patient with burn disease is at the risk of blood clots formation.
Thrombus formation may also take place in other diseases 
atherosclerosis, hypertension, myocardial infarction. What polysaccharide
is used to prevent formation of blood clots?
A. Heparin D. Hyaluronic acid
B. Amylose E. Chondroitin-4-sulfate
C. Starch

19. Results of blood and urine tests indicate that the content of glucuronic
and sialic acids are above normal. Catabolism of which of the following
compounds is enhanced?
A. Glucosamine glican D. Elastin
B. Glycoprotein E. Calmodulin
C. Collagen

20. A patient has symptoms of active forms of rheumatism. Clinical


examination did not confirm this diagnosis. It should be also emphasized
that rheumatism is a connective tissue damage, ie destruction
heteropolysaccharides within glycoproteins. What biochemical blood and
urine tests may be carried out to clarify the diagnosis?
A. Determine aminosugars, sialic acids D. Identify glucocorticoids, 17-
B. Determine glycoproteins, total nitrogen ketosteroids
C. Determine glucose, albumin, globulins E. Determine ketone bodies

21. For the resorption of keloids a hyaluronidase is used. What biochemical


process causes advantages of enzymotherapy?
A. Cleavage of hyaluronic acid. B. Cleavage of heparin.
C. Cleavage of chondroitin sulfate. E. Cleavage of collagen.
D. The synthesis of glycosaminoglycans.

22. Mukopolysaccharydoses are hereditary diseases that manifested


pathological changes during bones and joints formation. What urine index
indicates this disease?
A. Excessive excretion of C. Excessive excretion of lipids.
glycosaminoglycans. D. Excessive excretion of glucose.
B. Excessive excretion of amino acids. E. Excessive excretion of proteins.

23. A patient was hospitalized with impaired vascular permeability. Specify


the protein of connective tissue which is disturbed at such conditions.
A. Collagen D. Tropomyosin
B. Myoglobin E. Ceruloplasmin
C. Albumin

24. Examination of a 2 years old child revealed deformation of the spine,


joints; lagging in growth, progressive coarsening of the face, deafness,
thickening of the skin. Increased excretion of dermatine sulfate and heparin
sulfate were determined in urine. Deficit of which from the following
enzymes caused these changes?
A. α-L-iduronidase. D. Glucose-6-phosphatase.
B. N-sulfamidase. E. β-glucoseamine– N-
C. N-acetyl glucose aminidase. acetyltransferase.

25. In inflammatory diseases of joints decreasing in viscosity of synovial


fluid is observed. The reason of this disorder might be:
A. Reducing of the number and D. Reduction of protein
depolymerization of hyaluronic acid. concentration.
B. Reduction of the number and E. Reducing of the number and
depolymerization of proteoglycans. depolymerization of chondroitin
C. Reducing of the number and sulfate.
depolymerization of glycoproteins.

26. In a patient with scurvy a disorder of hydroxylation of proline and lysine


within the collagen molecule was observed. What biochemical process is
inhibited?
A. Microsomal oxidation D. Peroxidase oxidation
B. Oxidative phosphorylation E. Tissue respiration
C. Lipid peroxidation

27. The process of destruction of connective tissue due to collagenoses


was observed. What laboratory tests should be assigning to a patient with a
chronic form of collagenosis?
A. Content of oxyproline and oxylisine in B. The activity of LDH isoenzymes
blood serum and urine C. The level of urates in the blood
D. C-reactive protein E. Transaminase activity

28. A 35-year-old patient consulted a dentist about low density of dental


tissues, increase fragility of teeth after eating solid food. In order to
determine Ca/P relation a scrape of enamel was sent to the laboratory. What
value of this index is suggestive of intensified demineralization?
A. 0.9 D. 1.85
B. 1.67 E. 3.5
C. 1.5

29. Examination of a patient with Slaj syndrome revealed urinary excretion


of heparin sulphate and chondroitin sulfate. This condition is likely to be
caused by the deficiency of the following substance.
A. -Glucuronidase C. Lactate dehydrogenase
B. -amylase D. Arylsulfatase A
E. Cathepsin D

30. A patient diagnosed with defects of the formation of enamel and dentin
of teeth due to low content of calcium in his blood. Such changes are
caused by deficiency of the next hormone:
A. Parathormone D. Somatotropin
B. Thyrocalcitonin E. Thyroxine
C. Triiodothyronine

31. Treatment of enamel erosion with dissolved in oil vitamin D3 was


ineffective. The main reason for this may be a disorder of:
A. Bile production D. Digestion of proteins
B. Functioning of gonads E. Functioning of the pancreas
C. Digestion of carbohydrates

32. Analysis of a patient with frequent bleeding in internal organs and


mucous membranes revealed proline and lysine in collagen fibbers. This
might be caused by the deficiency of the next vitamin:
A. C D. A
B. E E. D
C. K
CLINICAL CASES AND SITUATIONAL TASKS

33. Some pathogens break down hyaluronic acid, releasing the enzyme
hyaluronidase. Explain their advantages over the microorganisms lacking
hyaluronidase activity.

Answer: Hyaluronidase allows certain pathogens to penetrate the body through


mucous membranes and skin, causing hydrolysis (depolymerization) of hyaluronic
acid, and thereby promotes their distribution in the human body. Determination of
degradation products of connective tissue such as aminotsugars, glucuronic and
sialic acids in blood and urine is widely used for indication of such
microorganisms. Gram-positive bacteria produce a protective capsule, consisting
from hyaluronic acid, which increases their pathogenicity.

34. Patient with burn disease is at the risk of formation of blood clots in
blood vessels. What glycosaminoglycan may be used to prevent formation
of blood clots?

Answer: Anticoagulant heparin belongs to derivatives of glycosaminoglycans. Its


biochemical mechanism of action is achieved due to the fact that it binds to
inhibitor of coagulation factor being antithrombin III. This leads to conformational
changes of antithrombin III and thereby increasing of its resistance to thrombin.
Preserved in such a way antithrombin prevents intravascular coagulation.

35. A patient revealed the fragility of blood vessel walls, increased bleeding,
decreasing strength and elasticity of the skin. What component of the
biosynthesis of connective tissue is affected? What is the most likely
diagnosis?

Answer: Vitamin C plays the role of a coenzyme in hydroxylation of proline and


lysine while protocollagen is converted to collagen (i.e. post-translational
modification). The hydroxylation reaction is catalysed by lysyl hydroxylase (for
lysine) and prolyl hydroxylase (for proline). This reaction is dependent on vitamin
C, molecular oxygen and a-ketoglutarate. Hydroxyproline and hydroxylysine are
essential for the collagen cross-linking and the strength of the fiber. In this way,
vitamin C is necessary for maintenance of normal connective tissue and wound
healing process. Deficiency of this vitamin leads to disruption of the formation of
cross-linking, resulting in declining density and mechanical properties of collagen
fibbers. Such symptoms are observed in scurvy.

36. The most abundant heteropolysaccharides of the connective tissue are


the glycosaminoglycans. These molecules are long unbranched
polysaccharides containing a repeating disaccharide unit. Indicate the
following glycosaminoglycans:
a b

c d

e f

Answer: a – hyaluronic acid, b – dermatan sulfate, c – chondroitin 4-sulfate, d –


chondroitin 6-sulfate, e – keratan sulfate, f – heparin.
SECTION XVIII
BIOCHEMISTRY OF SALIVA AND TOOTH TISSUE

1. What substance cause a viscosity of saliva and protect oral mucosa


against harmful agents, mechanical damage including ?
A. Mucine D. Kallicreine
B. Lysozyme E. Glucose
C. Amylase

2. Parodontitis is accompanied with activation of proteolytic degradation of


parodont tissue. The marked increase in content of what substances in oral
fluid indicates on activation of proteolysis?
A. Amino acids D. Glucose
B. Biogenic amines E. Cholesterol
C. Organic acids

3. Concentration of proton ions in saliva of healthy adults corresponds to


pH 6,4-7,8. What process in enamel is favored during shift of salivary pH to
acidic side (below pH 6,2)?
A. Surface enamel demineralization D. Calcification
B. Mineralization of enamel surface E. Fluorosis
C. Increase in resistancy to acids

4. Normal pH of saliva is 6.4-7.8. The shift of pH to alkaline region (pH 7.8)


favors the next process:
A. Deposition of Ca and phosphate ions D. Release of calcium from tooth
into surface of enamel tissue
B. Decrease in resistance of tooth tissue E. Release of phosphate from
to the action of cariesogenic factors tooth tissue
C. Enamel demineralization

5. One of the functions of saliva is a protective function. It consists in


formation of local resistance of mucosa toward bacteria due to secretion
by parotid glands of the following protein:
A. Secretory IgA D. Fibrinogene
B. Elastine E. Albumin
C. Collagene

6. Decrease in activity of what enzyme indicates on the hypofunction of


parotid gland?
A. Amylase D. Lysozyme
B. Glucokinase E. Phosphatase
C. Maltase
7. What component of saliva is produced in salivary as well as other glands
and provides a viscosity to saliva?
A. Mucine D. Hyaluronic acid
B. Amylase E. Phosphates
C. Maltase

8. Protective function of saliva is providet by several mechanisms, one of


them includes enzyme, which exhibits bactericidal effect by cleavage a
polysaccharide complex of cell wall of staphylococci and streptococci.
How is called this enzyme?
A. Lysozyme D. Collagenase
B. α-Amylase E. β-Glucuronidase
C. Oligo-1,6-glucosidase

9. Mucine is one of the principal constituents of saliva. According its


chemical composition it belongs to the next class of conjugated
proteins:
A. Glycoproteins D. Lipoproteins
B. Nucleoproteins E. Metalloproteins
C. Phosphoproteins

10. There exist a number of factors which change the pH in oral cavity. What
consequences have a decrease of pH of oral fluid below 6.4?
A. Enamel deminaralization and caries D. Lowering of protective
development properties of oral fluid
B. Activation of starch breakdown E. Deterioration of oral cavity
C. Enhancement of enamel hygiene
mineralization

11. Note the optimal range of oral fluid pH for favourable mineralization and
remineralization of tooth tissue.
A. 7,2-7,4 D. 6,2-6,0
B. 7,4-7,8 E. 5,8-6,0
C. 6,4-6,6

12. What proteins in saliva prevent the precipitation of calcium phosphate


salts and support them in a colloidal state?
A. Proline rich proteins of saliva D. Blood serum globulins
B. Lyzozyme E. Seromucoids
C. Blood serum albumin

13. It is known, that salivary glands produce several hormones. Which of


them favours the mineralization of teeth?
A. Parotin C. Epidermal growth factor
B. Calcitonin D. Nerve growth factor
E. Thymocytes transforming factor

14. What ion component of saliva which concentration corresponds


approximately to that in blood plasma is the most important in
mineralization of tooth tissues?
A. Calcium D. Phosphate
B. Sulphate E. Potassium
C. Bicarbonate

15. How is changed mineralization of hard tissues of tooth in patients with


salivary stone disease?
A. It is altered proportion of mineral C. It is increased
constituents of saliva D. It is no changes
B. It is decreased

16. Increase in content of what hormones in saliva characterizes the


development of stress reaction in the organism?
A. Cortisol, adrenalin D. Thyroid hormones
B. Testosteron, estradiol E. Insulin
C. Parathyroid hormone, calcitonin

17. In enzymes of oral cavity an important protective significance have


enzymes, which neutralize free radicals. Indicate these enzymes.
A. Myeloperoxidase and lactoperoxidase D. Hyaluronidase and collagenase
B. Lysozyme and amylase E. Acid and alkaline phosphatases
C. Proteinases and nucleases

18. In oral cavity occur initial stages of digestion. What enzyme of oral
cavity degrades polysaccharides?
A. Alpha-Amylase D. Peroxidase
B. Maltase E. Catalase
C. Saccharase

19. An important significance in mineralization of enamel plays salivary


alkaline phosphatase, which provides the following process:
A. Increase of inorganic phosphate D. Decrease of calcium and
concentration in saliva phosphate concentration in
B. Decrease of calcium concentration in saliva
saliva E. Decrease the resistance of
C. Decrease of inorganic phosphate enamel towards cariesogenic
concentration in saliva factors

20. For prevention of proteolysis of own proteins salivary glands produce


inhibitors of proteinases. Note an inhibitor of proteinases, which is
produced in salivary glands.
A. Acid stabile inhibitor of proteinases D. Alpha-2-macroglobulin
B. Alpha-1-antitrypsin E. Neuraminidase
C. Superoxide dismutase

21. What type of immunoglobulins is prevailing in saliva and provides local


immunity of mucosa layers in oral cavity?
A. IgA D. IgG
B. sIgA E. IgM
C. IgE

22. Daily produced amount of saliva depends from age, gender, nutrition
etc. In majority of adults it corresponds to the next mean value:
A. 1,5-2,0 L D. 0,3-0,7 L
B. 1,0-1,5 L E. 0,8-1,6 L
C. 0,5-1,0 L

23. Mucine is a mixture of glycoproteins of mucinous type, which is


secreted by different salivary glands. It fulfils the next function in oral
cavity:
A. All answers are correct C. Inhibits diffusion of ions into
B. It makes an envelope for mucose hard tissues of teeth
layer of oral cavity and teeth, thus D. Binds calcium ions
protecting them from damage. E. Supports buffer properties of
oral fluid.

24. Salivary amylase hydrolyses α-1-4 glucosidic bonds in molecules of


polysaccharides. What ion is necessary for stabilization of active centre
structure in this enzyme?
A. Chloride D. Magnesium
B. Calcium E. Selenium
C. Zinc

25. Saliva contains about 50 enzymes, including acid and alkaline


phosphatases. What is the significance of phosphatases in physiology of
oral cavity?
A. They release inorganic phosphate C. They change permeability of
from phosphorus containing organic tissues, including tooth enamel.
substances and favour mineralization D. They possess antioxidant
of bones and teeth. function and protect oral
B. They are an important protective mucosa
factor against pathogenic gram- E. They inhibit proteolytic
positive bacteria of oral cavity. cleavage of salivary proteins.

26. In saliva are present proteins which possess antibacterial properties


and participate in mineral metabolism.
A. All answers are correct D. Cystatins
B. Acidic proline rich proteins E. Histatins
C. Statherins and lactoferrin

27. Antioxidant enzymes in saliva play important protective role in


decomposition of reactive oxygen. Find from the list one which belongs to
selenium-dependent enzymes.
A. Glutathione peroxidase C. Catalase
B. Superoxide dismutase D. Myeloperoxidase

28. Metabolites of TCA cycle plays in the human body range of functions.
Which of these metabolites found in saliva (standard 0.002 - 0.02 g /l), is
involved in the regulation of blood calcium level and influences the
mineralization of teeth?

A. Citrate D. Fumarate
B. Isocitrate E. α-ketoglutarate
C. Malate

29. An important role in developing of tooth tissues plays parotyn. In which


of the following glands it is synthesized?

 A. In the salivary glands D. In the hypothalamus


B. In the thyroid gland E. In the thyroid gland
C. In the pancreas

30. Saliva contains proteins that play an important role in the oral cavity.
What is normal total content of proteins in the liquid?

 A. 2.0 - 5.0 g\l D. 2.0 - 4.0 g\l


B. 0.15 - 0.25 g\l E. 1.0 - 3.25 g\l
C. 0.6 - 0.8 g\l

31. It is known that phosphoproteins (proteins with postranscriptional


modification) are involved in the mineralization of bone and tooth.
Phosphoric acid in these proteins is mainly associated with…:
A. Serine D. Glutamate
B. Glycine E. GABA
C. Methionine

32. Human body obtains vitamins, macro-and micronutrients with dietary


uptake. Dental fluorosis is highly associated with the excess of the next
compound?
A. Fluorine D. Sodium
B. Phosphorus E. Potassium
C. Calcium
33. Enamel is highly resistant to various mechanical and chemical factors.
Synthesis of what component provides the resistance?
A. Fluorapatite D. Hydroxyapatite
B. Carbonate apatite E. Collagen
C. Chlorine apatite

34. The strength of tooth tissues depends on the ratio of organic, inorganic
compounds and water. Which of the following tissues contain the least
amount of water?
A. Enamel D. Pulp
B. Dentine E. Cement
C. Periodontium

35. Deposition of mineral salts occurs in the organic matrix of the tooth
prior to the accelerated synthesis of one of the following polysaccharides:
A. Chondroitin sulfate D. Ceratan sulfate
B. Heparin E. Dermatan sulfate
C. Glycogen

36. Citrate is involved in the mobilization of calcium. What tooth tissue


contains the highest concentration of citrate?
A. Dentine D. Enamel
B. Pulp E. Epithelium
C. Cement

37. Fluoride enters the human body mainly with water. What concentration
of fluoride in water can predict the development of dental caries?
A. Up to 0.5 mg / l D. Up to 2.0 mg / l
B. Up to 1.0 mg / l E. Up to 2.5 mg / l
C. Up to 1.5 mg / l

38. Indicate amino acid, essential for the formation of collagen and organic
matrix of the tooth.
A. Oxyproline D. Threonine
B. Tyrosine E. Homoserine
C. Serine

39. Organic matrix of enamel is represented by a variety of compounds,


including proteins. Specific proteins of enamel include...
A. Amelogenin, ameloblastin, enamelin D. Proteoglycans
B. Collagen E. Albumins
C. Elastine
40. Formation of organic matrix is one of the stages of enamel
mineralization. Name the protein that plays a leading role in this process.
A. Collagen D. Globulin
B. Fibronectin E. Elastin
C. Albumin

41. Mineral base of tooth tissue is formed with different apatite crystals.
What type of apatite is dominated in the mineral component of tooth tissue?
A. Hydroxyapatite D. Fluorapatite
B. Carbonate apatite E. Strontium apatite
C. Chlorine apatite

42. Antioxidant protection of oral mucosa provides a number of enzymes.


Select one of them:
A. Superoxide dismutase D. Aldolase
B. Cytochrome oxidase E. Glucose-6-phosphatase
C. Hexokinase

43. Proper metabolism of the periodontal tissue requires energy. Specify


the way of ATP formation in this tissue.
A. Anaerobic glycolysis D. Photosynthetic phosphorylation
B. Creatine phosphate pathway E. Oxidation of fatty acids
C. Oxidative phosphorylation in
mitochondria

EXAMPLES OF KROK 1 TESTS

44. Examination of a patient revealed erosion of enamel. What vitamin


should be prescribed in this case?
A. D D. A
B. C E. PP
C. K

45. Vitamin A was prescribed to a patient with periodontitis. What process


is activated under the influence of this vitamin and provides a therapeutic
effect?
A. Growth and differentiation of cells C. Carboxylation of acids
B. Hydroxylation of proline D. Color vision

46. Antioxidants of natural and artificial origin are used during the treatment
of periodontal disease. Which of the following is used as antioxidant agent?
A. Taurine C. Gluconate
B. Thiamine D. Pyridoxine
E. Choline

47. Excess of fluoride in water leads to fluorosis. What element can be used
for elimination of toxic effects of fluoride on the tooth?
A. Calcium D. Mercury
B. Potassium E. Selenium
C. Sodium

48. Human body obtains vitamins, macro-and micronutrients from dietary


uptake. Elevated quantities of what trace element has the most pronounced
effect on the formation of caries?
A. Iron D. Strontium
B. Barium E. Copper
C. Selenium

49. Examination of a child revealed the initial stages of caries. Which


products should predominate in the diet of the patient?
A. Protein, vitamins C, E A, minerals D. Nucleic acids and
B. Fats and fat-soluble vitamins micronutrients
C. Carbohydrates and water soluble E. Vegetable proteins, vitamins,
vitamins minerals

50. Disorders of the enamel and dentin formation were observed due to a
low content of calcium in the blood. What hormone deficiency may cause
such disorders?
A. Parathyroid hormone D. Somatotropin
B. Triiodothyronine E. Thyroxine
C. Insulin

51. Plaque - a pathological formation on the surface of the teeth. The main
sources of minerals in the formation of plaque are:
A. Gingival fluid D. Blood plasma
B. Oral liquid E. Blood serum
C. Saliva

52. Examination of a patient revealed plaque depositions. What compound


is essential for plaque formation?
A. Calcium phosphate D. Pigments
B. Calcium oxalate E. Oxaloacetate
C. Urate

53. In humans, substances with osteotropic effects are synthesized. Which of


the listed substances possess these properties?
A. Parotin C. Insulin
B. Thyroxin D. Nerve growth factor
E. Secretory immunoglobulin A

54. Hypovitaminosis of vitamin C reduces the formation of the organic matrix of


teeth, delayed remineralization processes; leads to disturbance of collagen
synthesis. This occurs, because this vitamin is involved in the process of...
A. Hydroxylation of proline and lysine D. Hydroxylation of proline
B. Carboxylation of proline E. Hydroxylation of lysine
C. Carboxylation of lysine

55. In dental practice special pasts are used. They are rich in fat-soluble vitamins
A and D, because:
A. These vitamins regulate metabolism of C. These vitamins activate energy
heteropolysaccharides of tooth and metabolism in tissues of the tooth
promote deposition of calcium salts D. Promote replacement of strontium
B. These vitamins help the transformation apatite by hydroxyapatite
of procollagen to collagen that results in E. Provides antioxidant properties of
remineralization tissues

56. In the treatment of periodontitis, hormones that stimulate the mineralization


of teeth and inhibits of bone resorption are used, namely:
A. Calcitonin D. Dopamine
B. Insulin E. Thyroxin
C. Adrenalin

57. A disorder in the structure of collagen fibers associated with a deficiency of


vitamin C leads to tooth loss. This can be explained, because vitamin C is a
cofactor of:
A. Lysine hydroxylase and proline C. Glycosil transferase
hydroxylase D. Procollagen peptidase
B. Lysine hydroxylase and lysine oxidase

58. To prevent gingivitis and to improve the regeneration of epithelial cells


special vitamins are added to the periodontal toothpaste. Choose from the list
one of the following vitamins that is used in this case:
A. Retinol D. Biotin
B. Thiamine E. Calciferol
C. Phylochinon

59. A 3 years old child was hospitalized with symptoms of severe stomatitis and
gingivitis, as well as dermatitis of open parts of the body. The examination
revealed violations of inherited transport of neutral amino acids in the small
intestine. What vitamin’s insufficiency leads to the above symptoms?
A. Niacin D. Biotin
B. Vitamin A E. Pantothenic acid
C. Cobalamin
60. In the complex treatment of periodontitis, tocopherol was used. What effect
causes the healing properties of this vitamin?
A. Antioxidant D. Osteotropic
B. Antiphlogistic E. Prooxydant
C. Antialergic

61. Examination of a patient revealed gingivitis accompanied by hypoxia. The


formation of what metabolite of carbohydrate metabolism is increased in the
periodontal tissues?
A. Lactate D. Glucose-6-phosphate
B. Ribose-5-phosphate E. NADPH
C. Glycogen

62. During the treatment of paradontosis antioxidants are used. What


natural compound, found in food, serves as antioxidant?
A. Tocopherol D. Pyridoxol
B. Thiamine E. Choline
C. Calcitriol

63. During inflammation of periodontal tissues hypoxia occurs. This leads


to changes of the ratio of ATP / ADP + Pi, resulting activation of
phosphofructokinase. What metabolic process is activated?
A. Glycolysis D. Oxidation of fatty acids
B. PPP E. Synthesis of purine nucleotides
C. Ornithine cycle

64. Examination of a patient revealed that five teeth were affected by caries.
What changes in the concentration of free amino acids (tryptophan,
threonine, glutamate, aspartate, lysine) in the mixed saliva are observed?
A. Concentration is increased C. Do not change
B. Significantly reduced D. Decreased

65. Admission of calcium and phosphorus are essential for mineralization


of tooth tissue. The main source of these macroelements in the tissues of
the tooth is:
A. Oral liquid D. Gingival fluid
B. Blood E. Interstitial fluid
C. Drinking water

66. Inadequate enforcement of oral care leads to increasing in teeth plaque


formation. What enzymes are activated in such conditions?
A. Liases D. Hydrolases
B. Transferases E. Ligases
C. Oxidoreductases
67. Mineralization of tooth tissue is regulated by many hormones. What
hormone of salivary glands contributes to this process?
A. Parotin C. Calcitonin
B. Parathyroid hormone D. Kalidin

68. The main role in the process of calcification of tooth tissue plays
osteocaltcin. This protein has the ability to bind calcium ions. Indicate
special amino acid that provides this function.
A. Carboxy glutamine C. Oxyproline
B. Alanine D. Proline

69. Decreased resistance of tooth enamel is observed. Which of the


following biochemical processes provides the highest resistance of tooth
enamel?
A. Synthesis fluorapatite D. Synthesis of collagen
B. Synthesis of hydroxyapatite E. Synthesis of carbonate apatite
C. Synthesis of chlorine apatite

70. Excessive intake of fluoride is harmful. What salt solution should be


used for removal of the toxic effects of fluoride on the tooth?
A. Solution of calcium salts C. Solution of sodium salts
B. Solution of mercury salts D. Solution of potassium salts

71. Phosphates are necessary for normal development of tooth tissue. What
is the most effective source of organic phosphorus, which is used by the
body for the mineralization processes?
A. Phosphatidylcholine D. DNA, RNA
B. Phosphoproteins E. Mononucleotides
C. Glycerophosphate

72. Metabolism of tooth tissue takes place with the participation of


hormonal regulation. Which of the following hormones stimulates the
inclusion of calcium ions in the osteoblasts of tooth tissue?
A. Calcitonin D. Insulin
B. Parathyroid hormone E. Thyroxin
C. Cortisone

73. Hypoxia occurs during periodontal inflammation. This disorder is


accompanied by accumulation of lactate in the periodontal tissues due to
conversion of pyruvic acid to…
A. Acetyl-CoA D. Glycerol
B. Oxaloacetic acid E. Glucose
C. Alanine
74. Examination of a patient revealed bleeding gums, and periodontitis.
What from the listed below drugs can be effectively used in treatment of
this disease?
A. Vitamins A and C D. Nicotinic acid
B. Biotin E. Glutamate
C. Vikasol

75. Demineralization of enamel is the main cause of tooth decay. Which of


the above factors causes demineralization of enamel?
A. Proteolytic enzymes, organic acids D. Urea, ammonia,
B. Saturated fatty acids E. Buffer systems of saliva
C. Glucose, starch

76. Examination of a patient revealed gingivitis. Overexpression of which of


the following enzymes is observed during this disease?
A. Collagenase, elastase D. Neuraminidase, phospholipase
B. Proteases, hyaluronidase C
C. DNAse, RNAse

77. Periodontitis is accompanied by activation of proteolysis in the


periodontal tissues. What component of oral fluid indicates activation of
proteolysis?
A. Amino acids D. Biogenic amines
B. Organic acids E. Cholesterol
C. Glucose

78. During investigation of oral cavity of patient dentist revealed dryness


and numerous erosions of mucosa. Deficiency of what vitamin may
cause these changes?
A. Retinol D. Rhutin and flavonoids
B. Biotin E. Pyridoxine
C. Menaquinon

79. Changes in chemical composition of saliva are observed not only in


diseases of the oral cavity, but also in diseases of the digestive tract. What
somatic pathology is accompanied by decreased activity of lysozyme in
saliva?

A. Stomach cancer D. Gastritis


B. Hepatitis E. Dysbacteriosis
C. Pancreatitis

80. What pathology can be suggested when the activity of amylase in saliva
of a patient is decreased?
A. Insufficient secretory function of C. Parotis hyperfunction
parotis D. Sublinqual gland hyperfunction
B. Submandibular gland hypofunction E. Sublinqual gland hypofunction

81. Many diseases are frequently accompanied with changes in content of


different metabolites in saliva. What substance concentration may be
increased in saliva of patient suffering from diabetes mellitus?
A. Glucose D. Albumin
B. Creatinine E. Zinc ions
C. Urea

82. What component of saliva significantly increases the risk of caries


development in case of diabetes mellitus?
A. Glucose D. Residual nitrogen
B. Urea E. Ammonia
C. Amino acids

83. Excessive content of glucose in saliva in persons with diabetes mellitus


leads to development of the following pathology:
A. Multiple teeth lesions with tooth decay D. Fluorosis
B. Enamel hyperplasia E. Increased calcification of
C. Enamel hypoplasia enamel

84. In a patient developed an intensive hypersalivation, which may cause


partial neutralization of hydrochloric acid in gastric juice. Digestion of
what substances will be damaged in this case?
A. Proteins D. Cholesterides
B. Nucleic acids E. Carbohydrates
C. Lipids

85. Somatic diseases as usually are accompanied with changes in


composition of saliva. What disease may cause the increase in residual
nitrogen in saliva?
A. Renal insufficiency D. Infectious hepatitis
B. Acute pancreatitis E. Perforation of gastric ulcer
C. Acute appendicitis

86. During investigation of saliva in patient with periodontitis an increase in


products of free radical oxidation was revealed. What is the influence of
excessive generation of these products upon tissues of oral cavity?
A. Damage of cells D. Hypersalivation
B. Hyposalivation E. Changes in chemical
C. Lowering of pH of saliva composition of saliva
87. In a patient with paradontitis the increase in activity of acid phosphatase
and hyaluronidase in saliva is observed. How can be interpereted this
changes?
A. Acceleration of catabolism of D. Activation of free radical
biomolecules in parodont processes
B. Enhancement of proteins excretion E. Insufficiency of protective
with saliva function of parodont tissue
C. Disorder of hormonal regulation of
metabolism in parodont

88. During investigation of mixed saliva of the patient with parodontitis


dentist detected an increase in content of free amino acids. How can be
interpreted this result?
A. Activation of proteolytic enzymes D. Negative nitrogen balance
B. Activation of nucleic acids degradation E. Insufficiency of water soluble
in tissues of parodont vitamins
C. Activation of free radical oxidation
processes

89. In a patient with erosive stomatitis was detected enhanced


concentration of chromium, nickel, cobalt, iron due to corrosion of
prosthesis from chromo-nickel alloys. The most probable cause of this
state can be:
B. Shift of pH in saliva to acidic side F. Excessive content of fluorine in
C. Hypersalivation drinking water
D. Shift pH in saliva to alkaline side
E. Poisonong with salts of heavy metals

90. In a patient a progressive demineralization of enamel is observed, pH of


saliva is 6.0. What type of diet is recommended to be restricted?
A. Enriched in carbohydrates D. Rich in lipids
B. Enriched in vitamins E. Enriched in unsaturated fatty
C. Rich in proteins acids.

91. Caries develops as a consequence of the effect of many factors. What is


the principal mechanism of multiple caries development in case of
hyposalivation?
A. Decrease in mineralization capacity of D. Insufficient mechanical
saliva processing of meal
B. Insufficiency of neuropeptides E. Decrease in vitamins
secretion providement
C. Decrease of hormones level
CLINICAL CASES AND SITUATIONAL TASKS

92. What is the purpose of a salivary gland during digestion?

Answer: The salivary gland produces amylase that helps to break down starches
and liquefy the food in the chewing phase of digestion. Once swallowed, the food
is exposed to more enzymes and processes that complete the digestion as it
travels through the various areas of the digestive tract.

93. What salivary glands are inflamed during mumps?

Anser: The most common salivary gland infection in children is mumps, which
involves the parotid glands. The parotid gland is the largest of the salivary glands.
It is found wrapped around the mandibular ramus, and it secretes saliva through
Stensen's duct into the oral cavity, to facilitate mastication and swallowing.
Parotid inflammation (or parotitis) occurs in 60-70% of infections and 95% of
patients with symptoms. Parotitis causes swelling and local pain, particularly
when chewing. It can occur on one side (unilateral) or both sides (bilateral).

94. Usually salivary gland cancer may not cause any symptoms. It is
sometimes found during a regular dental check-up or physical exam. How is
salivary gland cancer diagnosed? What is the treatment for salivary gland
cancer?

Answer: Typically these present as a lump of the parotid gland, submandibular


gland, mouth or throat. Adenoid cystic carcinoma may also include facial paralysis
or numbness. Salivary duct carcinoma patients more commonly have lymph node
metastasis.
A diagnosis is made after a biopsy of the affected tissue(s). Currently, the
standard practice is complete surgical excision. Depending on the histological
findings, some patients (most high grade tumors) may also undergo post-
operative radiation therapy. However, almost 25% of patients develop recurrence
and/or metastasis within 5 to 10 years.
SECTION XIX.
BIOCHEMICAL REACTIONS

Write biochemical conversions, shown below inserting missed metabolites


(x1-xn):
.
1. Stearate → х1 → Acetyl-CoA → х2 → Hydroxymethylglutaryl-CоА → х3 →
Acetone

2. Ethanol → х1 → Acetate → х2 → Palmitate → х3 → Diacylglycerole

3. Arginilglycine → х1 → Guanidineacetate → x2 → Creatinephosphate →


АТP → Carbamoil phosphate → Citrulline → Urea

4. Pyruvate → Alanine → NADH2 → Glycerol-3-phosphate → Phosphatidic


acid → x1 → Glucose-1-phosphate → x2 → Glucose

5. Galactose → x1 → Galactose-1-phosphate → x2 → Glucose-6-phosphate


→ NADPН2 → Mevalonate → Heme

6. α-Ketoglutarate → Oxaloacetate → x1 → 2-phosphoglycerate → x2 →


1,3-diphosphoglycerate → x3 → dioxyacetonephosphste → 3-phospoglycerate

7. Succinyl-CоА → x1 → Fumarate → x2 → Oxaloacetate → Aspartate →


Argininosuccinate → Arginine → Nitric oxide (NO)

8. Pyruvate → x1 → Phosphoenolpyruvate → АТP → Orthophosphate → x2


→ Glucose-6-phosphate

9. Glycerol → x1 → Dioxyaceton phosphate → x3 → Glucose-6-phosphate


→ Glucose → Glicogen

10. Pyruvate → α-Ketoglutarate → x1 → Glutamine → x2 → Carbamoyl


phosphate → Citrulline → Argininosuccinate → x3 → Nitric oxide

11. Maltose → x1 → Glucose-6-phosphste → x2 → UDP-glucose → x3 →


Glucose

12. Glicogen → x1 → Fructose-1,6-diphosphate → x2 → Phosphoglicerol →


Phosphatidic acid → Lecithin

13. Palmitate → x1 → Acetyl-CоА → x2 → cys-Aconitate → х3 → α-


Ketoglutarate → Glutamate → Glutamine

14. Succinate → х1 → Acetoacetyl-CоА → х2 → Citrate → х3 → Isocitrate →


х4 → х5 → Succinate → Fumarate → х6 → Oxaloacetate
15. Serine → х1 → Glutamine → Glutamate → Ammonia → Carbamoyl
phosphate → х2 → Argininosuccinate → х3 → Urea

16. GTP → GMP → х1 → Guanine → х2 → Uric acid

17. Arginine → х1 → Citrulline → х2 → Fumarate→ х3 → Oxaloacetate →


Aspartate → Asparagine → Ammonia salts

18. Oxaloacetate → х1 → Carbamoyl phosphate → х2 → Orotate →


Оrotidine-5’-phosphate → UMP

19. UMP → х1 → х2 → CTP → CDP-serine → Phosphatylserine

20. Asparagin → х1 → Carbamoyl phosphate → Carbamoyl aspartate → х2


→ Orotate → х3 → UMP

21. Hemoglobine → х1 → Biliverdin → х2 → Mesobilirubin→ Stercobilin

22. Alanine → х1 → Glutamine → Glutamate → Ammonia → Carbamoyl


phosphate → х2 → Argininosuccinate → х3 → Nitric oxide (NO)

23. Protein → Tyrosine→ х1 → Dopamine → х2 → Epinephrine

24. АTP → АМP → х1 → Inosine → х2 → Xantine → Uric acid

25. Argininosuccinate → Arginine → х1 → Citrulline → х2 → Fumarate → х3


→ Oxaloacetate → Aspartate → Asparagine → Carbamoyl phosphate

26. Pyruvate → Oxaloacetate → х1 → Citrate → х2 → Isocitrate → α-


Ketoglutarate → Glutamate

27. α- Ketoglutarate → Succinyl-CоА → δ-Аminolevulinic acid → х1 →


Protoporphin-ІX → Heme → Hemoglobin

28. UMP → х1 → х2 → UTP → UDP-choline → Phosphatidylecholine

29. Pehenylalanine → Тyrosine → Dioxyphenylalanine → х2 →


Norepinephrine → Epinephrine

30. Aspartic acid→ х1 → Carbamoyl phosphate → Carbamoyl aspartate →


X2 → Оrotic acid

31. Glycerol → х1 → Dioxyacetonephosphate → х2 → 1,3-


Dioxyacetonephosphate → АТP → Pyrophosphate
32. Protein → Alanine → х1 → Аcetyl-CоА → х2 → Acetoacetate →
Аcetoacetyl-CоА → 2-аcetyl-CоА → х3 → Аcetone

33. Pyrophosphate → х1 → Glucоse-1-phosphate → х2 → Оrthophosphate


→ 1,3-diphosphogycerate → АТP → Fructose-6-phosphate → х3 →
dioxyacetonephosphate

34. Cystine → Cysteine → Glutamate → х1 → α-Ketoglutarate →


Oxaloacetate → х2 → АТP

35. Alanine → Pyruvate → х1 → Carbamoyl phosphate → Citrulline → х2


→ Arginine → Creatine → Creatinine

36. Pyruvate → х1 → Acetyl-CоА → х2 → cys-Аconitate → х3 → NADН2 →


3АТP

37. Glutamate → х1 → α-Кetoglutarate → Oxaloacetate → х2 → АТP →


х3 → Glucose-6-phosphate

38. Glucose → Glucose-6-phosphate → NАDPН2 → Glutamate →


Glutamine → х1

39. АТP → х1 → 3-phosphogyceraldehyde → х2 → Phructose-1,6-


diphosphate → х3 → Glucose-6-phosphate → Glucose

40. Arginine → х1 → х2 → Phosphocreatine → Оrthophosphate → Glucose-


1-phosphate → х3 → Glucose → UDP-glucose

41. Fructose-1,6-diphosphate → x1 → Dioxyacetonephosphate → x2 →


Phosphatidic acid → Phosphatidylinositol

42. NАDН2 → Glycerol-3-phosphate → Phosphatidic acid → х1 → Glucose-


1-phosphate → х2 → Glucose → х3 → Glycogen

43. Fumaric acid → х1 → Oxaloacetate → х2 → Argininosuccinate → х3 →


Urea

44. Acetyl-CоА → Fatty acid → Acyl-CоА → Phosphatidic acid →


Оrthophosphate → АТP → х1 → Glucose-1-phosphate → х2 → Glycogen

45. Glucose → Glucose-6-phosphate → х1 → Tyrosine → х2 →


Homogentistic acid → х3 → Fumarylacetoacetate

46. β-Hydroxyburiric acid → х1 → Acetoacetyl-CоА → х2 → Citrate → х3 →


Isocitrate → α-ketoglutarate + СО2
47. GTP → Phosphoenolpyruvate → х1 → S-Adenosyl methionine →
Phosphatidylcholine → Diacylglicerole → х2 → Acyl-CоА → Acetyl-CоА

48. Aspartate → х1 → Аmmonia → Glutamate → α-Кеtoglutarate → СО2 →


Oxaloacetate → х2 → Citrate

49. Acetyl-КоА → х1 → cys-Аconitate → х2 → СО2 → х3 → Citrulline →


Argininosuccinate → Fumarate

50. 2-phosphoglycerate → х1 → 1,3-diphosphoglycerate → х2 →


Dioxyacetonephosphate → Fructose-1,6-diphosphate → Phosphogliceraldehyde
→ Dioxyacetonephosphate

51. Acetoacetate → СО2 → Malonyl-CоА → Fatty acid → х1 → Phosphatidic


acid → Cephaline

52. Fatty acid → х1 → Phosphatidic acid → х2 → Phosphatidylethanoamine


→ Ethanolamine

53. Fructose-6-phosphate → х1 → Dioxyacetonephosphate → х2 → х3 →


Diacylglicerol → х4 → Fatty acid → Acetyl-CoA → Acetoacetyl-CоА → х5 →
Hydroxybutirate

54. Citrate → х1 → х2 → α-Ketoglutarate → Glutamate → Aspartate → х3 →


Arginine → х4 → Creatine → Creatinine

55. Amino acid → х1 → Glutamine → х2 → GABA → х3 → NH4+

56. Alanine → х1 → Аcetyl-CоА → х2 → Hydroxymethylglutaryl-CоА → х3


→ х4 → Аcetyl-CоА → Fatty acid

57. Succinyl-CоА → х1 → Porphobilinogen → х2 → х3 →


Coproporphyrinogen ІІІ → х4 →х5 → Heme

58. Acetyl-CоА → х1 → cys-Aconitate → х2 → СО2 → х3 → Citrulline →


Аspartate → х4 → Аrginine → Nitric oxyde (NO)

59. Acetyl-CоА → х1 → Hydroxymethylglutaryl-CоА → х2 → Oxybutirate


REFERENCES:

1. Satyanarayana U., Chakrapani U. “Biochemistry”, Third Edition. – 2006. –


792 p.
2. Trudy McKee, James R. McKee “Biochemistry”. - 1999, 288 p.
3. Murray R. K., Granner D. K., Mayes P. A., Rodwell V. W. “Harper′s
Biochemistry”.(26th edition) - 2003, 927 p.
4. Brownie A.C., Kernohan J.C. “Biochemistry. A core text with self-
assessment” .-Churchill Livingstone. - 1999. - 288 p.
5. Devlin T.M., ed. Textbook of Biochemistry with Clinical Correlations, 5th ed.
New York: Wiley-Liss, 2002.
6. Geoffrey L. Zubay, William W. Parson, Dennis E. Vance. Principles of
Biochemistry – Wm. C. Brown Publishers. – 1995. – 864 p.
7. Koolman J., Rom K.-H. Color Atlas of Biochemistry. – Stuttgart. New York.
–Thieme Verlag. –– 1996. – 435 p.
8. Lehninger A. Principles of Biochemistry. – New York. – W.H.Freeman
and Company. – 2005. – 1010 p.
9. Champe P. C., Harvey R. A. “Biochemistry. 2 nd edition” Lippincott′s
Illustrated Reviews. – 1994. – 443 p.
10. Toy E.C., Seifert W. E., Strobel H.W., Harms K.P. “Case Files in
Biochemistry. 2nd edition ” – 2008. – 488 p.

Potrebbero piacerti anche